Location via proxy:   [ UP ]  
[Report a bug]   [Manage cookies]                

SUMA204

Download as pdf or txt
Download as pdf or txt
You are on page 1of 540

SUMA204

UNDERGRADUATE COURSE
B.Sc., MATHEMATICS
SECOND YEAR
FOURTH SEMESTER

CORE PAPER - VII

TRANSFORM TECHNIQUES

INSTITUTE OF DISTANCE EDUCATION


UNIVERSITY OF MADRAS
B.Sc., MATHEMATICS CORE PAPER - VII
SECOND YEAR - FOURTH SEMESTER TRANSFORM TECHNIQUES

WELCOME
Warm Greetings.

It is with a great pleasure to welcome you as a student of Institute of Distance


Education, University of Madras. It is a proud moment for the Institute of Distance education
as you are entering into a cafeteria system of learning process as envisaged by the University
Grants Commission. Yes, we have framed and introduced Choice Based Credit
System(CBCS) in Semester pattern from the academic year 2018-19. You are free to
choose courses, as per the Regulations, to attain the target of total number of credits set
for each course and also each degree programme. What is a credit? To earn one credit in
a semester you have to spend 30 hours of learning process. Each course has a weightage
in terms of credits. Credits are assigned by taking into account of its level of subject content.
For instance, if one particular course or paper has 4 credits then you have to spend 120
hours of self-learning in a semester. You are advised to plan the strategy to devote hours of
self-study in the learning process. You will be assessed periodically by means of tests,
assignments and quizzes either in class room or laboratory or field work. In the case of PG
(UG), Continuous Internal Assessment for 20(25) percentage and End Semester University
Examination for 80 (75) percentage of the maximum score for a course / paper. The theory
paper in the end semester examination will bring out your various skills: namely basic
knowledge about subject, memory recall, application, analysis, comprehension and
descriptive writing. We will always have in mind while training you in conducting experiments,
analyzing the performance during laboratory work, and observing the outcomes to bring
out the truth from the experiment, and we measure these skills in the end semester
examination. You will be guided by well experienced faculty.

I invite you to join the CBCS in Semester System to gain rich knowledge leisurely at
your will and wish. Choose the right courses at right times so as to erect your flag of
success. We always encourage and enlighten to excel and empower. We are the cross
bearers to make you a torch bearer to have a bright future.

With best wishes from mind and heart,

DIRECTOR

(i)
B.Sc., MATHEMATICS CORE PAPER - VII
SECOND YEAR - FOURTH SEMESTER TRANSFORM TECHNIQUES

COURSE WRITER

Dr. R. Sakthivel
Assistant Professor
Department of Mathematics
Pachaiyappa’s College
Chennnai - 600 030.

COORDINATION AND EDITING

Prof. V. THANGARAJ
Former Director and Head
Ramanujan Institute for
Advanced Study in Mathematics
University of Madras,
Chennai - 600 005.

© UNIVERSITY OF MADRAS, CHENNAI 600 005.

(ii)
B.Sc –Mathematics - Second Year- Fourth Semester
Core Paper 7 – TRANSFORM TECHNIQUES
Syllabus

UNIT I: Introduction – Properties of Laplace transform- Laplace transform of elementary


functions-Problems using properties-Laplace transform of special function, unit step function
and Dirac delta function - Laplace transform of derivatives and Integrals – Evaluation of integral
using Laplace Transform - Initial Value Theorem – Final Value Theorem and problems –Laplace
Transform of periodic function.
Chapter 2: Section 2.1 to 2.20

UNIT II: Introduction, Properties of inverse Laplace transform, Problems (usual types);
Convolution Theorem - Inverse Laplace Transform using Convolution theorem.
Chapter 3: Section 3.1 to 3.11

UNIT III: Introduction, Expansions of periodic function of period 2π; expansion of even and
odd functions; half range cosine and sine series – Fourier series of change of interval.
Chapter 1: Section 1.1 to 1.11

UNIT IV: Introduction of Fourier transform - Properties of Fourier Transforms - Inverse Fourier
transform – Problems, Fourier sine and cosine transforms and their inverse Fourier transform –
Problems, Convolution theorem, Parseval’s identity and problems using Parseval’s identity.
Chapter 4: Section 4.1 to 4.12

UNIT V: Applications of Laplace transform to solution of first and second order linear differential
equations (constant coefficients) and simultaneous linear ordinary differential equations –
Application of Laplace transform to partial differential equations. Application of Laplace
Transform and Fourier transform to Initial and Boundary Value Problems.
Chapter 5: Section 5.1, 5.3, 5.7 to 5.11

v
Recommended Text Book:
Content and treatment as in:
S. Sreenath, S. Ranganatham, M.V.S.S.N. Prasad and V. Ramesh Babu, Fourier Series and Integral
Transforms, S.Chand Publishing, New Delhi, 2014.

Books for study and reference:

1. M.K. Venkataraman, Engineering Mathematics(Volume 3), National Publishing Co.,


Chennai,2001.

2. P.Kandasamy and others, Engineering Mathematics,(Volume 3), S.Chand and Co., New
Delhi,2010.

3. Stanley I. Grossman and William R. Derrick, Advanced Engineering Mathematics, Harper and
Row, New York, 1988.

vi
Lessons

1 BASIC CONCEPTS AND LAPLACE TRANSFORM OF ELEMENTARY


FUNCTION 1
1.1 Introduction . . . . . . . . . . . . . . . . . . . . . . . . . . . . . . . . . . . . . 1
1.2 Definition . . . . . . . . . . . . . . . . . . . . . . . . . . . . . . . . . . . . . . 2
1.3 Sufficient Conditions For the Existence of the Laplace Transform of a Function . 2
1.4 Definitions . . . . . . . . . . . . . . . . . . . . . . . . . . . . . . . . . . . . . . 3
1.5 Linearity Property . . . . . . . . . . . . . . . . . . . . . . . . . . . . . . . . . . 6
1.6 Laplace Transform of Some Elementary Functions . . . . . . . . . . . . . . . . . 7
1.7 Worked out Problems . . . . . . . . . . . . . . . . . . . . . . . . . . . . . . . . 12
1.8 Exercises . . . . . . . . . . . . . . . . . . . . . . . . . . . . . . . . . . . . . . 25

2 FIRST SHIFTING THEOREM AND SECOND SHIFTING THEOREM 27


2.1 First Translation (or) First Shifting Theorem . . . . . . . . . . . . . . . . . . . . 27
2.2 Worked out Problems (Problems based on first shifting theorem) . . . . . . . . . 29
2.3 Unit Step Function(Heaviside Unit Step Function) . . . . . . . . . . . . . . . . . 33
2.4 Second Translation (or) Second Shifting Theorem . . . . . . . . . . . . . . . . . 34
2.5 Worked out Problems . . . . . . . . . . . . . . . . . . . . . . . . . . . . . . . . 36
2.6 Change of Scale of Property . . . . . . . . . . . . . . . . . . . . . . . . . . . . 37
2.7 Worked out Problems . . . . . . . . . . . . . . . . . . . . . . . . . . . . . . . . 38
2.8 Exercises . . . . . . . . . . . . . . . . . . . . . . . . . . . . . . . . . . . . . . 40

3 LAPLACE TRANSFORM OF DERIVATIVES AND INTEGRALS 41


3.1 Laplace Transform of Derivatives . . . . . . . . . . . . . . . . . . . . . . . . . . 41
vii
viii LESSONS

3.2 Initial Value Theorem . . . . . . . . . . . . . . . . . . . . . . . . . . . . . . . . 43


3.3 Final Value Theorem . . . . . . . . . . . . . . . . . . . . . . . . . . . . . . . . 44
3.4 Worked out Problems . . . . . . . . . . . . . . . . . . . . . . . . . . . . . . . . 45
3.5 Exercises . . . . . . . . . . . . . . . . . . . . . . . . . . . . . . . . . . . . . . 47
3.6 Laplace Transform of Integrals . . . . . . . . . . . . . . . . . . . . . . . . . . . 48
3.7 Worked out Problems . . . . . . . . . . . . . . . . . . . . . . . . . . . . . . . . 48
3.8 Note on Differentiation under the Integral Sign . . . . . . . . . . . . . . . . . . . 49
3.9 Laplace Transform of tn F(t) . . . . . . . . . . . . . . . . . . . . . . . . . . . . 50
3.10 Worked out Problems . . . . . . . . . . . . . . . . . . . . . . . . . . . . . . . . 52
3.11 Division by t . . . . . . . . . . . . . . . . . . . . . . . . . . . . . . . . . . . . 63
3.12 Worked out Problems . . . . . . . . . . . . . . . . . . . . . . . . . . . . . . . . 64
3.13 Evaluation of Integrals by using Laplace Transform . . . . . . . . . . . . . . . . 74
3.14 Worked out Problems . . . . . . . . . . . . . . . . . . . . . . . . . . . . . . . . 74
3.15 Exercises . . . . . . . . . . . . . . . . . . . . . . . . . . . . . . . . . . . . . . 80

4 LAPLACE TRANSFORM OF SOME SPECIAL FUNCTIONS AND PERIODIC


FUNCTIONS 83
4.1 Laplace Transform of Some Special Functions . . . . . . . . . . . . . . . . . . . 83
4.2 Worked out Examples . . . . . . . . . . . . . . . . . . . . . . . . . . . . . . . . 89
4.3 Laplace Transform of Periodic Functions . . . . . . . . . . . . . . . . . . . . . . 91
4.4 Worked out Problems . . . . . . . . . . . . . . . . . . . . . . . . . . . . . . . . 93
4.5 Exercises . . . . . . . . . . . . . . . . . . . . . . . . . . . . . . . . . . . . . . 97

5 THE INVERSE LAPLACE TRANSFORM 99


5.1 Introduction . . . . . . . . . . . . . . . . . . . . . . . . . . . . . . . . . . . . . 99
5.2 Worked out Problems . . . . . . . . . . . . . . . . . . . . . . . . . . . . . . . . 100

6 INVERSE LAPLACE TRANSFORM BY PARTIAL FRACTIONS 107


6.1 Introduction . . . . . . . . . . . . . . . . . . . . . . . . . . . . . . . . . . . . . 107
6.2 Worked out Problems . . . . . . . . . . . . . . . . . . . . . . . . . . . . . . . . 107
Transform Techniques B.Sc.(Mathematics)-IDE(UNOM)-II Year - IV Sem
LESSONS ix

6.3 Exercises . . . . . . . . . . . . . . . . . . . . . . . . . . . . . . . . . . . . . . 114

7 FIRST SHIFTING THEOREM 117


7.1 Introduction . . . . . . . . . . . . . . . . . . . . . . . . . . . . . . . . . . . . . 117
7.2 Worked out Problems . . . . . . . . . . . . . . . . . . . . . . . . . . . . . . . . 118
7.3 Exercises . . . . . . . . . . . . . . . . . . . . . . . . . . . . . . . . . . . . . . 126
7.4 Second Shifting Theorem . . . . . . . . . . . . . . . . . . . . . . . . . . . . . . 126
7.5 Worked out Problems . . . . . . . . . . . . . . . . . . . . . . . . . . . . . . . . 128
7.6 Change of Scale of Property . . . . . . . . . . . . . . . . . . . . . . . . . . . . 132
7.7 Worked out Problems . . . . . . . . . . . . . . . . . . . . . . . . . . . . . . . . 132
7.8 Exercises . . . . . . . . . . . . . . . . . . . . . . . . . . . . . . . . . . . . . . 134

8 INVERSE LAPLACE TRANSFORM OF DERIVATIVES AND INTEGRALS 135


8.1 Introduction . . . . . . . . . . . . . . . . . . . . . . . . . . . . . . . . . . . . . 135
8.2 Inverse Laplace Transform of Derivatives . . . . . . . . . . . . . . . . . . . . . 135
8.3 Worked out Problems . . . . . . . . . . . . . . . . . . . . . . . . . . . . . . . . 136
8.4 Inverse Laplace Transform of Integrals . . . . . . . . . . . . . . . . . . . . . . . 140
8.5 Worked out Problems . . . . . . . . . . . . . . . . . . . . . . . . . . . . . . . . 141
8.6 Exercises . . . . . . . . . . . . . . . . . . . . . . . . . . . . . . . . . . . . . . 143
8.7 Multiplication by Powers of p . . . . . . . . . . . . . . . . . . . . . . . . . . . 144
8.8 Worked out Problems . . . . . . . . . . . . . . . . . . . . . . . . . . . . . . . . 144
8.9 Division by p . . . . . . . . . . . . . . . . . . . . . . . . . . . . . . . . . . . . 148
8.10 Worked out Problems . . . . . . . . . . . . . . . . . . . . . . . . . . . . . . . . 148
8.11 Exercises . . . . . . . . . . . . . . . . . . . . . . . . . . . . . . . . . . . . . . 155

9 CONVOLUTION THEOREM 157


9.1 Convolution . . . . . . . . . . . . . . . . . . . . . . . . . . . . . . . . . . . . . 157
9.2 Worked out Problems . . . . . . . . . . . . . . . . . . . . . . . . . . . . . . . . 159
9.3 Exercises . . . . . . . . . . . . . . . . . . . . . . . . . . . . . . . . . . . . . . 172

Transform Techniques B.Sc.(Mathematics)-IDE(UNOM)-II Year - IV Sem


x LESSONS

10 FOURIER SERIES 175


10.1 Introduction . . . . . . . . . . . . . . . . . . . . . . . . . . . . . . . . . . . . . 175
10.2 Periodic Function . . . . . . . . . . . . . . . . . . . . . . . . . . . . . . . . . . 176
10.3 Euler’s Formulae . . . . . . . . . . . . . . . . . . . . . . . . . . . . . . . . . . 177
10.4 Conditions for Fourier Expansion . . . . . . . . . . . . . . . . . . . . . . . . . . 180
10.5 Worked out Problems . . . . . . . . . . . . . . . . . . . . . . . . . . . . . . . . 182
10.6 Exercises . . . . . . . . . . . . . . . . . . . . . . . . . . . . . . . . . . . . . . 207

11 FOURIER SERIES FOR DISCONTINUOUS FUNCTIONS 209


11.1 Introduction . . . . . . . . . . . . . . . . . . . . . . . . . . . . . . . . . . . . . 209
11.2 Worked out Problems . . . . . . . . . . . . . . . . . . . . . . . . . . . . . . . . 211
11.3 Exercises . . . . . . . . . . . . . . . . . . . . . . . . . . . . . . . . . . . . . . 232

12 FOURIER SERIES OF EVEN AND ODD FUNCTIONS 235


12.1 Introduction . . . . . . . . . . . . . . . . . . . . . . . . . . . . . . . . . . . . . 235
12.2 Even and Odd Functions . . . . . . . . . . . . . . . . . . . . . . . . . . . . . . 235
12.3 Fourier Series for Even and Odd Functions . . . . . . . . . . . . . . . . . . . . . 236
12.4 Worked out Problems . . . . . . . . . . . . . . . . . . . . . . . . . . . . . . . . 238
12.5 Exercises . . . . . . . . . . . . . . . . . . . . . . . . . . . . . . . . . . . . . . 256

13 HALF RANGE FOURIER SERIES 259


13.1 Introduction . . . . . . . . . . . . . . . . . . . . . . . . . . . . . . . . . . . . . 259
13.2 Worked out Problems . . . . . . . . . . . . . . . . . . . . . . . . . . . . . . . . 260
13.3 Exercises . . . . . . . . . . . . . . . . . . . . . . . . . . . . . . . . . . . . . . 283

14 OTHER FORMS OF FOURIER SERIES 285


14.1 Introduction . . . . . . . . . . . . . . . . . . . . . . . . . . . . . . . . . . . . . 285
14.2 Fourier Series for Even and Odd Functions in (−l, l) . . . . . . . . . . . . . . . 288
14.3 Worked out Problems . . . . . . . . . . . . . . . . . . . . . . . . . . . . . . . . 290
14.4 Exercises . . . . . . . . . . . . . . . . . . . . . . . . . . . . . . . . . . . . . . 317

Transform Techniques B.Sc.(Mathematics)-IDE(UNOM)-II Year - IV Sem


LESSONS xi

15 FOURIER COSINE AND SINE SERIES 321


15.1 Introduction . . . . . . . . . . . . . . . . . . . . . . . . . . . . . . . . . . . . . 321
15.2 Worked out Problems . . . . . . . . . . . . . . . . . . . . . . . . . . . . . . . . 323
15.3 Exercises . . . . . . . . . . . . . . . . . . . . . . . . . . . . . . . . . . . . . . 338

16 FOURIER TRANSFORMS 341


16.1 Introduction . . . . . . . . . . . . . . . . . . . . . . . . . . . . . . . . . . . . . 341
16.2 Dirichlet’s Conditions . . . . . . . . . . . . . . . . . . . . . . . . . . . . . . . . 343
16.3 Fourier sine and cosine integrals . . . . . . . . . . . . . . . . . . . . . . . . . . 345
16.4 Fourier Integral Theorem in Complex Form . . . . . . . . . . . . . . . . . . . . 345
16.5 Fourier Integral Representation of any Function . . . . . . . . . . . . . . . . . . 346
16.6 Finite and Infinite Fourier Transforms and Inverse Transforms . . . . . . . . . . 347
16.7 Relation Between Fourier Transform and Laplace Transform . . . . . . . . . . . 350
16.8 Properties of Fourier Transform . . . . . . . . . . . . . . . . . . . . . . . . . . . 351
16.9 Worked out Problems . . . . . . . . . . . . . . . . . . . . . . . . . . . . . . . . 358
16.10 Exercises . . . . . . . . . . . . . . . . . . . . . . . . . . . . . . . . . . . . . . 376

17 FOURIER SINE AND COSINE TRANSFORMS 379


17.1 Introduction . . . . . . . . . . . . . . . . . . . . . . . . . . . . . . . . . . . . . 379
17.2 Worked out Problems . . . . . . . . . . . . . . . . . . . . . . . . . . . . . . . . 379
17.3 Problems Related to Integral Equations (Fourier Transforms) . . . . . . . . . . . 408
17.4 Worked out Problems . . . . . . . . . . . . . . . . . . . . . . . . . . . . . . . . 408
17.5 Exercises . . . . . . . . . . . . . . . . . . . . . . . . . . . . . . . . . . . . . . 411

18 CONVOLUTION 413
18.1 Introduction . . . . . . . . . . . . . . . . . . . . . . . . . . . . . . . . . . . . . 413
18.2 Parseval’s Identity for Fourier Transforms . . . . . . . . . . . . . . . . . . . . . 414
18.3 Worked out Problems . . . . . . . . . . . . . . . . . . . . . . . . . . . . . . . . 416
18.4 Exercises . . . . . . . . . . . . . . . . . . . . . . . . . . . . . . . . . . . . . . 421
18.5 Worked out Problems Related to Finite Fourier Transform . . . . . . . . . . . . . 421
Transform Techniques B.Sc.(Mathematics)-IDE(UNOM)-II Year - IV Sem
xii LESSONS

18.6 Exercises . . . . . . . . . . . . . . . . . . . . . . . . . . . . . . . . . . . . . . 435

19 APPLICATIONS TO ORDINARY DIFFERENTIAL EQUATIONS WITH


CONSTANT COEFFICIENTS 437
19.1 Introduction . . . . . . . . . . . . . . . . . . . . . . . . . . . . . . . . . . . . . 437
19.2 Worked out Problems . . . . . . . . . . . . . . . . . . . . . . . . . . . . . . . . 438
19.3 Solution of Simultaneous Ordinary Differential Equations . . . . . . . . . . . . . 467
19.4 Worked out Problems . . . . . . . . . . . . . . . . . . . . . . . . . . . . . . . . 468
19.5 Exercises . . . . . . . . . . . . . . . . . . . . . . . . . . . . . . . . . . . . . . 478

20 APPLICATION OF LAPLACE TRANSFORMS TO PARTIAL DIFFERENTIAL


EQUATIONS 481
20.1 Introduction . . . . . . . . . . . . . . . . . . . . . . . . . . . . . . . . . . . . . 481
20.2 Worked out Problems . . . . . . . . . . . . . . . . . . . . . . . . . . . . . . . . 483
20.3 Application of Laplace Transform to Initial and Boundary Value Problems . . . . 496
20.4 Worked out Problems . . . . . . . . . . . . . . . . . . . . . . . . . . . . . . . . 497
20.5 Exercises . . . . . . . . . . . . . . . . . . . . . . . . . . . . . . . . . . . . . . 507

21 APPLICATIONS OF FOURIER TRANSFORMS TO INITIAL AND


BOUNDARY VALUE PROBLEMS 509
21.1 Introduction . . . . . . . . . . . . . . . . . . . . . . . . . . . . . . . . . . . . . 509
21.2 Worked out Problems . . . . . . . . . . . . . . . . . . . . . . . . . . . . . . . . 511
21.3 Application of Finite Fourier Transform . . . . . . . . . . . . . . . . . . . . . . 517
21.4 Worked out Problems . . . . . . . . . . . . . . . . . . . . . . . . . . . . . . . . 519
21.5 Applications of Laplace Transforms to Integral Equations . . . . . . . . . . . . . 524
21.6 Exercises . . . . . . . . . . . . . . . . . . . . . . . . . . . . . . . . . . . . . . 525

Model Question Paper 527

Transform Techniques B.Sc.(Mathematics)-IDE(UNOM)-II Year - IV Sem


UNIT-I

Lesson 1

BASIC CONCEPTS AND LAPLACE


TRANSFORM OF ELEMENTARY
FUNCTION
Learning Objectives
Upon completion of this lesson, students will be able to

• define Laplace transform and identify the condition for the existence of Laplace
transform

• obtain Laplace transform of some elementary functions

• find Laplace transform of simple functions using standard formula.

1.1 Introduction

T
here are many transform techniques which are used in the analysis and design of
engineering systems. Some of the transform techniques were introduced in the beginning
by great individuals which were vigorously defined and developed by mathematician in later days.
The technique of Laplace transform is one such. In this chapter and subsequent chapters to follow,
we introduce the idea of the Laplace transform and develop some useful results. Subsequently, we

B.Sc.(Mathematics)-IDE(UNOM)-II Year-IV Sem 1 Transform Techniques


2 1.2. Definition

display how the Laplace transform techniques is used in solving a class of problems in differential
equations.

1.2 Definition

Let F(t) be a given function and defined for all positive values of t. Then the Laplace transform
of F(t) is denoted by L [ f (t)] or f (p) and is defined by
Z ∞
L [F(t)] = f (p) = e−pt F(t)dt (1.2.1)
0

provided the integral exists. Here the parameter p is a real or complex number.
The operation of multiplying F(t) by e−pt and integrating from 0 to ∞ is called Laplace
transformation.
The relation (16.2.1) can also be written as

F(t) = L −1 [ f (p)].

In such a case, the function F(t) is said to be Inverse Laplace Transform of f (p). The symbol
L which transform F(t) into f (p) can be called the Laplace transform operator. The symbol
L −1 which transform f (p) into F(t) can be called the Inverse Laplace transform operator.

1.3 Sufficient Conditions For the Existence of the Laplace


Transform of a Function

While finding the Laplace transform of elementary functions, it can be noticed that the integral
exists under certain conditions, such as p > 0 or p > a etc. In general, the function F(t) must
satisfy the following conditions for the existence of the Laplace transform.

Transform Techniques B.Sc.(Mathematics)-IDE(UNOM)-II Year-IV Sem


1.4. Definitions 3

(i) The function F(t) must be piece - wise continuous or sectionally continuous in any limited
interval 0 < a ≤ t ≤ b.
(ii) The function F(t) is of exponential order.

1.4 Definitions

Definition 1.4.1 (Piece - wise continuous function). A function F(t) is said to be piece - wise
(or sectionally) continuous over the closed interval [a, b] it it is defined on that interval and is
such that the interval can be divided into a finite number of subintervals, in each of which F(t) is
continuous and has both right and left hand limits at every end point of the subinterval.

Example 1.4.1. The function



t , if 0 < t < 5


 2

F(t) = 


2t + 3, if t > 5


is sectionally continuous for t > 0.

Example 1.4.2. The function F(t) = 1


t
is not sectionally continuous in any interval containing
t = 0.

Definition 1.4.2 (Function of Exponential Order). A function F(t) is said to be exponential


order a if
lim e−at F(t) = a finite quantity
t→∞

i.e.,for a given number T, there exists a real number M > 0 such that

|e−at F(t)| < M, ∀ t ≥ T

or
|F(t)| < Meat , ∀ t ≥ T.

B.Sc.(Mathematics)-IDE(UNOM)-II Year-IV Sem Transform Techniques


4 1.4. Definitions

For example, F(t) = t2 , sinat, eat etc., are all of exponential order and also continuous. But
F(t) = et is not of exponential order and as such its Laplace transform does not exists.
2

Theorem 1.4.1 (Existence Theorem For Laplace Transform). If F(t) is a function which is
piece - wise continuous on every finite interval in the range t ≥ 0 and satisfies |F(t)| ≤ Meat for
all t ≥ 0 and for some constants a and M, then the Laplace transform of F(t) exists for all
p > a.

Proof. By the definition of Laplace transform, we have


Z ∞
L [F(t)] = e−pt F(t)dt
0

Z t0 Z ∞
L [F(t)] = e−pt
F(t)dt + e−pt F(t)dt (1.4.1)
0 t0
R t0
The integral 0
e−pt F(t)dt exists, since F(t) is piece - wise continuous on every finite interval
0 ≤ t ≤ t0 .
Now,

Z ∞ Z ∞
−pt
e F(t)dt ≤
|e−pt F(t)dt|
t0
Zt0∞
< e−pt Meat , ( since |F(t) ≤ Meat |)
Zt0∞
= Me−(p−a)t dt
t0
h e−(p−a)t i∞
= M
−(p − a) t0
e−(p−a)t0
= M , (p > a)
p−a
Z ∞ e−(p−a)t0
∴ e−pt F(t)dt ≤ M , (p > a).
t0 p−a

−(p−a)t0
But M e p−a
, (p > a) can be made as small as we please by taking t0 sufficiently large. Thus
from (1.4.1), we conclude that L [F(t)] exists for all p > a. 

Transform Techniques B.Sc.(Mathematics)-IDE(UNOM)-II Year-IV Sem


1.4. Definitions 5

Note. 1 The above theorem can also be stated as follows:


If F(t) is a function which is piece - wise continuous on every finite interval in the range t ≥ 0
and is of exponential order a as t → ∞, then the Laplace transform of F(t) exists for all p > a.
(or)
If F(t) is a function of class A, then the Laplace transform of F(t) exists for all p > a.

Note. 2 The condition of the theorem are sufficient but not necessary for the existence of the
Laplace transform. If the conditions are satisfied, the Laplace transform will exists. But if these
conditions are not satisfied, Laplace transform may or may not exists.

Example. Consider the function F(t) = 1



t

We know that
1
lim+ = lim+ √ = ∞.
t→0 t→0 t
Thus the function F(t) = 1

t
is not piece - wise continuous on every finite interval in the range
t ≥ 0. But F(t) is integrable from 0 to any positive value t0 .

Also,
F(t) ≤ Meat ∀ t > 1

with M = 1, a = 0.

Now Z ∞ Z ∞
1
L [F(t)] = e−pt
F(t)dt = e−pt √ dt
0 0 t
This converges for p > 0.
Put
√dt 2
pt = x ⇒ √ = √ dx
t p

π π
Z ∞ r
2 x2 2
∴ L [F(t)] = √ e dx = √ . = ,
p 0 p 2 p

π
R∞
e−x dx = .
2
since 0 2

Thus L [ √1t ] exists for p > 0, though 1



t
is not piece - wise continuous in the range of t ≥ 0.

B.Sc.(Mathematics)-IDE(UNOM)-II Year-IV Sem Transform Techniques


6 1.5. Linearity Property

General Properties of Laplace Transform: A very important property is that the Laplace
transform is a linear operator, just as differentiation and integration.

1.5 Linearity Property

Theorem 1.5.1. If L [F(t)] = f (p) and L [G(t)] = g(p) then

L [c1 F(t) + c2G(t)] = c1 L [F(t)] + c2 L [G(t)] = c1 f (p) + c2 g(p),

where c1 and c2 are constants.

Proof. By definition,
Z ∞
L [c1 F(t) + c2G(t)] = e−pt [c1 F(t) + c2G(t)]dt
0
Z ∞ Z ∞
= e c1 F(t)dt +
−pt
e−ptG(t)dt
0 0
Z ∞ Z ∞
= c1 e F(t)dt + c2
−pt
e−ptG(t)dt
0 0

L [c1 F(t) + c2G(t)] = c1 L [F(t)] + c2 L [G(t)]

L [c1 F(t) + c2G(t)] = c1 f (p) + c2 g(p)

Transform Techniques B.Sc.(Mathematics)-IDE(UNOM)-II Year-IV Sem


1.6. Laplace Transform of Some Elementary Functions 7

1.6 Laplace Transform of Some Elementary Functions

Elementary functions include algebraic and transcendental functions. From the definition of
Laplace transform, we obtain the following results:

1. L [k] = k
p
(p > 0), where k is a constant.

Proof. By definition,
Z ∞
L [F(t)] = e−pt F(t)dt
Z0 ∞
L [k] = e−pt kdt
0
 e−pt ∞
= k
−p 0
k
= − (0 − 1)
p
k
L [k] = , if p > 0.
p

Note. 1 For k = 1, L [1] = 1p , p > 0.


Note. 2 The above Laplace transform does not exist for p ≤ 0. It follows:
(i) For k = 0, L [0] = 0 (ii) For k < 0, the integral does not exists.

2. L [t] = 1
p2

Proof. By definition,
Z ∞
L [F(t)] = e−pt F(t)dt
Z0 ∞
L [t] = e−pt .tdt
0

B.Sc.(Mathematics)-IDE(UNOM)-II Year-IV Sem Transform Techniques


8 1.6. Laplace Transform of Some Elementary Functions

h  e−pt   e−pt i∞


L [t] = t − (1)
−p (−p)2 0
h  1 i
= t(0) − 0 − 0 − 2
p
1
L [t] = 2 , if p > 0.
p

3. L [tn ] = n!
pn+1
, where n is a positive integer.

Proof. By definition,
Z ∞
L [F(t)] = e−pt F(t)dt
Z0 ∞
L [tn ] = e−pt .tn dt
0

Put pt = u, pdt = du. When t = 0 ⇒ u = 0 and when t = ∞ ⇒ u = ∞

Z ∞  u n du
L [t ] =
n
e−u
0 p p
Z ∞
1
= n+1
e−u un du
p
Z0 ∞
1
= e−u u(n+1)−1 du
pn+1 0
Z ∞
1
= n+1
Γ(n + 1) since Γ(n) = e−x xn−1 dx, n > 0
p 0
Γ(n + 1)
L [tn ] = , if p > 0 and n + 1 > 0.
pn+1

In particular, if n is a positive integer, Γ(n + 1) = n!

n!
∴ L [tn ] = , if p > 0 and n is a positive integer.
pn+1


Transform Techniques B.Sc.(Mathematics)-IDE(UNOM)-II Year-IV Sem


1.6. Laplace Transform of Some Elementary Functions 9

Definition 1.6.1. Gamma Function. If n > 0 then the Gamma function Γ(n) is defined by
Z ∞
Γ(n) = e−x xn−1 dx.
0

The following are some important properties of the Gamma function:


(i) Γ(n + 1) = nΓ(n), if n > 0.
(ii) Γ(n + 1) = n! if n is a positive integer.
√ √
(iii) Γ(1) = 1, Γ( 21 ) = π, Γ(− 12 ) = −2 π
(iv) Γ(0), Γ(−1), Γ(−2), ... are all not defined.
Note. L [tn ] can also be expressed in terms of Gamma function.
We have Z ∞
L [F(t)] = e−pt F(t)dt
0

Putting x = pt, we get

∞ ∞
Γ(n + 1)
Z Z
1 1
L [t ] =
n
e x dx =
−x n
e−x x(n+1)−1 dx = ,
pn+1 0 pn+1 0 pn+1
when n > −1.
If n is a positive integer, Γ(n + 1) = n! in particular.
Hence
Γ(n + 1) n!
L [tn ] = n+1
= n+1 ,
p p
where n is a positive integer.

4. L [eat ] = 1
p−a
, (p − a) > 0

Proof.
Z ∞
We have L [F(t)] = e−pt F(t)dt
Z0 ∞
L [eat ] = e−pt eat dt
0

B.Sc.(Mathematics)-IDE(UNOM)-II Year-IV Sem Transform Techniques


10 1.6. Laplace Transform of Some Elementary Functions

Z ∞
= e−(p−a)t dt
0
h e−(p−a)t i∞
=
−(p − a) 0
1
L [eat ] = , if p > a
p−a

5. L [e−at ] = 1
p+a
, (p + a) > 0.

6. L [sinh at] = a
p2 −a2
, if p > |a|

Proof.

h eat − e−at i
L [F(t)] = L
2
1h i
= L [e ] − L [e−at ] , by linear property
at
2
1h 1 1 i
= −
2 p−a p+a
a
= 2
p − a2

7. L [cosh at] = p
p2 −a2
, if Re(p) > a. Here for the existence of Laplace Transform, we require
p > a or more precisely Re(p) > a

Proof.

h eat + e−at i
L [F(t)] = L
2
1h i
= L [eat ] + L [e−at ] , by linear property
2

Transform Techniques B.Sc.(Mathematics)-IDE(UNOM)-II Year-IV Sem


1.6. Laplace Transform of Some Elementary Functions 11
1h 1 1 i
= +
2 p−a p+a
p
= 2
p − a2

8. L [sin at] = a
p2 +a2
, if p > 0.

Proof. By the definition of Laplace transform, we have


Z ∞
L [sin at] = e−pt sin atdt
0
h e−pt i∞ Z
eax
= (−p sin at − a cos at) , since eax
sin bx = (a sin bx − b cos bx)
p2 + a2 0 a2 + b2
a
= 2
p + a2

9. L [cos at] = p
p2 +a2
, if p > 0.

Proof. By the definition of Laplace transform, we have


Z ∞
L [cos at] = e−pt cos atdt
0
h e−pt i∞ Z
eax
= (−p cos at + a sin at) , since e ax
cos bx = (a cos bx + b sin bx)
p2 + a2 0 a2 + b2
p
= 2 L [cos at]
p + a2

Alternate method to find L [sin at] and L [cos at] :


We know that
1
L [aat ] =
p−a

B.Sc.(Mathematics)-IDE(UNOM)-II Year-IV Sem Transform Techniques


12 1.7. Worked out Problems

Replacing a by ia, we get

1 p + ia
L [eiat ] = =
p − ia (p − ia)(p + ia)

p + ia
L [cos at + i sin at] =
p2 + a2
Equating the real and imaginary parts on both sides, we have
L [cos at] = p
p2 +a2
and L [sin at] = a
p2 +a2
.

1.7 Worked out Problems

Example 1.7.1. Find the Laplace transform of the following functions:


e−at −1
(i) (t2 + 1)2 ; (ii) a
; (iii) sin 2t cos t;

Solution. (i)

L [(t2 + 1)2 ] = L [t4 + 2t2 + 1]

= L [t4 ] + 2L [t2 ] + L [1], using linear property

4! 2! 1
L [(t2 + 1)2 ] = 5
+2 3 +
p p p
24 4 1
= 5+ 3+
p p p
1
L [(t2 + 1)2 ] = 5 (p4 + 4p2 + 24), p > 0.
p

(ii)

h e−at − 1 i 1 h −at i
L = L e −1
a a
1h i
= L [e−at ] − L [1] by linear property
a

Transform Techniques B.Sc.(Mathematics)-IDE(UNOM)-II Year-IV Sem


1.7. Worked out Problems 13
1h 1 1i
= −
a p+a p
−1
= .
p(p + a)

(iii)

Since 2 sin A cos B = sin(A + B) + sin(A − B)


1
sin 2t cos t = [sin 3t + sin t]
2
1
∴ L [sin 2t cos t] = L [sin 3t] + L [sin t]
2
1h 3 1 i
= +
2 p2 + 9 p2 + 1
2(p2 + 3)
L [sin 2t cos t] = .
(p2 + 1)(p2 + 9)

Example 1.7.2. Find the Laplace transform of the following functions:


(i) cosh2 2t; (ii) cos3 2t; (iii) (sin t + cos t)2

(i) Since cosh2 2t = 21 (1 + cosh 4t)

1h
L [cosh2 2t] = L [1] + L [cosh 4t]
2
1h 1 p i
= + 2
2 p p − 16
p2 − 8
L [cosh2 2t] =
p(p2 − 16)

(ii) Since cos 3A = 4 cos3 A − 3 cos A ⇒ cos3 A = 14 cos 3A + 43 cos A

1 3
cos3 2t = cos 6t + cos 2t
4 4
1 3
L [cos3 2t] = L [cos 6t] + L [cos 2t]
4 4

B.Sc.(Mathematics)-IDE(UNOM)-II Year-IV Sem Transform Techniques


14 1.7. Worked out Problems

1 p 3 p
= +
4 p2 + 36 4 p2 + 4
ph 1 3 i
= +
4 p2 + 36 p2 + 4
ph 4p2 + 112 i
=
4 (p2 + 36)(p2 + 4)

(iii)
(sin t + cos t)2 = sin2 t + cos2 t + 2 sin t cos t = 1 + sin 2t

L [(sin t + cos t)2 ] = L [1 + sin 2t]


1 2
= + 2
p p +4
p2 + 2p + 4
L [(sin t + cos t)2 ] = .
p(p2 + 4)

Example 1.7.3. Find the Laplace transform of the following functions:


(i) cos t cos 2t cos 3t; (ii) sinh3 2t; (iii) sinh2 3t.

(i)

1
cos t cos 2t cos 3t = cos t(2 cos 2t cos 3t)
2
1
= cos t(cos 5t + cos t)
2
1
= (cos t cos 5t + cos2 t)
2
1h  1 + cos 2t i
= (cos 6t + cos 4t) + 2
4 2
1h i
= cos 6t + cos 4t + 1 + cos 2t
4
1h p p 1 p i
L [cos t cos 2t cos 3t] = + + +
4 p2 + 36 p2 + 16 p p2 + 4

(ii)
et − e−t
sinh t =
2

Transform Techniques B.Sc.(Mathematics)-IDE(UNOM)-II Year-IV Sem


1.7. Worked out Problems 15
h et − e−t i3
sinh3 2t =
2
1 6t
sinh 2t =
3
[e − 3e2t + 3e−2t − e−6t ]
8
1
L [sinh3 2t] = L [e6t − 3e2t + 3e−2t − e−6t ]
8
1h 1 3 3 1 i
= − + −
8 p−6 p−2 p+2 p+6
1h 1 1 i 3h 1 1 i
= − − −
8 p−6 p+6 8 p−2 p+2
48
L [sinh3 2t] =
(p2 − 4)(p2 − 36)

(iii)

h e3t − e−3t i2
sinh2 3t =
2
1 9t
= [e + e−9t − 2]
4
1
L [sinh2 3t] = L [e9t + e−9t − 2]
4
1h 1 1 2i
= + −
4 p−9 p+9 p

Example 1.7.4. Find the Laplace transform of the following:


(i) e2t + 4t3 − 2 sin 3t + 3 cos 3t; (ii) (sin t − cos t)3 ; (iii) f (t) = |t − 1| + |t + 1|, t ≥ 0.

Solution. (i)

L [e2t + 4t3 − 2 sin 3t + 3 cos 3t] = [e2t ] + 4[t3 ] − 2[ sin 3t] + 3[ cos 3t]
1 3! 3 p
=
+ 4. 4 − 2. 2 +3 2
p−2 p p +9 p +9
1 24 6 3p
= + 4− 2 + 2
p−2 p p +9 p +9
1 24 3(p − 2)
L [e2t + 4t3 − 2 sin 3t + 3 cos 3t] = + 4+ 2 .
p−2 p p +9

(ii)
(sin t − cos t)3 = sin3 t − cos3 t − 3 sin2 t cos t + 3 sin t cos2 t

B.Sc.(Mathematics)-IDE(UNOM)-II Year-IV Sem Transform Techniques


16 1.7. Worked out Problems

(sin t − cos t)3 = sin3 t − cos3 t − 3(1 − cos2 t) cos t + 3 sin t(1 − sin2 t)

= sin3 t − cos3 t − 3 cos t + 3 cos3 t + 3 sin t − 3 sin3 t

(sin t − cos t)3 = 3 sin t − 3 cos t + 2 cos3 t − 2 sin3 t

1 3
cos 3A = 4 cos3 A − 3 cos A ⇒ cos3 A = cos 3A + cos A
4 4
3 1
sin 3A = 3 sin A − 4 sin3 A ⇒ sin3 A = sin A − sin 3A
4 4

h1 3 i h3 1 i
(sin t − cos t)3 = 3 sin t − 3 cos t + 2 cos 3t +
cos t − 2 sin t − sin 3t
4 4 4 4
3 3 1 1
= sin t − cos t + cos 3t + sin 3t
2 2 2 2
h3 3 1 1 i
L [(sin t − cos t)3 ] = L sin t − cos t + cos 3t + sin 3t
2 2 2 2
3 1 3 p 1 p 1 3
= − + +
2 p + 1 2 p + 1 2 p + 9 2 p2 + 9
2 2 2

1 h 3(1 − p) p+3i
L [(sin t − cos t)3 ] = +
2 p2 + 1 p2 + 9

(iii) We know that


When, 0 < t < 1, |t − 1| = 1 − t.
Now t > 0 ⇒ t + 1 > 1 > 0. Also |t + 1| = t + 1.

∴ F(t) = (1 − t) + t + 1 = 2, when 0 < t < 1

When t > 1, |t − 1| = t − 1, since t − 1 > 0.


Also |t + 1| = t + 1. since t + 1 > 2 > 0.

∴ F(t) = (t − 1) + t + 1 = 2t, when t > 1

Transform Techniques B.Sc.(Mathematics)-IDE(UNOM)-II Year-IV Sem


1.7. Worked out Problems 17

Thus,
Z ∞
L [ f (t)] = f (t)e−pt dt,
0
Z 1 Z ∞
= 2e dt +
−pt
2te−pt dt,
0 1
 e−pt 1 h  e−pt   e−pt i∞
= 2 +2 t − ,
−p 0 −p p2 1
2 h  e−p e−p i
= − (e−p − 1) + 2 0 − − − 2 ,
p p p
2 2 2 2
= − e−p + + e−p + 2 e−p ,
p p p p
−p 
2  e
L [ f (t)] = 1+ .
p p

Example 1.7.5. Find the Laplace transform of the following:


(i) sin(ωt + α); (ii) sin 2t cos 3t; (iii) cosh3 2t

Solution. (i)

sin(ωt + α) = sinωt cos α + cos ωt sin α,


ω p
L [sin(ωt + α)] = cos α. 2 + sin α. 2 ,
p +ω 2 p + ω2
1
L [sin(ωt + α)] = 2 (ω cos α + p sin α).
p + ω2

(ii)We know that

2 sin A cos B = sin(A + B) + sin(A − B)


1
sin 2t cos 3t = [sin 5t + sin(−t)],
2
1
= [sin 5t − sin t],
2

B.Sc.(Mathematics)-IDE(UNOM)-II Year-IV Sem Transform Techniques


18 1.7. Worked out Problems

1h 5 1 i
L [sin 2t cos 3t] = − ,
2 p2 + 25 p2 + 1
1 h 5(p2 + 1) − (p2 + 25) i
= ,
2 (p2 + 25)(p2 + 1)
2(p2 − 5)
L [sin 2t cos 3t] = .
(p2 + 25)(p2 + 1)

(iii) We have

cosh 3A = 4 cosh3 A − 3 cosh A,


1 3
cosh3 A = cosh 3A + cosh A,
4 4
1 3
cosh3 2t = cosh 6t + cosh 2t,
4 4
1 p 3 p
L [cosh3 2t] = . 2 + . 2 ,
4 p − 36 4 p − 4
p h (p2 − 4) + 3(p2 − 36) i
= ,
4 (p2 − 4)(p2 − 36)
p(p2 − 28)
L [cosh3 2t] = .
(p2 − 4)(p2 − 36)

Example 1.7.6. Find L [7e2t + 9e−2t + 5 cos t + 7t3 + 5 sin 3t + 2]

Solution.

1 1 p 3 1
L [7e2t + 9e−2t + 5 cos t + 7t3 + 5 sin 3t + 2] = 7. + 9. + 5. 2 + 5. 2 + 2. .
p−2 p+2 p +1 p +9 p

Example 1.7.7. Find L [(5e2t − 3)2 ]

Solution.
L [(5e2t − 3)2 ] = L [25e4t + 9 − 30e2t ]
1 1 1
= 25 + 9. − 30. .
p−4 p p−2

Example 1.7.8. Prove that the function f (t) = t2 is of exponential order 3.

Transform Techniques B.Sc.(Mathematics)-IDE(UNOM)-II Year-IV Sem


1.7. Worked out Problems 19

Solution.
lim e−3t f (t) = lim e−3t t2
t→∞ t→∞
t2 ∞
= lim form
t→∞ e3t ∞
2t
= lim 3t , by L’Hôpital Rule
t→∞ 3e
2
= lim 3t , by L’Hôpital Rule
t→∞ 93
2
= ,

lim e−3t f (t) = 0, a finite quantity.
t→∞

Hence t2 is of exponential order 3.

3
Example 1.7.9. Prove that the Laplace transform of et does not exist.

Solution.
3 3 −at)
lim e−at f (t) = lim e−at et = lim e(t =∞
t→∞ t→∞ t→∞

∴ f (t) = et is not of exponential order.


3

Hence its Laplace transform does not exist.

Example 1.7.10. Obtain the Laplace transform of the function



(t − 1) , if t > 1


 2

f (t) = 


if 0 < t < 1

0,

Solution. By definition,
Z ∞
L [ f (t)] = e−pt f (t)dt,
0
Z 1 Z ∞
= −pt
e f (t)dt + e−pt f (t)dt,
0 1

B.Sc.(Mathematics)-IDE(UNOM)-II Year-IV Sem Transform Techniques


20 1.7. Worked out Problems
Z 1 Z 1
L [ f (t)] = −pt
e .0dt + e−pt (t − 1)2 dt,
0 0
h  e−pt   e−pt   e−pt i∞
= (t − 1)2 − 2(t − 1) −2 , by using Bernoulli’s formula,
−p p2 p3 1
  e−p 
= [0 − 0 − 0 − 2 3 ],
p
2 −p
L [ f (t)] = 3 e .
p

Example 1.7.11. Find the Laplace transform of



sin t, if 0 < t < π




f (t) = 


0, if t > π


Solution. By definition,
Z ∞
L [ f (t)] = e−pt f (t)dt,
Z0 π Z ∞
= e −pt
f (t)dt + e−pt f (t)dt,
Z0 π Zπ ∞
= e−pt sin tdt + e−pt .0 dt,
0 π
h e−pt iπ
= (−p sin t − cos t) ,
p2 + 1 0

1 + e−πp
L [ f (t)] = .
p2 + 1

eat
eat sin btdt =
R
Note: a2 +b2
(a sin bt − b cos bt).

Example 1.7.12. Find the Laplace transform of



e , if 0 < t < 5


 t

f (t) = 


if t > 5

3,

Transform Techniques B.Sc.(Mathematics)-IDE(UNOM)-II Year-IV Sem


1.7. Worked out Problems 21

Solution. By definition,
Z ∞
L [ f (t)] = e−pt f (t)dt,
0
Z 5 Z ∞
= f (t)dt +
e−pt
e−pt f (t)dt,
0 5
Z 5 Z ∞
L [ f (t)] = e e dt +
−pt t
e−pt 3dt,
0 5

Z 5 Z ∞
L [ f (t)] = e −(p−1)t
dt + e−pt 3dt,
0 5
h e−(p−1)t i5  e−pt ∞
= +3
−(p − 1) 0 −p 5
1 3
= − (e−5(p−1) − 1) − (0 − e−5p )
p−1 p
−5(p−1)
1−e 3
= + e−5p .
p−1 p

Example 1.7.13. Find the Laplace transform of


0<t<2





1, if



2<t<4

2, if



f (t) = 


4<t<6





3, if



t>6

0, if

Solution. By definition,
Z ∞
L [ f (t)] = e−pt f (t)dt,
0
Z 2 Z 4 Z 6 Z ∞
= e .1dt +
−pt
e .2 dt +
−pt
e .3 dt +
−pt
e−pt .0 dt,
0 2 4 6

B.Sc.(Mathematics)-IDE(UNOM)-II Year-IV Sem Transform Techniques


22 1.7. Worked out Problems

 e−pt 2  e−pt 4  e−pt 4


= +2 +3
−p 0 −p 2 −p 4
1 2 3
= − (e−2p − 1) − (e−4p − e−2p ) − (e−6p − e−4p )
p p p
1
L [ f (t)] = (1 + e−2p + e−4p − 3e−6p ).
p

Example 1.7.14. Find the Laplace transform of


0, 0 < t < 1








f (t) = 

t, 1 < t < 2






0, t > 2


Solution. By definition,
Z ∞
L [ f (t)] = e−pt f (t)dt,
0
Z 1 Z 2 Z ∞
= e .0 dt +
−pt
e .t dt +
−pt
e−pt .0 dt,
0 1 2

Z 2
L [ f (t)] = e−pt .tdt
1
h  e−pt   e−pt i2
= t − 1. 2
−p p 1
1 1
= − (2e−2p − e−p ) − 2 (e−2p − e−p )
p p
2 −2p 1 −p 1 −2p 1 −p
= − e + e − 2e + 2e
p p p p
1 −p
L [ f (t)] = (e − e−2p − 2pe−2p + pe−p ).
p2

Transform Techniques B.Sc.(Mathematics)-IDE(UNOM)-II Year-IV Sem


1.7. Worked out Problems 23

Example 1.7.15. Find L [ f (t)], where



e , t>a


 t−a

f (t) = 


t<a

0,

Solution. By definition,
Z ∞
L [ f (t)] = e−pt f (t)dt,
Z0 a Z ∞
= e .0 dt +
−pt
e−pt .et−a dt,
Z0 ∞ a

= e−pt .et e−a dt,


a
Z ∞
= e−a
e−(p−1)t dt,
a
 e−(p−1)t ∞
= e−a ,
−(p − 1) a
e−a
= − (0 − e−(p−1)a ),
(p − 1)
e−pa
L [ f (t)] = , p > 1.
p−1
h i
Example 1.7.16. Show that L √1
πt
= √1 .
p

Solution.
h 1 i 1 1
L √ = √ [t− 2 ]
πt π
1 Γ( 2 + 1)
−1
= √
π p− 12 +1

B.Sc.(Mathematics)-IDE(UNOM)-II Year-IV Sem Transform Techniques


24 1.7. Worked out Problems

1 Γ( 2 )
h 1 i 1
L √ = √
πt π p 12

1 π  1 √ 
= √ √ , Γ( ) = π
π p 2
h 1 i 1
L √ = √ .
πt p

x3 x5 x7
Example 1.7.17. Using the expression sin x = x − 3!
+ 5!
− 7!
+ · · · , show that


√ π 1
L [sin t] = 3
e− 4p .
2p 2

Solution. √ √ √
√ √ ( t)3 ( t)5 ( t)7
sin t = t − + − + ···
3! 5! 7!
3 5 7
√ 1 t 2 t 2 t 2
sin t = t 2 − + − + · · ·
3! 5! 7!
√ 1
h t 32 i h t 52 i h t 72 i
L [sin t] = L [t 2 ] − L +L −L + ···
  3!
  5!  7!
Γ 32 1Γ 2
5
1Γ 2
7

= 3
− + + ···
p2 3! p 25 5! p 27
    
1
2
Γ( 1
2
.
3 1
1 2 2 2 Γ 1
. .
5 3 1
1 2 2 2 2 Γ 1

= 3
− + 7
+ · · · , Γ(n + 1) = nΓ(n)
p2 3! p 52 5! p2
√ h
π 3 5.3 7.5.3 i
= 3
1− + 2
− 3
+ ···
2p 2 2.3!p 2.2.5!p 2.2.2.7!p
√ h
π 1 1 1 i
= 3
1 − + − + · · ·
2p 2 22 p (22 p)2 2! (22 p)3 3!

√ π − 221 p
L [sin t] = 3
e
2p 2

√ π − 4p1
L [sin t] = 3
e
2p 2

Transform Techniques B.Sc.(Mathematics)-IDE(UNOM)-II Year-IV Sem


1.8. Exercises 25

1.8 Exercises

A. Find the Laplace transform of the following functions:


(1) t2 + at + b; (2) t3 + 5 cos t; (3) 2e3t − e−3t ; (4) 2t ;
(5) sin t cos t; (6) sin2 t; (7) sin2 at; (8) cos2 t;
(9) 3 cosh 5t − 4 sinh 5t; (10) cosh2 (2t); (11) sinh2 (2t); (12) sinh at − sin at;
(13) cos(ωt + θ); (14) cos3 t; (15) cos3 3t; (16) sinh3 (2t);
(17) sin3 2t; (18) sin 2t sin 3t; (19) cos 4t sin 2t; (20) cos 5t cos 2t;
√ 3 √ 3
(21) t − √1t ; (22) t + √1t .
B. Find the Laplace transform of the following function:

2t, 0 < t < 5




f (t) = 


1, t > 5


C. Find the Laplace transform of the following function:


0, 0 < t < 1








f (t) = 

1, 1 < t < 2






2, t > 2


D. Find the Laplace transform of the following function:



e , 0 < t < 1


 t

f (t) = 


0, t > 1


B.Sc.(Mathematics)-IDE(UNOM)-II Year-IV Sem Transform Techniques


26 1.8. Exercises

E. Find the Laplace transform of the following function:



cos t, 0 < t < 2π




f (t) = 


0, t > 2π.


Answers
A.(1) p23 + a
p2
+ bp ; (2) 6
p4
+ 5p
p2 +1
; (3) p+9
p2 −9
;
1 1 2
(4) p−log 2
; (5) p2 +4
; (6) p(p2 +4);
2a2 p +2
2 3p−20
(7) p(p2 +4a2 )
; (8) p(p2 +4)
; (9) p2 −25
;
2
2a3
(10) p(pp2−8
−16)
; (11) 8
p(p2 −16)
; (12) p4 −a4
;
(13) p cosp2θ−ω
+ω2
sin θ
; (14) p2 +7p
(p2 +1)(p2 +9)
; (15) p(p2 +63)
(p2 +9)(p2 +81)
;
48 48 12p
(16) (p2 −4)(p2 −36)
; (17) (p2 +4)(p2 +36)
; (18) (p2 +1)(p2 +25)
;
√ h
2 p(p +29)
2 π 3 √ i
(20) 2(p −12)
(p2 +36)(p2 +4)
; (21) (p2 +9)(p2 +49)
; (22) 4 5 −
6
3 + 12

p
+8 p ;
p 2 p2
√ h
π 3 √ i
5 + + −8 p .
6 12
(23) 4 3 √
p
p2 p2

(B). 2
p2
(1 − e−5p ) − 9p e−5p . (C) 1p (e−p + e−2p ).
p(1−e−2πp )
(D). 1
1−p
(e1−p − 1). (D) 1+p2
.

Transform Techniques B.Sc.(Mathematics)-IDE(UNOM)-II Year-IV Sem


UNIT-I

Lesson 2

FIRST SHIFTING THEOREM


AND SECOND SHIFTING THEOREM
Learning Objectives
Upon completion of this lesson, students will be able to

• identify the first and second shifting theorem

• solve problems using first and second shifting theorem

• find the Laplace transform of unit step function

• implement the change of scale of property.

2.1 First Translation (or) First Shifting Theorem

I
n this lesson, we study first and second shifting theorems.

Theorem 2.1.1. If L [F(t)] = f (p), then [eat F(t)] = f (p − a), p − a > 0.

B.Sc.(Mathematics)-IDE(UNOM)-II Year-IV Sem 27 Transform Techniques


28 2.1. First Translation (or) First Shifting Theorem

Proof. By definition,
Z ∞
L [F(t)] = e−pt F(t)dt
Z0 ∞
L [eat F(t)] = e−pt eat F(t)dt
Z0 ∞
= e−(p−a)t F(t)dt
0

Z ∞
L [F(t)] = e−(p−a)t F(t)dt
Z0 ∞
= e−ut F(t)dt, where u = p − a
0

= f (u)

L [eat f (t)] = f (p − a)

Corollary 2.1.1. Using the above theorem, we have [e−at F(t)] = f (p + a), p + a > 0.

Working Rule:

L [e−at F(t)] = f (p + a) = f [p] p→p+a = L [ f (t)] p→p+a .

p → p + a means that p is replaced by (p + a).


i.e., to find the Laplace transform of the product of two functions, one of which is e−at , we ignore
e−at , find the Laplace transform of the other factors as a function of p and change p into p + a
in it.

Transform Techniques B.Sc.(Mathematics)-IDE(UNOM)-II Year-IV Sem


2.2. Worked out Problems (Problems based on first shifting theorem) 29

2.2 Worked out Problems (Problems based on first shifting


theorem)

Example 2.2.1. Find the Laplace transform of the following functions:


(i) e−t cos 2t; (ii) e−3t (2 cos 5t − 3 sin 5t);
(iii) e−t (3 sin 2t − 5 cosh 2t).

Solution. (i)

L [e−t cos 2t] = L[cos 2t] p→p+1 , by first shifting theorem


h p i
=
p2 + 4 p→p+1
p+1
=
(p + 1)2 + 4
p+1
L [e−t cos 2t] = 2 .
p + 2p + 5

(ii)

L [e−3t (2 cos 5t − 3 sin 5t)] = L [2 cos 5t − 3 sin 5t] p→p+3


h 2p 3.5 i
= −
p2 + 25 p2 + 25 p→p+3
h 2p − 15 i
=
p2 + 25 p→p+3
2(p + 3) − 15
=
(p + 3)2 + 25
2p − 9
L [e−3t (2 cos 5t − 3 sin 5t)] = 2 .
p + 6p + 34

B.Sc.(Mathematics)-IDE(UNOM)-II Year-IV Sem Transform Techniques


30 2.2. Worked out Problems (Problems based on first shifting theorem)

(iii)

L [e−t (3 sin 2t − 5 cosh 2t)] = L [3 sin 2t − 5 cosh 2t] p→p+1


h 3.2 5.p i
= −
p2 + 4 p2 − 4 p→p+1
h 6 5(p + 1) i
= −
(p + 1)2 + 4 (p + 1)2 − 4
6 5(p + 1)
= 2 − 2 .
p + 2p + 5 p + 2p − 3

Example 2.2.2. Find the Laplace transform of the following functions:


(i) e−t sinh bt; (ii) e−at cosh bt;

Solution. (i)

L [e−at sinh bt] = L [sinh bt] p→p+a


h b i
=
p2 − b2 p→p+a
b
=
(p + a)2 − b2

(ii)

L [e−at cosh bt] = L [cosh bt] p→p+a


h p i
=
p2 − b2 p→p+a
p
L [e−at cosh bt] =
(p + a)2 − b2
h i
Example 2.2.3. Evaluate L et (cos 2t + 21 sinh 2t) .

Transform Techniques B.Sc.(Mathematics)-IDE(UNOM)-II Year-IV Sem


2.2. Worked out Problems (Problems based on first shifting theorem) 31

Solution.
h 1 i h 1 i
L et (cos 2t + sinh 2t) = L cos 2t + sinh 2t
2 2 p→p−1
h p 1 2 i
= + . 2
p + 4 2 p − 4 p→p−1
2

p−1 1
= +
(p − 1) + 4 (p − 1)2 − 4
2
h 1 i p−1 1
L et (cos 2t + sinh 2t) = 2 + 2 .
2 p − 2p + 5 p − 2p − 3

2a2 p
Example 2.2.4. Show that L [sinh at sin at] = p4 +4a4
.

Solution.
h eat − e−at  i
L [sinh at sin at] = L sin at
2
1
= L [eat sin at − e−at sin at]
2
1h i
= L [sin at] p→p−a − L [sin at] p→p+a
2
1h a i h a i
= −
2 p2 + a2 p→p−a p2 + a2 p→p+a
1 h a a i
= −
2 (p − a)2 + a2 (p + a)2 + a2
a h (p + a)2 + a2 − ((p − a)2 + a2 ) i
=
2 (p − a)2 + a2 )(p + a)2 + a2 )
ah 4ap i
=
2 (p2 − 2ap + 2a2 )(p2 + 2ap + 2a2 )
2a2 p
=
(p2 + 2a2 )2 − (2ap)2
2a2 p
L [sinh at sin at] =
p4 + 4a4

Example 2.2.5. Find L [sinh at cos at].

B.Sc.(Mathematics)-IDE(UNOM)-II Year-IV Sem Transform Techniques


32 2.2. Worked out Problems (Problems based on first shifting theorem)

Solution.
h eat − e−at  i
L [sinh at cos at] = L cos at
2
1
= L [eat cos at − e−at cos at]
2
1h i
L [sinh at cos at] = L [cos at] p→p−a − L [cos at] p→p+a
2

1h p i h p i
L [sinh at cos at] = −
2 p2 + a2 p→p−a p2 + a2 p→p+a
1h p−a p+a i
= −
2 (p − a)2 + a2 (p + a)2 + a2
1 h (p − a)((p + a)2 + a2 ) − (p + a)((p − a)2 + a2 ) i
=
2 (p − a)2 + a2 )(p + a)2 + a2 )
1h 4p2 a − 2a(p2 + 2a2 ) i
=
2 (p2 − 2ap + 2a2 )(p2 + 2ap + 2a2 )
2p2 a − ap2 − 2a3
=
(p2 + 2a2 )2 − (2ap)2
a(p2 − 2a2 )
L [sinh at sin at] = .
p4 + 4a4

Example 2.2.6. Find L [cosh at cos at].

Solution.
h eat + e−at  i
L [cosh at cos at] = L cos at
2
1
= L [eat cos at + e−at cos at]
2
1h i
= L [cos at] p→p−a + L [cos at] p→p+a
2
1h p i h p i
= +
2 p2 + a2 p→p−a p2 + a2 p→p+a

Transform Techniques B.Sc.(Mathematics)-IDE(UNOM)-II Year-IV Sem


2.3. Unit Step Function(Heaviside Unit Step Function) 33
1h p−a p+a i
= +
2 (p − a)2 + a2 (p + a)2 + a2
1 h (p − a)((p + a)2 + a2 ) + (p + a)((p − a)2 + a2 ) i
=
2 (p − a)2 + a2 )(p + a)2 + a2 )
1h 2p(p2 + 2a2 ) − 4pa2 i
=
2 (p2 − 2ap + 2a2 )(p2 + 2ap + 2a2 )
p(p2 + 2a2 ) − 2pa2 p3
= = .
(p2 + 2a2 )2 − (2ap)2 p4 + 4a4

Example 2.2.7. Find L [(1 + te−t )3 ].

Solution.
L [(1 + te−t )3 ] = L [1 + 3te−t + 3t2 e−2t + t3 e−3t ]
1 h1i h 2! i h 3! i
= + 3. 2 + 3. 3 + 4
p p p→p+1 p p→p+2 p p→p+3
1 3 6 6
L [(1 + te−t )3 ] = + + + .
p (p + 1)2 (p + 2)3 (p + 3)4

2.3 Unit Step Function(Heaviside Unit Step Function)

The function 
0, if t < a




u(t − a) (or) H(t − a) (or) ua (t) = 


1, if t > a


is called unit step function or Heaviside unit step function.


To find the Laplace transform of unit step function:
By the definition of Laplace transform,
Z ∞
L [F(t)] = e−pt F(t)dt
Z0 a Z ∞
L [ua (t)] = e −pt
.0 dt + e−pt .1 dt
0 a

B.Sc.(Mathematics)-IDE(UNOM)-II Year-IV Sem Transform Techniques


34 2.4. Second Translation (or) Second Shifting Theorem

 e−pt ∞
=
−p a
1
= − (e−∞ − e−ap )
p
−ap
e
=
p

Note: If a = 0, then the unit step function is



0, if t < 0




u(t) = 


1, if t > 0


The Laplace transform of u(t) is 1s .

2.4 Second Translation (or) Second Shifting Theorem

Theorem 2.4.1. If L [F(t)] = f (p) and



if t > a

F(t − a),



g(t) = 


if t < a

0,

then L [g(t)] = e−ap f (p)

Proof. By definition,
Z ∞
L [F(t)] = e−pt F(t)dt
Z0 ∞
L [g(t)] = e−pt g(t)dt
0
Z a Z ∞
= −pt
e .0 dt + e−pt F(t − a) dt
0 a

Transform Techniques B.Sc.(Mathematics)-IDE(UNOM)-II Year-IV Sem


2.4. Second Translation (or) Second Shifting Theorem 35

Put t − a = u so that dt = du.


When t = a, u = 0 and when t → ∞, u → ∞.
Z ∞
L [g(t)] = e−p(u+a) F(u)du
0
Z ∞
= e−ap
e−pu F(u)du
0

= e−ap L [F(t)]

L [g(t)] = e−ap f (p).

Another form of Second Shifting Theorem.


If L [F(t)] = f (p) and a > 0, then L [F(t − a)u(t − a)] = e−ap f (p), where u(t − a) is the unit
step function.

Proof.
Z ∞
L [F(t)] = e−pt F(t)dt
Z0 ∞
L [F(t − a)u(t − a)] = e−pt F(t − a)u(t − a)dt
0

Put t − a = v so that dt = dv.


When t = 0, v = −a, and when t → ∞, u → ∞.
Z ∞
L [F(t − a)u(t − a)] = e−p(v+a) F(v)u(v)dv
−a
Z 0 Z ∞
=F(v)u(v)dv + −p(v+a)
e e−p(v+a) F(v)u(v)dv
−a 0
Z 0 Z ∞
L [F(t − a)u(t − a)] = e−p(v+a)
F(v).0 dv + e−p(v+a) F(v).1 dv, by the definition of u(t − a)
−a 0

B.Sc.(Mathematics)-IDE(UNOM)-II Year-IV Sem Transform Techniques


36 2.5. Worked out Problems
Z ∞
L [F(t − a)u(t − a)] = e −ap
e−pv F(v).1 dv
0

= e−ap L [F(t)]

L [F(t − a)u(t − a)] = e−ap f (p).

2.5 Worked out Problems

Example 2.5.1. Find the Laplace transform of



π π
cos(t − 3 ), if t >



 3
g(t) = 

π

if t <

0,


3

Solution. Let f (t) = cos t.

p
∴ L [ f (t)] = L [cos t] = = F(p).
p2 +1
Now  
π π π π
 f (t − 3 ), if t > if t >
 
cos(t − 3 ),


 


3 3
g(t) =  =
 
π π
 
if t < if t <
 
0,


3
0,


3

Applying second shifting theorem, we get


πp
− πp
h p i pe− 3
L [g(t)] = e 3 = .
p2 + 1 p2 + 1

Example 2.5.2. Find the Laplace transform of



if t >
 2π 2π
cos(t − ),


 3 3
g(t) = 


if t < 2π

0,


3

Transform Techniques B.Sc.(Mathematics)-IDE(UNOM)-II Year-IV Sem


2.6. Change of Scale of Property 37

Solution. Let f (t) = cos t.

p
∴ L [ f (t)] = L [cos t] = = F(p).
p2 +1
Now  
π
if t > if t >
 2π
 2π 2π
 f (t − ), cos(t − ),


 


3 3 3 3
g(t) =  =
 
π
 
if t < if t < 2π
 
0,


3
0,


3

Applying second shifting theorem, we get

2πp
− 2πp
h p i pe− 3
L [g(t)] = e 3 = 2 .
p2 + 1 p +1

Example 2.5.3. Find the Laplace transform of (i) (t − 2)3 u(t − 2); (ii) e−3t u(t − 2).

Solution. Let F(t) = t3 .


3! 6
L [F(t)] = L [t3 ] = 4
= 4 = f (p).
p p
Applying second shifting theorem, we get

6
L [(t − 2)3 u(t − 2)] = e−2p . .
p4
(ii)
L [e−3t u(t − 2)] = L [e−3(t−2) e−6 u(t − 2)].

Let F(t) =−3t . Then L [F(t)] = 1


p+3
.
Applying second shifting theorem, we have

1 e−2(p+3)
L [e−3t u(t − 2)] = e−6 L [e−3t u(t − 2)] = e−6 e−2p . = .
p+3 p+3

2.6 Change of Scale of Property


h pi
Theorem 2.6.1. If L [F(t)] = f (p), then (i) L [F(at)] = 1a f a
, (ii) L [F( at )] = a f (ap).

B.Sc.(Mathematics)-IDE(UNOM)-II Year-IV Sem Transform Techniques


38 2.7. Worked out Problems

Proof. (i) By definition,


Z ∞
L [F(t)] = e−pt F(t)dt
Z0 ∞
L [F(at)] = e−pt F(at)dt
0

Put at = u, so that dt = 1a du. When t = 0, u = 0 and when t → ∞, u → ∞.


Z ∞
u du
L [F(at)] = e−p( a ) F(u)
0 a
1 ∞ −( ap )u
Z
= e F(u)du
a 0
1 h pi
L [F(at)] = f .
a a

(ii) Replace a by t
a
in (i), we get L [F( at )] = a f (ap). 

2.7 Worked out Problems

Example 2.7.1. Find L [sin2 at] using change of scale of property.

Solution.
1 − cos 2at
sin2 at =
2
1
L [sin2 at] = L [(1 − cos 2at)]
2
1 1 1
= . − L [cos 2at]
2 p 2

Let f (t) = cos t, then F(p) = p


p2 +1
, and using change of scale of property, we have

p
1 p
L [cos 2at] = 2a
= 2 .
2a ( 2a ) + 1 p + 4a2
p 2

Hence
1 1 p 2a2
L [sin2 at] = − . 2 = .
2p 2 p + 4a2 p(p2 + 4a2 )

Transform Techniques B.Sc.(Mathematics)-IDE(UNOM)-II Year-IV Sem


2.7. Worked out Problems 39
3
− p+1
1
Example 2.7.2. If L [F(t)] = 1p e− p , prove that L [e−t F(3t)] = e
p+1
.

Solution.
1 − 1p
L [F(t)] = e
p
By the change of scale of property, we have

1  p  1 3 − 3p 1 − 3p
L [F(3t)] = f = . e = e .
3 3 3 p p
Now,
h1 3 i
L [e−t F(3t)] = e− p ,
p p→p+1

by first shifting theorem.


1 − p+1
3
∴ L [e−t F(3t)] = e .
p+1
9p2 −12p+15
Example 2.7.3. If L [F(t)] = (p−1)3
, find L [F(3t)] using change of scale property.

9p2 −12p+15
Solution. Given L [F(t)] = (p−1)3
= f (p).
By change of scale of property,

1  p  1 9( 3 )2 − 12( 3 ) + 15 9(p2 − 4p + 15)


p p
L [F(3t)] = F = = .
3 3 3 ( 3p − 1)3 (p − 3)3

p2 −p+1
Example 2.7.4. Applying change of scale of property, if L [F(t)] = (2p+1)2 (p−1)
then show that
p2 −2p+4
L [F(2t)] = 4(p+1)2 (p−2)
.

p2 −p+1
Solution. Given L [F(t)] = (2p+1)2 (p−1)
= f (p).

By change of scale of property,

1  p  1 ( 2 )2 − ( 2 ) + 1
p p
1 (p2 − 2p + 4)
L [F(2t)] = f = = .
2 2 2 (2 2p + 1)2 ( 2p − 1) 4 (p + 1)2 (p − 2)

B.Sc.(Mathematics)-IDE(UNOM)-II Year-IV Sem Transform Techniques


40 2.8. Exercises

2.8 Exercises

A. Find the Laplace transform of the following functions:


(1)t3 e−3t ; (2) e−t (3 cos 5t − 4 sin 5t); (3) e2t (3 sinh 2t − 5 cosh 2t);
e−at tn−1
(4) (n−1)!
; (5) e−at sinh bt; (6) e−t cos2 t;
(7) cosh at cos at; (8) sinh 3t cos2 t; (9) (1 + te−t )3 ;
(10) e−t sin t cos t; (11) et cos 4t sin t; (12) (t − 1)3 u(t − 1);
(13) e−2t 1 − u(t − 1).
B. Find the Laplace transform of

π π
cos(t − 4 ), if t >



 4
g(t) = 

π

if t <

0,


4

C. Find the Laplace transform of



π π
sin(t − 3 ), if t >



 3
f (t) = 

π

if t <

0,


3

D.If L [ f (t)] = 20−4p


p2 −4p+20
then find (i) L [ f (3t)] (ii) L [e−t f (2t)].

Answers
6 3p−17 16−5p
A. (1) (p+3)4; (2) p2 +2p+26
; (3) p2 −4p
;
1 b p +2p+3
2
(4) (p+a)n
; (5) (p+a)2 −b2
; (6) (p+1)(p2 +2p+5)
;
p3 p2 −13
h i
(7) p4 +4a4
; (8) 3 1
2 p2 −9
+ p4 −10p 2 +169 ; (9) 1
p
+ 3
(p+1)2
+ 6
(p+2)3
+ 6
(p+3)4
;
1 3 1 6e−p
(10) p2 +2p+5
; (11) p2 −2p+37
− p2 −2p+5
; (12) p4
;
1−e−(p+2)
(13) p+2
.
3πp −πp
e− 4
B. p2 +1
. C. e 3
p2 +1
.
D. (i) 60−4p
p2 −12p+80
; (ii) 4(9−p)
p2 −6p+73
.

Transform Techniques B.Sc.(Mathematics)-IDE(UNOM)-II Year-IV Sem


UNIT-I

Lesson 3

LAPLACE TRANSFORM OF
DERIVATIVES AND INTEGRALS
Learning Objectives
Upon completion of this lesson, students will be able to

• find Laplace transform of derivatives and integrals

• solve problems using Laplace of derivatives and integrals

• use initial and final value theorems

• solve problems using initial and final value theorems


h F(t) i
• solve problems based on L [tn F(t) and L t

• evaluate integrals using Laplace transform.

3.1 Laplace Transform of Derivatives

I
n this Lesson, we study the Laplace transform of derivatives, initial and final value theorems
with worked out sample problems.

Theorem 3.1.1. If F(t) is continuous and of exponential order and if F 0 (t) is sectionally

B.Sc.(Mathematics)-IDE(UNOM)-II Year-IV Sem 41 Transform Techniques


42 3.1. Laplace Transform of Derivatives

continuous then the Laplace transform of F 0 (t) is given by L [F 0 (t)] = p f (p) − F(0), where
L [F(t)] = f (p).

Proof. By definition,
Z ∞
L [F(t)] = e−pt F(t)dt
Z0 ∞
L [F (t)] =
0
e−pt F 0 (t)dt
0
Z q
= lim e−pt d(F(t))
q→∞ 0

Let u = e−pt , then du = −pe−pt dt.


dv = d(F(t)), v = F(t)dt

h q Z q i
L [F (t)] = lim e F(t) −
0 −pt
(−pe−pt )F(t)dt
q→∞ 0
0
h Z q i
= lim e F(t) − F(0) + p
−pq
e−pt F(t)dt .
q→∞ 0

Since F(t) is of exponential order, limt→∞ e−pt F(t) = 0.


Z ∞
∴ L [F (t)] = 0 − F(0) + p
0
e−pt F(t)dt
0

= −F(0) + pL [F(t)]

L [F 0 (t)] = p f (p) − F(0).

Note. The Laplace transform of the second order derivative F 00 (t) is similarly obtained.

Theorem 3.1.2. L [F 00 (t)] = p2 L [F(t)] − pF(0) − F 0 (0)

Proof. We have L [G0 (t)] = pL [G(t)] − G(0).

Transform Techniques B.Sc.(Mathematics)-IDE(UNOM)-II Year-IV Sem


3.2. Initial Value Theorem 43

Let G(t) = F 0 (t). Then

L [F 00 (t)] = pL [F 0 (t)] − F 0 (0)

= p[pL [F(t)] − F(0)] − F 0 (0)

= p2 L [F(t)] − pF(0) − F 0 (0)

L [F 00 (t)] = p2 f (p) − pF(0) − F 0 (0).

Similarly we can prove that L [F n (t)] = pn f (p) − pn−1 F(0) − pn−2 F 0 (0) − .... − F n−1 (0).

Note. L [F n (t)] = pn f (p), if F 0 (0) = 0, F 00 (0) = 0, ...F n−1 (0) = 0.

3.2 Initial Value Theorem

Theorem 3.2.1. Let F(t) be a continuous for all t ≥ 0 and be of exponential order as t → ∞. If
F 0 (t) is of class A then limt→0 F(t) = lim p→∞ pL [F(t)] = lim p→∞ p f (p).

Proof. By the theorem of Laplace transform of derivatives, we have


Z ∞
L [F (t)] =
0
e−pt F 0 (t)dt = p f (p) − f (0).
0

Since F 0 (t) is sectionally continuous and of exponential order, we have


Z ∞ Z ∞   Z ∞
lim −pt
e F (t)dt =
0
lim e −pt
F (t)dt =
0
0. F 0 (t)dt = 0.
p→∞ 0 0 p→∞ 0

B.Sc.(Mathematics)-IDE(UNOM)-II Year-IV Sem Transform Techniques


44 3.3. Final Value Theorem

∴ lim L [F 0 (t)] = lim p f (p) − F(0)


p→∞ p→∞

0 = lim p f (p) − F(0)


p→∞

F(0) = lim p f (p)


p→∞

i.e., lim F(t) = lim p f (p)


t→0 p→∞

Note. If F(t) fails to be continuous at t = 0, but limt→0 F(t) exists then the result still holds.

3.3 Final Value Theorem

Theorem 3.3.1. Let F(t) be a continuous for all t ≥ 0 and be of exponential order as t → ∞. If
F 0 (t) is of class A then limt→∞ F(t) = lim p→0 L [F(t)] = lim p→0 p f (p).

Proof. By the theorem of Laplace transform of derivatives, we have


Z ∞
L [F (t)] =0
e−pt F 0 (t)dt = p f (p) − F(0).
0

Taking limit as p → 0 on both sides, we get

Z ∞
lim e−pt F 0 (t)dt = lim p f (p) − F(0)
p→0 0 p→0
Z ∞ 
lim e−pt F 0 (t)dt = lim p f (p) − F(0)
0 p→0 p→0
Z ∞
F 0 (t)dt = lim p f (p) − F(0)
0 p→0

Transform Techniques B.Sc.(Mathematics)-IDE(UNOM)-II Year-IV Sem


3.4. Worked out Problems 45

0 = lim p f (p) − F(0)


[F(t)]∞
p→0

lim F(t) − F(0) = lim p f (p) − F(0)


t→∞ p→0

lim F(t) = lim p f (p).


t→∞ p→0

Note. If F(t) fails to be continuous at t = 0, but limt→0 F(t) exists then the result still holds.

3.4 Worked out Problems

Example 3.4.1. Using the theorem on transform of derivatives, find the Laplace transform of the
following functions:
(i) eat ; (ii) cos at; (iii) t sin at.

Solution. (i) Let F(t) = eat . Then F 0 (t) = aeat and F(0) = 1.

Now

L [F 0 (t)] = p f (p) − F(0)

L [aeat ] = pL [eat ] − 1

aL [eat ] − pL [eat ] = −1

(a − p)L [eat ] = −1
1
L [eat ] = −
a− p
1
L [eat ] = .
p−a

(ii) Let F(t) = cos at. Then F 0 (t) = −a sin at, F 00 (t) = −a2 cos at.

L [F 00 (t)] = p2 L [F(t)] − pF(0) − F 0 (0)

B.Sc.(Mathematics)-IDE(UNOM)-II Year-IV Sem Transform Techniques


46 3.4. Worked out Problems

L [−a2 cos at] = p2 L [cos at] − p.1 − 0, since F(0) = 1 and F 0 (0) = 0.

−a2 L [cos at] − p2 L [cos at] = −p

(p2 + a2 )L [cos at] = p


p
L [cos at] = .
p2 + a2

(iii) Let F(t) = t sin at. Then F 0 (t) = sin at + at cos at, F 00 (t) = a cos at + a[cos at − at sin at] =
2a cos at − a2 t sin at.
F(0) = 0 and F 0 (0) = 0.

L [F 00 (t)] = p2 L [F(t)] − pF(0) − F 0 (0)

L [2a cos at − a2 t sin at] = p2 L [t sin at] − 0 − 0

2aL [cos at] − a2 L [t sin at] − p2 L [t sin at] = 0

2aL [cos at] − (a2 + p2 )L [t sin at] = 0


p
(a2 + p2 )L [t sin at] = 2a.
+ a2 p2
2ap
L [t sin at] = .
(p + a2 )2
2

√ √ √
π 1

Example 3.4.2. Evaluate L [ cos√t t ] (or) If L [sin t] = 3 e
− 4p
, find L [ cos√t t ].
2p 2
√ √
Solution. Let F(t) = sin t. Then F 0 (t) = 1√
2 t
cos t. Also F(0) = 0.
Now

L [F 0 (t)] = pL [F(t)] − F(0)


q√
h cos ti √
L √ = pL [sin t] − 0
2 t

Transform Techniques B.Sc.(Mathematics)-IDE(UNOM)-II Year-IV Sem


3.5. Exercises 47
q√

1 hcos ti π 1
L √ = p. 3 e− 4p
2 t 2p 2
q√

hcos ti π − 4p1
L √ = 1
e
t 2p 2
q√
ti π − 4p1
hcos r
L √ = e .
t p

h q i
Example 3.4.3. If L 2 πt = 1
3 show that L [ √1tπ ] = √1 .
p
p2
q
Solution. Let F(t) = 2 t
π
. Then F 0 (t) = √2 . 1√
π 2 t
= √1
πt
and F(0) = 0.

L [F 0 (t)] = pL [F(t)] − F(0)


r
h 1 i t
L √ = pL [2 ]−0
πt π
2p √
= √ L [ t]
π
 
2p 2Γ 3

= √
π p 32
 
1
2p 2 2Γ 1

= √
π p 32

p π
= √ 3
πp 2
h 1 i 1
L √ = √ .
πt p

3.5 Exercises

Using the theorem on transform of derivatives, evaluate:


2
(1) If L [t sin at] = (p22ap
+a2 )2
, then prove that L [sin at + at cos at] = (p2ap 2 +a2 )2
h t(1−cos ωt) i 2 +ω2
ωt
h i
(2) Given L 1−cos ω2
= p(p21+ω2 ) , show that L ω2
= p2 (p2 +ω2 )2 .
3p

B.Sc.(Mathematics)-IDE(UNOM)-II Year-IV Sem Transform Techniques


48 3.6. Laplace Transform of Integrals

3.6 Laplace Transform of Integrals


hR t i
Theorem 3.6.1. If L [F(t)] = f (p) then L 0
F(u)du = 1
p
f (p).

Rt
Proof. Let G(t) = 0
F(u)du. Then

Z t Z 0
dh i
g (t) =
0
F(u)du = F(t) and G(0) = f (u)du = 0.
dt 0 0

Taking Laplace transform on both sides, we get

L [G0 (t)] = L [F(t)].

But
L [G0 (t)] = pL [G(t)] − G(0) = pL [G(t)] − 0

∴ L [G0 (t)] = L [F(t)]

pL [G(t)] = L [F(t)]
1
L [G(t)] =
L [F(t)]
p
hZ t i 1 1
L F(u)du = L [F(t)] = f (p).
0 p p

hR t R t i
Note. Similarly if L [F(t)] = f (p) then L 0 0 F(u)dudu = 1
p2
f (p).
hR t R t R t i
In general, L 0 0 .... 0 F(u)dudu.....du = p1n f (p).

3.7 Worked out Problems


hR t i
Example 3.7.1. Find L 0
e−t cos tdt .

Transform Techniques B.Sc.(Mathematics)-IDE(UNOM)-II Year-IV Sem


3.8. Note on Differentiation under the Integral Sign 49

Solution. Let F(t) = e−t cos t. Then

h p i p+1
L [F(t)] = L [e−t cos t] = L [cos t] p→p+1 = = = f (p)
p2 + 1 p→p+1 (p + 1)2 + 1

Using the Laplace transform of integral, we have

hZ t
i 1
L F(u)du = f (p)
0 p
hZ t i 1 p+1
L e−u cos u du = . 2 .
0 p p + 2p + 2
hR t R t i
Example 3.7.2. Find L 0 0
cosh au du du .

Solution. Let F(t) = cosh at. Then

p
L [F(t)] = L [cosh at] = = f (p).
p2 − a2

Using the theorem on Laplace transform of integral, we have

hZ t i 1 p
L cosh au du = . 2
0 p p − a2

hZ t i 1
L cosh au du =
0 p2 − a2
hZ t Z t i 1 1 1
Applying again, L cosh au dudu = . 2 = .
0 0 p p − a2 p(p2 − a2 )

3.8 Note on Differentiation under the Integral Sign


Rb
The value of a definite integral f (x, α)dx is a function of α (called parameter), F(α) say.
a
Rb
To find the derivative of F(α), first we have to evaluate the definite integral a f (x, α)dx and
then differentiate F(α) with respect to α. However, it is not always possible to first evaluate
the integral and then to find its derivatives. Such problem are solved by reversing the order of

B.Sc.(Mathematics)-IDE(UNOM)-II Year-IV Sem Transform Techniques


50 3.9. Laplace Transform of tn F(t)

integration and differentiation i.e., first differentiate f (x, α) partially with respect to α and then
integrate it.

Leibnitz’s Rule: If f (x, α) and ∂α
[ f (x, α)] be continuous functions of x and α, then

b b

Z Z
dh i
f (x, α)dx = [ f (x, α)]dx.
dx a a ∂x

3.9 Laplace Transform of tn F(t)

1. Multiplication by t.

Theorem 3.9.1. If F(t) is sectionally continuous and of exponential order and if L [F(t)] = f (p)
then L [tF(t)] = − ddp f (p) = − f 0 (p).

Proof. We have Z ∞
L [F(t)] = f (p) = e−pt F(t)dt.
0

Then by Leibnitz’s rule for differentiation under the integral sign, we have
Z ∞
df d
= e−pt F(t)dt
dp dp 0
∂ −pt
Z ∞
= e F(t)dt
∂p
Z0 ∞
df
= −te−pt F(t)dt
dp 0

Z ∞
df
= − e−pt [tF(t)]dt
dp 0

= −L [tF(t)]
df
Thus L [tF(t)] = − = − f 0 (p).
dp

Transform Techniques B.Sc.(Mathematics)-IDE(UNOM)-II Year-IV Sem


3.9. Laplace Transform of tn F(t) 51

2. Multiplication by tn

Theorem 3.9.2. If F(t) is sectionally continuous and of exponential order and if L [F(t)] = f (p)
n
then L [tn F(t)] = (−n)n ddpn f (p), where n = 1, 2, 3, ...

Proof. We will use mathematical induction to prove the result.


R∞
By definition, we have f (p) = L [F(t)] = 0 e−pt F(t)dt.
Differentiating with respect to p, we get
Z ∞
d d
[ f (p)] = e−pt F(t)dt
dp dp
Z ∞ 0
∂ −pt
= e F(t)dt, by Leibnitz’s rule for differentiation under the integral sign
0 ∂p
Z ∞
= −te−pt F(t)dt
0
Z ∞
= − e−pt [tF(t)]dt
0

= −L [tF(t)]
d
T hus L [tF(t)] = (−1) [ f (p)],
dp

which proves the result for n = 1.


Now assume that the result is true for n = m (say), so that


dm
Z
e−pt [tm F(t)]dt = (−1)m [ f (p)].
0 d pm

Again differentiating with respect to p,

d h ∞ −pt m dm+1
Z i
e [t F(t)]dt = (−1)m m+1
dp 0 dp
∂ −pt m
Z ∞
dm+1
[e ][t F(t)]dt = (−1)m m+1 , by Leibnitz’s rule
0 ∂p dp

B.Sc.(Mathematics)-IDE(UNOM)-II Year-IV Sem Transform Techniques


52 3.10. Worked out Problems


dm+1
Z
[−te−pt ][tm F(t)]dt = (−1)m
0 d pm+1
Z ∞
dm+1
− e−pt [tm+1 F(t)]dt = (−1)m m+1
dp
Z0 ∞
dm+1
e−pt [tm+1 F(t)]dt = (−1)m+1 m+1
0 dp

This shows that, if the result is true for n = m, it is also true for n = m + 1. Thus, by the method
of mathematical induction the result is true for all values of n, where n is a positive integer.

dn
∴ L [tn F(t)] = (−n)n f (p).
d pn

3.10 Worked out Problems

Problems based on L [tn F(t)]

Example 3.10.1. Find (i) L [t sin at] (ii) L [t cos at].

Solution. (i)

d
L [t sin at] = − L [sin at]
dp
d h a i
= −
d p p2 + a2
h 1 i
= −a − 2 (2p)
(p + a2 )2
2ap
L [t sin at] = .
(p + a2 )2
2

Transform Techniques B.Sc.(Mathematics)-IDE(UNOM)-II Year-IV Sem


3.10. Worked out Problems 53

(ii)

d
L [t cos at] = − L [cos at]
dp
d h p i
= −
d p p2 + a2
h (p2 + a2 )(1) − p(2p) i
= −
(p2 + a2 )2
a2 − p2
L [t cos at] = − 2
(p + a2 )2

p2 − a2
L [t cos at] = .
(p2 + a2 )2

Example 3.10.2. Find (i) L [sin at − at cos at] (ii) L [t sin 3t cos 2t].

Solution. (i)

L [sin at − at cos at] = L [sin at] − aL [t cos at]


a d
= − a(−1) L [cos at]
p2 +a 2 dp
a d h p i
= + a
p2 + a2 d p p2 + a2
a h (p2 + a2 )(1) − p(2p) i
= + a
p2 + a2 (p2 + a2 )2
a h a2 − p2 i
= + a
p2 + a2 (p2 + a2 )2
ap2 + a3 + a3 − ap2
=
(p2 + a2 )2
2a3
L [sin at − at cos at] = .
(p2 + a2 )2

B.Sc.(Mathematics)-IDE(UNOM)-II Year-IV Sem Transform Techniques


54 3.10. Worked out Problems

(ii)

d
L [t sin 3t cos 2t] = − L [sin 3t cos 2t]
dp
d h sin 5t + sin t i
= − L
dp 2
1 d h 5 1 i
= − +
2 d p p2 + 25 p2 + 1
1h  1  1 i
= − 5. − 2 (2p) − (2p)
2 (p + 25)2 (p2 + 1)2
5p p
L [t sin 3t cos 2t] = + 2 .
(p + 25)
2 2 (p + 1)2

Example 3.10.3. Find (i) L [t2 cos 3t]; (ii) L [t2 sin 2t].

Solution. (i)

d2
L [t2 cos 3t] = L [cos 3t]
d p2
d2 h p i
=
d p2 p2 + 9

d h (p2 + 9)(1) − p(2p) i


L [t2 cos 3t] =
dp (p2 + 9)2
d h 9 − p2 i
=
d p (p2 + 9)2
(p2 + 9)2 (−2p) − (9 − p2 ).2(p2 + 9)(2p)
=
(p2 + 9)4
(p2 + 9)[(p2 + 9)(−2p) − 4p(9 − p2 )]
=
(p2 + 9)4
−2p3 − 18p − 36p + 4p3
=
(p2 + 9)3
2p3 − 54p
L [t2 cos 3t] = .
(p2 + 9)3

Transform Techniques B.Sc.(Mathematics)-IDE(UNOM)-II Year-IV Sem


3.10. Worked out Problems 55

(ii)

d2
L [t2 sin 2t] = L [sin 2t]
d p2
d2 h 2 i
=
d p2 p2 + 4
d h 1 i
= 2 − 2 (2p)
dp (p + 4)2
h (p2 + 4)2 (1) − p(2(p2 + 4)2p) i
= −4
(p2 + 4)4
h (p2 + 4)[(p2 + 4) − 4p2 ] i
= −4
(p2 + 4)4
2
h 4 − 3p i
= −4 2
(p + 4)3
4(3p2 − 4)
L [t2 sin 2t] = .
(p2 + 4)3

Example 3.10.4. Find the Laplace transform of the following functions:


(i) t(3 sin 2t − 2 cos 2t) (ii) (t2 − 3t + 2) sin 3t.

Solution. (i)

d
L [t(3 sin 2t − 2 cos 2t)] = − L [3 sin 2t − 2 cos 2t]
dp
d h 6 2p
= − − ]
d p p2 + 4 (p2 + 4)
d  6 − 2p i
L [t(3 sin 2t − 2 cos 2t)] = −
d p p2 + 4

h (p2 + 4)(−2) − (6 − 2p)(2p) i


L [t(3 sin 2t − 2 cos 2t)] = −
(p2 + 4)2
h −2p − 8 − 12p + 4p2 i
2
= −
(p2 + 4)2
h 2p2 − 12p − 8 i
= −
(p2 + 4)2
2(6p + 4 − p2 )
L [t(3 sin 2t − 2 cos 2t)] = .
(p2 + 4)2

B.Sc.(Mathematics)-IDE(UNOM)-II Year-IV Sem Transform Techniques


56 3.10. Worked out Problems

(ii)
L [(t2 − 3t + 2) sin 3t] = L [t2 sin 3t] − 3L [t sin 3t] + 2L [sin 3t] (3.10.1)
3
L [sin 3t] = .
p2 +9

d
L [t sin 3t] = − L [sin 3t]
dp
d h 3 i
= −
d p p2 + 9
h 1 i
= −3 − 2 (2p)
(p + 9)2
6p
L [t sin 3t] = .
(p + 9)2
2

d2
L [t2 sin 3t] = L [sin 3t]
d p2
d2 h 3 i
=
d p2 p2 + 9
d h 6p i
= − 2
dp (p + 9)2
(p2 + 9)2 (−6) + 6p.2(p2 + 9)2p
=
(p2 + 9)4
(p2 + 9)[(p2 + 9)(−6) + 24p2 ]
=
(p2 + 9)4
−6p2 − 54 + 24p2
=
(p2 + 9)3
18p2 − 54
L [t2 sin 3t] = .
(p2 + 9)3

Using L [sin 3t], L [t sin 3t] and L [t2 sin 3t] in (3.10.1), we get

18p2 − 54 6p 3
L [(t2 − 3t + 2) sin 3t] = + 2 + 2 .
(p + 9)
2 3 (p + 9) 2 p +9

Problems based on tn eat F(t)

Transform Techniques B.Sc.(Mathematics)-IDE(UNOM)-II Year-IV Sem


3.10. Worked out Problems 57

Example 3.10.5. Find the Laplace transform of the following functions:


(i) te−t cosh t; (ii) te2t sin 3t.

(i)
L [te−t cosh t] = L [t cosh t] p→p+1 , by first shifting theorem (3.10.2)

d h p i
L [t cosh t] = −
d p p2 − 1
(p2 − 1)(1) − p(2p)
= −
(p2 − 1)2
p +1
2
L [t cosh t] =
(p2 − 1)2

h p2 + 1 i (p + 1)2 + 1 p2 + 2p + 2
(3.10.2) ⇒ L [te−t cosh t] = = = .
(p2 − 1)2 p→p+1 ((p + 1)2 − 1)2 (p2 + 2p)2
Example 3.10.6. Find the Laplace transform of the following functions:
(i) (1 + te−t )2 ; (ii) te−t sin t.

Solution. (i)
(1 + te−t )2 = 1 + 2te−t + t2 e−2t

L [(1 + te−t )2 ] = L [1 + 2te−t + t2 e−2t ]

= L [1] + 2L [te−t + L [t2 e−2t ]


1 d h 1 d2 h 1 i
= −2 ]+ 2
p dp p + 1 dp p + 2
1 1 d h 1
= − 2[− ] + − ]
p (p + 1)2 dp (p + 2)2
1 2 2
L [(1 + te−t )2 ] = + + .
p (p + 1) 2 (p + 2)3

(ii)
L [te−t sin t] = L [t sin t] p→(p+1) (3.10.3)

B.Sc.(Mathematics)-IDE(UNOM)-II Year-IV Sem Transform Techniques


58 3.10. Worked out Problems

d
L [t sin t] = − L [sin t]
dp
d h 1 i
= −
d p p2 + 1
h 1 i
= − − 2 (2p)
(p + 1)2
2p
L [t sin t] =
(p + 1)2
2

h 2p i 2)(p + 1) 2(p + 1)
(3.10.3) ⇒ L [te−t sin t] = = = 2 .
(p + 1) p→(p+1) ((p + 1) + 1)
2 2 2 2 (p + 2p + 2)2
Example 3.10.7. Find the Laplace transform of the following functions:
(i) t2 e−2t cos t; (ii) t3 e2t sin t.

Solution. (i)
L [t2 e−2t cos t] = L [t2 cos t] p→p+2 (3.10.4)

d2
L [t2 cos t] = L [cos t]
d p2
d2 h p i
=
d p2 p2 + 1
d h (p2 + 1)(1) − p(2p) i
=
dp (p2 + 1)2
d h 1 − p2 i
=
d p (p2 + 1)2
(p2 + 1)2 (−2p) − (1 − p2 )2(p2 + 1)(2p)
=
(p2 + 1)4
(p + 1)[(p + 1)(−2p) − 4p(1 − p2 )]
2 2
=
(p2 + 1)4
−2p − 2p − 4p + 4p3
3
=
(p2 + 1)3
2p3 − 6p
L [t2 cos t] =
(p2 + 1)3

Transform Techniques B.Sc.(Mathematics)-IDE(UNOM)-II Year-IV Sem


3.10. Worked out Problems 59
h 2p3 − 6p i
(3.10.4) ⇒ L [t2 e−2t cos t] =
(p2 + 1)3 p→p+2
2(p + 2)3 − 6(p + 2)
L [t2 e−2t cos t] =
((p + 2)2 + 1)3

2(p + 2)((p + 2)2 − 3)


L [t2 e−2t cos t] =
(p2 + 4p + 5)3
2(p + 2)(p2 + 4p + 1)
L [t2 e−2t cos t] = .
(p2 + 4p + 5)3

(ii)
L [t3 e2t sin t] = L [t3 sin t] p→p−2 (3.10.5)

d3
L [t3 sin t] = (−1)3 L [sin t]
d p3
d3 h 1 i
= −1
d p3 p2 + 1
d2 h 2p i
= −1 2 − 2
dp (p + 1)2
d h (p2 + 1)2 (1) − p.2(p2 + 1)(2p) i
= 2
dp (p2 + 1)4
d h (p + 1)[p + 1 − 4p2 ] i
2 2
= 2
dp (p2 + 1)4
d h 1 − 3p2 i
= 2
d p (p2 + 1)3
h (p2 + 1)3 (−6p) − (1 − 3p2 )3(p2 + 1)2 (2p) i
= 2
(p2 + 1)6
h (p2 + 1)2 [(p2 + 1)(−6p) − 6p(1 − 3p2 )] i
= 2
(p2 + 1)6
h −6p3 − 6p − 6p + 18p3 i
= 2
(p2 + 1)4
h 12p3 − 12p i
= 2
(p2 + 1)4
24p(p2 − 1)
L [t3 sin t] =
(p2 + 1)4

B.Sc.(Mathematics)-IDE(UNOM)-II Year-IV Sem Transform Techniques


60 3.10. Worked out Problems

h 24p(p2 − 1) i
(3.10.5) ⇒ L [t3 e2t sin t] =
(p2 + 1)4 p→p−2
24(p − 2)((p − 2)2 − 1)
=
((p − 2)2 + 1)4
24(p − 2)(p2 − 4p + 3)
L [t3 e2t sin t] = .
(p2 − 4p + 5)4

Example 3.10.8. Find L [teat sin bt].

Solution.
L [teat sin bt] = L [t sin at] p→p−a (3.10.6)

d
L [t sin bt] = − L [sin bt]
dp
d h b i
= −
d p p2 + b2
h 1 i
= − −b 2 (2p)
(p + b2 )2
2bp
L [t sin bt] =
(p + b2 )2
2
h 2bp i
(3.10.6) ⇒ L [teat sin bt] =
(p2 + b2 )2 p→p−a
2b(p − a)
L [teat sin bt] =
((p − a)2 + b2 )2
hR t i
Example 3.10.9. Find L 0
te−t sin 4tdt .

Solution.
hZ t i 1
L te−t sin 4tdt = .L [te−t sin 4t] (3.10.7)
0 p

L [te−t sin 4t] = L [t sin 4t] p→p+1 (3.10.8)

Transform Techniques B.Sc.(Mathematics)-IDE(UNOM)-II Year-IV Sem


3.10. Worked out Problems 61
d
L [t sin 4t] = − L [sin 4t]
dp
d h 4 i
= −
d p p2 + 16
h 1 i
= − −4 2 (2p)
(p + 16)2
8p
L [t sin bt] =
(p + 16)2
2
h 8p i
(3.10.8) ⇒ L [te−t sin 4t] =
(p2 + 16)2 p→p+1
8(p + 1)
=
((p + 1)2 + 16)2
8(p + 1)
L [te−t sin 4t] =
(p + 2p + 17)2
2

hZ t i 1 8(p + 1)
(3.10.7) ⇒ L te−t sin 4tdt = . 2
0 p (p + 2p + 17)2
h t 8(p + 1)
Z i
L te−t sin 4tdt = .
0 p(p + 2p + 17)2
2

hR t i
Example 3.10.10. Find L 0
te−t sin 2tdt .

Solution.
hZ t i 1
L te−t sin 2tdt = .L [te−t sin 2t] (3.10.9)
0 p

L [te−t sin 2t] = L [t sin 2t] p→p+1 (3.10.10)

d
L [t sin 2t] = − L [sin 2t]
dp
d h 2 i
= −
d p p2 + 4
h 1 i
= − −2 2 (2p)
(p + 4) 2

4p
L [t sin bt] =
(p + 4)2
2

B.Sc.(Mathematics)-IDE(UNOM)-II Year-IV Sem Transform Techniques


62 3.10. Worked out Problems

h 4p i
(3.10.10) ⇒ L [te−t sin 2t] =
(p2 + 4)2 p→p+1
4(p + 1)
=
((p + 1)2 + 4)2
8(p + 1)
L [te−t sin 2t] =
(p + 2p + 5)2
2

hZ t i 1 4(p + 1)
(3.10.9) ⇒ L te−t sin 2tdt = . 2
0 p (p + 2p + 5)2
hZ t i 4(p + 1)
L te−t sin 2tdt = .
0 p(p + 2p + 5)2
2


1 π 1
Example 3.10.11. If L [t 2 ] = 3 then find L [t− 2 ].
2p 2

1
Solution. Let F(t) = t− 2 .

We have L [tF(t)] = − ddp [ f (p)]

1 d
L [t.t− 2 ] = − ( f (p))
dp

1 d
L [t 2 ] = − ( f (p))
dp

π d
3
= − ( f (p))
2p 2 dp

d π
( f (p)) = − 3
dp 2p 2
√ Z
π dp
f (p) = − 3
2 p2

π 2
=
2 p 12
π
r
L [F(t)] =
p
π
r
1
i.e., L [t− 2 ] = .
p

Transform Techniques B.Sc.(Mathematics)-IDE(UNOM)-II Year-IV Sem


3.11. Division by t 63

3.11 Division by t
h F(t) i R∞
Theorem 3.11.1. If L [F(t)] = f (p) then L t
= 0
f (p)d p, provided the integral exists.

Proof. By the definition, Z ∞


f (p) = L [F(t)] = e−pt F(t)dt
0

Integrating on both sides with respect to p from p = p to p = ∞, we get


Z ∞ Z ∞ hZ ∞ i
f (p)d p = e−pt F(t)dt d p
p p 0

Interchanging the order of integration in the repeated integrals as p and t are independent
variables, we have
Z ∞ Z ∞ Z ∞
f (p)d p = F(t) e−pt d pdt
p 0 p
Z ∞ hZ ∞ i
= F(t) e−pt d p dt
0 p
Z ∞ Z ∞ h e−pt i∞
f (p)d p = F(t) dt
p 0 −t p

Z ∞ Z ∞ h 0 − e−pt i
f (p)d p = F(t) dt
p 0 −t
Z ∞
e−pt
= F(t) dt
0 t
Z ∞ h F(t) i
= e−pt dt
0 t
Z ∞ h F(t) i
f (p)d p = L
p t
h F(t) i Z ∞
Hence L = f (p)d p.
t p

B.Sc.(Mathematics)-IDE(UNOM)-II Year-IV Sem Transform Techniques


64 3.12. Worked out Problems

Note. The result can be extended to repeated integration of the transformation.

3.12 Worked out Problems


h F(t) i
Problems based on L t

Example 3.12.1. Find the Laplace transform of the following functions:


(i) sin t
t
; (ii) sin at
t
.

Solution. We know that


h F(t) i Z ∞
L = f (p)d p.
t p

(i)

h sin t i Z ∞
L = L [sin t]dt
t p
Z ∞
1
= dt
p p2 +1
= [tan−1 p]∞
p

= tan−1 (∞) − tan−1 (p)


π
= − tan−1 (p)
2
= cot−1 (p)
h sin t i 1
L = tan−1 .
t p

(ii)
h sin at i h sin at i
L = aL (3.12.1)
t at
Let F(t) = sin t
t
. Then
h sin t i
L [F(t)] = L = cot−1 (p),
t

Transform Techniques B.Sc.(Mathematics)-IDE(UNOM)-II Year-IV Sem


3.12. Worked out Problems 65

by using (i).
h sin at i 1 −1  p 
L = cot ,
at a a
by change of scale of property.

h sin at i 1  p  p
∴ (3.12.1) ⇒ L = a. cot−1 = cot−1 .
t a a a
h i
e−at −e−bt
Example 3.12.2. Find L t
.

Solution. We know that


h F(t) i Z ∞
L = f (p)d p.
t p

h e−at − e−bt i Z ∞
L = L [e−at − e−bt ]d p
t p
Z ∞ h 1 1 i
= − dp
p p+a p+b
h i∞
= log(p + a) − log(p + b)
p
h  p + a i∞
= log
p+b p
h p(1 + ap ) i∞
= log
p(1 + bp ) p
h (1 + ap ) i∞
= log
(1 + bp ) p

(1 + ap )
= log 1 − log[ ]
(1 + bp )
h p + ai
= − log
p+b
h e−at − e−bt i h p + bi
L = log .
t p+a
h i
Example 3.12.3. Show that L cos at
t
does not exist.

B.Sc.(Mathematics)-IDE(UNOM)-II Year-IV Sem Transform Techniques


66 3.12. Worked out Problems

Solution.
h cos at i Z ∞
L = L [cos at]d p
t p
Z ∞
p
= dp
p p2 + a2
1
= log(p2 + a2 )∞
p
2
h cos at i 1
L = [log(∞) − log(p2 + a2 )],
t 2

does not exist.


h i
Example 3.12.4. Find L sin 3t cos t
t
.

Solution.
h sin 3t cos t i Z ∞
L = L [sin 3t cos t]d p
t p
Z ∞
1
= L [sin 4t + sin 2t]d p
p 2
1 ∞h 4
Z
2 i
= + dp
2 p p2 + 16 p2 + 4
1 h 1 −1  p  1  p i∞
= 4. tan + 2. tan−1
2 4 4 2 2 p
1 h   p  p i
= tan−1 (∞) + tan−1 (∞) − tan−1 + tan−1
2 4 2
1 h π π  −1  p   p i
= + − tan + tan−1
2 2 2 4 2
h sin 3t cos t i π 1  −1  p   p 
L = − tan + tan−1 .
t 2 2 4 2
h i
Example 3.12.5. Find L cos 2t−cos 3t
t
.

Solution.
h cos 2t − cos 3t i Z ∞
L = L [cos 2t − cos 3t]d p
t p

Transform Techniques B.Sc.(Mathematics)-IDE(UNOM)-II Year-IV Sem


3.12. Worked out Problems 67
Z ∞
p p i h
− = dp
p p2 + 4 p2 + 9
1 ∞ h 2p
Z
2p i
= − dp
2 p p2 + 4 p2 + 9
h cos 2t − cos 3t i 1
L = [log(p2 + 4) − log(p2 + 9)]∞
p
t 2

h cos 2t − cos 3t i 1h  p2 + 4 i∞


L = log 2
t 2 p +9 p
1h  p2 (1 + p42 ) i∞
= log 2
2 p (1 + p29
) p
1h 1 + 4
p2 ∞
i
= log
2 1+ 9
p2
p

1h  1 + p2 i
4

= log 1 − log
2 1 + p2
9

1h  p2 + 4 i
= 0 − log 2
2 p +9
1  p + 9
2
= log 2
2 p +4
h cos 2t − cos 3t i  p2 + 9  12
L = log 2 .
t p +4
h i
1−et
Example 3.12.6. Find L t
.

Solution.
h 1 − et i Z ∞
L = L [1 − et ]d p
t p
Z ∞ h1
1 i
= dp −
p p p−1
h i∞
= log p − log(p − 1)
p
h  p i∞
= log
p−1 p

B.Sc.(Mathematics)-IDE(UNOM)-II Year-IV Sem Transform Techniques


68 3.12. Worked out Problems

h p  i∞
= log
p(1 − 1p ) p
h  1 i∞
= log
(1 − 1p ) p
 1 
= log 1 − log
1 − 1p
 p 
= 0 − log
p−1
h 1 − et i  p − 1
L = log .
t p
hR t i
1−e−t
Example 3.12.7. Find L 0 t
dt .

Solution.
h Z t 1 − e−t i 1 h 1 − e−t i
L dt = .L (3.12.2)
0 t p t

h 1 − e−t i Z ∞
L = L [1 − e−t ]d p
t p
Z ∞ h1 1 i
= − dp
p p p+1
h i∞
= log p − log(p + 1)
p
h  p i∞
= log
p+1 p
h  p i∞
= log
p(1 + 1p ) p
h  1 i∞
= log
(1 + 1p ) p
 1 
= log 1 − log
1 + 1p
 p 
= 0 − log
p+1
h 1 − e−t i  p + 1
L = log
t p

Transform Techniques B.Sc.(Mathematics)-IDE(UNOM)-II Year-IV Sem


3.12. Worked out Problems 69

h Z t 1 − e−t i 1  p + 1
∴ (3.12.2) ⇒ L dt = log .
0 t p p
hR t i
et sin t
Example 3.12.8. Find L 0 t
dt .

Solution.
h Z t et sin t i 1 h et sin t i
L dt = .L (3.12.3)
0 t p t

h et sin t i h sin t i
L =L (3.12.4)
t t p→(p−1)

h sin t i Z ∞
L = L [sin t]d p
t p

h sin t i h i∞
L = tan−1 (p)
t p

= tan−1 (∞) − tan−1 (p)


π
= − tan−1 (p)
2
h sin t i
L
= cot−1 (p)
t
h et sin t i
∴ (3.12.4) ⇒ L = cot−1 (p) p→(p−1)
t
= cot−1 (p − 1)
h Z t et sin t i 1
∴ (3.12.3) ⇒ L dt = cot−1 (p − 1).
0 t p
h i
Example 3.12.9. Find L 1−cos at
t
.

Solution.
h 1 − cos at i Z ∞
L = L [1 − cos at]d p
t p

B.Sc.(Mathematics)-IDE(UNOM)-II Year-IV Sem Transform Techniques


70 3.12. Worked out Problems
Z ∞ h1 p i
= − dp
p p p2 + a2
Z ∞h
1 1 2p i
= − dp
p p 2 p2 + a2
h 1 i∞
= log p − log(p2 + a2 )
2 p
h p i∞
= log p − log( p2 + a2 )
p
h p i∞
= log p
p + a2 p
2
h p i∞
= log q
2 p
p 1 + ap2
h 1 i∞
= log q
2 p
1 + ap2
h 1 i
= log 1 − log q
1 + ap2
2

h p i
= 0 − log p
p2 + a2
h p2 + a2 i
p
h 1 − cos at i
L = log .
t p
h i
Example 3.12.10. Find L 1−cos t
t2
.

Solution.
h 1 − cos at i Z ∞ Z ∞
L = L [1 − cos t]d p
t2 p p
h p2 + 1 i
Z ∞ p
= log , putting a = 1 in the above problem
p p
Z ∞ p
= [log p2 + 1 − log p]d p
p
Z ∞
1
= [ log(p2 + 1) − log p]d p
p 2
Z ∞
1
= [log(p2 + 1) − 2 log p]d p
2 p

Transform Techniques B.Sc.(Mathematics)-IDE(UNOM)-II Year-IV Sem


3.12. Worked out Problems 71

Let u = log[(p2 + 1) − 2 log p] and dv = d p Then du = 2p


p2 +1
− 2
p
and v = p.

h 1 − cos at i 1 h ∞ Z ∞  2p 2 i
L = log((p + 1) − 2 log p).p −
2
p. 2 − dp
t2 2 p
p p +1 p
hp  p + 1 i∞ 1
2 Z ∞
1
= log − −2. 2 dp
2 p 2 p 2 p p +1
hp  p2 (1 + p12 ) i∞ Z ∞ 1
= log + dp
2 p2 p
p p2 + 1
p  1
= − log 1 + 2 + (tan−1 p)∞ p
2 p
p  p2 + 1 
= − log + tan−1 (∞) − tan−1 (p)
2 p2
p  p2  π
= log 2 + − tan−1 (p)
2 p +1 2
h 1 − cos at i p  p +1
2 
L = log + cot−1 (p).
t2 2 p2
h i
e−3t sin 2t
Example 3.12.11. Find L t
.

Solution.
h e−3t sin 2t i h sin 2t i
L =L . (3.12.5)
t t p→p+3

h sin 2t i Z ∞
L = L [sin 2t]d p
t p

h sin 2t i Z ∞
2
L = dp
t p +4 p2
h1 p i∞
= 2 tan−1
2 2 p
 p
= tan−1 (∞) − tan−1
2
π −1 p
 
= − tan
2 2
h sin 2t i  p
L = cot−1
t 2

B.Sc.(Mathematics)-IDE(UNOM)-II Year-IV Sem Transform Techniques


72 3.12. Worked out Problems

h e−3t sin 2t i h  p i
∴ (3.12.5) ⇒ L = cot−1
t 2 p→p+3
he −3t
sin 2t i  p + 3
L = cot−1 .
t 2
h i
Example 3.12.12. Find L cos 4t sin 2t
t
.

Solution.
h cos 4t sin 2t i Z ∞
L = L [cos 4t sin 2t]d p
t p
Z ∞
1
= L [sin 6t − sin 2t]d p
p 2
1 ∞h 6
Z
2 i
= − dp
2 p p2 + 36 p2 + 4
Z ∞h
3 1 i
= − dp
p p2 + 36 p2 + 4
h 1  p 1  p i∞
= 3. tan−1 − tan−1
6 6 2 2 p
1 −1 1 −1 1  p 1  p 
= tan (∞) − tan (∞) − tan−1 − tan−1
2 2 2 6 2 2
1 h  p   p i
= tan−1 − tan−1 .
2 2 6
h Rt i
Example 3.12.13. Find L e−3t 0 sin t
t
dt .

Solution. Z t
h sin t i h Z t sin t i
L e −3t
dt = L dt . (3.12.6)
0 t 0 t p→p+3

h Z t sin t i 1 h sin t i
L dt = L
0 t p t
Z ∞
1
= L [sin t]d p
p p
1 ∞ 1
Z
= dp
p p p2 + 1

Transform Techniques B.Sc.(Mathematics)-IDE(UNOM)-II Year-IV Sem


3.12. Worked out Problems 73
1 h −1 i∞
= tan (p)
p p

1 π
h i
= − tan−1 (p)
p 2
1
= cot−1 (p)
p

Z t
h sin t i h1 i
∴ (3.12.6) ⇒ L e−3t
dt = cot−1 (p)
t p p→p+3
Z0 t
h sin t i 1
L e−3t dt = cot−1 (p + 3).
0 t p + 3
h i
Example 3.12.14. Find L sinh at
t
.

Solution.
h sinh at i Z ∞
L = L [sinh at]d p
t p
Z ∞
a
= dp
p − a2p2
1 p − a ∞
= a. log
2a p+a p
1 1 − a ∞
p
= log
2 1+ p a p

1h 1 − ap i
= log 1 − log
2 1 + ap
1h  p − a i
= 0 − log
2 p+a
h sinh at i 1  p + a
L = log .
t 2 p−a
h i
Example 3.12.15. Show that L cosh at
t
does not exist.

B.Sc.(Mathematics)-IDE(UNOM)-II Year-IV Sem Transform Techniques


74 3.13. Evaluation of Integrals by using Laplace Transform

Solution.
h cosh at i Z ∞
L = L [cosh at]d p
t p
Z ∞
p
= dp
p p2 − a2
1
= log[p2 − a2 ]∞
p
2
h cosh at i 1
L = [log(∞) − log(p2 − a2 )].
t 2

This does not exist.

3.13 Evaluation of Integrals by using Laplace Transform

Sometimes, evaluation of improper integrals can be done easily by using Laplace transform
technique.

3.14 Worked out Problems

Example 3.14.1. Evaluate the following:


R∞ R∞
(i) 0 te−3t dt (ii) 0 e−4t sin 3tdt.

Solution. (i) By definition,


Z ∞
L [F(t)] = e−pt F(t)dt = f (p).
0

Z ∞ Z ∞
te dt =
−3t
e−pt tdt
0 0 p=3

= L [t] p=3
h1i 1
= 2
= .
p p=3 9

Transform Techniques B.Sc.(Mathematics)-IDE(UNOM)-II Year-IV Sem


3.14. Worked out Problems 75

(ii)
Z ∞ Z ∞
e −4t
sin 3tdt = e−pt sin 3tdt
0 0 p=4

= L [sin 3t] p=4

Z ∞
3 i h
e−4t sin 3tdt =
p2 + 9 p=4
Z0 ∞
3
e−4t sin 3tdt = .
0 25
R∞ sin 2t
Example 3.14.2. Evaluate 0 t
dt.

Solution. Z ∞ Z ∞
sin 2t sin 2t
dt = e−pt dt
0 t 0 t p=0
Z ∞
sin 2t h sin 2t i
dt = L (3.14.1)
0 t t p=0

h sin 2t i Z ∞
L = L [sin 2t]d p
t p
Z ∞
2
= dp
p +4 p2
h 1  p i
= 2. tan−1
2 2
π  p
= − tan−1
2 2

π
Z
sin 2t  p 
∴ (3.14.1) dt = − tan−1
t 2 2 p=0
Z0 ∞
sin 2t π
dt = .
0 t 2
R∞ R∞
Example 3.14.3. Show that (i) 0
te−3t sin tdt = 3
50
; (ii) 0
t3 e−t sin tdt = 0.

Solution. (i) Z ∞ Z ∞
te −3t
sin tdt = e−pt [t sin t]dt = L [t sin t] p=3 . (3.14.2)
0 0 p=3

B.Sc.(Mathematics)-IDE(UNOM)-II Year-IV Sem Transform Techniques


76 3.14. Worked out Problems

d
L [t sin t] = − L [sin t]
dp
d h 1 i
= −
d p p2 + 1
h 2p i
= − − 2
(p + 1)2
2p
L [t sin t] =
(p + 1)2
2

Z ∞
2p i h
∴ (3.14.2) ⇒ te−3t sin tdt =
(p2 + 1)2 p=3
Z0 ∞
3
te−3t sin tdt = .
0 50

(ii) Z ∞ Z ∞
t e sin tdt =
3 −t
e−pt
[t sin t]dt = L [t3 sin t] p=1 .
3
(3.14.3)
0 0 p=1

d3
L [t3 sin t] = (−1)3 L [sin t]
d p3
d3 h 1 i
= − 3 2
dp p + 1
d2 h 2p i
= − 2 − 2
dp (p + 1)2
d h (p2 + 1)2 (1) − p.2(p2 + 1)2p i
= 2
dp (p2 + 1)4
d h (p + 1)[p2 + 1 − 4p2 ] i
2
= 2
dp (p2 + 1)4
d h 1 − 3p2 i
= 2
d p (p2 + 1)3
h (p2 + 1)3 (−6p) − (1 − 3p2 ).3(p2 + 1)2 (2p) i
= 2
(p+ 1)6
h (p2 + 1)2 [(p2 + 1)(−6p) − 6p(1 − 3p2 )] i
= 2
(p2 + 1)6
h (p2 + 1)(−6p) − 6p(1 − 3p2 ) i
= 2
(p2 + 1)4

Transform Techniques B.Sc.(Mathematics)-IDE(UNOM)-II Year-IV Sem


3.14. Worked out Problems 77

h −6p3 − 6p − 6p + 18p3 i
= 2
(p2 + 1)4
h 12p3 − 12p i
= 2
(p2 + 1)4
24p(p2 − 1)
=
(p2 + 1)4
Z ∞ h 24p(p2 − 1) i
∴ (3.14.3) ⇒ t3 e−t sin tdt = = 0.
0 (p2 + 1)4 p=1
R∞ e−at −e−bt
R∞ e−3t −e−6t
R∞ e−t −e−2t
Example 3.14.4. Evaluate (i) 0 t
dt; (ii) 0 t
dt; (iii) 0 t
dt.

Solution. (i)

∞ ∞
e−at − e−bt h e−at − e−bt i h e−at − e−bt i
Z Z
dt = e−pt dt = L . (3.14.4)
0 t 0 t p=0 t p=0

h e−at − e−bt i h p + bi
L = log , by Example 3.12.2.
t p+a
Z ∞
e−at − e−bt h p + bi b
∴ (3.14.4) ⇒ dt = log = log .
0 t p + a p=0 a
(ii) Put a = 3, b = 6 in (i), we get


e−3t − e−6t
Z 6
dt = log = log 2.
0 t 3

(iii) Put a = 1, b = 2 in (i), we get


e−1t − e−2t
Z 2
dt = log = log 2.
0 t 1
R∞ cos at−cos bt
Example 3.14.5. Evaluate 0 t
dt, by using Laplace transform.

Solution.
Z ∞ Z ∞
cos at − cos bt h cos at − cos bt i h cos at − cos bt i
dt = e−pt dt = L . (3.14.5)
0 t 0 t p=0 t p=0

B.Sc.(Mathematics)-IDE(UNOM)-II Year-IV Sem Transform Techniques


78 3.14. Worked out Problems

h cos at − cos bt i Z ∞
L = L [cos at − cos bt]d p
t p
Z ∞ h p p i∞
= −
p p2 + a2 p2 + b2 p
1
= [log(p2 + a2 ) − log(p2 + b2 )]∞
p
2
1  p +b 
2 2
= log
2 (p2 + a2 )
Z ∞
cos at − cos bt 1  p2 + b2 
∴ (3.14.5) ⇒ dt = log
0 t 2 (p2 + a2 ) p=0
1  b2 
= log 2
2 a
Z ∞
cos at − cos bt b
dt = log .
0 t a
R∞ e−at sin2 t
Example 3.14.6. Using Laplace transform, evaluate 0 t
dt.

Solution.
∞ ∞
e−at sin2 t h sin2 t i h sin2 t i
Z Z
dt = −pt
e dt = L
. (3.14.6)
0 t 0 t p=a t p=a

h sin2 t i Z ∞
L = L [sin2 t]d p
t p
Z ∞ h 1 − cos 2t i
= L dp
p 2
1 ∞ h1
Z
p i
= − 2 dp
2 p p p +4
1h 1 i∞
= log p − log(p2 + 4)
2 2 p
1 h  p i ∞
= log p
2 p +4 p
2

1h  p i
= log 1 − log p
2 p2 + 4
 p2 + 4 
p
1
= log
2 p

Transform Techniques B.Sc.(Mathematics)-IDE(UNOM)-II Year-IV Sem


3.14. Worked out Problems 79

 p2 + 4 

p
e−at sin2 t
Z
1
∴ (3.14.6) ⇒ dt = log
0 t 2 p p=a

Z ∞
e−at sin2 t 1  a2 + 4 
dt = log .
0 t 2 a

Note: Put a = 1 in the above result, we have



Z ∞
e−t sin2 t 1  1 + 4 1 √ 1
dt = log = log 5 = log 5.
0 t 2 1 2 4
R∞
Example 3.14.7. Using Laplace transform, evaluate 0
te−t sin tdt.

Solution. Z ∞ Z ∞
te sin tdt =
−t
e−pt [t sin t]dt = L [t sin t] p=1 . (3.14.7)
0 0 p=1

d
L [t sin t] = − L [sin t]
dp

d h 1 i
L [t sin t] = −
d p p2 + 1
h 2p i
=
(p2 + 1)2
2p
=
(p + 1)2
2
Z ∞ h 2p i
∴ (3.14.7) ⇒ te−t sin tdt =
(p2 + 1)2 p=1
Z0 ∞
1
te−t sin tdt = .
0 2

B.Sc.(Mathematics)-IDE(UNOM)-II Year-IV Sem Transform Techniques


80 3.15. Exercises

3.15 Exercises

1. Find the Laplace transform of the following functions:


(i) t sin2 t; (ii) t sin2 3t; (iii) t sinh at;
(iv) t2 cos t; (v) t2 sin at; (vi) t3 e−3t ;
(vii) t3 cos at; (viii) t cos(at + b); (ix) t(1−cos at)
a2
.
2. Find the Laplace transform of the following functions:
(i) teat sin at; (ii) te−3t cos 2t; (iii) te−t cosh t;
(iv) t2 e−t cos2 t; (v) te−4t sin 3t; (vi) t cos(at + b).
3. Find the Laplace transform of the following functions:
sin ht sin at cos at
(i) t
; (ii) t
; (iii) t
;
1−e2t 1−e−t cos at−cos bt
(iv) t
; (v) t
; (vi) t
;
eat −cos bt sin2 t
(vii) t
; (viii) t
; (ix) sin 3t cos t
t
.
4. Using Laplace transform, show that
R∞ R∞
(i) 0 te−2t cos tdt = 25
3
; (ii) 0 t2 e−4t sin 2tdt = 11
500
.

5. Evaluate the following:


R∞ R∞ R∞ 2
(i) 0 sint t dt (ii) 0 cos 6t−cos
t
4t
dt; (iii) 0
e−t sint t dt.

6. Evaluate the following:


hR t u i hR t i
1−e−2t
(i) L 0 1−eu
du ; (ii) L 0 t
dt .

7. Evaluate the following:


hR t i hR t i hR t i
(i) L 0 eat sinh btdt ; (ii) L 0 et cosh tdt ; (iii) L 0
te−t sin tdt .

8. Evaluate the following:


hR t R t i hR t R t i
(i) L 0 0 cos atdtdt ; (ii) L 0 0 sin atdtdt .

Answers
2 +4) 54(p2 +12)
1. (i) p2(3p
2 (p2 +4)2 ; (ii) p2 (p2 +36)2
; (iii) 2ap
(p2 −a2 )2
;
2p3 −6p 2
2a(3p −a ) 2
−6
(iv) (p2 +1)3
; (v) (p2 +a2 )3
; (vi) (p+3)4
;
(vii) 6(p (p+a2 +a)−6a 3p +a2
4 4 2 p2 2
2 )4 ; (viii) 1
(p2 +a2 )2
[(p2 − a2 ) cos b − 2ap sin b]; (ix) p2 (p2 +a2 )2
.

Transform Techniques B.Sc.(Mathematics)-IDE(UNOM)-II Year-IV Sem


3.15. Exercises 81

2a(p−a) p2 +6p+5 p2 +2p+2


2. (i) (p2 −2ap+2a2 )2
; (ii) (p2 +6p+13)2
; (iii) (p2 +2p)2
;
(v) 6(p+4)
(p2 +8p+25)2
.
   
3. (i) 12 log p+1p−1
; (ii) cot−1 ap ; (iii) Does not exists;
(vi) 21 log pp2 +b
 p−2   p+1   2 2
(iv) log p ; (v) log p ; +a2
;
 p2 +b2   p2 +4 
(vii) 21 log (p−a)2 ; (viii) 14 log p2 ; (ix) π2 − 12 tan−1 4p + tan−1 2p .
h    i

5. (i) π2 ; (ii) log 23 ; (iii) 21 log 5.


 

   
6. (i) 1p log 1 − 1p ; (ii) 1p log 1 + 2p .
7. (i) b
p(p−a)2 −b2
; (ii) p−1
p2 (p−2)
; (iii) 2(p+1)
p(p2 +2p+2)2
.
8. (i) 1
p(p2 +a2 )
; (ii) a
p2 (p2 +a2 )
.

B.Sc.(Mathematics)-IDE(UNOM)-II Year-IV Sem Transform Techniques


82
UNIT-I

Lesson 4

LAPLACE TRANSFORM OF
SOME SPECIAL FUNCTIONS AND
PERIODIC FUNCTIONS
Learning Objectives
Upon completion of this lesson, students will be able to

• find Laplace transform of some special functions

• solve Laplace transform of periodic functions.

4.1 Laplace Transform of Some Special Functions

I
n this Lesson, we study laplace Transform of some special functions.

1. Unit Impulse Function (or Dirac Delta Function)


The idea of a very large force (like earthquake) acting for a very short duration of time is of
practical importance as, for instance, in the collision of two bodies. To deal with such situations,
we introduce a function called the unit impulse function which is a discontinuous function. Thus

B.Sc.(Mathematics)-IDE(UNOM)-II Year-IV Sem 83 Transform Techniques


84 4.1. Laplace Transform of Some Special Functions

unit impulse function is considered as the limiting form of the function


0, for t < a








f (t − a) = 

, for a ≤ t ≤ a + 
1


 ε


0, for t > a


In other words, 
 ε , for a ≤ t ≤ a + 


 1

f (t − a) = 



0, otherwise

As ε > 0 it is clear from figure that as ε → 0, the height of shaded strip increases indefinitely
and the width decreases in such a way that its area is always unity.
The limit of fε (t − a) as ε → 0 is denoted by δ(t − a) and is called Dirac delta function
(sometimes the unit impulse function).
Thus, the unit impulse function δ(t − a) is defined as follows:

∞, for t = a




δ(t − a) = 



0, t , a

Laplace Transform of Dirac Delta Function.


Z ∞
L [F(t)] = e−pt F(t)dt
Z0 ∞
L [ fε (t − a)] = e−pt fε (t − a)dt
Z0 a+ε Z ∞
= fε (t − a)dt +
e−pt
e−pt fε (t − a)dt
Za a+ε Z ∞ a+ε
1
= e−pt dt + e−pt (0)dt
a ε a+ε
1  e−pt a+ε
=
ε −p a
1
L [ fε (t − a)] = − (e−p(a+ε) − e−ap )
εp

Transform Techniques B.Sc.(Mathematics)-IDE(UNOM)-II Year-IV Sem


4.1. Laplace Transform of Some Special Functions 85

e−ap (1 − e−εp )
L [ fε (t − a)] = .
εp

L [δ(t − a)] = lim L [ fε(t−a) ]


ε→0
e−ap (1 − e−εp )  0 
= lim
ε→0 εp 0
−εp
p
= e−ap lim
ε→0 p

= e−ap .1

Hence L [δ(t − a)] = e−ap .

2. Error Function. The error function denoted by erf(t) is defined by


Z t
2 2
erf(t) = √ e−u du.
π 0

Result. Find L [erf(t)] and hence prove that

√ 3p + 8
L [terf(2 t)] = .
+ 4) 2
3
p2 (p

Solution. By definition of error function, we have



Z t
2 2
erf(t) = √ e−u du
π 0

t
u4 u6
Z
2 i
= √ [1 − u2 + − + · · · du
π 0 2! 3!
2 h u 3
u 5
u7 i √t
erf(t) = √ u − + − + ···
π 3 5.2! 7.3! 0
3 5 7
2 h 1 t2 t2 t2 i
= √ t2 − + − + ···
π 3 5.2! 7.3!

B.Sc.(Mathematics)-IDE(UNOM)-II Year-IV Sem Transform Techniques


86 4.1. Laplace Transform of Some Special Functions

2 h 1 1 3 1 5 1 7
∴ L [erf(t)] = √ L [t 2 ] − L [t 2 ] + L [t 2 ] − L [t 2 ] + · · ·
π 3 5.2! 7.3!
2 h Γ( 2 ) 1 Γ( 2 ) 1 Γ( 2 ) 1 Γ( 2 )
3 5 7 9 i Γ(n + 1)
= √ − + − + · · · since L [tn ] =
π p2 3
3 p25
5.2! p 27
7.3! p 29
pn+1
√ √ √ √
2 h π 1 π 1 1 π 1 3 1 π 1 3 5 1 i
L [erf(t)] = √ . 3 − . . 5 + . . . 7 − . . . 9 + · · · , Γ(n + 1) = nΓ(n)
π 2 p2 2 2 p2 2 2 4 p2 2 2 4 6 p2

√ h
2 π 1 1 1 3 1 1 3 5 1 i
L [erf(t)] = √ . 3 1 − . + . 2 − . . 3 + · · ·
π 2p 2 2 p 2 4p 2 4 6p
1
1h 1 i− 2
= 3 1+
p2 p
1
∴ L [erf(t)] = p .
p p+1
 p
Using change of scale of property, L [F(at)] = 1a f a
, where L [F(t)] = f (p). We have

√ √
L [erf(2 t)] = L [erf( 4t)]
1 1
= . q
4 1 p +1
4 4
√ 2
L [erf(2 t)] =
p p+4
p
√ d h 2 i
L [t erf( 4t)] = −
dp p p + 4
p
h 1  1 i
= + p + 4.1
p
−2. − 2 p. p
p (p + 4) 2 p + 4
2  p + 2(p + 4) 
=
p2 (p + 4) 2 p + 4
p
√ 3p + 8
L [t erf( 4t)] = .
p2 (p + 4) 2
3

Transform Techniques B.Sc.(Mathematics)-IDE(UNOM)-II Year-IV Sem


4.1. Laplace Transform of Some Special Functions 87

3. Bessel Function.
Bessel function of order n is defined by

tn h t2 t4 i
Jn (t) = 1 − + − · · ·
2n Γ(n + 1) 2.(2n + 2) 2.4(2n + 2)(2n + 4)
∞ r
X (−1)  t n+2r
Jn (t) = .
r=0
r!Γ(n + r + 1) 2

Laplace Transform of Bessel Function.


Result.1 Prove that
(i) L [J0 (t)] = √ 1 ; (ii) L [J0 (at)] = √ 12 ;
1+p2 p +a2

(iii) L [tJ0 (at)] = 2 p 2 3 ; (iv) L [e−at J0 (at)] = √ 1


;
(p +a ) 2 p2 ++2ap+a2
R∞
(v) 0 J0 (t)dt = 1.


X (−1)r  1 n+2r
1. Jn (t) =
r=0
r!Γ(n + r + 1) 2

X (−1)r  t 2r
∴ J0 (t) =
r=0
(r!)2 2
t2 t4 t6
=1− + − + ··· .
22 22 .42 22 .42 .62
1 1 1
∴ L [J0 (t)] = L [1] − 2 L [t2 ] + 2 2 L [t4 ] − 2 2 2 L [t6 ] + · · · .
2 2 .4 2 .4 .6
1 1 2! 1 4! 1 6!
= − 2. 3 + 2 2. 5 − 2 2 2. 7 + ··· .
p 2 p 2 .4 p 2 .4 .6 p
1 h 1 1 1.3 1 2 1.3.5  1 3
  i
= 1− . 2 + − + · · ·
p 2 p 2.4 p2 2.4.6 p2
1h 1 i− 12
= 1+ 2
p p
1
= q
2
p 1+p p2
1
L [J0 (t)] = p .
1 + p2

B.Sc.(Mathematics)-IDE(UNOM)-II Year-IV Sem Transform Techniques


88 4.1. Laplace Transform of Some Special Functions

2. Using change of scale of property, we have

1  p
L [F(at)] = f
a a
1 1
L [J0 (at)] = q
a p2
1+ a2
1
L [J0 (at)] = p .
a2 + p2

d
3. L [tJ0 (t)] = − L [J0 (at)]
dp
d h 1 i
=−
d p a2 + p2
p
h 1 1 i
L [tJ0 (t)] = − − 1 2 . .2p
a + p2 2 a2 + p2
p

p
L [tJ0 (t)] = .
(p2 + a2 ) 2
3

4. L [e−at J0 (at)] = L [J0 (at)] p→p+a


1
= p
(p + a)2 + a2
1
L [e−at J0 (at)] = p .
p2 + 2ap + 2a2
Z ∞ Z ∞
5. J0 (t)dt = e−pt J0 (t)dt
0 0 p=0

= L [J0 (t)] p=0


h 1 i
= p = 1.
1 + p2 p=0

Result 2. Prove that (i) L [J1 (t)] = 1 − √ p2 ; (ii) L [tJ1 (t)] = 1


3 where J1 (t) is a Bessel
p +1 (p2 +1) 2
function of order one.

Solution. (i) From the theorem on Laplace transform of derivatives, we have

L [F 0 (t)] = pL [F(t)] − F(0).

Transform Techniques B.Sc.(Mathematics)-IDE(UNOM)-II Year-IV Sem


4.2. Worked out Examples 89

We know that J00 (t) = −J1 (t).

L [J1 (t)] = L [−J00 (t)]

= −L [J00 (t)]

= −[pL [J0 (t)] − J0 (0)]


h 1 i
= − pp − 1 , since J0 (0) = 1
p2 + 1
p
L [J1 (t)] = 1 − p .
p +1
2

(ii)

d
L [tJ1 (t)] = − L [J1 (t)]
dp

d h p i
L [tJ1 (t)] = − 1− p
dp p2 + 1
p2 + 1 − p. √1 2 2p
p
2 p +1
=
p2 +1
1
L [tJ1 (t)] = .
(p2 + 1) 2
3

4.2 Worked out Examples

Example 4.2.1. Evaluate L [t2 u(t − 2)].

Solution. The unit step function u(t − 2) is defined by



0, if t < 2




u(t − 2) = 


1, if t > 2


B.Sc.(Mathematics)-IDE(UNOM)-II Year-IV Sem Transform Techniques


90 4.2. Worked out Examples
Z ∞
L [u(t − 2)] = e−pt u(t − 2)dt
0
Z 2 Z ∞
= u(t − 2)dt +
−pt
e e−pt u(t − 2)dt
0 2
Z 2 Z ∞
= e−pt .0 dt + e−pt .1 dt
0 2
 e−pt ∞
=
−p 2
1
= − (0 − e−2p )
p
−2p
e
L [u(t − 2)] =
p
d2
Hence L [t2 (u − 2)] = (−1)2 2 L [u(t − 2)]
dp
2 h −2p i
d e
=
d p2 p
d h p(−2e−2p ) − e−2p .1 i
=
dp p2
d h e−2p (2p + 1) i
L [t2 (u − 2)] = −
dp p2

h p2 [e−2p (2) − 2e−2p (2p + 1)] − e−2p (2p + 1)(2p) i


L [t (u − 2)] = −
2
p4
2e−2p 2
= − 4
[p − p2 (2p + 1) − p(2p + 1)]
p
2e−2p
= − 4 [p2 − 2p3 − p2 − 2p2 − p]
p
p2e−2p
= [2p2 + 2p + 1]
p4
2e−2p
L [t2 (u − 2)] = [2p2 + 2p + 1].
p3

Example 4.2.2. Find L [et−3 u(t − 3)].

Transform Techniques B.Sc.(Mathematics)-IDE(UNOM)-II Year-IV Sem


4.3. Laplace Transform of Periodic Functions 91
e−3p
Solution. Since L [u(t − 3)] = p
.

L [et−3 u(t − 3)] = L [e−3 et u(t − 3)]

= e−3 L [u(t − 3)] p→p−1


h −3p i
−3 e
= e
p p→p−1
h e−3(p−1) i
= e−3
p−1
−3p 3
e e
= e−3
p−1
−3p
e
L [et−3 u(t − 3)] = .
p−1

4.3 Laplace Transform of Periodic Functions

Definition 4.3.1. Periodic Function. A function F(t) is said to be periodic if and only if
F(t + T ) = F(t) for some value of T and for every value of t. The smallest positive value of
T for which this equation is true for all every value of t is called the period of the function.

Alternatively, a function F(t) is said to be periodic function of period T > 0, if

F(t) = F(t + T ) = F(t + 2T ) = .....F(t + nT ).

Example. Let F(t) = sin t. Then

F(t + 2π) = F(t + 4π) = ... = F(t).

Therefore sin t is a periodic function with period 2π.


Similarly, cos t is a periodic function with period 2π and tan t is a periodic function with period
π.
Note. If T is the period of F(t) then the period of F(at + b) is T
a
.

B.Sc.(Mathematics)-IDE(UNOM)-II Year-IV Sem Transform Techniques


92 4.3. Laplace Transform of Periodic Functions

Theorem 4.3.1. If F(t) is a periodic function with period T, then


Z T
1
L [F(t)] = e−pt F(t)dt.
1 − e−pT 0

Proof. By definition, we have


Z ∞ Z T Z 2T Z 3T
L [F(t)] = e −pt
F(t)dt = −pt
e F(t)dt + −pt
e F(t)dt + e−pt F(t)dt + · · ·
0 0 T 2T

Put t = u + T in the second integral, we have

dt = du, when t = T ⇒ u = 0 and when t = 2T ⇒ u = T.

Put t = u + 2T in the third integral, we have

dt = du, when t = 2T ⇒ u = 0 and when t = 3T ⇒ u = T.

Put t = u + 2T in the fourth integral and so on.Then


Z T Z T Z T
L [F(t)] = e −pt
F(t)du + e −p(u+T )
F(u + T )du + e−p(u+2T ) F(u + 2T )du + · · ·
0 0 0
Z T Z T Z T
= e −pt
F(t)dt + e −pT
e −pu
F(u)du + e −2pT
e−ut F(u)du + · · ·
0 0 0

since F(u) = F(u + T ) = F(u + 2T ) = · · ·


Z T Z T Z T
L [F(t)] = −pt
e F(t)dt + e −pT −pt
e F(t)dt + e −2pT
e−pt F(t)dt + · · · ,
0 0 0

by changing the dummy variable u by t.


Z T Z T
L [F(t)] = (1 + e−pT
+e −2pT
+ · · · ∞) e F(t)dt = (1 − e )
−pt −pT −1
e−pt F(t)dt
0 0
Z T
1
L [F(t)] = −pT
e−pt F(t)dt.
1−e 0

Transform Techniques B.Sc.(Mathematics)-IDE(UNOM)-II Year-IV Sem


4.4. Worked out Problems 93

4.4 Worked out Problems

Example 4.4.1. Find the Laplace transform of the square - wave function of period 2a defined as

k, when 0 < t < a




F(t) = 


−k, when a < t < 2a


Solution. Given that F(t) is a periodic function with period T = 2a.


Z 2a
1
L [F(t)] = −2ap
e−pt F(t)dt
1−e 0
1 hZ a Z a i
= e−pt
(k)dt + e −pt
(−k)dt
1 − e−2ap 0 0
k h e−pt a  e−pt 2a i
= −
1 − e−2ap −p 0 −p a
k h 1 −pa 1 −2ap i
= − (e − 1) + (e − e−ap
)
1 − e−2ap p p
k
= −2ap
[1 − e−pa + e−2ap − e−ap ]
p(1 − e )
k
= (1 − e−ap )2
p(1 − e−2ap )
k
= (1 − e−ap )2
p(1 + e )(1 − e−ap )
−ap

k(1 − e−ap )
=
p(1 + e−ap )
−ap ap ap
ke 2 (e 2 − e− 2 )
= ap ap ap
pe− 2 (e 2 + e− 2 )
k  ap 
L [F(t)] = tanh .
p 2

B.Sc.(Mathematics)-IDE(UNOM)-II Year-IV Sem Transform Techniques


94 4.4. Worked out Problems

Example 4.4.2. Find L [F(t)], where F(t) is a periodic function of period 2π and it is given by

sin t, 0 < t < π




F(t) = 


0, π < t < 2π


Solution. Since F(t) is a periodic function with period 2π.


Z 2π
1
L [F(t)] = −2πp
e−pt F(t)dt
1−e 0
Z π Z 2π
1 h i
= e−pt
F(t)dt + e −pt
F(t)dt
1 − e−2πp 0 π
Z π Z 2π
1 h i
= e−pt
sin tdt + e −pt
(0)dt
1 − e−2πp 0 π
Z π
1
= e−pt sin tdt
1 − e−2πp 0
1 h e−pt iπ Z
eax
= (−p sin t − cos t) , since eax
sin bx = (a sin bx − b cos bx)
1 − e−2πp p2 + 1 0 a2 + b2
1 h e−πp 1 i
= (1) − (−1)
1 − e−2πp p2 + 1 p2 + 1
1+e −πp
=
(p2 + 1)(1 − e−2πp )
1 + e−πp
=
(p2 + 1)(1 + e−πp )(1 − eπp )
1
L [F(t)] = .
(p + 1)(1 − e−πp )
2

Example 4.4.3. If F(t) = t2 , 0 < t < 2 and F(t + 2) = F(t), find L [F(t)].

Solution. Clearly F(t) is a periodic function with period T = 2.


Z T
1
L [F(t)] = −pT
e−pt F(t)dt
1−e 0
Z 2
1
= e−pt t2 dt
1 − e−2p 0
1 h 2  e−pt   e−pt   e−pt i2
= t − 2t + 2
1 − e−2p −p p2 −p3 0

Transform Techniques B.Sc.(Mathematics)-IDE(UNOM)-II Year-IV Sem


4.4. Worked out Problems 95

1 h e−2p e−2p e−2p 2 i


=
− 4 − 4 − 2 − (0 − 0 − )
1 − e−2p p p2 p3 p3
1 h 2 − e−2p (4p2 + 4p + 2) i
L [F(t)] =
1 − e−2p p3

Example 4.4.4. Find L [| sin t|].

Solution. Since | sin t| is a periodic function with period π.


Z π
1
L [F(t)] = e−pt F(t)dt
1 − e−πp 0

Z π
1
L [F(t)] = −πp
e−pt | sin t|dt
1−e
Z0 π
1
= −πp
e−pt sin tdt since | sin t| = sin t in 0 < t < π
1−e 0
1 h e−pt iπ
= (−p sin t − cos t)
1 − e−πp p2 + 1 0
−πp
1 h e 1 i
= (0 + 1) − (0 − 1)
1 − e−πp p2 + 1 p2 + 1
1 + e−πp
L [F(t)] = .
(1 + p2 )(1 − e−πp )

Example 4.4.5. Find L [F(t)], where F(t) is given by



t, 0 < t < b




F(t) =  ,


2b − t, b < t < 2b


2b being the period of F(t).

Solution. Since F(t) is a periodic function with period 2b.


Z 2b
1
L [F(t)] = e−pt F(t)dt
1 − e−2bp 0
hZ b Z 2b
1 i
= e−pt
tdt + e−pt
(2b − t)dt
1 − e−2bp 0 b

B.Sc.(Mathematics)-IDE(UNOM)-II Year-IV Sem Transform Techniques


96 4.4. Worked out Problems

1 h  e−pt   e−pt b  e−pt e−pt 2b i


= t − + (2b − t) − (−1)
1 − e−2bp −p p2 0 −p p2 b
1 h be−bp e−bp  1 e−2bp  e−bp e−bp i
= − − − 0 − + 0 + − − b + 2
1 − e−2bp p p2 p2 p2 p p
1 h 1 − 2e + e
bp −2bp i
= −2bp
1−e p2
1 (1 − e−bp )2
=
(1 + e−bp )(1 − e−bp ) p2
1 − e−bp
L [F(t)] =
p2 (1 + e−bp )
−bp bp bp
1 e 2 (e 2 − e− 2 )
= 2 bp bp
p e− 2 (e 2 + e− bp2 )
1  bp 
L [F(t)] = 2 tanh .
p 2

Example 4.4.6. Find the Laplace transform of the saw - toothed wave of period T, given

k
F(t) = t, when 0 < t < T.
T

Solution. Since F(t) is a periodic function with period T.


Z T
1
L [F(t)] = e−pt F(t)dt
1 − e−pT 0
Z T
1 k
= −pT
e−pt tdt
1−e 0 T
Z T
k
= te−pt dt
T (1 − e−pT ) 0
k h  e−pt   e−pt iT
= t −
T (1 − e−pT ) −p p2 0
k h e−pT e−T p  1 i
= −pT
− T − 2 − 0− 2
T (1 − e ) p p p
h 1 − pT e −pT −pT
k −e i
L [F(t)] =
T (1 − e−pT ) p2

Transform Techniques B.Sc.(Mathematics)-IDE(UNOM)-II Year-IV Sem


4.5. Exercises 97

Example 4.4.7. Find the Laplace transform of the rectified semi - wave function defined by

π
sin ωt, 0 < t <



ω

F(t) = 

0, ωπ < t < 2π






ω

Solution. Since F(t) is a periodic function with period T.

Z 2π
1 ω
L [F(t)] = e−pt F(t)dt
− 2πp
1−e ω 0
Z π
1 ω
= e−pt sin ωt dt
− 2πp
1−e ω 0
1 h e−pt i ωπ
= (−p sin ωt − ω cos ωt)
1 − e− ω p + ω
2πp 2 2 0
πp
1 h e− ω 1 i
= (0 + ω) − (0 − ω)
1 − e− ω p + ω p2 + ω2
2πp 2 2
πp
1 h ωe− ω + ω i
=
1 − e− ω p + ω
2πp 2 2
πp
ω(1 + e− ω )
L [F(t)] = πp πp
(1 + e− ω )(1 − e− ω )(p2 + ω2 )

ω
L [F(t)] = − πp
.
(1 − e ω )(p2 + ω2 )

4.5 Exercises

1. If 
3t, 0 < t < 2




F(t) = 


6, 2 < t < 4


find L [F(t)], where F(t) has period 4.

B.Sc.(Mathematics)-IDE(UNOM)-II Year-IV Sem Transform Techniques


98 4.5. Exercises

2. Find the Laplace transform of the function of period 2a defined as



k, 0 < t < a




F(t) = 


0, a < t < 2a


3. Find the Laplace transform of the square wave function



1, 0 < t < 2


 a

F(t) = 


−1, a2 < t < a


with period T = a.
4. Find the Laplace transform of the function of period T defined as F(t) = Tt , 0 < t < T.

Answers. h
k ap
 ap 
3
1. p2 (1−e −4p ) 1 − e
−2p
(1 + 2e2p ). 2. 2p
e2 sec h 2
 
e−pT
3. 1p tanh pa 4
. 4. 1
p2 T
− p(1−e−pT )
.

Transform Techniques B.Sc.(Mathematics)-IDE(UNOM)-II Year-IV Sem


UNIT-II

Lesson 5

THE INVERSE LAPLACE TRANSFORM


Learning Objectives
Upon completion of this lesson, students will be able to

• define the concept of inverse Laplace transform

• solve simple problems based on inverse Laplace transform.

5.1 Introduction

I
n the first Unit we have considered Laplace transform of some functions F(t). Let us now
consider the converse namely, given f (p) how to determine F(t) such that L [F(t)] =
f (p).

The inverse Laplace transform is useful in solving differential equations without finding the
general solution and arbitrary constants.

Definition 5.1.1. If f (p) is the Laplace transform of a function F(t), then F(t) is called the
inverse Laplace transform of f (p) and is denoted by L −1 [ f (p)] i.e,

F(t) = L −1 [ f (p)].

L −1 is called the inverse Laplace transform operator.

B.Sc.(Mathematics)-IDE(UNOM)-II Year-IV Sem 99 Transform Techniques


100 5.2. Worked out Problems
h i
Thus if L [eat ] = 1
p−a
then L −1 1
p−a
= eat
h p i
and if L [cos at] = p
p2 +a2
then L −1 p2 +a2
= cos at, etc.
We list some standard inverse Laplace transforms as follows:

Table of Inverse Laplace Transforms


S.No f (p) L −1 [ f (p)] = F(t)
1
1 p
1
1 tn
2 pn+1
,n is a positive integer n!
tn
3 1
pn+1
, n > −1 Γ(n+1)
1
4 p−a
eat
1
5 p+a
e−at
1 1
6 p2 +a2 a
sin at
p
7 p2 +a2
cos at
1 1
8 p2 −a2 a
sinh at
p
9 p2 −a2
cosh at

Note. Inverse Laplace transform of a given function f (p) can be obtained either by use of the
above standard results or by splitting the given function into its partial fractions and then applying
the above results.

General property of inverse Laplace transform


For each property on Laplace transform, there is a corresponding property of Inverse Laplace
transform which readily follows from the definition.

Linear Property. If f1 (p) and f2 (p) are Laplace transform of F1 (t) and F2 (t) respectively, then

L −1 [c1 f1 (p) + c2 f2 (p)] = c1 L −1 [ f1 (p)] + c2 L −1 [ f2 (p)],

where c1 and c2 are any constants.

5.2 Worked out Problems

Example 5.2.1. Find L −1 [ p23p


−25
]

Transform Techniques B.Sc.(Mathematics)-IDE(UNOM)-II Year-IV Sem


5.2. Worked out Problems 101

Solution.
h 3p i h p i
L −1 = 3L −1
= 3 cosh 5t.
p2 − 25 p2 − 25
Example 5.2.2. Find the inverse Laplace transform of the following functions:
(i) 2p−5
p2 −4
; (ii) 2p+1
p(p+1)
; (iii) 3p−8
4p2 +25
.
h 2p − 5 i h 2p 5 i
Solution. (i) L −1 = L −1− (ii)
p2 − 4 p2 − 4 p2 − 4
h p i h 1 i
= 2L −1 2 − 5L −1 2
p −4 p −4
h 2p − 5 i 1
L −1 2 = 2 cosh 2t − 5. sinh 2t.
p −4 2
h 2p + 1 i h p + (p + 1) i
L −1 = L −1 (iii)
p(p + 1) p(p + 1)
h 1 1i
= L −1 +
p+1 p
h 1 i h1i
= L −1 + L −1
p+1 p
h 2p + 1 i
L −1 = e−t + 1.
p(p + 1)
h 3p − 8 i h 3p 8 i
L −1 = L −1

4p2 + 25 4p2 + 25 4p2 + 25
3 h p i 8 h 1 i
= L −1 2 − L −1
4 p + ( 25 )2 4 p2 + ( 25 )2
3  5t  2  5t 
= cos −2 sin
4 2 5 2
h 3p − 8 i 3  5t  4  5t 
L −1 = cos − sin .
4p2 + 25 4 2 5 2
h i
Example 5.2.3. Find L −1 3p−2 5 − p+2 7
.
p2

Solution.
h 3p − 2 7 i h pi h1i 7 h 1 i
L −1 − = 3L −1 5 − 2L −1 5 − L −1
p2
5
p+2 p2 p2 3 p + 23
1 3
t2 t2 7 2
= 3 3 − 2 5 − e− 3 (t) .
Γ( 2 ) Γ( 2 ) 3

h 2p+1 i
Example 5.2.4. Find L −1 p2 −4
.

B.Sc.(Mathematics)-IDE(UNOM)-II Year-IV Sem Transform Techniques


102 5.2. Worked out Problems

Solution.
h 2p + 1 i p i h h 1 i
L −1 = 2L −1 + L −1
p2 − 4 p2 − 4 p2 − 4
1
= 2 cosh(2t) + sinh(2t).
2
h p2 −3p+4 i h 3(p2 −2)2 i
Example 5.2.5. Find (i) L −1 p3
. (ii) L −1 2p5
.

Solution.
h p2 − 3p + 4 i h p2 p 4i
(i) L −1 = L −1 + −3
p3 p3 p3 p3
h1 1 1i
= L −1 − 3. 2 + 4 3
p p p
2
t
= 1 − 3t + 4.
2!
= 1 − 3t + 2t2 .

h 3(p2 − 2)2 i 3 −1 h (p2 − 2)2 i


(ii) L −1 = L
2p5 2 p5
3 −1 h (p4 − 4p2 + 4) i
= L
2 p5
3 −1 h 1 4 4i
= L − 3+ 5
2 p p p
2 4i
3 h t t
= 1−4 +4
2 2! 4!
4i
3h t
= 1 − 2t2 +
2 6
3 h 6 − 12t2 + t4 i
=
2 6
h 3(p2 − 2)2 i 1
L −1
= (6 − 12t2 + t4 ).
2p5 2
h i h p2 +9p−9 i
Example 5.2.6. Find (i) L −1 2p−5
4p2 +25
+ 4p−18
9−p2
. (ii) L −1 p3 −9p
.

Solution.
h 2p − 5 4p − 18 i −1 2p − 5 −1 4p − 18
h i h i
(i) L −1 + = L + L
4p2 + 25 9 − p2 4p2 + 25 9 − p2

Transform Techniques B.Sc.(Mathematics)-IDE(UNOM)-II Year-IV Sem


5.2. Worked out Problems 103
h 2p − 5 4p − 18 i h 2p i h 5 i h 4p i h 18 i
L −1 + = L −1
− L −1
+ L −1
− L −1
4p2 + 25 9 − p2 4p2 + 25 4p2 + 25 9 − p2 9 − p2
2 h p i 5 h 1 i
= L −1 2 25 − L −1 2 25
4 p + 4 4 p + 4
h p i h 1 i
− 4L −1 2 + 18L −1 2
p −9 p −9
1  5t  5 2  5t  1
= cos − . sin − 4 cosh 3t + 18. sinh 3t
2 2 4 5 2 3
h 2p − 5 4p − 18 i 1  5t  1  5t 
L −1 + = cos − sin − 4 cosh 3t + 6 sinh 3t.
4p2 + 25 9 − p2 2 2 2 2

h p2 + 9p − 9 i h p2 − 9 + 9p i
(ii) L −1 = L −1
p3 − 9p p(p2 − 9)
h p2 − 9 i h 9p i
= L −1 + L −1
p(p2 − 9) p(p2 − 9)
h1i h 1 i
= L −1 + 9L −1 2
p p −9
1
= 1 + 9. sinh 3t
3
h p2 + 9p − 9 i
L −1 = 1 + 3 sinh 3t.
p3 − 9p

h  √ p−1 2 i
Example 5.2.7. Find L −1 5
p2
+ p
− 7
3p+2
.

Solution.
 √ p − 1 2 h p − 2 √p + 1
−1 5 7 i −1 1
h h i h 1 i
L + − = 5L + L −1
− 7L −1
p2 p 3p + 2 p2 p2 3p + 2
h1i h1i h1i 7 h 1 i
= 5t + L −1 − 2L −1 3 + L −1 2 − L −1
p p2 p 3 p + 23
1
t2 7 2
= 5t + 1 − 2 3 + t − e− 3 t
Γ( 2 ) 3
h 5  √ p − 1 2 7 i 2t 2
1
7 − 23 t
L −1
+ − = 6t + 1 − − e .
p2 p 3p + 2 Γ( 23 ) 3

√ i
6−30 p
h
Example 5.2.8. Find L −1 3
p2 −3
+ 3p+2
p3
− 3p−27
p2 +9
+ p4 .

B.Sc.(Mathematics)-IDE(UNOM)-II Year-IV Sem Transform Techniques


104 5.2. Worked out Problems

Solution. √
h 3 3p + 2 3p − 27 6 − 30 p i h 3 i
−1 3p + 2
h i
−1 3p − 27
h i
L −1
+ − + = L −1
+ L − L
p2 − 3 p3 p2 + 9 p4 p2 − 3 p3 p2 + 9

h 6 − 30 p i
+ L −1
p4
h 1 i h 3p i h2i
= 3L −1 2 + L −1 3 + L −1 3
p −3 p p
h p i h 1 i h1i
− 3L −1 2 + 27L −1 2 + 6L −1 4
p +9 p +9 p
h1i
− 30L −1 7
p2
1 √ t2
= 3. √ sinh( 3t) + 3t + 2. − 3 cos 3t
3 2!
5
1 t3 t2
+ 27. sin 3t + 6. − 30 7
3 3! Γ( 2 )

h 3 3p + 2 3p − 27 6 − 30 p i √ √
L −1 2 + − + = 3 sinh 3t + 3t + t2 − 3 cos 3t + 9 sin 3t
p −3 p 3 p +9
2 p 4
5
t2
+ t − 30 7 .
3
Γ( 2 )

h i
Example 5.2.9. Find L −1 6
2p−3
− 3+4p
9p2 −16
+ 8−6p
16p2 +9
.
h 6 3 + 4p 8 − 6p i
Solution. L −1 − 2 +
2p − 3 9p − 16 16p2 + 9

h 3 + 4p i
6 i −1 8 − 6p
h h i
= L −1 − L −1 + L
2p − 3 9p2 − 16 16p2 + 9
6 h 1 i h 3 i h p i h 8 i h p i
= L −1 − L −1
− 4L −1
+ L −1
− 6L −1
2 p − 32 9p2 − 16 9p2 − 16 16p2 + 9 16p2 + 9
3 3 h 1 i 4 h p i 8 h 1 i 6 h p i
= 3e 2 t − L −1 2 16 − L −1 2 16 + L −1 2 9 − L −1 2 9
9 p − 9 9 p − 9 16 p + 16 16 p + 16
3 1 3 4  4 4  1 4 3  3 3 
= 3e 2 t − . sinh t − cosh t + . sin t − cos t
3 4 3 9 3 2 3 4 8 4

h 6 3 + 4p 8 − 6p i 3 1 4  4 4  2 3  3 3 
L −1 − 2 + = 3e 2t − sinh t − cosh t + sin t − cos t.
2p − 3 9p − 16 16p2 + 9 4 3 9 3 3 4 8 4

Transform Techniques B.Sc.(Mathematics)-IDE(UNOM)-II Year-IV Sem


5.2. Worked out Problems 105
h  i
t2 t4 t6
Example 5.2.10. Show that L −1 1p . cos 1p = 1 − (2!)2
+ (4!)2
− (6!)2
+ ....

Solution.
p2 p4 p6
S incecos p = 1 − + − + .....
2! 4! 6!
1 1 1h 1  1 2 1  1 4 1  1 6 i
cos = 1− + − + ...
p p p 2! p 4! p 6! p
1 1 1 1 1 1 1
= − 3
+ 5
− + ....
p 2! p 4! p 6! p7
h1  1 i h1i 1 h1i 1 h1i 1 h1i
L −1 cos = L −1 − L −1 3 + L −1 5 − L −1 7
p p p 2! p 4! p 6! p
2 4 6
1 t  1 t  1 t 
= 1− + − + ....
2! 2! 4! 4! 6! 6!
h1  1 i t2 t4 t6
L −1 cos = 1− + − + .....
p p (2!)2 (4!)2 (6!)2
h  i
t3 t5 t7
Example 5.2.11. Show that L −1 1p . sin 1p = t − (3!)2
+ (5!)2
− (7!)2
+ ....

Solution.
p3 p5 p7
S incesin p = p − + − + .....
3! 5! 7!
1 1 1h 1  1 3 1  1 5 1  1 7 i
sin = p− + − + ...
p p p 3! p 5! p 7! p
1 1 1 1 1 1 1
= 2− + − + ....
p 3! p4 5! p6 7! p8
h1  1 i h1i 1 h1i 1 h1i 1 h1i
L −1 sin = L −1 2 − L −1 4 + L −1 6 − L −1 8
p p p 3! p 5! p 7! p
3 5 7
1 t  1 t  1 t 
= t− + − + ....
3! 3! 5! 5! 7! 7!
h1  1 i t3 t5 t7
L −1 sin = t− + − + .....
p p (3!)2 (5!)2 (7!)2

B.Sc.(Mathematics)-IDE(UNOM)-II Year-IV Sem Transform Techniques


106
UNIT-II

Lesson 6

INVERSE LAPLACE TRANSFORM BY


PARTIAL FRACTIONS
Learning Objectives
Upon completion of this lesson, students will be able to

• find inverse Laplace transform using partial fraction method

6.1 Introduction

I
f f (p) is given in the form g(p)
h(p)
, where g and h are polynomial in p, then F(t) can be
obtained by resolving f (p) into partial fractions and manipulating term by term.

6.2 Worked out Problems


h i
Example 6.2.1. Find L −1 p
p2 −a2
.

Solution. Let
p p A B
= = + (6.2.1)
p2 −a 2 (p − a)(p + a) p − a p + a
Then
p = A(p + a) + B(p − a) (6.2.2)

B.Sc.(Mathematics)-IDE(UNOM)-II Year-IV Sem 107 Transform Techniques


108 6.2. Worked out Problems

To find A, put p = a in (17.2.2), we get A = 12 .


To find B, put p = −a in (17.2.2), we get B = 21 .
Substituting these values of A and B in (20.2.1), we get

p 1 1
= +
−ap2
2 2(p − a) 2(p + a)
h p i h 1 1 i
L −1 2 = L −1
+
p − a2 2(p − a) 2(p + a)
1 −1 h 1 i 1 −1 h 1 i
= L + L
2 p−a 2 p+a
1 at 1 −at
= e + e
2 2
1 at
= (e + e−at )
2
p
= cosh at.
p2 − a2

Example 6.2.2. Find the inverse Laplace transform of 4


(p+1)(p+2)
.

Solution.
h 4 i h 1 i
L −1 = 4L −1
(p + 1)(p + 2) (p + 1)(p + 2)
h 1 1 i
= 4L −1 − (Resolving into partial f ractions)
p+1 p+2
h 1 i h 1 i
= 4L −1 − 4L −1
p+1 p+2
h 4 i
L −1 = 4e−t − 4e−2t
(p + 1)(p + 2)
h i
Example 6.2.3. Find L −1 3p+1
p2 −2p−3
.

Solution. Let
3p + 1 3p + 1 A B
= = + (6.2.3)
p2 − 2p − 3 (p + 1)(p − 3) p + 1 p − 3
Then
3p + 1 = A(p − 3) + B(p + 1)

Put p = −1, we have A = 1


2

Transform Techniques B.Sc.(Mathematics)-IDE(UNOM)-II Year-IV Sem


6.2. Worked out Problems 109

Put p = 3, we have B = 52 . Substituting these values of A and B in (20.2.3), we get

3p + 1 1 5
= +
p2 − 2p − 3 2(p + 1) 2(p − 3)
h 3p + 1 i 1 h 1 i 5 h 1 i
L −1 2 = L −1 + L −1
p − 2p − 3 2 p+1 2 p−3
h 3p + 1 i 1 5
L −1 2 = e−t + e3t .
p − 2p − 3 2 2
h i
Example 6.2.4. Find L −1 p−2
p2 +5p+6
.

Solution. Let
p−2 p−2 A B
= = + (6.2.4)
p2 + 5p + 6 (p + 2)(p + 3) p + 2 p + 3
Then
p − 2 = A(p + 3) + B(p + 2)

Put p = −2, we get A = −4


Put p = −3, we get B = 5. Substituting these values of A and B in (20.2.4), we get

p−2 4 5
= − +
+ 5p + 6p2 p+2 p+3
h p−2 i h 4 5 i
L −1 2 = L −1 − +
p + 5p + 6 p+2 p+3
h 1 i h 1 i
= −4L −1 + 5L −1
p+2 p+3
h p−2 i
L −1 2 = −4e−2t + 5e−3t .
p + 5p + 6
h i
Example 6.2.5. Find L −1 1
p(p+1)(p+2)
.

Solution. Let
1 A B C
= + + (6.2.5)
p(p + 1)(p + 2) p p + 1 p + 2
Then
1 = A(p + 1)(p + 2) + Bp(p + 2) + C p(p + 1)

B.Sc.(Mathematics)-IDE(UNOM)-II Year-IV Sem Transform Techniques


110 6.2. Worked out Problems

Put p = 0, we get 1 = 2A ⇒ A = 1
2

Put p = −1, we get B = −1.


Put p = −2, we get 1 = 2C ⇒ C = 21 . Substituting these values of A and B in (20.2.6), we get

1 1 1 1
= − +
p(p + 1)(p + 2) 2p p + 1 2(p + 2)
h 1 i 1 h1i h 1 i 1 h 1 i
L −1 = L −1 − L −1 + L −1
p(p + 1)(p + 2) 2 p p+1 2 p+2
1 1
= − e−t + e−2t
2 2
1
= (1 − 2e + e−2t )
−t
2
h 1 i 1
L −1
= (1 − e−t )2 .
p(p + 1)(p + 2) 2

p2
h i
Example 6.2.6. Evaluate L −1 (p+1)(p+2)(p+3)

Solution. Let
p2 A B C
= + + (6.2.6)
(p + 1)(p + 2)(p + 3) p + 1 p + 2 p + 3
Then
p2 = A(p + 2)(p + 3) + B(p + 1)(p + 3) + C(p + 2)(p + 1)

Put p = −1, we get 1 = A(−1 + 2)(−1 + 3) ⇒ A = 1


2

Put p = −2, we get 4 = B(−2 + 1)(−2 + 3) ⇒ B = −4


Put p = −3, we get 9 = C(−3 + 1)(−3 + 2) ⇒ C = 92 . Substituting these values of A, B and C
in (20.2.8), we get

p2 1 4 9
= − +
(p + 1)(p + 2)(p + 3) 2(p + 1) p + 2 2(p + 3)
2
h p i 1 h 1 i h 1 i 9 h 1 i
L −1 = L −1 − 4L −1 + L −1
(p + 1)(p + 2)(p + 3) 2 p+1 p+2 2 p+3
2
h p i 1 9
L −1 = e−t − 4e−2t + e−3t
(p + 1)(p + 2)(p + 3) 2 2

Transform Techniques B.Sc.(Mathematics)-IDE(UNOM)-II Year-IV Sem


6.2. Worked out Problems 111
h i
Example 6.2.7. Evaluate L −1 1
(p+1)(p2 +1)

Solution. Let
1 A Bp + C
= + 2 (6.2.7)
(p + 1)(p + 1) p + 1
2 p +1
Then
1 = A(p2 + 1) + (Bp + C)(p + 1) = (A + B)p2 + (B + C)p + (A + C)

Comparing the coefficients of p2 , p, and constant terms on both sides, we get


A + B = 0, B + C = 0 and A + C = 0.
Solving these, we get A = 12 , B = − 21 and C = 12 . Substituting these values of A, B and C in
(20.2.9), we get

1 1 p 1
= − +
(p + 1)(p + 1)
2 2(p + 1) 2(p + 1) 2(p + 1)
2 2
h 1 i 1 −1 h 1 i 1 −1 h p i 1 −1 h 1 i
L −1 = L − L + L
(p + 1)(p2 + 1) 2 p+1 2 p2 + 1 2 p2 + 1
h 1 i 1 −t 1 1
L −1 = e − cos t + sin t.
(p + 1)(p2 + 1) 2 2 2

p2 +p−2
h i
Example 6.2.8. Find the inverse Laplace transformation of p(p+3)(p−2)

Solution. Let
p2 + p − 2 A B C
= + + (6.2.8)
p(p + 3)(p − 2) p p + 3 p − 2
Then
p2 + p − 2 = A(p + 3)(p − 2) + Bp(p − 2) + C p(p + 3)

Put p = 0, we get −2 = −6A ⇒ A = 31 .


Put p = −3, we get 4 = 15B ⇒ B = 4
15

Put p = 2, we get 4 = 10C ⇒ C = 2


5

B.Sc.(Mathematics)-IDE(UNOM)-II Year-IV Sem Transform Techniques


112 6.2. Worked out Problems

Substituting these values of A, B and C in (20.2.10), we get

p2 + p − 2 1 4 2
= + +
p(p + 3)(p − 2) 3p 15(p + 3) 5(p − 2)
h p + p−2 i
2
1 h1i 4 h 1 i 2 h 1 i
L −1 = L −1 + L −1 + L −1
p(p + 3)(p − 2) 3 p 15 p+3 5 p−2
h p + p−2 i
2
1 4 2
L −1 = + e−3t + e2t .
p(p + 3)(p − 2) 3 15 5

p2 +2p−4
h i
Example 6.2.9. Evaluate L −1 (p2 +9)(p−5)

Solution. Let
p2 + 2p − 4 A Bp + C
= + (6.2.9)
(p2 + 9)(p − 5) p − 1 p2 + 9
Then
p2 + 2p − 4 = A(p2 + 9) + (Bp + C)(p − 5)

Put p = 5, we have 31 = 34A ⇒ A = 31


34

Comparing the coefficients of p2 , and p, on both sides, we get


A + B = 1, −5B + C = 2
Therefore B = 3
34
and C = 83
34

Substituting these values of A, B and C in (6.2.9), we get

p2 + 2p − 4 31 3p + 83
= +
(p2 + 9)(p − 5) 34(p − 5) 34(p2 + 9)
h p2 + 2p − 4 i 31 3 h p i 83 h 1 i
L −1 2 = + L −1 2 + L −1 2
(p + 9)(p − 5) 34(p − 5) 34 p +9 34 p +9
h p2 + 2p − 4 i 31 5t 3 83 sin 3t
L −1 2 = e + cos 3t +
(p + 9)(p − 5) 34 34 34 3
h p2 + 2p − 4 i 1 h 5t 83 i
L −1 2 = 31e + 3 cos 3t + sin 3t .
(p + 9)(p − 5) 34 3

p2
h i
Example 6.2.10. Evaluate L −1 (p2 +4)(p2 +25)

Solution. Let
p2 Ap + B C p + D
= + 2 (6.2.10)
(p2 + 4)(p2 + 25) p2 + 4 p + 25

Transform Techniques B.Sc.(Mathematics)-IDE(UNOM)-II Year-IV Sem


6.2. Worked out Problems 113

Then
p2 = (Ap + B)(p2 + 25) + (C p + D)(p2 + 4)

Comparing the coefficients of p3 , and p2 , p and the constant term on both sides, we get

A + C = 0, B + D = 1, 25A + 4C = 0 and 25B + D = 0

Solving these equations, we get A = 0, C = 0, B = − 21


4
and D = 25
21
.

Substituting these values of A, B, C and D in (6.2.10), we get

p2 −4 25
= +
(p2 + 4)(p2 + 25) 21(p2 + 4) 21(p2 + 25)
2
h p i 4 −1 h 1 i 25 −1 h 1 i
L −1 2 = − L + L
(p + 4)(p2 + 25) 21 p2 + 4 21 p2 + 25
4 sin 2t 25 sin 5t
= − +
21 2 21 5
2 5
= − sin 2t + sin 5t
21 21
h p2 i 1h i
L −1 2 = 5 sin 5 − 2 sin 2t .
(p + 4)(p2 + 25) 21
h i
Example 6.2.11. Evaluate L −1 p
(p2 +1)(p2 +9)(p2 +25)

h i
Solution. We have 1
(p2 +1)(p2 +9)
= 1 1
8 p2 +1
− 1
p2 +9
, by resolving into partial fractions.

1 1h 1 1 i
= −
(p2 + 1)(p2 + 9)(p2 + 25) 8 (p2 + 1)(p2 + 25) (p2 + 9)(p2 + 25)
1h 1  1 1  1 1 1 i
= − − −
8 24 p2 + 1 p2 + 25 16 p2 + 9 p2 + 25
1h 1 1 1 1 1 1
= . 2 − . 2 + . 2
8 24 p + 1 16 p + 9 48 p + 25
1 h 16 24 8 i
= − +
8 × 24 × 16 p2 + 1 p2 + 9 p2 + 25
p 1 h 16p 24p 8p i
= − +
(p2 + 1)(p2 + 9)(p2 + 25) 3092 p2 + 1 p2 + 9 p2 + 25
h p i 1 h  p   p   p i
L −1 2 = 16.L −1
− 24.L −1
+ 8.L −1
(p + 1)(p2 + 9)(p2 + 25) 3092 p2 + 1 p2 + 9 p2 + 25
1  
= 16 cos t − 24 cos 3t + 8 cos 5t .
3092

B.Sc.(Mathematics)-IDE(UNOM)-II Year-IV Sem Transform Techniques


114 6.3. Exercises
h i
Example 6.2.12. Find L −1 2p+3
p3 −6p2 +11p−6
.

Solution. Let

2p + 3 2p + 3 A B C
= = + + (6.2.11)
p3 − 6p + 11p − 6 (p − 1)(p − 2)(p − 3) p − 1 p − 2 p − 3
2

Then
2p + 3 = A(p − 2)(p − 3) + B(p − 1)(p − 3) + C(p − 1)(p − 2)

Put p = 1, we have A = 5
2

Put p = 2, we have B = −7
Put p = 3, we get C = 29 .
Substituting these values of A and B in (6.2.11), we get

2p + 3 5 7 9
= − +
− 6p + 11p − 6
p3 2 2(p − 1) p − 2 2(p − 3)
h 2p + 3 i 5 h 1 i h 1 i 9 h 1 i
L −1 3 = L −1
− 7L −1
+ L −1
p − 6p2 + 11p − 6 2 p−1 p−2 2 p−3
h 2p + 3 i 5 9
L −1 3 = et − 7e2t + e3t .
p − 6p + 11p − 6
2 2 2

6.3 Exercises

Find the inverse Laplace transform of the following:


p2 1 2p2 −6p+5
1. (p2 +4)(p2 +9)
; 2. p(p2 +1)(p2 +4)(p2 +16)
; 3. p3 −6p2 +11p−6
;
1 1 1
4. (p+1)(p−2)
; 5. (p+1)(p+3)
; 6. p2 −5p+6
;
p +2p−3
2
7. p23p+7
−2p−3
; 8. p(p+2)(p−3) ; 9. 2p−1
p3 −p
;
2 2
10. 3(p2p−1)
5 ; 11. 2p2p+8 ; 12. 2p2p−8 ;
3
p+1
13. p2 −2p+2 ; 14. 1
p4 −1
; 15. p2p
4 −1 ;
2
4 +4 .
11p −2p+5 5p+3
16. (p+1)(p−2)(2p−1) ; 17. (p−1)(p2 +2p+5)
; 18. p2p

Transform Techniques B.Sc.(Mathematics)-IDE(UNOM)-II Year-IV Sem


6.3. Exercises 115

Answers
1. 15 (3 sin 3t − 2 sin 2t); 2. 1
768
(4 cos t − 6 cos 3t + 2 cos 5t); 4. 31 (e2t − e−t );
5. 21 (e−t − e−3t ); 6. e3t − e2t ; 7. 4e3t − e−t ;
et 3t2 3t4
8. 1
2
+ 54 e3t − 3 −2t
10
e ; 9. 1 + 2
− 32 e−t ; 10. 3
2
− 2
+ 48
;
11. 1
2
cos 2t; 12. 1
2
cosh 2t; 13. et (cos t + 2 sin t);
t
14. 21 (sinh t − sin t); 15. cos t + cosh t; 16. 2e−t + 5e2t − 23 e 2 ;
 
17. et + e−t 23 sin 2t − cos 2t ; 18. sin t sinh t.

B.Sc.(Mathematics)-IDE(UNOM)-II Year-IV Sem Transform Techniques


116
UNIT-II

Lesson 7

FIRST SHIFTING THEOREM


Learning Objectives
Upon completion of this lesson, students will be able to

• identify the first shifting theorem in inverse Laplace transform

• find inverse Laplace transform based on first shifting theorem

• identify the second shifting theorem in inverse Laplace transform

• find inverse Laplace transform based on second shifting theorem.

7.1 Introduction

T
his Lesson contains the first shifting theorem, the second shifting theorem, change of scale
property and how to apply these ideas to find out the inverse Laplace Transform.

Theorem 7.1.1. (First Shifting Theorem) If L −1 [ f (p)] = F(t), then L −1 [ f (p − a)] = eat F(t).

Proof. We know that

L [eat f (t)] = f (p − a) ⇒ L −1 [ f (p − a)] = eat F(t) = eat L −1 [ f (p)].

B.Sc.(Mathematics)-IDE(UNOM)-II Year-IV Sem 117 Transform Techniques


118 7.2. Worked out Problems

Note: Similarly, we have


1. L −1 [ f (p + a)] = e−at F(t) = e−at L −1 [ f (p)].
2. L −1 [ f (p)] = eat L −1 [ f (p + a)].
3. L −1 [ f (p)] = e−at L −1 [ f (p − a)].

7.2 Worked out Problems

(Problems base on first shifting theorem)


h i
Example 7.2.1. Find L −1 1
p2 +2p+5
.

Solution.
h 1 i h 1 i
L −1 = L −1
p2 + 2p + 5 (p + 1)2 + 4
since p2 + 2p + 5 = (p + 1)2 − 1 + 5 = (p + 1)2 + 4 by completing square method
h 1 i h 1 i
L −1 2 = e−t L −1 2 , by first shifting theorem
p + 2p + 5 p + 22
sin 2t
= e−t .
2
h i
Example 7.2.2. Find L −1 p+3
p2 −10p+29
.

Solution.
h p+3 i h p+3 i
L −1 = L −1
p2 − 10p + 29 (p − 5)2 + 4
h p − 5 + 5 + 3i
= L −1
(p − 5)2 + 4
h p−5 i h 8 i
= L −1 + L −1
(p − 5)2 + 4 (p − 5)2 + 4
h p i h 1 i
= e5t L −1 2 + 8e5t L −1 2 , by first shifting theorem
p +4 p +4
sin 2t
= e5t cos 2t + 8e5t
2
h p + 3 i  
L −1 2 = e5t cos 2t + 4 sin 2t .
p − 10p + 29

Transform Techniques B.Sc.(Mathematics)-IDE(UNOM)-II Year-IV Sem


7.2. Worked out Problems 119
h i
Example 7.2.3. Find L −1 p
p2 +4p+5
.

Solution.
h p i h p i
L −1 = L −1
p2 + 4p + 5 (p + 2)2 + 1
h p+2−2 i
= L −1
(p + 2)2 + 1

h p i h p+2 i h 2 i
L −1 = L −1
− L −1
p2 + 4p + 5 (p + 2)2 + 1 (p + 2)2 + 1
h p i h 1 i
= e−2t L −1 2 − 2e−2t L −1 2 , by first shifting theorem
p +1 p +1
= e−2t cos t − 2e−2t sin t
h p i  
L −1 = e−2t cos t − 2 sin t .
p2 + 4p + 5
h i h i
Example 7.2.4. Find (i) L −1 1
(p+2)2 +16
; (ii) L −1 √ 1 ;
2p−3

Solution. (i)
h 1 i h 1 i
−2t sin 4t
L −1 = e −2t
L −1
= e .
(p + 2)2 + 16 p2 + 16 4
(ii)

1
h 1 i h 1 i 1 3t h1i 1 3t t 2 −1 1 3t
L −1 p = L −1 q = √ e 2 L −1 1 = √ e 2 1 = √ e 2 .
2p − 3 2(p − 32 ) 2 p2 2 Γ( 2 ) 2πt

h i h i
Example 7.2.5. Find (i) L −1 2p+3
p2 +2p+2
; (ii) Find (i) L −1 1
(p+a)n
.

Solution.
h 2p + 3 i h 2p + 3 i
L −1 = L −1
p2 + 2p + 2 (p + 1)2 + 1
h 2(p + 1) − 2 + 3 i
= L −1
(p + 1)2 + 1

B.Sc.(Mathematics)-IDE(UNOM)-II Year-IV Sem Transform Techniques


120 7.2. Worked out Problems

h 2(p + 1) i h 1 i
= L −1 + L −1
(p + 1)2 + 1 (p + 1)2 + 1
h p i h 1 i
= 2e−t L −1 2 + e−t L −1 2 , by first shifting theorem
p +1 p +1
= 2e−t cos t + e−t sin t
h 2p + 3 i  
L −1 = e−t 2 cos t + sin t .
p2 + 2p + 2

(ii)
n−1
1 i −1 1
h i
h
−at t
L =e L
−at
−1
=e .
(p + a)n pn Γ(n)
h 3p−2 i
Example 7.2.6. Find the inverse Laplace transform of L −1 p2 −4p+20 .

Solution.
h 3p − 2 i h 3p − 2 i
L −1 = L −1
p2 − 4p + 20 (p − 2)2 + 16

3p − 2 i −1 3(p − 2) + 6 − 2
h h i
L −1 = L
p2 − 4p + 20 (p − 2)2 + 16
h 3(p − 2) i h 4 i
= L −1 + L −1
(p − 2)2 + 16 (p − 2)2 + 16
h p i h 1 i
= 3e2t L −1 2 + 4e2t L −1 2 , by first shifting theorem
p + 16 p + 16
= 3e2t cos 4t + e2t sin 4t
h 3p − 2 i  
L −1 = e2t 3 cos 4t + sin 4t .
p2 − 4p + 20
h i
Example 7.2.7. Find the inverse Laplace transform of L −1 1
(p+1)2 (p2 +4)
.

Solution. Let
1 A B Cp + D
= + + (7.2.1)
(p + 1)2 (p2 + 4) p + 1 (p + 1)2 p2 + 4

Transform Techniques B.Sc.(Mathematics)-IDE(UNOM)-II Year-IV Sem


7.2. Worked out Problems 121

Then

1 = A(p + 1)(p2 + 4) + B(p2 + 4) + (C p + D)(p + 1)2

1 = (A + C)p3 + (A + B + 2C + D)p2 + (4A + C + 2D)p + (4A + 4B + D)

Comparing the likewise coefficients, we get

A + C = 0, A + B + 2C + D = 0, 4A + C + 2D = 0 and 4A + 4B + D = 1

Solving these equations, we get

2 1 2 3
A= , B= ,C=− , D=− .
25 5 25 25

Substituting these values of A, B C and d in (7.2.1), we get

1 2 1 2 3
= + − −
(p + 1)2 (p2 + 4) 25(p + 1) 5(p + 1) 2 25(p + 4) 25(p2 + 4)
2
h 1 i 2 −1 h 1 i 1 −1 h 1 i 2 −1 h p i 3 −1 h 1 i
L −1 = L + L − L − L
(p + 1)2 (p2 + 4) 25 p+1 5 (p + 1)2 25 p2 + 4 25 p2 + 4
2 −t −1 h 1 i 1 −t −1 h 1 i 2 3 sin 2t
= e L + e L 2
− cos 2t −
25 p 5 p 25 25 2
2 −t 1 2 3 sin 2t
= e .1 + e−t .t − cos 2t −
25 5 25 25 2
h 1 i 2 −t 1 −t 2 3
L −1 = e + e t− cos 2t − sin 2t.
(p + 1)2 (p2 + 4) 25 5 25 50
h i h i
Example 7.2.8. Find (i) L −1 p+2
(p−2)3
; (ii) Find (i) L −1 p
(p+3)2
.

B.Sc.(Mathematics)-IDE(UNOM)-II Year-IV Sem Transform Techniques


122 7.2. Worked out Problems

Solution.
h p+2 i
−1 p − 2 + 2 + 2
h i
L −1 = L
(p − 2)3 (p − 2)3
h p−2 i h 1 i
= L −1 + 4L −1
(p − 2)3 (p − 2)3
h 1 i
−1 1
h i
= L −1 + 4e 2t
L , by first shifting theorem
(p − 2)2 p3
t2
= e2t .t + 4e2t
2!
h p+2 i  
L −1 = te2t 1 + 2t .
(p − 2)3

(ii)

p i −1 p + 3 − 3
h h i
L −1 = L
(p + 3)2 (p + 3)2
h p+3 i h 3 i
= L −1 − L −1
(p + 3)2 (p + 3)2
h 1 i h1i
= L −1 − 3e−3t L −1 2
p+3 p
h1i
= e−3t L −1 − 3e−3t .t
p
= e−3t .1 − 3e−3t t
h p i
L −1 = e−3t (1 − 3t).
(p + 3)2
h i
Example 7.2.9. Find the inverse Laplace transform of L −1 4p+5
(p−1)2 (p+2)
.

Solution. Let
4p + 5 A B C
= + + (7.2.2)
(p − 1) (p + 2) p − 1 (p − 1)
2 2 p+2
Then
4p + 5 = A(p − 1)(p + 2) + B(p + 2) + C(p − 1)2 (7.2.3)

= A(p2 + p − 2) + Bp + 2B + C(p2 − 2p + 1)

4p + 5 = (A + C)p2 + (A + B − 2C)p + (−2A + 2B + C)

Transform Techniques B.Sc.(Mathematics)-IDE(UNOM)-II Year-IV Sem


7.2. Worked out Problems 123

Put p = 1 in (7.2.3), we get B = 3.


Put p = −2 in (7.2.3), we get C = − 31 .
Comparing the coefficient of p2 , we get

1
A+C = 0 ⇒ A = .
3
Substituting these values of A, B, C and D in (7.2.2), we get

4p + 5 1 3 1
= + −
(p − 1) (p + 2)
2 3(p − 1) (p − 1) 2 3(p + 2)
h 4p + 5 i 1 −1 1h i h 1 i 1 −1 h 1 i
L −1 = L + 3L −1
− L
(p − 1)2 (p + 2) 3 p−1 (p − 1)2 3 p+2
1 t h1i 1
= e + 3et L −1 2 − e−2t
3 p 3
h 4p + 5 i 1 1
L −1 = et + 3et .t − e−2t .
(p − 1) (p + 2)
2 3 3
h i
Example 7.2.10. Find L −1 3p−4
p2 −4p+8

Solution.
p2 − 4p + 8 = (p − 2)2 − 4 + 8 = (p − 2)2 + 4,

by completing square method.

3p − 4 i
h h 3p − 4 i
L −1 = L −1
p2 − 4p + 8 (p − 2)2 + 4
h 3(p − 2) + 6 − 4 i
= L −1
(p − 2)2 + 4
h 3(p − 2) i h 1 i
= L −1 + 2L −1
(p − 2)2 + 4 (p − 2)2 + 4
h p i h 1 i
= 3e2t L −1 2 + 2e2t L −1 2
p +4 p +4
sin 2t
= 3e−2t cos 2t + 2e2t
2
h 3p − 4 i
L −1 2 = 3e−2t cos 2t + e2t sin 2t.
p − 4p + 8

B.Sc.(Mathematics)-IDE(UNOM)-II Year-IV Sem Transform Techniques


124 7.2. Worked out Problems

p2
h i
Example 7.2.11. Prove that L −1 (p+2)3
= e−2t (1 − 4t + 2t2 ).

Solution.
p2 i −1 (p + 2) − 4p − 4)
h h 2 i
L −1 = L
(p + 2)3 (p + 2)3
h (p + 2)2 i h p+1 i
= L −1 − 4L −1
(p + 2)3 (p + 2)3
h 1 i h p + 2 − 1i
= L −1 − 4L −1
p+2 (p + 2)3
h p2 i h p+2 i
−1 1
h i h 1 i
L −1 = e−2t
L − 4L −1
+ 4L −1
(p + 2)3 p (p + 2)3 (p + 2)3

h p2 i h 1 i
−1 1
h i
L −1 = e−2t
.1 − 4L −1
+ 4e−2t
L
(p + 2)3 (p + 2)2 p3
h1i t2
= e−2t − 4e−2t L −1 2 + 4e−2t
p 2!
= e−2t − 4e−2t .t + 2e−2t t2
h p2 i
L −1 = e−2t (1 − 4t + 2t2 ).
(p + 2)3
h i
Example 7.2.12. Find L −1 p+2
p2 −2p+5
.

Solution. p2 − 2p + 5 = (p − 1)2 − 1 + 5 = (p − 1)2 + 4, by completing square method.

p+2 i
h h p+2 i
L −1 = L −1
p2 − 2p + 5 (p − 1)2 + 4
h p − 1 + 1 + 2i
= L −1
(p − 1)2 + 4
h p−1 i h 1 i
= L −1 + 3L −1
(p − 1)2 + 4 (p − 1)2 + 4
h p i h 1 i
= et L −1 2 + 3et L −1 2
p +4 p +4
sin 2t
= et cos 2t + 3et
2
h p+2 i  3 
L −1 2 = et cos 2t + sin 2t .
p − 2p + 5 2

Transform Techniques B.Sc.(Mathematics)-IDE(UNOM)-II Year-IV Sem


7.2. Worked out Problems 125
h i
Example 7.2.13. Find L −1 p
p4 +4a4
.

Solution.

p4 + 4a4 = (p2 + 2a2 )2 − 4a2 p2 = (p2 + 2a2 − 2ap)(p2 + 2a2 + 2ap).

p p
=
+ 4a
p4 4 (p + 2a − 2ap)(p2 + 2a2 + 2ap)
2 2

1h 1 1 i
= −
4a p2 − 2ap + 2a2 p2 + 2ap + 2a2
1h 1 1 i
= −
4a (p − a)2 + a2 (p + a)2 + a2
h p i 1  −1 h 1 i h 1 i
L −1 4 = L − L −1
p + 4a4 4a (p − a)2 + a2 (p + a)2 + a2

h p i 1  at −1 h 1 i h 1 i
L −1 = e L − e −at
L −1
p4 + 4a4 4a p2 + a2 p2 + a2
1 h at sin at sin at i
= e − e−at
4a a a
1
= sin at(eat − a−at )
4a2
1
= sin at. 2 sinh at
4a2
h p i 1
L −1 4 = sin at sinh at.
p + 4a 4 2a2
h i
Example 7.2.14. Find L −1 p
5
(p+1) 2

Solution.
p p+1−1
=
(p + 1) (p + 1) 2
5 5
2

p+1 1
= −
(p + 1) (p + 1) 2
5 5
2

1 1
= −
(p + 1) 2 (p + 1) 2
3 5

B.Sc.(Mathematics)-IDE(UNOM)-II Year-IV Sem Transform Techniques


126 7.3. Exercises

h p i h 1 i h 1 i
L −1 = L −1 − L −1
(p + 1) (p + 1) (p + 1)
5 3 5
2 2 2

h1i h1i
= e−t L −1 3
− e−t L −1
5
p 2 p2
h p i h t 21 3
t2 i
L −1
= e −t
− .
(p + 1) 2
5
Γ( 32 ) Γ( 25 )

7.3 Exercises

Find the inverse Laplace transform of:


1 p+1 4p+5
1. p(p2 −1)(p2 +1) ; 2. p2 +6p+25
; 3. (p−4)2 (p+3)
;
p p 2p+1
4. 3 ; 5. p4 +p2 +1
; 6. (p−1)2 +(p+2)2
;
(p+1) 2
e−3p pe−ap
7. p2 −2p+5
; 8. p2 −9
.

Answers  
1. 12 (cos ht + cos t − 2); 2. e−3t cos 2t − 12 sin 2t ; 3. 17 e4t + 3e4t t − 71 e−3t ;
q √
e−t t √2 3t
4. πt − 2e
√ −t
π
; 5. 3
sin 2
sinh 2t ; 6. 3t (et − e−2t );
7. 21 e(t−3) sin 2(t − 3)H(t − 3); 8. cosh 3(t − a)H(t − a).

7.4 Second Shifting Theorem

Theorem 7.4.1. If L −1 [ f (p)] = F(t), then L −1 [e−ap f (p)] = G(t) where



F(t − a), if t > a




G(t) = 


if t < a

0,

Transform Techniques B.Sc.(Mathematics)-IDE(UNOM)-II Year-IV Sem


7.4. Second Shifting Theorem 127

Proof. By definition,
Z ∞
L [G(t)] = e−ptG(t)dt
Z0 a Z a
= G(t)dt +
e −pt
e−ptG(t)dt
0 0
Z a Z ∞
= e .0 dt +
−pt
e−pt F(t − a)dt
0 a

Put t − a = x so that dt = dx. Also when t = a, x = 0, when t = ∞, x = ∞.


Z ∞
L [G(t)] = e−p(x+a) F(x)dx
0
Z ∞
= e−ap
e−px F(x)dx
Z0 ∞
= e−ap e−pt F(t)dt
0

= e L [F(t)]
ap

= e−ap f (p)

∴ G(t) = L −1 [e−ap f (p)].

Note. The above result can also be expressed as



F(t − a), if t > a




L [e f (p)] = 
−1 −ap


if t < a

0,

or
L −1 [e−ap f (p)] = F(t − a)H(t − a),

where H(t − a) is Heaviside unit step function.

B.Sc.(Mathematics)-IDE(UNOM)-II Year-IV Sem Transform Techniques


128 7.5. Worked out Problems

7.5 Worked out Problems

(Based on second shifting theorem)


h i
e−5p
Example 7.5.1. Find L −1 (p−2)4

Solution.
h 1 i t3
L −1 =
p4 3!
3
h 1 i 2t t
L −1 = e = F(t),
(p − 2)4 6
by first shifting theorem.
 
3
e2(t−5)(t−5)
>  6 , if t > 5
 
F(t − 5), if t 5
 
1
h i 
 

L −1 e−5p = =

 
(p − 2)4

 

if t < 5  if t < 5
 
0,

 0,

by second shifting theorem. (or)

h e−5p i 1
L −1 = e2(t−5) (t − 5)3 H(t − 5),
(p − 2)4 6

where 
if t > 5

1,



H(t − 5) = 


if t < 5

0,


h i
e−3p
Example 7.5.2. Find L −1 p3
.

Solution.
h 1 i t2
S ince L −1 = = F(t),
p3 2!
by first shifting theorem. Using second shifting theorem, we have

h e−3p i (t − 3)2
L −1 = F(t − 3)H(t − 3) = H(t − 3),
p3 2

Transform Techniques B.Sc.(Mathematics)-IDE(UNOM)-II Year-IV Sem


7.5. Worked out Problems 129

where H(t − 3) is the Heaviside unit step function.


h −4p i
Example 7.5.3. Find L −1 (p−3)
e
4 .

Solution. Since
3
h 1 i −1 1
h i
3t t
L −1 = e3t
L = e = F(t),
(p − 3)4 p4 6
by first shifting theorem. Using second shifting theorem, we have

h e−4p i e3(t−4) (t − 4)3


L −1 = F(t − 4)H(t − 4) = H(t − 4),
(p − 3)4 6

where H(t − 4) is the Heaviside unit step function.


h −uπp i
Example 7.5.4. Find L −1 pep2 +25
5

Solution. Since
h p i
L −1 = cos 5t = F(t).
p2 + 25
Using second shifting theorem, we have

h pe −uπp
5 i  uπ   uπ   uπ   uπ 
L −1
= F t − H t − = cos 5 t− H t−
p2 + 25 5 5 5 5
 

where H t − is the Heaviside unit step function.
5
h −2p i
Example 7.5.5. Find L −1 p2 +4p+5
e
.

Solution. Since

h 1 i h 1 i h 1 i
L −1 = L −1
= e−2t
L −1
== e−2t sin t = F(t),
p2 + 4p + 5 (p + 2)2 + 1 p2 + 1

by first shifting theorem. Using second shifting theorem, we have

h e−2p i
L −1
= F(t − 2)H(t − 2) = e−2(t−2) sin(t − 2)H(t − 2),
p + 4p + 5
2

where H(t − 2) is the Heaviside unit step function.

B.Sc.(Mathematics)-IDE(UNOM)-II Year-IV Sem Transform Techniques


130 7.5. Worked out Problems
h 3+5p i
Example 7.5.6. Find L −1 p2 e2p
.

Solution. Let
3 + 5p 3 5
f (p) = 2
= 2+
p p p
h3 5i
L −1 [ f (p)] = L −1 + = 3t + 5 = F(t),
p2 p
by first shifting theorem. Using second shifting theorem, we have

h 3 + 5p i h e−2p (3 + 5p) i
L −1 = L −1 = F(t − 2)H(t − 2) = (3(t − 2) + 5)H(t − 2),
p2 e2p p2

where H(t − 2) is the Heaviside unit step function.


h (p+1)e−πp i
Example 7.5.7. Find L −1 p2 +p+1
.

Solution. Let
p+1
f (p) =
p2 + p+1
Then

p+1 p+1
=
p2 + p+1 (p + 21 )2 + 3
4
p + 21 + 1
= 2
(p + 21 )2 + 3
4
p+ 1 1
= 2
+ 2
(p + ) + 34
1 2
2
(p + 1 2
2
) + 34
h p+1 i h p + 12 i 1 h 1 i
L −1 = L −1 + L −1
p2 + p+1 (p + 12 )2 + 43 2 (p + 12 )2 + 34
1
h p i 1 1 h 1 i
= e− 2 t L −1 2 3 + e− 2 t L −1 2 3
p +4 2 p +4
√ 
√ 
 3  1 1 sin 23 t
− 12 t
= e cos t + e− 2 t √
2 2 3
2
√ √
 3 1  3 i
− 12 t
h
= e cos t + √ sin t
2 3 2

Transform Techniques B.Sc.(Mathematics)-IDE(UNOM)-II Year-IV Sem


7.5. Worked out Problems 131

Using second shifting theorem, we have


√ √
h (p + 1)e−πp i  3 1  3
− 12 (t−π)
h  i
L −1
= F(t − π)H(t − π) = e cos (t − π) + √ sin (t − π) H(t − π),
p2 + p + 1 2 3 2

where H(t − π) is the Heaviside unit step function.


h 4−3p i
Example 7.5.8. Find L −1 e 5 .
(p+4) 2

Solution.
h 1 i h1i
L −1 = e−4t L −1
(p + 4) 2
5 5
p2
3
t2
= e −4t
Γ( 52 )
3
t2
= e−4t 3
. 1 Γ( 12 )
2 2
3
4 t2
= e−4t √
3 π
= F(t)

Using second shifting theorem, we have

3
h e4−3p i 4 (t − 3) 2
L −1
= e F(t − 3)H(t − 3) = e−4(t−3) √ H(t − 3),
4
(p + 4) 2
5
3 π

where H(t − 4) is the Heaviside unit step function.


h −πp i
Example 7.5.9. Find L −1 1+ep2 +1
.

Solution.
h 1 + e−πp i 1 i h −πp i
−1 e
h
L −1 = L −1 + L
p2 + 1 p2 + 1 p2 + 1
= sin t + sin(t − π)H(t − π), by second shi f ting theorem

= sin t − sin tH(t − π)


h1 + e −πp i
L −1 = sin t(1 − H(t − π)).
p2 + 1

B.Sc.(Mathematics)-IDE(UNOM)-II Year-IV Sem Transform Techniques


132 7.6. Change of Scale of Property

7.6 Change of Scale of Property


h i
Theorem 7.6.1. If L [F(t)] = f (p), then L −1 [ f (ap)] = a1 F t
a
, a > 0.

Proof. By definition,
Z ∞
f (p) = e−pt F(t)dt
Z0 ∞
f (ap) = e−apt F(t)dt
0

Put at = u, then dt = du
a
.
When t = 0, u = 0 and t = ∞, u = ∞.
Z ∞ h u i du
f (ap) = e−pu F
0 a a
Z ∞
1 hui
= e−pu F du
a 0 a
Z ∞
1 h ti
= e−pt F dt, on changing the dummy variable u by t
a 0 a
1 h t i
= L
a a
h 1  t i
f (ap) = L F
a a
1 t
Hence L −1 [ f (ap)] = F .
a a

7.7 Worked out Problems


h i h i
Example 7.7.1. If L −1 p
(p2 +1)2
= 12 t sin t, find L −1 8p
(4p2 +1)2
.

Solution. Given,
h p i 1
L −1 = t sin t.
(p2 + 1)2 2

Transform Techniques B.Sc.(Mathematics)-IDE(UNOM)-II Year-IV Sem


7.7. Worked out Problems 133

Replace p by ap, we get

h ap i 1 1 t t
L −1 = . . . sin ,
((ap)2 + 1)2 2 a a a

by change of scale of property.

Putting a = 2, we get
h 2p i t t
L −1 = sin
(4p2 + 1)2 8 2
8p i t h t
∴ L −1 = sin .
(4p2 + 1)2 2 2
h − 1p i √ h − ap i
Example 7.7.2. If L −1 e 1 = cos√2πt t , find L −1 e 1
p2 p2

Solution. Given, √
h e− 1p i cos 2 t
L −1 = √ .
πt
1
p2
Replace p by ap, we get q
t
1
− ap
i 1 cos 2
−1 e
h a
L 1
= q ,
(ap) 2 a π at

by change of scale of property.

q
t
1 1 cos1
2
− ap
−1 e
h i a
√ L = √ √ ,
πt
1
a p2 a

Putting a = a1 , we get

h e− ap i cos 2 at
L −1 √ = √ .
p πt
Example 7.7.3. For a > 0, prove that L −1 [ f (p)] = F(t) implies that

1 −bt  t 
L −1 [ f (ap + b)] = e a F .
a a

B.Sc.(Mathematics)-IDE(UNOM)-II Year-IV Sem Transform Techniques


134 7.8. Exercises

Solution. By definition,
Z ∞
f (p) = L [F(t)] = e−pt F(t)dt
0
Z ∞
f (ap + b) = e−(ap+b)t F(t)dt
Z0 ∞
= e−apt e−bt F(t)dt
0

Put at = x, so that dt = dx
a
.

Also t = 0 ⇒ x = 0 and t = ∞ ⇒ x = ∞.

 x Z ∞
bx
f (ap + b) = dx e−px e− a F
a
Z0 ∞ h 1 bt  t 
= e−pt e− a F dt
0 a a
h 1 bt  t i
f (ap + b) = L e− a F
a a
1 −bt
t
Hence L −1 [ f (ap + b)] = e a F .
a a

7.8 Exercises
h i h i  
1. Given L −1 p
(p2 +1)2
= 2t sin t, show that L −1 32p
(16P2 +1)2
= 4t sin t
4
.

p2 −1 9p2 −1
h i h i  
2. Given L −1 (p2 +1)2
= t cos t, show that L −1 (9P2 +1)2
= 9t cos t
3
.

Transform Techniques B.Sc.(Mathematics)-IDE(UNOM)-II Year-IV Sem


UNIT-II

Lesson 8

INVERSE LAPLACE TRANSFORM


OF DERIVATIVES AND INTEGRALS
Learning Objectives
Upon completion of this lesson, students will be able to

• find inverse Laplace transform of derivatives and integrals

• solve problems based on inverse Laplace transform of derivatives and integrals


h f (p) i
• find L −1 [pn f (p)] and L −1 p

8.1 Introduction

T
his Lesson focuses on finding Inverse Laplace Transform of derivatives, integrals,
multiplication and division by powers of p.

8.2 Inverse Laplace Transform of Derivatives

dn
Theorem 8.2.1. If L −1 [ f (p)] = F(t), then L −1 [ f n (p)] = (−1)n tn F(t) where f n (p) = d pn
[ f (p)].

B.Sc.(Mathematics)-IDE(UNOM)-II Year-IV Sem 135 Transform Techniques


136 8.3. Worked out Problems

Proof. We know that

dn
L [tn F(t)] = (−1)n ( f (p))
d pn
= (−1)n f n (p)

∴ tn F(t) = (−1)n L −1 [ f n (p)]

i.e., (−1)n tn F(t) = (−1)2n L −1 [ f n (p)]

= L −1 [ f n (p)]

∴ L −1 [ f n (p)] = (−1)n tn F(t).

8.3 Worked out Problems


h i
Example 8.3.1. Find the inverse Laplace transform of log 1 + 1
p2
.

Solution.
1
L −1 [ f (p)] = − L −1 [ f 0 (p)] (8.3.1)
t

h 1i h p2 + 1 i
Let f (p) = log 1 + 2 = log 2
, f (p) = log(p2 + 1) − log p2
p p

2p 2
f 0 (p) = −
p2 + 1 p
2h  p   1 i
(4.2.1) ⇒ L −1 [ f (p)] = − L −1 2 − L −1 2
t p +1 p
2
= − [cos t − 1]
t
h  1 i 2
L −1 log 1 + 2 = (1 − cos t).
p t

Transform Techniques B.Sc.(Mathematics)-IDE(UNOM)-II Year-IV Sem


8.3. Worked out Problems 137
h p+3 i
Example 8.3.2. Find the inverse Laplace transform of log p+4
.

Solution.
h p + 3i
Let f (p) = log = log(p + 3) − log(p + 4).
p+4

1
f 0 (p) = −
p+3 p+4
1h  1   1 i
(4.2.1) ⇒ L −1 [ f (p)] = − L −1 − L −1
t p+3 p+4
1
= − [e−3t − e−4t ]
t
h  p + 3 i 1
L −1 log = [e−4t − e−3t ].
p+4 t
h p+1 i
Example 8.3.3. Find the inverse Laplace transform of log p−1
.

Solution.
h p + 1i
Let f (p) = log = log(p + 1) − log(p − 1).
p−1

1 1
f 0 (p) = −
p+1 p−1
1h  1   1 i
(4.2.1) ⇒ L −1 [ f (p)] = − L −1 − L −1
t p+1 p−1
1
= − [e−t − et ]
t
h  p + 1 i 1 t
L −1 log = [e − e−t ]
p−1 t
h  p + 1 i 2
i.e., L −1 log = sinh t.
p−1 t
h 1+p i
Example 8.3.4. Find the inverse Laplace transform of log p2
.

Solution.
h1 + pi
Let f (p) = log = log(p + 1) − log(p2 ),
p2
f (p) = log(p + 1) − 2 log p

B.Sc.(Mathematics)-IDE(UNOM)-II Year-IV Sem Transform Techniques


138 8.3. Worked out Problems

1 1
f 0 (p) = −2
p+1 p
1 h  1   1 i
(4.2.1) ⇒ L −1 [ f (p)] = − L −1 − 2L −1
t p+1 p
1
= − [e−t − 2]
t
h  1 + p i 1
L −1 log = [2 − e−t ]
p2 t

p2 +1
h i
Example 8.3.5. Find the inverse Laplace transform of log (p−1)2
.

Solution.

h p2 + 1 i
Let f (p) = log = log(p2 + 1) − log(p − 1)2 , f (p) = log(p2 + 1) − 2 log(p − 1)
(p − 1)2

2p 1
f 0 (p) = −2
+1p2 p−1
1 h  2p   1 i
(4.2.1) ⇒ L −1 [ f (p)] = − L −1 2 − 2L −1
t p +1 p−1
1
= − [2 cos t − 2et ]
t
h  p2 + 1 i 2
L −1 log 2
= [et − cos t]
(p − 1) t
h    i
Example 8.3.6. Find L −1 tan−1 ap + cot−1 bp .

Solution.
a  p
Let f (p) = tan−1 + cot−1 .
p b
Then
1  −a  1 1
f 0 (p) = − . .
1+ a2
p2
p2 1+ p2
b2
b

Transform Techniques B.Sc.(Mathematics)-IDE(UNOM)-II Year-IV Sem


8.3. Worked out Problems 139
−a b
f 0 (p) = −
p2 + a2 p2 + b2
1 h −1  −a   b i
(4.2.1) ⇒ L −1 [ f (p)] = −L − L −1
t p2 + a2 p2 + b2
1
= − [− sin at − sin bt]
t
h a  p i 1
L −1 tan−1 + cot−1 = [sin at + sin bt].
p b t
h  i
Example 8.3.7. Find L −1 cot−1 p+a
b
.

Solution.
 p + a
Let f (p) = cot−1 .
b
Then
1 1
f 0 (p) = − (p+a)2
. .
1+ b2
b

−b
f 0 (p) = −
(p + a)2 + b2
1 h −1  b i
(4.2.1) ⇒ L −1 [ f (p)] = − L −
t (p + a)2 + b2
1 h b i
= e−at L −1 2
t p + b2
h  p + a i e −at
L −1 cot−1 = sin bt.
b t
h i
Example 8.3.8. Find L −1 tan−1 (p + 1) .

Solution. Let f (p) = tan−1 (p + 1). Then f 0 (p) = 1


1+(p+1)2

1 h −1  1 i
(4.2.1) ⇒ L −1 [ f (p)] = − L
t (p + 1)2 + 1
1 h 1 i
= − e−t L −1 2
t p +1
h  p + a i e −t
L −1 cot−1 = − sin t.
b t

B.Sc.(Mathematics)-IDE(UNOM)-II Year-IV Sem Transform Techniques


140 8.4. Inverse Laplace Transform of Integrals
h i
Example 8.3.9. Find L −1 p
(p2 −25)2
.

Solution.
1
Let f (p) = .
p2 − 25
Then

1
f 0 (p) = − .2p
(p2 − 25)2
1
Since L −1 [ f (p)] = − L −1 [ f 0 (p)]
t
i.e., L −1 [ f 0 (p)] = −tL −1 [ f (p)]
h −2p i h 1 i
L −1 2 = −tL −1
(p − 25)2 p2 − 25
t h 1 i
= L −1 2
2 p − 25
h p i t
L −1 2 2
= sinh 5t.
(p − 25) 10

8.4 Inverse Laplace Transform of Integrals


hR ∞ i
Theorem 8.4.1. If L −1 [ f (p)] = F(t), then L −1 p
f (p)d p = F(t)
t
.

Proof. We know that


h F(t) i Z ∞
L = f (p)d p,
t p

provided the integral exists.


hZ ∞ i F(t)
∴L −1
f (p)d p = .
p t
The above result can also be written as

hZ ∞ Z ∞
i1 h i
L −1
f (p)d p = L −1 [ f (p)] or L −1 [ f (p)] = tL −1 f (p)d p
p t p

Transform Techniques B.Sc.(Mathematics)-IDE(UNOM)-II Year-IV Sem


8.5. Worked out Problems 141

8.5 Worked out Problems


h i
Example 8.5.1. Find L −1 2p
(p2 −4)2
.

Solution.
2p
Let f (p) =
(p2 − 4)2

hZ ∞ i
L [ f (p)] = tL
−1 −1
f (p)d p
p
hZ ∞
h 2p i 2p i
L −1 = tL −1 d p
(p − 4)2
2 2
p (p − 4)
2

h −1 ∞ i
= tL −1 2
p −4 p
h 1 i
= tL −1 0 + 2
p −4
sinh 2t
= t
2
h 2p i t
L −1 = sinh 2t
(p2 − 4)2 2
h i
Example 8.5.2. Find L −1 p+2
((p2 +4p+5)2
.

Solution.
p+2
Let f (p) =
(p2 + 4p + 5)2

hZ ∞ i
L [ f (p)] = tL
−1 −1
f (p)d p
p
h p+2 i hZ ∞ p+2 i
L −1 = tL −1
d p
(p2 + 4p + 5)2 p (p + 4p + 5)
2 2

2p + 4
h1 Z i
= tL −1 d p
2 p (p2 + 4p + 5)2

B.Sc.(Mathematics)-IDE(UNOM)-II Year-IV Sem Transform Techniques


142 8.5. Worked out Problems

t −1 h 1 ∞ i
L − 2 =
2 p + 4p + 5 p
t h 1 i
= L −1
2 (p + 2)2 + 1
t h 1 i
= e−2t L −1 2
2 p +1
h p + 2 i t
L −1 2 = e−2t sin t.
(p + 4p + 5)2 2
h i
Example 8.5.3. Find L −1 p
(p2 −a2 )2
.

Solution.
p
Let f (p) =
(p2 − a2 )2

hZ ∞ i
L [ f (p)] = tL
−1 −1
f (p)d p
p
h p i hZ ∞ p i
L −1
= tL −1
d p
(p2 − a2 )2 2
p (p − a )
2 2

h1 Z ∞
2p i
= tL −1 d p
2 p (p2 − a2 )2
t h −1 ∞ i
= L −1 2
2 p − a2 p
t h 1 i
= L −1 0 + 2
2 p − a2
t sinh at
=
2 a
h p i t
L −1 2 2 2
= sinh at.
(p − a ) 2a
hR ∞   i
Example 8.5.4. Find L −1 p
log u−1
u+1
du .

Solution.
u − 1
Let f (u) = log = log(u − 1) − log(u + 1)
u+1
1 1
f 0 (u) = −
u−1 u+1

Transform Techniques B.Sc.(Mathematics)-IDE(UNOM)-II Year-IV Sem


8.6. Exercises 143

By inverse Laplace transform of derivatives,

1
L −1 [ f (u)] = − L −1 [ f 0 (u)]
t
1 h 1 1 i
L −1 [log(u − 1) − log(u + 1)] = − L −1 −
t u−1 u+1
1 −1 h 1 1 i
= − L −
t u−1 u+1
1 t
= − (e − e−t )
t
1 −t
L [log(u − 1) − log(u + 1)] = (e − et ) = F(t)
−1
t

Now applying inverse Laplace transform of integrals,

hZ ∞ i F(t)
L −1
f (u)du = .
p t


hZ u − 1 i 1 1 e−t − et
∴L −1
log du = (e−t − et ) = .
p u+1 t t t2
h i h i
Note. (1) L −1 2ap
(p2 −a2 )
= t sinh at; (2) L −1 p
(p2 +a2 )
= t
2a
sin at.

8.6 Exercises

A. Find the inverse Laplace transform of the following:


 p   p+a   
1. log p−1 ; 2. log p+b ; 3. log p+ap−a
;
 2
  2
  
4. log 1 + ap2 ; 5. log 1 − ap2 ; 6. log 1 − p12 ;
 p2 +1     
7. log p(p+1) ; 8. log p(p+1)
p2 +4
; 9. cot −1 p
2
;
 
10. cot−1 (p + 1); 11. tan−1 p22 ; 12. p2 −a p
2.

hR ∞  i
B. Find L −1 1
u
− 1
u−1
du .
h R p∞   i
C. Find L −1 p
a
u2 −a2
b
− u2 −b 2 du .
hR ∞  i
D. Find L −1 p u2 +a u
2
u
− u2 +b 2 du .

B.Sc.(Mathematics)-IDE(UNOM)-II Year-IV Sem Transform Techniques


144 8.7. Multiplication by Powers of p

Answers
t e−bt −e−at
A. 1. e −1
t
; 2. t
; 3. 2
at
sinh at;
4. 2t (1 − cos at); 5. 2t (1 − cosh at); 6. 2t (1 − cosh t);
7. 1t (1 + e−t − 2 cos t); 8. 1t (2 cos 2t − e−t − 1); 9. sin 2t
t
;
e−t sin t 2 t
10. t
; 11. t
sinh t sin t; 12. 2a
sinh at.

1−et
B. t
.
C. 1t (sinh at − sinh bt).
D. 1t (cos at − cos bt).

8.7 Multiplication by Powers of p

Theorem 8.7.1. If L −1 [ f (p)] = F(t) and F(0) = 0, then L −1 [p f (p)] = F 0 (t).

Proof. We know that


L [F 0 (t)] = p f (p) − F(0)

∴ L [F 0 (t)] = p f (p), since F(0) = 0

or
L −1 [p f (p)] = F 0 (t).

Note: In general, L −1 [pn f (p)] = F n (t), if F n (0) = 0 for n = 1, 2, 3, .., n − 1.

8.8 Worked out Problems


h i
Example 8.8.1. Find L −1 p
2p2 −1
.

Transform Techniques B.Sc.(Mathematics)-IDE(UNOM)-II Year-IV Sem


8.8. Worked out Problems 145

Solution. Let
1
f (p) =
2p2 −1

Since F(t) = L −1 [ f (p)]


h 1 i
= L −1
2p2 − 1
1h 1 i
=
2 p2 − 12
 
sinh √t
1 2
=
2 √1
2
1  t 
F(t) = √ sinh √
2 2

Clearly, F(0) = 0. Thus,

h p i h 1 i
Thus L −1 = L −1
p.
2p2 − 1 2p2 − 1
= L −1 [p f (p)]

= F 0 (t)
1  t  1
= √ cosh √ . √
2 2 2
h p i 1  t 
L −1 = cosh √ .
2p2 − 1 2 2
h i
Example 8.8.2. Find L −1 p
(p−4)5
.

Solution. Let
1
f (p) =
(p − 4)5

B.Sc.(Mathematics)-IDE(UNOM)-II Year-IV Sem Transform Techniques


146 8.8. Worked out Problems

Since F(t) = L −1 [ f (p)]


1
= L −1 [ ]
(p − 4)5
h1i
= e4t L −1 5
p
4
t
= e4t
4!
t4 e4t
F(t) =
24

Clearly F(0) = 0.

L −1 [p f (p)] = F 0 (t)
h p i d h t4 e4t i
L −1 =
(p − 4)5 dt 24
1 4 4t
= [t e + e4t 4t3 ]
24
h p i t3 e4t (t + 1)
L −1 = .
(p − 4)5 6

p2
h i
Example 8.8.3. Find L −1 (p−3)3
.

Solution. Let
1
f (p) =
(p − 3)3

S ince F(t) = L −1 [ f (p)]


1
= L −1 [ ]
(p − 3)3
h1i
= e3t L −1 3
p
2
t
F(t) = e3t
2

Transform Techniques B.Sc.(Mathematics)-IDE(UNOM)-II Year-IV Sem


8.8. Worked out Problems 147

Clearly F(0) = 0, F 0 (0) = 0

L −1 [p2 f (p)] = F 00 (t)


h p2 i d2 h t2 3t i
L −1 = e
(p − 3)3 dt2 2
1 d h 2 3t i
= t .3e + 2te3t
2 dt
1 h 2 3t i
= 9t e + 6te3t + 6te3t + 2e3t
2
h p2 i 1 h 2 3t i
L −1 = 9t e + 12te 3t
+ 2e3t
.
(p − 3)2 2

p2
h i
Example 8.8.4. Find L −1 (p2 +a2 )2
.

Solution. Let
p
f (p) =
(p2 + a2 )2

hZ ∞
p i
L [ f (p)] = tL
−1 −1
d p
p (p + a )
2 2 2

h1 ∞Z
2p i
= tL −1 d p
2 p (p2 + a2 )2
h1 1 ∞ i
= tL −1 − 2
2 p + a2 p
t −1 h 1 i
= L
2 p2 + a2
t
L −1 [ f (p)] = sin at.
2a

Clearly F(0) = 0.

L −1 [p f (p)] = F 0 (t)
h p2 i h p.p i
L −1 2 = L −1
(p + a2 )2 (p2 + a2 )2

B.Sc.(Mathematics)-IDE(UNOM)-II Year-IV Sem Transform Techniques


148 8.9. Division by p

dh t i
=
sin at
dt 2a
1
= [t.a cos at + sin at]
2a
h p2 i 1
L −1 2 = [at cos at + sin at].
(p + a )
2 2 2a

8.9 Division by p
h f (p) i Rt
Theorem 8.9.1. If L −1 [ f (p)] = F(t), then L −1 p
= 0
F(u)du.

Proof. We know that


hZ t i 1
L F(u)du = f (p)
0 p
h f (p) i Z t
∴ L −1 = F(u)du.
p 0

h f (p) i RtRt
Note.If L −1 [ f (p)] = F(t), then L −1 p2
= 0 0
F(u)dudu.

8.10 Worked out Problems

Example 8.10.1. Find the inverse Laplace transform of 1


p(p+2)
.

Solution. Let f (p) = 1


p+2
.
h 1 i
F(t) = L −1 = e−2t .
p+2

h f (p) i Z t
Since L −1
= F(u)du
p 0
Z t
h 1 i
L −1
= e−2t dt
p(p + 2) 0

Transform Techniques B.Sc.(Mathematics)-IDE(UNOM)-II Year-IV Sem


8.10. Worked out Problems 149
 e−2t t
=
−2 0
h 1 i 1
L −1 = (1 − e−2t ).
p(p + 2) 2

Example 8.10.2. Find the inverse Laplace transform of 1


p(p2 +a2 )
.

Solution. Let f (p) = 1


p2 +a2
.
h 1 i sin at
F(t) = L −1 = .
p2 + a2 a

h f (p) i Z t
Since L −1
= F(u)du
p 0
Z t
h 1 i sin at
L −1
= dt
p(p2 + a2 ) 0 a
1  − cos at t
=
a a 0
1
(cos at − 1) = −
a2
h 1 i 1
L −1 = 2 (1 − cos at).
p(p + a )
2 2 a

Example 8.10.3. Find the inverse Laplace transform of 1


p2 (p2 +a2 )
.

Solution. Let f (p) = 1


p(p2 +a2 )
.

h 1 i 1
L −1 = 2 (1 − cos at).
p(p2 + a2 ) a

by the previous example.

h f (p) i Z t
Since L −1
= F(u)du
p 0
Z t
h 1 1i
L −1
=
(1 − cos at)dt
p2 (p2 + a2 )
0 a
2

1  sin at t
= 2 t−
a a 0
h 1 i 1  1 
L −1 2 2 = t − sin at .
p (p + a2 ) a2 a

B.Sc.(Mathematics)-IDE(UNOM)-II Year-IV Sem Transform Techniques


150 8.10. Worked out Problems

Example 8.10.4. Find the inverse Laplace transform of 1


p3 (p2 +a2 )
.

Solution. Let f (p) = 1


p2 (p2 +a2 )
.

h 1 i 1 1 
L −1 = t − sin at .
p2 (p2 + a2 ) a2 a

by the previous example.

h f (p) i Z t
S ince L −1
= F(u)du
p 0
Z t
h 1 1 1
i 
L −1
=t − sin at dt
p3 (p2 + a2 )
0 a
2 a
1  t2 1  − cos at t
= 2 −
a 2 a a 0
2
1 t
h 1 1i
= 2 + 2 cos at − 2
a 2 a a
2
h 1 i 1 t
 1 
L −1 3 2 = + (cos at − 1) .
p (p + a2 ) a2 2 a2

Example 8.10.5. Find the inverse Laplace transform of 1


p3 (p2 +1)
.

Solution. Taking a = 1 in the previous example, we have

h 1 i t2
L −1 = + (cos t − 1).
p2 (p2 + 1) 2

Example 8.10.6. Find the inverse Laplace transform of 1


(p2 +a2 )2
.

Solution. We know that

h p i t
L −1 = sin at = F(t), say
(p + a )
2 2 2 2a

h 1 i h p i
∴ L −1 = L −1
(p2 + a2 )2 p(p2 + a2 )2

Transform Techniques B.Sc.(Mathematics)-IDE(UNOM)-II Year-IV Sem


8.10. Worked out Problems 151
h f (p) i Z t
Since L −1
= F(u)du
p 0
Z t
u
= sin audu
0 2a
Z t
1 u
= sin audu
2a 0 2a
1 h  cos au   sin au it
= u − −1 −
2a a a2 0
1 h u cos au sin au t i
= − +
2a a a2 0
1
= (sin au − au cos au)t0
2a3
h p i 1
L −1 = (sin at − at cos at).
(p2 + a2 )2 2a3

Example 8.10.7. Find the inverse Laplace transform of 1


p(p2 +2p+2)
.

Solution.
1 1
Let f (p) = = .
p2 + 2p + 2 (p + 1)2 + 1
Then
h 1 h 1 i
L −1 [ f (p)] = L −1 = e−t
L −1
= e−t sin t = F(t)
(p + 1)2 + 1 p2 + 1

h f (p) i Z t
L −1
= F(u)du
p 0
Z t
h 1 i
L −1
= e−u sin udu
p(p2 + 2p + 2) 0
h e−u it
= (− sin u − cos u)
1+1 0
1 h i
= − e−t (sin t + cos t) − (0 + 1)
2
h 1 i 1
L −1
= [1 − e−t (sin t + cos t)].
p(p + 2p + 2)
2 2
 p+2 
Example 8.10.8. Find the inverse Laplace transform of 1
p
log p+1
.

Solution.
 p + 2
Let f (p) = log = log(p + 2) − log(p + 1)
p+1

B.Sc.(Mathematics)-IDE(UNOM)-II Year-IV Sem Transform Techniques


152 8.10. Worked out Problems

1 1
f 0 (p) = −
p+2 p+1
h 1 1 i
L −1 [ f 0 (p)] = L −1 −
p+2 p+1
= e−2t − e−t
1
L −1 [ f (p)] = − L −1 [ f 0 (p)]
t
h  p + 2 i 1 −2t
∴ L −1 log = − (e − e−t )
p+1 t
h f (p) i Z t
Since L −1 = F(u)du
p 0
h1  p + 2 i Z t −2u
e − e−u
L −1 log = − du.
p p+1 0 u
h1  p + 2 i Z t −u
e − e−2u
L −1 log = du.
p p+1 0 u

p2 +2

Example 8.10.9. Find the inverse Laplace transform of p(p2 +4)
.

Solution.
h p2 + 2 i
−1 (p + 4) − 2
h 2 i
L −1 = L
p(p2 + 4) p(p2 + 4)
h p2 + 4 2 i
= L −1

p(p2 + 4) p(p2 + 4)
h1i h 2 i
= L −1 − L −1
p p(p2 + 4)

h p2 + 2 i h 2 i
L −1 = 1 − L −1
(8.10.1)
p(p2 + 4) p(p2 + 4)
2
Let f (p) =
p2 + 4
L −1 [ f (p)] = sin 2t = F(t).

Transform Techniques B.Sc.(Mathematics)-IDE(UNOM)-II Year-IV Sem


8.10. Worked out Problems 153
h f (p) i Z t
L −1
= F(u)du
p 0
Z t
h 2 i
L −1
= sin 2udu
p(p2 + 4) 0
 cos 2u t
= −
2 0
h 2 i 1
L −1 = − (cos 2t − 1)
p(p + 4)
2 2

h p2 + 2 i 1
∴ (8.10.1) ⇒ L −1 = 1 + (cos 2t − 1)
p(p + 4)
2 2
h p2 + 2 i 1
L −1 = (1 + cos 2t).
p(p + 4)
2 2

Example 8.10.10. Find the inverse Laplace transform of p+1


p2 (p2 +1)
.

Solution.
h p+1i h p 1 i
L −1 = L −1
+
p2 + 1 p2 + 1 p2 + 1
= cos t + sin t

h f (p) i Z t
L −1
F(u)du =
p 0
h p+1 i Z t
L −1
= (cos u + sin u)du
p(p2 + 1) 0

= (sin u − cos u)t0


h p+1 i
L −1 = 1 + sin t − cos t
p(p2 + 1)
h p+1 i Z t
Finally L −1
= (1 + sin u − cos u)du
p2 (p2 + 1) 0

= (u − cos u − sin u)t0


h p+1 i
L −1 = 1 + t − cos t − sin t.
p2 (p2 + 1)

Example 8.10.11. Find the inverse Laplace transform of 1


p4 −2p3
.

B.Sc.(Mathematics)-IDE(UNOM)-II Year-IV Sem Transform Techniques


154 8.10. Worked out Problems

Solution.
h 1 i h 1 i
L −1 = L −1
p4 − 2p3 p3 (p − 2)
h 1 i
L −1 = e2t = F(t)
p−2

h f (p) i Z t
L −1
= F(u)du
p 0
Z t
h 1 i
L −1
= e2u du
p(p − 2) 0
 e2u t
=
2 0

1 h i 1
L −1 = (e2t − 1)
p(p − 2) 2
Z t
h 1 i 1
Similarly L −1 2 = ( (e2u − 1))du
p (p − 2) 0 2
1  e2u t
= −u
2 2 0
2t
1 e
 1
= −t−
2 2 2
Z t  e2u
h 1 i 1 1
Finally L −1 3 = ( − t − du
p (p − 2) 0 2 2 2
2u 2
1 e
 u u t

= − −
2 4 2 2 0
1 1 2t t2 t 1 i
h
= e − − −
2 4 2 2 4
1 h 1 2t 1 i
= (e − 1) − (t2 + t)
2 4 2
h 1 i 1
L −1 3 = (e2t − 2t2 − 2t − 1)
p (p − 2) 8
h i
Example 8.10.12. Find L −1 1
p(p2 −1)(p2 +1)
.

Transform Techniques B.Sc.(Mathematics)-IDE(UNOM)-II Year-IV Sem


8.11. Exercises 155

Solution.
h 1 i
−1 1
h  1 1 i
L −1 = L −
p(p2 − 1)(p2 + 1) 2 p2 − 1 p2 + 1
1  −1 h 1 i h 1 i
= L − L −1
2 p2 − 1 p2 + 1
1
= (sinh t − sin t)
2
= F(t)
Z t
h 1 i 1
∴L −1
= (sinh u − sin u)du
p(p − 1)(p + 1)
2 2
0 2
1
= (cosh u + cos u)t0
2
1
= (cosh t + cos t − (1 + 1))
2
h 1 i 1
L −1 = (cosh t + cos t − 2).
p(p − 1)(p + 1)
2 2 2

8.11 Exercises

Find the inverse Laplace transform of the following:


 2 +a2  p2
1. 2p2p−1 ; 2. p log pp2 +b2 ; 3. (p2 −a 2 )2 ;

p2 −w2 p2 p2
4. (p2 +w2 )2
; 5. (p−1)3
; 6. (p+4)3
;
p 2
1 1
7. (p−1)4
; 8. p(p2 +9)
; 9. p(p2 −16)
;
1 1 1
10. p(p+2)3
; 11. p(p−a)4
; 12. p(p2 +4p+5)
;
1 8 1
13. p2 (p−a)
; 14. p4 −4p2
; 15. p2 (p+1)2
;
 p−2 
16. 1
p2 p2 +4
; 17. 1
p3 (p−2)
.

Answers

B.Sc.(Mathematics)-IDE(UNOM)-II Year-IV Sem Transform Techniques


156 8.11. Exercises
 
1. 1
2
cosh √t
2
; 2. t22 [(cos at − cos bt) + t(a sin at − b sin bt)];
3. 1
2a
(sinh at + at cosh at); 4. t cos wt;
et
5. 2 (t2 + 4t + 2); 6. e−4t (8t2 − 8t + 1);
3
7. et (t + t2 + t6 ); 8. 19 (1 − cos 3t);
9. 1
16
(cosh 4t − 1); 10. 18 [1 − e−2t (2t2 + 2t + 1)];
eat
11. 1
a4
+ 6a4
(a3 t3 − 3a2 t2 + 6at − 6); 12. 51 [1 − e−2t (2 sin t + cos t)];
1
13. a2 (eat −at−1)
; 14. sinh 2t − 2t;
15. te−t + 2e−t + t − 2; 16. 14 (1 − 2t − cos 2t + sin 2t);
17. 18 (e2t − 2t2 − 2t − 1).

Transform Techniques B.Sc.(Mathematics)-IDE(UNOM)-II Year-IV Sem


UNIT-II

Lesson 9

CONVOLUTION THEOREM
Learning Objectives
Upon completion of this lesson, students will be able to

• find convolution of two functions

• identify convolution theorem

• solve problems by using convolution theorem.

9.1 Convolution

C
onvolution is useful for obtaining inverse Laplace transform of a product of two transforms
and solving ordinary differential equations.

Definition 9.1.1. Let F(t) and G(t) be two functions defined for t > 0. We define
Z t
F(t) ∗ G(t) = F(u)G(t − u)du
0

assuming that the integral on the right hand side exists. F(t)∗G(t) is called the convolution product
of F(t) and G(t).

It can be proved that

B.Sc.(Mathematics)-IDE(UNOM)-II Year-IV Sem 157 Transform Techniques


158 9.1. Convolution

(i) Convolution product is commutative. i.e., F(t) ∗ G(t) = G(t) ∗ F(t);


(ii) Convolution product is associative. i.e., F(t) ∗ (G(t) ∗ H(t)) = (F(t) ∗ G(t)) ∗ H(t);
(iii) F(t) ∗ 0 = 0 ∗ F(t) = 0.

Note. In general 1 ∗ F(t) = F(t).

Theorem 9.1.1. If L [F(t)] = f (p) and L [G(t)] = g(p) then

L [F(t) ∗ G(t)] = f (p).g(p) or L −1 [ f (p).g(p)] = F(t) ∗ G(t).

Proof. Let Z t
φ(t) = F(t) ∗ G(t) = F(u)G(t − u)du.
0

Then
Z ∞ hZ t i
L [φ(t)] = e −pt
F(u)G(t − u)du dt
0 0
Z ∞Z t
= e−pt F(u)G(t − u)dudt
0 0

The double integral is considered within the region enclosed by the lines u = 0 and u = t.
On changing the order of integration, we get
Z ∞ Z ∞
L [φ(t)] = e−pt F(u)G(t − u)dtdu
Z0 ∞ u hZ ∞ i
= −pu
e F(u) e−p(t−u)G(t − u)dt du
Z0 ∞ u
hZ ∞ i
= e−pu F(u) e−pvG(v)dv du on putting t − u = v
Z0 ∞ 0

= e−pu F(u)[g(p)]du
0
Z ∞
= g(p) e−pu F(u)du
0

Transform Techniques B.Sc.(Mathematics)-IDE(UNOM)-II Year-IV Sem


9.2. Worked out Problems 159

L [φ(t)] = g(p). f (p)

or φ(t) = L −1 [ f (p).g(p)]

or F(t) ∗ G(t) = L −1 [ f (p).g(p)].

Hence the theorem follows. 

9.2 Worked out Problems


h i
Example 9.2.1. Find L −1 1
(p+a)(p+b)
by using convolution theorem.

Solution. Let f (p) = 1


p+a
and g(p) = 1
p+b
. Then

h 1 i
F(t) = L −1 [ f (p)] = L −1 = e−at .
p+a
and
h 1 i
G(t) = L −1 [g(p)] = L −1 = e−bt .
p+b
∴ by convolution theorem,

h 1 i h 1 1 i
L −1 = L −1 .
(p + a)(p + b) (p + a) p + b
= L −1 [ f (p).g(p)]

= F(t) ∗ G(t)
Z t
= F(u)G(t − u)du
0
Z t
= e−au e−b(t−u) du
0
Z t
= e −bt
e−b(t−u) du
0

B.Sc.(Mathematics)-IDE(UNOM)-II Year-IV Sem Transform Techniques


160 9.2. Worked out Problems

 e−(a−b)u t
= e−bt
−(a − b) 0
1 −bt  −(a−b)t 
= − e e −1
a−b
1
= (e−at − e−bt ).
b−a
h i
Example 9.2.2. Find L −1 1
p(p2 +4)
by using convolution theorem.

Solution. Let f (p) = 1


p
and g(p) = 1
p2 +4
. Then

h1i
F(t) = L −1 [ f (p)] = L −1 = 1.
p
and
h 1 i 1
G(t) = L −1 [g(p)] = L −1 = sin 2t.
p2 + 4 2
∴ by convolution theorem,

h 1 i
−1 1
h 1 i
L −1 = L .
p(p2 + 4) p p2 + 4
= L −1 [ f (p).g(p)]

= F(t) ∗ G(t)
Z t
= F(u)G(t − u)du
0
Z t
h 1 i 1
L −1
= 1. sin 2(t − u)du
p(p + 4)
2
0 2

1 t
Z
h 1 i
L −1
= sin 2(t − u)du
p(p2 + 4) 2 0
1  − cos 2(t − u) 
=
2 −2
1
= (cos 0 − cos 2t)
4
h 1 i 1
L −1 = (1 − cos 2t).
p(p + 4)
2 4

Transform Techniques B.Sc.(Mathematics)-IDE(UNOM)-II Year-IV Sem


9.2. Worked out Problems 161
h i
Example 9.2.3. Find L −1 1
p(p2 +2p+2)
by using convolution theorem.

Solution. Let f (p) = 1


p
and g(p) = 1
p2 +2p+2
. Then

h1i
F(t) = L −1 [ f (p)] = L −1 = 1.
p
and
h 1 i h 1
G(t) = L −1 [g(p)] = L −1 = L −1
= e−t sin t.
p2 + 2p + 2 (p + 1)2 + 1
∴ by convolution theorem,

h 1 i
−1 1
h 1 i
L −1 = L .
p(p2 + 2p + 2) p p2 + 2p + 2
= L −1 [ f (p).g(p)]

= F(t) ∗ G(t)

= G(t) ∗ F(t)
Z t
= G(u)F(t − u)du
0
Z t
= e−u sin u.1du
0
 e−u t
= (− sin u − cos u)
1+1 0
1
= − [e−t (sin t + cos t) − 1.(0 + 1)]
2
h 1 i 1
L −1 = [1 − et (sin t + cos t)].
p(p2 + 2p + 2) 2
h i
Example 9.2.4. Find L −1 1
(p2 +a2 )2
by using convolution theorem.

Solution. Let f (p) = 1


p2 +a2
and g(p) = 1
p2 +b2
. Then

h 1 i 1
F(t) = L −1 [ f (p)] = L −1 = sin at.
p2 + a2 a
and
h 1 i 1
G(t) = L −1 [g(p)] = L −1 = sin at.
p2 + b2 a

B.Sc.(Mathematics)-IDE(UNOM)-II Year-IV Sem Transform Techniques


162 9.2. Worked out Problems

∴ by convolution theorem,

h 1 i h 1 1 i
L −1 = L −1
.
p(p2 + 2p + 2) p+ a2 p2 + b2
= L −1 [ f (p).g(p)]

= F(t) ∗ G(t)
Z t
= F(u)G(t − u)du
0
Z t
1 1
= sin au. sin a(t − u)du
0 a a
Z t
1
= 2 sin au sin(at − au)du
a 0
Z t
1 cos(2au − at) − cos at
= 2 du
a 0 2
1 h sin(2au − at) it
= − cos at.u
2a2 2a 0
1 1h 1 i
= sin at − t cos at + sin at
2a2 2a 2a
h 1 i 1
L −1 2 = [sin at − at cos at].
(p + a2 )2 2a3
h i
Example 9.2.5. Find L −1 1
(p−2)(p+2)2
by using convolution theorem.

Solution. Let f (p) = and g(p) = p−2


1
(p+2)2
1
. Then
h i h i
F(t) = L −1 [ f (p)] = L −1 (p+2)
1
2 = e−2t t. and G(t) = L −1 [g(p)] = L −1 p−2
1
= e2t .

∴ by convolution theorem,

h 1 i h 1 1 i
L −1 = L −1
.
(p − 2)(p + 2)2 (p + 2)2 p − 2
= L −1 [ f (p).g(p)]

= F(t) ∗ G(t)

Z t
h 1 i
L −1
= F(u)G(t − u)du
(p − 2)(p + 2)2 0

Transform Techniques B.Sc.(Mathematics)-IDE(UNOM)-II Year-IV Sem


9.2. Worked out Problems 163
Z t
h 1 i 1
L −1
= 1. e−2u .u.e2(t−u) du
(p − 2)(p + 2)2 0 2
Z t
= e2t ue−4u du
0
h  e−4u   e−4u it
= e2t u − 1. by integration by parts
−4 16 0
h 1 i 1
L −1 = [e2t − (4t + 1)e−2t ]
(p − 2)(p + 2)2 16
h i
Example 9.2.6. Find L −1 p
(p2 +a2 )2
by using convolution theorem.

Solution. Let f (p) = p


p2 +a2
and g(p) = 1
p2 +a2
. Then

h p i
F(t) = L −1 [ f (p)] = L −1 = cos at.
p2 + a2
and
h 1 i 1
G(t) = L −1 [g(p)] = L −1 = sin at.
p2 + a2 a
∴ by convolution theorem,

h p i h p 1 i
L −1 = L −1 2 . 2
(p2 +a )
2 2 p + a p + a2
2

= L −1 [ f (p).g(p)]

= F(t) ∗ G(t)
Z t
= F(u)G(t − u)du
0
Z t
= cos au. sin a(t − u)du
0
1 t
Z
= cos au sin(at − au)du
a 0
1 t sin(au + at − au) − sin(au − at + au)
Z
= du
a 0 2
Z t
1
= [sin at − sin(2au − at)]du
2a 0

B.Sc.(Mathematics)-IDE(UNOM)-II Year-IV Sem Transform Techniques


164 9.2. Worked out Problems

1h 1 it
=
sin at.u + cos(2au − at)
2a 2a 0
1 h 1 1 i
= t sin at + cos at − cos at
2a 2a 2a
h p i t
L −1
= sin at.
(p2 + a2 )2 2a

p2
h i
Example 9.2.7. Find L −1 (p2 +a2 )(p2 +b2 )
by using convolution theorem.

Solution. Let f (p) = p


p2 +a2
and g(p) = p
p2 +b2
. Then

h p i
F(t) = L −1 [ f (p)] = L −1 = cos at.
p2 + a2
and
h p i
G(t) = L −1 [g(p)] = L −1 = cos bt.
p2 + b2
∴ by convolution theorem,

h p2 i h p p i
L −1 = L −1
.
(p2 + a2 )(p2 + b2 ) p2 + a2 p2 + b2
= L −1 [ f (p).g(p)]

= F(t) ∗ G(t)
Z t
= F(u)G(t − u)du
0
Z t
= cos au. cos b(t − u)du
0
Z t
= cos au cos(bt − bu)du
0
cos(au + bt − bu) + cos(au − bt + bu)
Z t
= du
0 2
1 th
Z i
= cos[(a − b)u + bt] + cos[(a + b)u − bt] du
2 0
1 h sin[(a − b)u + bt] sin[(a + b)u − bt] it
= +
2 a−b a+b 0
1h 1 1 i
= (sin at − sin bt) + (sin at + sin bt)
2 (a − b) a+b

Transform Techniques B.Sc.(Mathematics)-IDE(UNOM)-II Year-IV Sem


9.2. Worked out Problems 165
1h  1 1   1 1 i
sin at + = + sin bt −
2 a−b a+b a+b a−b
1h  2a   2b i
= sin at 2 + sin bt − 2
2 a − b2 a − b2
2
h p i 1
L −1 2 = 2 (a sin at − b sin bt)
(p + a )(p + b )
2 2 2 a − b2

p2
h i
Example 9.2.8. Find L −1 (p2 +4)2
by using convolution theorem.

Solution. Let f (p) = p


p2 +4
and g(p) = p
p2 +4
. Then

h p i
F(t) = L −1 [ f (p)] = L −1 = cos 2t.
p2 + 4
and
h p i
G(t) = L −1 [g(p)] = L −1 = cos 2t.
p2 + 4
∴ by convolution theorem,

h p2 i h p p i
L −1 = L −1
.
(p2 + 4)2 p2 + 4 p2 + 4
= L −1 [ f (p).g(p)]

= F(t) ∗ G(t)
Z t
= F(u)G(t − u)du
0
Z t
= cos 2u. cos 2(t − u)du
0
Z t
= cos 2u cos(2t − 2u)du
0
cos(2u + 2t − 2u) + cos(2u − 2t + 2u)
Z t
= du
0 2
1 t
Z
= [cos 2t + cos(4u − 2t)]du
2 0

B.Sc.(Mathematics)-IDE(UNOM)-II Year-IV Sem Transform Techniques


166 9.2. Worked out Problems

1h sin(4u − 2t) it
=
cos 2t.u +
2 4 0
1 h 1 i
= t cos 2t + (sin(4t − 2t) − sin(−2t))
2 4
h p2 i 1  sin 2t 
L −1 2 = t cos 2t + .
(p + 4)2 2 2
h i
Example 9.2.9. Find L −1 p+1
(p2 +2p+2)2
by using convolution theorem.

Solution.
h p+1 i h p+1 i
L −1 = L −1
(p2 + 2p + 2)2 (p + 1)2 + 1)2
h p+1 i h p i
L −1 2 = e−t
L −1
(p + 2p + 2)2 (p2 + 1)2

by first shifting theorem.


We now evaluate
h p i
L −1
(p2 + 1)2
Let f (p) = p
p2 +1
and g(p) = 1
p2 +1
. Then

h p i
F(t) = L −1 [ f (p)] = L −1 = cos t.
p2 + 1
and
h 1 i
G(t) = L −1 [g(p)] = L −1 = sin t.
p2 + 1
∴ by convolution theorem,

h p i h p 1 i
L −1 = L −1
.
(p2 + 1)2 p2 + 1 p2 + 1
= L −1 [ f (p).g(p)]

= F(t) ∗ G(t)
Z t
= F(u)G(t − u)du
0
Z t
= cos u. sin(t − u)du
0

Transform Techniques B.Sc.(Mathematics)-IDE(UNOM)-II Year-IV Sem


9.2. Worked out Problems 167
Z t
= cos u sin(t − u)du
0
Z t
sin t − sin(2u − t)
= du
0 2
1h cos(2u − t) it
= sin t.u +
2 2 0
1 h 1 i
= t sin t + (cos t − cos t)
2a 2
h p i t
L −1
= sin t.
(p2 + 1)2 2

h p+1 i t
∴ L −1 = e−t . sin t.
(p + 2p + 2)
2 2 2
h i
Example 9.2.10. Find L −1 1
p(p2 +4)2
by using convolution theorem.

Solution. Let f (p) = 1


p2 +4
and g(p) = 1
p2 +4
. Then

h 1 i 1
F(t) = L −1 [ f (p)] = L −1 = sin 2t.
p2 + 4 2
and
h 1 i 1
G(t) = L −1 [g(p)] = L −1 = sin 2t.
p2 + 4 2
∴ by convolution theorem,

h 1 i h 1 1 i
L −1 = L −1
.
(p2 + 4)2 p2 + 4 p2 + 4
= L −1 [ f (p).g(p)]

h1 i
L −1 = F(t) ∗ G(t)
(p2 + 4)2
Z t
h 1 i
L −1
= F(u)G(t − u)du
(p2 + 4)2 0
1 t
Z
= sin 2u. sin 2(t − u)du
4 0
1 t
Z
= sin 2u sin(2t − 2u)du
4 0

B.Sc.(Mathematics)-IDE(UNOM)-II Year-IV Sem Transform Techniques


168 9.2. Worked out Problems

1 t cos(4u − 2t) − cos 2t


Z
= du
4 0 2
1 t
Z
= [cos(4u − 2t) − cos 2t]du
8 0
1 h sin(4u − 2t) it
= − cos 2t.u
8 4 0
1 sin 2t
h  sin(−2t) i
= − t cos 2t − −0
8 4 4
1 h 2 sin 2t i
= − t cos 2t
8 4
h 1 i sin 2t t cos 2t
L −1 = − .
(p2 + 4)2 16 8

Z t
h 1 i sin 2x x cos 2x 
L −1
= − dx
p(p2 + 4)2 0 16 8
h cos 2x x 1 it
= − − sin 2x − cos 2x
32 16 32 0
cos 2t t 1 1 1
= − − sin 2t − cos 2t + +
32 16 32 32 32
1 t 1
= − cos 2t − sin 2t +
16 16 16
h 1 i 1
L −1 = (1 − cos 2t − t sin 2t).
p(p2 + 4)2 16
h i
Example 9.2.11. Prove that L −1 8
(p2 +1)3
= (3 − t2 )2 sin t − 3t cos t.

Solution. We have
h 8 i h 1 1 i
L −1 = 8L −1
.
(p2 + 1)3 (p2 + 1)2 p2 + 1
First we find
h 1 i
L −1
(p2 + 1)2
by using convolution theorem.

Let f (p) = 1
p2 +1
and g(p) = 1
p2 +1
. Then

Transform Techniques B.Sc.(Mathematics)-IDE(UNOM)-II Year-IV Sem


9.2. Worked out Problems 169

h 1 i
F(t) = L −1 [ f (p)] = L −1 = sin t.
p2 + 1
and
h 1 i
G(t) = L −1 [g(p)] = L −1 = sin t.
p2 + 1
∴ by convolution theorem,

h 1 i h 1 1 i
L −1 = L −1
.
(p2 + 1)2 p2 + 1 p2 + 1
= L −1 [ f (p).g(p)]

= F(t) ∗ G(t)
Z t
= F(u)G(t − u)du
0
Z t
= sin u. sin(t − u)du
0
Z t
cos(2u − t) − cos t
= du
0 2
1 h sin(2u − t) it
= − cos t.u
2 2 0
1h1 i
= (sin(2t − t) − t cos t)
2 2
h 1 i 1
L −1 2 = (sin t − t cos t).
(p + 1)2 2

Again using convolution theorem, taking f (p) = 1


(p2 +1)2
and g(p) = 1
p2 +1
, we have

h 1 i h 1 1 i
L −1 = L −1
.
(p2 + 1)3 (p2 + 1)2 p2 + 1
Z t
1
= [sin u − u cos u] sin(t − u)du
0 2
1 t
Z
= [sin u sin(t − u) − u(cos u sin(t − u))]du
2 0
1 t h cos(2u − t) − cos t i
Z h sin t + sin(2u − t) i
= −u du
2 0 2 2
1 t
Z
h 1 i
L −1
= [cos(2u − t) − cos t − u sin t + u sin(2u − t)]du
(p2 + 1)3 4 0

B.Sc.(Mathematics)-IDE(UNOM)-II Year-IV Sem Transform Techniques


170 9.2. Worked out Problems

h 1 i 1 h sin(2u − t)  u2  u cos(2u − t) sin(2u − t) it


L −1 = − cos t.u − sin t − +
(p2 + 1)3 4 2 2 2 4 0

1h  t2  t cos t sin t i
= sin t − t cos t − sin t − +
4 2 2 2
1
= (3 sin t − 3t cos t − t2 sin t)
8
h 8 i
L −1 2 = (3 − t2 ) sin t − 3t cos t.
(p + 1)3
h i
Example 9.2.12. Find (i) L −1 √ 1
by the convolution integral and hence deduce the value
p(p−a)
 i
of p √1p+a . (ii) Applying convolution theorem, find L −1 √1 .
p p+1

Solution. Let f (p) = √1


p
and g(p) = 1
p−a
. Then

1
h 1 i t2 1
F(t) = L [ f (p)] = L
−1
√ = 1 = √ √.
−1
p Γ( 2 ) π t

and
h 1 i
G(t) = L −1 [g(p)] = L −1 = eat .
p−a
∴ by convolution theorem,

h 1 i h 1 1 i
L −1 √ = L −1 √ .
p(p − a) p p−a
= L −1 [ f (p).g(p)]

= F(t) ∗ G(t)
Z t
= F(u)G(t − u)du
0
Z t
1
= √ √ ea(t−u) du
0 π u
at Z t −au
e e
= √ √ du
π 0 u

Transform Techniques B.Sc.(Mathematics)-IDE(UNOM)-II Year-IV Sem


9.2. Worked out Problems 171

Put au = x2 ⇒ adu = 2xdx u = 0 ⇒ x = 0 and u = t ⇒ x = at.

at 2
e−at e−x 2x
Z
h 1 i
L −1
√ = √ dx
π
q
p(p − a) 0 x2 a
a
at Z t
h 1 i 2e 2
L −1 √ = √ √ e−x dx
p(p − a) a π 0
h 1 i eat √
L −1 √ = √ er f ( at).
p(p − a) a

Deduction:

h1 i h 1 i
L −1 √ = L −1 √
p p+a (p + a − a) p + a
h 1 i
= e−at L −1 √ by f irst shi f ting theorem
(p − a) P
eat √
= e−at √ er f ( at)
a
h 1 i 1 √
L −1 √ = √ er f ( at).
p p+a a

(ii) Putting a = 1 in the above result, we get

h 1 √
L −1 = er f ( t).
p p+1
p

h i
Example 9.2.13. Find L −1 1
(p−2)(p2 +1)
by using convolution theorem.

Solution. Let f (p) = 1


(p+2)2
and g(p) = 1
p−2
. Then

h 1 i
F(t) = L −1 [ f (p)] = L −1 = e2t .
p−2
and
h 1 i
G(t) = L −1 [g(p)] = L −1 = sin t.
p2 + 1

B.Sc.(Mathematics)-IDE(UNOM)-II Year-IV Sem Transform Techniques


172 9.3. Exercises

∴ by convolution theorem,

h 1 i h 1 1 i
L −1 = L −1
.
(p − 2)(p2 + 1) p − 2 p2 + 1
= L −1 [ f (p).g(p)]

= F(t) ∗ G(t)
Z t
= F(u)G(t − u)du
0
Z t
h 1 i
L −1 = e2u sin(t − u)du
(p − 2)(p2 + 1) 0

h 1 i  e2u t
L −1 = [2 sin(t − u) − cos(t − u)]
(p − 2)(p2 + 1) 5 0
Z ax
h e i
S ince eax sin(bx + c) = 2 [a sin(bx + c) − b cos(bx + c)]
a + b2
h 1 i e2t e0
L −1 = [2 sin 0 − cos 0] − [2 sin t − cos t]
(p − 2)(p2 + 1) 5 5
h 1 i 1
L −1 = (cos t − 2 sin t − e2t )
(p − 2)(p + 1)
2 5

9.3 Exercises

Using convolution theorem, evaluate the following:


h i h i h i
1. L −1 (p−1)(p+2)
1
; 2. L −1 p(p21−a2 ) ; 3. L −1 p2 (p12 −a2 ) ;
h i h p i h i
4. L −1 (p2 +9)(p+3)
1
; 5. L −1 (p2 +1) 2 ; 6. L −1 p(p2 p−a2 )2 ;
h i h p2 i h p i
7. L −1 (p2 +1)
1
3 ; 8. L −1
(p2 +a2 )2
; 9. L −1
p(p2 +a2 )3
;
h p2 i h i h i
10. L −1 p4 −a4 ; 11. L −1 p(p21+4)2 ; 12. L −1 p3 (p+1)
1
;
h i h i h i
13. L −1 (p2 +1)(p+1)
1
; 14. L −1 (p+2)12 (p−2) ; 15. L −1 (p2 +1)(p p
2 +4) ;

Transform Techniques B.Sc.(Mathematics)-IDE(UNOM)-II Year-IV Sem


9.3. Exercises 173

Answers
1. 13 (et − e−2t ); 2. 1
a2
(cosh at − 1);→

3. 1
a3
(sinh at − at); 4. 1
18
(sin 3t − cos 3t + e−3t );
t t
5. 2
sin t; 6. 2a
sinh at;
7. 81 [(3 − t2 ) sin t − 3t cos t]; 8. 1
2a
(sin at + at cos at);
9. t
8a3
(sin at − at cos at); 10. 1
2a
(sin at + sinh at);
t2
11. 1
16
(1 − t sin 2t − cos 2t); 12. 1 − t + 2
− e−t ;
13. 12 (sin t − cos t + e−t ); 14. 1
16
[e2t −)(4t + 1)e−2t ];
15. 31 (cos t − cos 2t).

B.Sc.(Mathematics)-IDE(UNOM)-II Year-IV Sem Transform Techniques


174
UNIT-III

Lesson 10

FOURIER SERIES
Learning Objectives
Upon completion of this lesson, students will be able to

• define Fourier series and the conditions for Fourier Expansions

• find the method for obtaining Fourier series in the interval [c, c + 2π]

• find Fourier series of functions in the interval [c, c + 2π].

10.1 Introduction

F
ourier series is an infinite series representation of a function in terms of sines and cosines of
an angle and its multiples. We know that Taylor’s series expansion is valid only functions
which are continuous and differentiable. But Fourier series is possible for continuous functions,
periodic functions and functions discontinuous in their values and derivatives. Fourier series
is useful to solve ordinary and partial differential equations particularly with periodic functions
appearing as non - homogeneous terms.

Suppose that a given function f (x) defined in [−π, π] or [0, 2π] or in any other interval can be
expressed as a trigonometric series as

a0
f (x) = + a1 cos x + a2 cos 2x + .... + an cos nx + ... + b1 sin x + b2 sin 2x + .... + bn sin nx + ...
2

B.Sc.(Mathematics)-IDE(UNOM)-II Year-IV Sem 175 Transform Techniques


176 10.2. Periodic Function

a0
f (x) = + (a1 cos x + b1 sin x) + (a2 cos 2x + b2 sin 2x) + (a3 cos 3x + b3 sin 3x)
2
+ ..... + (an cos nx + bn sin nx) + ...

a0 X
f (x) = + (an cos nx + bn sin nx)
2 n=1

where the a0 s and b0 s are constants within a desired range of values of the variable. Such series
is known as the Fourier series for f (x) and the constants a0 , an and bn (n = 1, 2, 3, ..) are
called Fourier coefficients of f (x).

10.2 Periodic Function

A function f (x) is said to be of period of T or to be periodic with period T > 0 if for all real x,

f (x + T ) = f (x)

and T is the least of all such values.


In other words, if the value of each ordinate f (x) repeats itself at equal intervals in the abscissa,
f (x) is said to be a periodic function. If

f (x) = f (x + T ) = f (x + 2T ) = ....

then T is called the period of the function f (x).

Example.1 Since
sin x = sin(x + 2π) = sin(x + 4π) = ...

the function sin x is periodic with period 2π. There is no positive value T, 0 < T < 2π such
that sin(x + T ) = sin x for all x.

Example.2 The period of tan x is π.


 
Example.3 sin 3x = sin(2π + 3x) = sin 3 2π
3
+ x . Note that sin 3x is periodic with period 2π
3
.

Transform Techniques B.Sc.(Mathematics)-IDE(UNOM)-II Year-IV Sem


10.3. Euler’s Formulae 177

Note. 1 If T is the period of f (x) then nT is also period of f for any integer n.
2. If T is period of f (x) and g(x) then the function h(x) = a f (x) + bg(x) has period T , where
a and b are constants.
3. If f (x) is a periodic function of period T, then f (ax) with a , 0, is a periodic function of
period T
a
.
4. The period of sum of a number of periodic functions is the L.C.M of the periods.
5. A constant function is periodic for any positive T.

10.3 Euler’s Formulae

Theorem 10.3.1. If f (x) is a periodic function with period 2π and if f (x) can be represented
by trigonometric series in (−∞, ∞)


a0 X
f (x) = + (an cos nx + bn sin nx)
2 n=1

then

1 c+2π
Z
a0 = f (x)dx
π c
1 c+2π
Z
an = f (x) cos nxdx (10.3.1)
π c
1 c+2π
Z
bn = f (x) sin nxdx
π c

These values of a0 , an bn are known as Euler’s formulae.

Proof. Let f (x) be represented in the interval [c, c + 2π] by the Fourier series

∞ ∞
a0 X X
f (x) = + an cos nx + bn sin nx (10.3.2)
2 n=1 n=1

Let us assume that the series is uniformly convergent in the interval c ≤ x ≤ c + 2π. Then the

B.Sc.(Mathematics)-IDE(UNOM)-II Year-IV Sem Transform Techniques


178 10.3. Euler’s Formulae

series can be integrated term by term.

To evaluate a0 . Integrating both sides of (16.3.2) from x = c to x = c + 2π, we get

Z c+2π Z c+2π ∞ Z c+2π ∞ Z c+2π


a0 X X
f (x)dx = dx + an cos nxdx + bn sin nxdx
c 2 c n=1 c n=1 c

Z c+2π ∞
X h  sin nx c+2π  cos nx c+2π i
a0
f (x)dx = (c + 2π − c) + an + bn −
c 2 n=1
n c n c
X∞
= a0 π + [an .0 − bn .0]
n=1
Z c+2π
f (x)dx = a0 π
c
Z c+2π
1
Hence a0 = f (x)dx.
π 0

To evaluate an :
Multiplying both sides of (16.3.2) by cos mx and then integrating from x = c to x = c + 2π, we
get

c+2π ∞
a0 c+2π
Z Z X Z c+2π
f (x) cos mxdx = cos mxdx + an cos nx cos mxdx
c 2 c n=1 c
X∞ Z c+2π
+ bn sin nx cos mxdx
n=1 c

The first and third integral on the right - hand side are always zero, but second integral is equal to

Transform Techniques B.Sc.(Mathematics)-IDE(UNOM)-II Year-IV Sem


10.3. Euler’s Formulae 179

π when m = n; otherwise it also vanishes when m , n. Hence


Z c+2π ∞ Z c+2π
1 Xh
f (x) cos mxdx = an cos2 nxdx
c 2 n=1 c
Z c+2π
an
= (1 + cos 2nx)dx
2 c
an  sin 2nx c+2π
= x+
2 2n c
an
= (2π)
2
1 c+2π
Z
∴ an = f (x) cos nxdx.
π 0

To evaluate bn : Multiplying both sides of (16.3.2) by sin mx and then integrating from x = c to
x = c + 2π, we get

c+2π ∞
a0 c+2π
Z Z X Z c+2π
f (x) sin mxdx = sin mxdx + an cos nx sin mxdx
c 2 c n=1 c
X∞ Z c+2π
+ bn sin nx sin mxdx
n=1 c

The first two integral on the right - hand side are always zero, but third integral is equal to π when
m = n; otherwise it also vanishes when m , n. Hence
Z c+2π ∞ Z c+2π
1 Xh
f (x) sin nxdx = bn 2 sin2 nxdx
c 2 n=1 c
Z c+2π
bn
= (1 − cos 2nx)dx
2 c
an  sin 2nx c+2π
= x−
2 2n c
bn
= (2π)
2
Z c+2π
1
∴ bn = f (x) sin nxdx.
π 0

The results a0 , an and bn are called Euler’s formulae. 

B.Sc.(Mathematics)-IDE(UNOM)-II Year-IV Sem Transform Techniques


180 10.4. Conditions for Fourier Expansion

Corollary 10.3.1. If f (x) is to be expanded as a Fourier series in the interval 0 ≤ x ≤ 2π, put
c = 0 then the Euler’s formulae reduces to

1 2π
Z
a0 = f (x)dx
π 0
1 2π
Z
an = f (x) cos nxdx (10.3.3)
π 0
1 2π
Z
bn = f (x) sin nxdx
π 0

Corollary 10.3.2. If f (x) is to be expanded as a Fourier series in the interval [−π, π], put c = −π
then the Euler’s formulae reduces to

1 π
Z
a0 = f (x)dx
π −π
1 π
Z
an = f (x) cos nxdx (10.3.4)
π −π
Z π
1
bn = f (x) sin nxdx
π −π

In many of the problems, we come across expansions of f (x) in [0, 2π] or [−π, π] and hence
formulae (16.3.3) and (10.3.4) to be remembered carefully.

Note. Because of the periodic nature, Fourier series constructed for one period is valid for all
values.

10.4 Conditions for Fourier Expansion

Dirichlet has formulated certain conditions known as Dirichlet conditions under which certain
functions possess valid Fourier expansions.
A function f (x) has a valid Fourier series expansion of the form


a0 X
+ (an cos nx + bn sin nx)
2 n=1

Transform Techniques B.Sc.(Mathematics)-IDE(UNOM)-II Year-IV Sem


10.4. Conditions for Fourier Expansion 181

where a0 , an and bn are constants,provided


(i) f (x) is well defined, periodic, single - valued function and finite;
(ii) f (x) has a finite number of finite discontinuities in any one period;
(iii) f (x) has at most a finite number of maxima and minima in the interval of definition.

Note. The above conditions are sufficient but not necessary.

Some Important Results:

1. 




0, for m , n
Z π 


cos mx cos nxdx = 

π, for m = n > 0


−π 



for m = n = 0


2π,

2. 
for m , n and m = n = 0

π
0,
Z 


sin mx sin nxdx = 


−π
π, for m = n > 0


3. sin nπ = 0, sin 2nπ = 0, for nZ, where Z is the set of all integers.
4. cos nπ = (−1)n , cos 2nπ = 1, nZ.
 
5. sin n + 12 π = (−1)n , nZ.
 
6. cos n + 12 π = 0, nZ.
7. 
n−1
(−1) 2 ,

if n is odd

nπ 

=

sin
2 

0,

 if n is even

8. 

0, if n is odd

nπ 

=

cos
2  n
(−1) 2 , if n is even


B.Sc.(Mathematics)-IDE(UNOM)-II Year-IV Sem Transform Techniques


182 10.5. Worked out Problems

Method of Obtaining Fourier Series of f (x) in the interval [c, c + 2π] :

Step.1 Fourier series of f (x) is given by

∞ ∞
a0 X X
f (x) = + an cos nx + bn sin nx
2 n=1 n=1

Step.2 Compute a0 and an using the formulae


Z c+2π
1
a0 = f (x)dx
π c

and Z c+2π
1
an = f (x) cos nxdx; n = 1, 2, 3, ...
π c

Step.3 Compute bn using the formula


Z c+2π
1
bn = f (x) sin nxdx; n = 1, 2, 3, ...
π c

Step.4 Substitute the values of a0 , an and bn in f (x) in Step.1 to get the required Fourier series.

10.5 Worked out Problems

Example 10.5.1. Express f (x) = x − π as Fourier series in the interval −π < x < π.

Solution. Let f (x) = x − π be represented by the Fourier series

∞ ∞
a0 X X
x−π= + an cos nx + bn sin nx (10.5.1)
2 n=1 n=1

1 π
Z
a0 = f (x)dx
π −π
1 π
Z
= (x − π)dx
π −π

Transform Techniques B.Sc.(Mathematics)-IDE(UNOM)-II Year-IV Sem


10.5. Worked out Problems 183

1h π
Z Z π i
= xdx − π dx
π −π −π
Z π
1h i
= 0 − π.2 dx since x is an odd function
π 0
1
= (−2π)(x)π0
π
a0 = −2π

1 π
Z
an = f (x) cos nxdx
π −π
1 π
Z
= (x − π) cos nxdx
π −π
1h π
Z Z π i
= x cos nxdx − π cos nxdx
π −π −π
Z π
1h i
= 0 − π.2 cos nxdx since x cos nx is an odd function
π 0
1  sin nx π
= (−2π)
π n 0
an = −2(sin nπ − sin 0)

an = 0.

1 π
Z
bn = f (x) sin nxdx
π −π
1 π
Z
= (x − π) sin nxdx
π −π
1h π
Z Z π i
= x sin nxdx − π sin nxdx
π −π −π
Z π
1h i
bn = 2 x sin nxdx − π(0) since sin nx is an odd function
π 0
2 h  − cos nx   − sin nx iπ
bn = x − 1.
π n n2 0
2 π cos nπ
h  i −2
= + 0 − (0 + 0) = cos nπ
π n π

B.Sc.(Mathematics)-IDE(UNOM)-II Year-IV Sem Transform Techniques


184 10.5. Worked out Problems

−2
bn = (−1)n
π
2
bn = (−1)n+1 , for n = 1, 2, 3, ..
π

Substituting theses values of a0 , an and bn in (19.4.1),we get the required Fourier series of f (x)
in (−π, π) as


X 2
x − π = −π + (−1)n+1 sin nx
n=1
n
 1 1 1 i
x − π = −π + 2 sin x − sin 2x + sin 3x − sin 4x + ..... .
2 3 4

Example 10.5.2. Express f (x) = π − x as Fourier series in the interval (0, 2π).

Solution. Let f (x) = x − π be represented by the Fourier series

∞ ∞
a0 X X
x−π= + an cos nx + bn sin nx (10.5.2)
2 n=1 n=1

1 2π
Z
a0 = f (x)dx
π 0
1 2π
Z
= (π − x)dx
π 0
1h x2 i2π
= πx −
π 2 0
a0 = 0.

1 2π
Z
an = f (x) cos nxdx
π 0
1 2π
Z
= (π − x) cos nxdx
π 0
1h  sin nx   − cos nx i2π
= (π − x) − (−1)
π n n2 0

an = 0.

Transform Techniques B.Sc.(Mathematics)-IDE(UNOM)-II Year-IV Sem


10.5. Worked out Problems 185

1 2π
Z
bn = f (x) sin nxdx
π 0
1 2π
Z
= (π − x) sin nxdx
π 0
1h  − cos nx   − sin nx i2π
= (π − x) − (−1)
π n n2 0
1 h cos 2nπ 1  i
= − (−π) + (π) + 0 + 0
π n n
1 h 2π i
=
π n
2
bn =
n

Substituting theses values of a0 , an and bn in (10.5.2),we get the required Fourier series of f (x)
in (−π, π) as


X 2
π−x = sin nx
n=1
n
 1 1 1 
π − x = 2 sin x + sin 2x + sin 3x + sin 4x + ..... .
2 3 4

Example 10.5.3. Find a Fourier series to represent f (x) = x2 in the interval (0, 2π).

Solution. Let f (x) = x2 be represented by the Fourier series

∞ ∞
a0 X X
x2 = + an cos nx + bn sin nx (10.5.3)
2 n=1 n=1

1 2π
Z
a0 = f (x)dx
π 0
1 2π 2
Z
= x dx
π 0
1 h x3 i2π
=
π 3 0
8 2
a0 = π.
3

B.Sc.(Mathematics)-IDE(UNOM)-II Year-IV Sem Transform Techniques


186 10.5. Worked out Problems
Z 2π
1
an = f (x) cos nxdx
π 0

1 2π 2
Z
an = x cos nxdx
π 0
1 h 2  sin nx   − cos nx   − sin nx i2π
= x − 2x +2
π n n2 n3 0
1h 1 i
= 0 + 2(2π) 2 + 0 − (0 + 0 + 0)
π n
1  4π 
=
π n2
4
an = 2 .
n

1 2π
Z
bn = f (x) sin nxdx
π 0
1 2π 2
Z
= x sin nxdx
π 0
1 h 2  − cos nx   − sin nx   cos nx i2π
= x − 2x + 2
π n n2 n3 0
1 h 1 1  2 i
= − 4π2 . + 0 + 2 3 − 0 + 0 + 3
π n n n
1  −4π2 
=
π n
−4π
bn = .
n

Substituting theses values of a0 , an and bn in (10.5.3),we get the required Fourier series of f (x)
in (−π, π) as

∞ ∞
4 2 X 4 X 4π
x2 = π + 2
cos nx − sin nx
3 n=1
n n=1
n
4  1 1   1 1 
x2 = π2 + 4 cos x + 2 cos 2x + 2 cos 3x + ..... − 4π sin x + sin 2x + sin 3x + ...
3 2 3 2 3

Example 10.5.4. Determine the Fourier series expansion of the function f (x) = 1
12
(3x2 −6xπ+2π2 )
in the interval (0, 2π).

Transform Techniques B.Sc.(Mathematics)-IDE(UNOM)-II Year-IV Sem


10.5. Worked out Problems 187

Solution.
∞ ∞
1 a0 X X
(3x2 − 6xπ + 2π2 ) = + an cos nx + bn sin nx (10.5.4)
12 2 n=1 n=1

Z 2π
1
a0 = f (x)dx
π 0

1 2π 1
Z
a0 = (3x2 − 6xπ + 2π2 )dx
π 0 12
1 h 3 i2π
= x − 3x2 π + 2π2 x
12π 0
1
a0 = (8π3 − 12π3 + 4π3 )
12π
a0 = 0.

1 2π
Z
an = f (x) cos nxdx
π 0
1 2π 1
Z
= (3x2 − 6xπ + 2π2 ) cos nxdx
π 0 12
1 h 2  sin nx   − cos nx   − sin nx i2π
= (3x − 6xπ + 2π2 ) − (6x − 6π) + 6
12π n n2 n3 0
1 h 1 1 i
= 0 + 6π 2 + 0 − (0 − 6π 2 )
12π n n
1  12π 
=
12π n2
1
an = .
n2

1 2π
Z
bn = f (x) sin nxdx
π 0
1 2π 1
Z
= (3x2 − 6xπ + 2π2 ) sin nxdx
π 0 12

B.Sc.(Mathematics)-IDE(UNOM)-II Year-IV Sem Transform Techniques


188 10.5. Worked out Problems

1 h 2  − cos nx   − sin nx   cos nx i2π


= (3x − 6xπ + 2π2 ) − (6x − 6π) + 6
12π n n2 n3 0
1 h 1 1  1 6 i
= − 2π2 . + 0 + 6 3 − − 2π2 + 0 + 3
12π n n n n
bn = 0

Substituting theses values of a0 , an and bn in (10.5.4), we get the required Fourier series of f (x)
in (−π, π) as


1 X 1
(3x − 6xπ + 2π ) =
2 2
cos nx.
12 n=1
n2

Example 10.5.5. Determine the Fourier series expansion of the function f (x) = e x in the interval
(0, 2π).

Solution.
∞ ∞
a0 X X
e =
x
+ an cos nx + bn sin nx (10.5.5)
2 n=1 n=1

1 2π
Z
a0 = f (x)dx
π 0
1 2π x
Z
= e dx
π 0
1 x 2π
= (e )0
π
1 2π
a0 = (e − 1).
π

1 2π
Z
an = f (x) cos nxdx
π 0
1 2π x
Z
= e cos nxdx
π 0
1 h ex i2π
= (cos nx + n sin nx)
1π 1 + n2 0

Transform Techniques B.Sc.(Mathematics)-IDE(UNOM)-II Year-IV Sem


10.5. Worked out Problems 189

1 h e2π 1 i
= (cos 2nπ + 0 − (1 + 0)
π 1 + n2 1 + n2

e −1
an = .
π(1 + n2 )

1 2π
Z
bn = f (x) sin nxdx
π 0
1 2π x
Z
= e sin nxdx
π 0
1 h ex i2π
= (sin nx − n cos nx)
π 1 + n2 0

1 e
h 1 i
= (0 − n) − (0 − n)
π 1 + n2 1 + n2
e2π − 1
bn = (−n) .
π(1 + n2 )

Substituting theses values of a0 , an and bn in (10.5.5), we get the required Fourier series of f (x)
in (−π, π) as

∞ ∞
1 2π X e2π − 1 X e2π − 1
ex = (e − 1) + cos nx + (−n) sin nx
2π n=1
π(1 + n 2)
n=1
π(1 + n2)

2π ∞ ∞
e −1 1h X cos nx X n sin nx i
ex = + − .
π 2 n=1 1 + n2 n=1 1 + n2

Example 10.5.6. Find the Fourier series representing f (x) = x, 0 < x < 2π. Sketch the graph of
f (x) from −4π to 4π.

Solution.
∞ ∞
a0 X X
x= + an cos nx + bn sin nx (10.5.6)
2 n=1 n=1

B.Sc.(Mathematics)-IDE(UNOM)-II Year-IV Sem Transform Techniques


190 10.5. Worked out Problems

1 2π
Z
a0 = f (x)dx
π 0
1 2π
Z
= xdx
π 0
1  x2 2π
=
π 2 0
a0 = 2π.

1 2π
Z
an = f (x) cos nxdx
π 0
1 2π
Z
= x cos nxdx
π 0
1 h  sin nx   − cos nx i2π
= x − 1.
1π n n2 0
1 h 1  1 i
= 0+ 2 − 0+ 2
π n n
an = 0.

1 2π
Z
bn = f (x) sin nxdx
π 0
1 2π
Z
= x sin nxdx
π 0
1 h  − cos nx   − sin nx i2π
bn = x − 1.
1π n n2 0

1 h 2π i
bn = − + 0 − (0 + 0)
π n
2
bn = −
n

Substituting theses values of a0 , an and bn in (10.5.6), we get the required Fourier series of f (x)
in (−π, π) as

Transform Techniques B.Sc.(Mathematics)-IDE(UNOM)-II Year-IV Sem


10.5. Worked out Problems 191


X 1
x = π−2 sin nx
n=1
n
 1 1 
x = π − 2 sin x + sin 2x + sin 3x + ... .
2 3

Example 10.5.7. Find a Fourier series to represent f (x) = x − x2 in the interval (−π, π). Hence
show that

X (−1)n−1 π2
=
n=1
n2 12

(or)
1 1 1 π2
+ + + ..... = .
12 22 32 12
Solution. Let f (x) = x − x2 be represented by the Fourier series

∞ ∞
a0 X X
x−x =
2
+ an cos nx + bn sin nx (10.5.7)
2 n=1 n=1

1 π
Z
a0 = f (x)dx
π π
1 π
Z
= (x − x2 )dx
π −π
1h π
Z Z 2π
= xdx − x2 dx
π −π
Z π −π
1h
= 0−2 x2 dx
π 0
1  x3 π
= (−2).
π 3 0
3
1  2π
a0 = −
π 3
2π2
a0 = − .
3

B.Sc.(Mathematics)-IDE(UNOM)-II Year-IV Sem Transform Techniques


192 10.5. Worked out Problems

1 π
Z
an = f (x) cos nxdx
π −π
Z π
1
= (x − x2 ) cos nxdx
π −π
1h π
Z Z π i
= x cos nxdx − x2 cos nxdx
π −π π
Z π
1h i
= 0−2 x2 cos nx
π 0
2 h 2  sin nx   − cos nx   − sin nx iπ
= − x − 2x + 2
π n n2 n3 0
2 h cos nπ i
= − 0 + 2(π) 2 + 0 − (0 + 0 + 0)
π n
2  2π(−1)n 
= − (n , 0)
π n2
4(−1)n+1
an = .
n2

1 2π
Z
bn = f (x) sin nxdx
π −π
1 2π
Z
= (x − x2 ) sin nxdx
π −π
1h π
Z Z π i
= x sin nxdx − x2 sin nxdx
π −π −π
Z π
1h i
= 2. x sin nxdx − 0
pi 0
2 h  − cos nx   −sinnx iπ
= x − 1.
pi n n2 0
n
2 h (−1) i
= −π + 0 − (0 + 0)
π n
(−1)n
= −
n
(−1)n+1
bn = .
n

Substituting theses values of a0 , an and bn in (10.5.7), we get the required Fourier series of f (x)
in (−π, π) as

Transform Techniques B.Sc.(Mathematics)-IDE(UNOM)-II Year-IV Sem


10.5. Worked out Problems 193

∞ ∞
−π2 X 4(−1)n+1 X 2(−1)n+1
x−x = 2
+ cos nx + sin nx
3 n=1
n2 n=1
n

−π2  1 1   1 1 
x − x2 = + 4 cos x − 2 cos 2x + 2 cos 3x − ..... + 2π sin x − sin 2x + sin 3x − ... .
3 2 3 2 3

Deduction. x = 0 is a point of discontinuity of f (x). Hence the Fourier series of f (x) at x = 0


converges to f (0).
Putting x = 0 in the above series, we have

−π2  1 1 1 
0 =
+ 4 1 − 2 + 2 − 2 + ...
3 2 3 4
 1 1 1  π2
i.e., 1 − 2 + 2 − 2 + ... =
2 3 4 12

X (−1) n−1
π2
or 2
= .
n=1
n 12

Example 10.5.8. Obtain the Fourier series to represent f (x) = 1


4
(π − x)2 in the interval
0 < x < 2π.
2
π−x π2

(or) If f (x) = in the interval (0, 2π), show that f (x) = + .
P∞ cos nx
2 12 n=1 n2
Hence obtain

1 1 1 1 π2
− + − + ... = .
12 22 32 42 12

(π−x)2
Solution. Let f (x) = 4
be represented by the Fourier series

∞ ∞
(π − x)2 a0 X X
= + an cos nx + bn sin nx (10.5.8)
4 2 n=1 n=1

B.Sc.(Mathematics)-IDE(UNOM)-II Year-IV Sem Transform Techniques


194 10.5. Worked out Problems

1 2π
Z
a0 = f (x)dx
π 0
1 2π (π − x)2
Z
= dx
π 0 4
1 h (π − x)3 i2π
= −
4π 3 0
1
= − [(−π)3 − π3 ]
12π
π2
a0 = .
6

Z 2π
1
an = f (x) cos nxdx
π 0

1 2π (π − x)2
Z
an = cos nxdx
π 0 4
1h  sin nx   − cos nx   − sin nx i2π
= (π − x)2 − 2(π − x)(−1) + 2
4π n n2 n3 0
1h 1 1 i
= 0 + 2(π) 2 + 0 − (0 − 2π. 2 + 0)
4π n n
1  4π 
= (n , 0)
4π n2
1
an = , i f n , 0.
n2

1 2π
Z
bn = f (x) sin nxdx
π 0
1 2π (π − x)2
Z
= sin nxdx
π 0 4
1h  − cos nx   − sin nx   cos nx i2π
= (π − x)2 − 2(π − x)(−1) + 2
4π n n2 n3 0
1 h 1 2  1 2 i
= − π2 + 3 − − π2 . + 0 + 3
4π n n n n
1
= (0) = 0.

Transform Techniques B.Sc.(Mathematics)-IDE(UNOM)-II Year-IV Sem


10.5. Worked out Problems 195

Substituting theses values of a0 , an and bn in (10.5.8), we get the required Fourier series of f (x)
in (0, 2π) as


(π − x)2 π2 X 1
= + cos nx
4 12 n=1 n2
(π − x)2 π2 1 1
= + cos x + 2 cos 2x + 2 cos 3x + .....
4 12 2 3

Deduction.
Putting x = π in the above series, we have

π2  1 1 1 
0 =
− 1 − 2 + 2 − 2 + ...
12 2 3 4
1 1 1 π2
i.e., 1 − 2 + 2 − 2 + ...... = .
2 3 4 12

Example 10.5.9. Obtain the Fourier series expansion of f (x) given that f (x) = (π − x)2 in the
π2
interval 0 < x < 2π. Hence show that 1
12
+ 1
22
+ 1
32
+ 1
42
+ ... = 6
.

Solution. From the above problem, we have

h π2 1 1 i
(π − x)2 = 4 + cos x + cos 2x + cos 3x + .....
12 22 32

Deduction: Putting x = 0, we have

π4 π2 1 1 1
= + 2 + 2 + 2 + .....
4 12 1 2 3
1 1 1 π2 π2
2
+ 2+ 2 + ..... = −
1 2 3 4 12
1 1 1 π2
2
+ 2+ 2 + ..... = .
1 2 3 6

Example 10.5.10. Find the Fourier series to represent the function f (x) = e−ax in the interval
x = −π to x = π. Deduce from that

π h 1 1 1 i
=2 2 − 2 + 2 − ....
sinh π 2 +1 3 +1 4 +1

B.Sc.(Mathematics)-IDE(UNOM)-II Year-IV Sem Transform Techniques


196 10.5. Worked out Problems

Solution.
∞ ∞
a0 X X
e−ax
= + an cos nx + bn sin nx (10.5.9)
2 n=1 n=1

1 π
Z
a0 = f (x)dx
π −π
1 π −ax
Z
= e dx
π −π
1  e−ax π
=
π −a −π
1
= − (e−aπ − a−aπ )

1
a0 = 2 sinh aπ.

1 π
Z
an = f (x) cos nxdx
π −π
1 π −ax
Z
= e cos nxdx
π −π
1 h e−ax iπ
an = (−a cos nx + n sin nx)
π a2 + n2 −π

1 h e−aπ eaπ i
an = (−a cos nπ + 0) − (−a cos nπ + 0)
π a2 + n2 a2 + n2
a
= (eaπ − e−aπ ) cos nπ
π(a + n2 )
2

2a(−1)n sinh aπ
an = .
π(a2 + n2 )

Transform Techniques B.Sc.(Mathematics)-IDE(UNOM)-II Year-IV Sem


10.5. Worked out Problems 197

1 π
Z
bn = f (x) sin nxdx
π −π
Z π
1
= e−ax sin nxdx
π −π
1 h e−ax iπ
= (−a sin nx − n cos nx)
1π a2 + n2 −π
1 h e−aπ eaπ i
= (0 − n cos nπ) − (0 − n cos nπ)
π a2 + n2 a2 + n2
n
= (eaπ − e−aπ ) cos nπ
π(a2 + n2 )
2n(−1)n sinh aπ
an = .
π(a2 + n2 )

Substituting theses values of a0 , an and bn in (10.5.9),we get the required Fourier series of f (x)
in (−π, π) as

∞ ∞
sinh aπ X 2a(−1)n sinh aπ X 2n(−1)n sinh aπ
e−ax = + cos nx + (−n) sin nx
aπ n=1
π(a 2 + n2 )
n=1
π(a 2 + n2 )

2 sinh aπ h 1 a cos ax a cos 2x a cos 3x i h sin x 2 sin 2x 3 sin 3x i


= − + − + .... − − + − ....
π 2a 1 + a2 22 + a2 32 + a2 12 + a2 22 + a2 32 + a2

which is the required Fourier series.

Deduction: Putting x = 0 and a = 1 in the above series, we get

2 sinh π h 1 1 1 1 1 i
1 = − + 2 − 2 + 2 − ........
π 2 2 2 +1 3 +1 4 +1
π  1 1 1 
= 2 2 − 2 + 2 − .... .
sinh π 2 +1 3 +1 4 +1

Note that x = 0 is a point of discontinuity.

Example 10.5.11. Find the Fourier series representing f (x) = x sin x, 0 < x < 2π.

Solution.
∞ ∞
a0 X X
x sin x = + an cos nx + bn sin nx (10.5.10)
2 n=1 n=1

B.Sc.(Mathematics)-IDE(UNOM)-II Year-IV Sem Transform Techniques


198 10.5. Worked out Problems

1 2π
Z
a0 = f (x)dx
π 0
1 2π
Z
= x sin xdx
π 0
1 2π
= x(− cos x) − 1.(− sin x)
π 0
1
= [−2π + 0 − (0 + 0)]
π
a0 = −2.

1 2π
Z
an = f (x) cos nxdx
π 0
1 2π
Z
= x sin x cos nxdx
π 0
1 2π sin(n + 1)x − sin(n − 1)x
Z
= x
π 0 2
Z 2π
1
= x[sin(n + 1)x − sin(n − 1)x]
π 0
1 h  cos(n + 1)x cos(n − 1)x   sin(n + 1)x sin(n − 1)x i2π
= x − + − 1. − + (n , 1)
2π n+1 n−1 (n + 1)2 (n − 1)2 0
1 h  cos 2(n + 1)π cos 2(n − 1)π i
= 2π − + , n,1
2π n+1 n−1
1 1
= − + n,1
n+1 n−1
2
an = 2
, n , 1.
n −1

If n = 1, we have
Z 2π
1
a1 = x sin 2x
2π 0
1 h  − cos 2x   − sin 2x i2π
= x − 1.
2π 2 4 0
1 1
= (−π) = − .
2π 2

Transform Techniques B.Sc.(Mathematics)-IDE(UNOM)-II Year-IV Sem


10.5. Worked out Problems 199

1 2π
Z
bn = f (x) sin nxdx
π 0
1 2π
Z
= x sin x sin nxdx
π 0
1 2π cos(n − 1)x − cos(n + 1)x
Z
= x
π 0 2
Z 2π
1
= x[sin(n + 1)x − sin(n − 1)x]
π 0
1 h  sin(n − 1)x sin(n + 1)x   cos(n − 1)x cos(n + 1)x i2π
= x − − 1. − + (n , 1)
2π n−1 n+1 (n − 1)2 (n + 1)2 0
1h cos 2(n − 1)π cos 2(n + 1)π  1 1 i
= 0+ − − 0 + − , n,1
2π (n − 1)2 (n + 1)2 (n − 1)2 (n + 1)2
1h 1 1 1 1
= − − + n,1
2π (n − 1) 2 (n + 1)2 (n − 1)2 (n + 1)2
bn = 0, n , 1.

If n = 1, then
Z 2π
1
b1 = x 2sin2 xdx
2π 0
Z 2π
1
= x(1 − cos 2x)dx
2π 0
1h  sin 2x   x2 cos 2x i2π
= x x− − 1. +
2π 2 2 4 0
2
1  4π 1 1 i
= 2π.2π − − +
2π 2 4 4
b1 = π.

Substituting theses values of a0 , an and bn in (10.5.10),we get the required Fourier series of
f (x) in (−π, π) as


1 X 2
x sin x = −1 − + cos nx + π sin x
2 n=2 n2 − 1

B.Sc.(Mathematics)-IDE(UNOM)-II Year-IV Sem Transform Techniques


200 10.5. Worked out Problems


1 X cos nx
x sin x = −1 + π sin x − cos x + 2 2−1
.
2 n=2
n

Example 10.5.12. Obtain the Fourier series expansion of f (x) given that f (x) = kx(π − x) in the
interval (0, 2π), where k is a constant.

Solution. Let f (x) = k(πx − x2 ) be represented by the Fourier series

∞ ∞
a0 X X
k(πx − x2 ) = + an cos nx + bn sin nx (10.5.11)
2 n=1 n=1

1 2π
Z
a0 = f (x)dx
π 0
1 π
Z
= k(πx − x2 )dx
π 0
k  x2 x3 2π
= π −
π 2 3 0
k  8 
a0 = 2π3 − π3
π 3
2kπ2
a0 = − .
3

1 2π
Z
an = f (x) cos nxdx
π 0
1 2π
Z
= k(πx − x2 ) cos nxdx
π 0
kh  sin nx   − cos nx   − sin nx i2π
= (πx − x2 ) − (π − 2x) + (−2)
π n n 2 n3 0
kh 1  π i
= − 0 − 3(π) 2 + 0 − 0 + 2 + 0
π n n
k −4π
 
= − (n , 0)
π n2
4k
an = − 2 n , 0.
n

Transform Techniques B.Sc.(Mathematics)-IDE(UNOM)-II Year-IV Sem


10.5. Worked out Problems 201

1 2π
Z
bn = f (x) sin nxdx
π 0
1 2π
Z
bn = k(πx − x2 ) sin nxdx
π 0

kh  − cos nx   − sin nx   cos nx i2π


bn = (πx − x2 ) − (π − 2x) + (−2)
π n n 2 n3 0
kh 2 1 1  1 i
= 2(π ) + 0 − 2. 3 − 0 + 0 − 2 3
π n n n
2
k 2π

= (n , 0)
π n
2kπ
bn = n , 0.
n

Substituting theses values of a0 , an and bn in (10.5.11),we get the required Fourier series of
f (x) in (−π, π) as

∞ ∞
−kπ2 X 1 X 1
k(πx − x ) = 2
− 4k 2
cos nx + 2kπ sin nx
3 n=1
n n=1
n

Example 10.5.13. Find the Fourier series to represent the function f (x) = eax in the interval
0 < x < 2π.

Solution.
∞ ∞
a0 X X
e =
ax
+ an cos nx + bn sin nx (10.5.12)
2 n=1 n=1

1 2π
Z
a0 = f (x)dx
π 0
1 2π ax
Z
= e dx
π 0
1  eax 2π
=
π a 0
1 2aπ
a0 = (e − 1)

B.Sc.(Mathematics)-IDE(UNOM)-II Year-IV Sem Transform Techniques


202 10.5. Worked out Problems

1 2π
Z
an = f (x) cos nxdx
π 0
1 2π ax
Z
= e cos nxdx
π 0
1 h eax i2π
= (a cos nx + n sin nx)
π a2 + n2 0
2aπ
1 e
h e0 i
= (a cos 2nπ + 0) − (a + 0)
π a2 + n2 a2 + n2
a
an = (e2aπ − 1)
π(a2 + n2 )

1 2π
Z
bn = f (x) sin nxdx
π 0
1 2π ax
Z
= e sin nxdx
π 0
1 h eax i2π
= (a sin nx − n cos nx)
1π a2 + n2 0
2aπ 0
1h e e i
= (0 − n) − (0 − n)
π a2 + n2 a2 + n2
n
bn = (1 − e2aπ )
π(a2 + n2 )

Substituting theses values of a0 , an and bn in (10.5.12),we get the required Fourier series of
f (x) in (−π, π) as

∞ ∞
e2aπ − 1 a(e2aπ − 1) X 1 e2aπ−1 X n
e ax
= + cos nx − sin nx
2aπ π n=1
a2 + n2 π n=1 a2 + n2

which is the required Fourier series.

Example 10.5.14. Find the Fourier series representing f (x) = x cos x, 0 < x < 2π.

Solution.
∞ ∞
a0 X X
x sin x = + an cos nx + bn sin nx (10.5.13)
2 n=1 n=1

Transform Techniques B.Sc.(Mathematics)-IDE(UNOM)-II Year-IV Sem


10.5. Worked out Problems 203

1 2π
Z
a0 = f (x)dx
π 0
1 2π
Z
= x cos xdx
π 0
1 2π
= x(sin x) − 1.(− cos x)
π 0
1
= [0 + 1 − (0 + 1)]
π
a0 = 0.

1 2π
Z
an = f (x) cos nxdx
π 0
1 2π
Z
an = x cos x cos nxdx
π 0

1 2π cos(1 + n)x + cos(1 − n)x


Z
an = x
π 0 2
1 h  sin(n + 1)x sin(1 − n)x   cos(n + 1)x cos(1 − n)x i2π
= x + − 1. − − (n , 1)
2π n+1 1−n (n + 1)2 (1 − n)2 0
1h  cos(n + 1)2π cos(1 − n)2π   1 1 i
= 0+ + − 0 + + n,1
2π (n + 1)2 (1 − n)2 (1 + n)2 (1 − n)2
an = 0 n , 1.

If n = 1, we have

1 2π
Z
a1 = x cos2 xdx
π 0
Z 2π
1
= x(1 + cos 2x)dx
2π 0
1h  sin 2x   x2 cos 2x i2π
= x x+ − 1. −
2π 2 2 4 0

B.Sc.(Mathematics)-IDE(UNOM)-II Year-IV Sem Transform Techniques


204 10.5. Worked out Problems

1 4π2 1 1 i
= 2π.2π − + −
2π 2 4 4
1
= (2π2 )

a1 = π.

1 2π
Z
bn = f (x) sin nxdx
π 0
1 2π
Z
= x cos x sin nxdx
π 0
1 2π sin(1 + n)x − sin(1 − n)x
Z
= x
π 0 2
Z 2π
1
= x[sin(1 + n)x − sin(1 − n)x]
2π 0
1 h  − cos(1 + n)x cos(1 − n)x   sin(1 + n)x sin(1 − n)x i2π
= x + − 1. − + (n , 1)
2π 1+n 1−n (1 + n)2 (1 − n)2 0
1h  cos 2(1 + n)π cos 2(1 − n)π  i
= − 2π + − (0 + 0) , n , 1
2π 1+n 1−n
1 1
= + n,1
1+n 1−n
2n
bn = , n , 1.
1 − n2

If n = 1, then
Z 2π
1
b1 = x sin 2xdx
2π 0
1 h  − cos 2x   − sin 2x i2π
= x − 1.
2π 2 4 0
1 h −2π  i
= + 0 − (0 + 0)
2π 2
1
b1 = − .
2

Substituting theses values of a0 , an and bn in (10.5.13),we get the required Fourier series of
f (x) in (−π, π) as

Transform Techniques B.Sc.(Mathematics)-IDE(UNOM)-II Year-IV Sem


10.5. Worked out Problems 205


1 X n
x cos x = π cos x − sin x + 2 sin nx
2 n=2,3,..
1 − n2

Example 10.5.15. Find a Fourier series to represent f (x) = x2 − x in the interval (−π, π). Hence
show that
1 1 1 π2
+ + + ..... = .
12 22 32 12
Solution. Let f (x) = x2 − x be represented by the Fourier series

∞ ∞
a0 X X
x −x=
2
+ an cos nx + bn sin nx (10.5.14)
2 n=1 n=1

1 π
Z
a0 = f (x)dx
π π
1 π 2
Z
= (x − x)dx
π −π
1h π 2
Z Z 2π
= x dx − xdx
π −π −π
Z π
1h i
= 2 x2 dx − 0
π 0
1  x3 π
= (2).
π 3 0
1  2π3 
a0 =
π 3
2π2
a0 = .
3

1 π
Z
an = f (x) cos nxdx
π −π
Z π
1
= (x2 − x) cos nxdx
π −π
1h π 2
Z Z π i
= x cos nxdx − x cos nxdx
π −π π

B.Sc.(Mathematics)-IDE(UNOM)-II Year-IV Sem Transform Techniques


206 10.5. Worked out Problems
Z π
1h i
= 2 x2 cos nx − 0
π 0
2 h 2  sin nx   − cos nx   − sin nx iπ
= x − 2x + 2
π n n2 n3 0
2h cos nπ i
= 0 + 2(π) 2 + 0 − (0 + 0 + 0)
π n
n
2 2π(−1)

= (n , 0)
π n2
4(−1)n
an = .
n2

1 2π
Z
bn = f (x) sin nxdx
π −π
1 2π 2
Z
= (x − x) sin nxdx
π −π
1h π 2
Z Z π i
= x sin nxdx − x sin nxdx
π −π −π
Z π
1h i
= 0 − 2. x sin nxdx
pi 0
2 h  − cos nx   −sinnx iπ
= − x − 1.
pi n n2 0
n
2 h (−1) i
= − −π + 0 − (0 + 0)
π n
(−1)n
= 2
n
2(−1)n
bn = .
n

Substituting theses values of a0 , an and bn in (10.5.14),we get the required Fourier series of
f (x) in (−π, π) as

∞ ∞
π2 X 4(−1)n X 2(−1)n
x −x =
2
+ cos nx + sin nx
3 n=1 n2 n=1
n

Deduction. x = 0 is a point of discontinuity of f (x). Hence the Fourier series of f (x) at x = 0


converges to f (0).

Transform Techniques B.Sc.(Mathematics)-IDE(UNOM)-II Year-IV Sem


10.6. Exercises 207

Putting x = 0 in the above series, we have

π2  1 1 1 
0 =
+ 4 − 1 + 2 − 2 + 2 + ...
3 2 3 4
 1 1 1  π2
i.e., − 1 + 2 − 2 + 2 − ... = −
2 3 4 12
1 1 1 π2
1 − 2 + 2 − 2 + .. = .
2 3 4 12

10.6 Exercises

1. Find the Fourier series of f (x) = x − π in (−π, π).


π−x
2. Obtain the Fourier series of f (x) = 2
in the interval [0, 2π]. Hence deduce that
1 − 31 + 51 − 71 + ... = π4 .
3. Find the Fourier series of π2 − x2 in (−π, π).
4. Show that in the range −π to π , x + x2 as a Fourier series is

π2  cos x cos 2x cos 3x   sin x sin 2x sin 3x 


x + x2 = −4 − + − .. + 2 − + − .... .
3 12 22 32 1 2 3

Hence show that



X 1 π2
2
= .
n=1
n 6

5. Express f (x) = x(2π − x) as Fourier series in (0, 2π). Deduce that

1 1 1 π2
+ + + ... = .
12 22 32 6

6. Obtain the Fourier series for the function


(i) e x − 1 in (0, 2π); (ii) e−x in the interval (0, 2π);
(iii) eax in (−π, π); (iv) eax in (0, 2π).

B.Sc.(Mathematics)-IDE(UNOM)-II Year-IV Sem Transform Techniques


208 10.6. Exercises

Answers.
h i
1. x − π = −π + 2 sin x − 12 sin 2x + 31 sin 3x − 14 sin 4x + ...
π−x
2. 2
= sin x + 12 sin 2x + 13 sin 3x + ...
P (−1)n
3. π2 − x2 = 23 π2 − 4 ∞ n=1 n2 cos nx.

5. f (x) = 32 π2 − 4 ∞ 1
P
n=1 n2 cos nx.
2π−1
h P cos nx P∞ n sin nx i
6. (i) e x − 1 = e π 21 + ∞ n=1 n2 +1 − n=1 n2 +1 − 1;

h i
(ii) e−x = 1−eπ 12 + ∞ cos nx
+ ∞ n sin nx
P P
n=1 n +1
2 n=1 n +1 ;
2
h n P (−1)n n sin nx i
(iii) eax = 2 sinh aπ 1
+ ∞ (−1) cos nx
+ ∞
P
π 2a n=1 a2 +n2 n=1 a2 +n2
;
2aπ 2aπ P cos nx 1−e2aπ P∞ n sin nx i
(iv) eax = e 2−1 + ae π −1 ∞ n=1 a2 +n2 + π n=1 a2 +n2 .

Transform Techniques B.Sc.(Mathematics)-IDE(UNOM)-II Year-IV Sem


UNIT-III

Lesson 11

FOURIER SERIES FOR


DISCONTINUOUS FUNCTIONS
Learning Objectives
Upon completion of this lesson, students will be able to

• identify Fourier series for discontinuous functions

• find Fourier series of discontinuous functions of periodicity 2π

11.1 Introduction

I
n deriving the Euler’s formulae a0 , an and bn it was assumed that f (x) is continuous.
If a function have finite number of discontinuities then such a function is expressible as a
Fourier series.
For instance, let the function f (x) be defined by

φ(x), if c < x < x0




f (x) = 


ψ(x), ifx0 < x < c + 2π


B.Sc.(Mathematics)-IDE(UNOM)-II Year-IV Sem 209 Transform Techniques


210 11.1. Introduction

where x0 is the point of discontinuity in (c, c + 2π). In such cases also we obtain the Fourier
series for f (x) in the usual way. The values of a0 , an and bn are given by
Z x0 Z c+2π
1h i
a0 = φ(x)dx + ψ(x)dx
π c x0

1 h x0
Z Z c+2π i
an = φ(x) cos nxdx + ψ(x) cos nxdx
π c x0
Z x0 Z c+2π
1 h i
bn = φ(x) sin nxdx + ψ(x) sin nxdx
π c x0

It may be seen from the graph, that at a point of finite discontinuity x = x0 , there is a finite jump
in the value of the function f (x) at x = x0 . Both the limits on the right and left exists and are
different. At such a point, the Fourier series converges to

1
[ f (x0 − 0) + f (x0 + 0)].
2

In fact,if f (x) satisfies Dirichlet’s conditions and


a0 X
f (x) = + (an cos nx + bn sin nx)
2 n=1

in [c, c + 2π] then the right hand side series converges to f (x) if x is a point of continuity of
f (x) and converges to
1
[ f (x + 0) + f (x − 0)]
2
if x is a point of discontinuity of f (x).
This result is useful to determine the sum of certain series.

Transform Techniques B.Sc.(Mathematics)-IDE(UNOM)-II Year-IV Sem


11.2. Worked out Problems 211

11.2 Worked out Problems

Example 11.2.1. Expand 


1, if 0 < x < π




f (x) = 


0, if π < x < 2π


as a Fourier series.

Solution. The Fourier series of f (x) in the interval (0, 2π) is given by

∞ ∞
a0 X X
f (x) = + an cos nx + bn sin nx (11.2.1)
2 n=1 n=1

Where

1 2π
Z
a0 = f (x)dx
π 0
1h π
Z Z 2π
= f (x)dx + f (x)dx
π 0 π
1h π
Z Z 2π
= 1. dx + 0. dx
π 0 π
1 π
= (x)
π 0
1
= (π)
π
a0 = 1.

B.Sc.(Mathematics)-IDE(UNOM)-II Year-IV Sem Transform Techniques


212 11.2. Worked out Problems

1 2π
Z
an = f (x) cos nxdx
π 0
1h π
Z Z 2π
= f (x) cos nxdx + f (x) cos nxdx
π 0 π
1h π
Z Z 2π
= 1. cos nxdx + 0. cos nxdx
π 0 π
1  sin nx π
=
π n 0
1
= (0 − 0)

an = 0.

1 2π
Z
bn = f (x) sin nxdx
π 0
1h π
Z Z 2π
= f (x) sin nxdx + f (x) sin nxdx
π 0 π
1h π
Z Z 2π
= 1. sin nxdx + 0. sin nxdx
π 0 π
1  − cos nx π
=
π n 0
1
= − (cos nπ − 1)



0, when n is even



bn = 


2
, when n is odd



 nπ

Substituting these values of a0 , an and bn in (11.2.1), we get


1 2X1 1 2 1 1 
f (x) = + sin nx = + sin x + sin 3x + sin 5x + .....
2 π n=1 n 2 π 3 5

Example 11.2.2. Find the Fourier series to represent the function f (x) given by Expand

if − π < x < 0

−k,



f (x) = 


if 0 < x < π

k,

Transform Techniques B.Sc.(Mathematics)-IDE(UNOM)-II Year-IV Sem


11.2. Worked out Problems 213

Hence show that 1 − 13 + 51 − 17 + .... = π4 . .

Solution. The Fourier series of f (x) in the interval (0, 2π) is given by

∞ ∞
a0 X X
f (x) = + an cos nx + bn sin nx
2 n=1 n=1

Where

1 π
Z
a0 = f (x)dx
π −π
Z π
1h 0
Z
= f (x)dx + f (x)dx
π −π 0
Z π
1h 0
Z
= (−k).dx + k.dx
π −π 0
k
= [−(x)0−π + (x)π0 ]
π
a0 = 0.

1 π
Z
an = f (x) cos nxdx
π −π
Z π
1h 0
Z
= f (x) cos nxdx + f (x) cos nxdx
π −π 0
Z π
1h 0
Z
= (−k). cos nxdx + k. cos nxdx
π −π 0
k h − sin nx 0  sin nx π i
= +
π n −π n 0
k
= (0 + 0)
π
an = 0.

B.Sc.(Mathematics)-IDE(UNOM)-II Year-IV Sem Transform Techniques


214 11.2. Worked out Problems

1 π
Z
bn = f (x) sin nxdx
π −π
Z π
1h 0
Z
= f (x) sin nxdx + f (x) sin nxdx
π −π 0
Z π
1h 0
Z
= (−k). sin nxdx + k. sin nxdx
π −π 0
k h cos nx 0  − cos nx π i
= +
π n −π n 0
n n
k 1 − (−1)
h (−1) − 1 i
= −
π n n
2k
= (1 − (−1)n )



0, when n is even



bn = 


4k
,



 nπ when n is odd

Substituting these values of a0 , an and bn in (11.2.1), we get


X 4k 4k  1 1 
f (x) = sin nx = sin x + sin 3x + sin 5x + ..... .
n=1
nπ π 3 5

Deduction. At x = π2 , f (x) is continuous. Hence the Fourier series converges to f π


 
2
. Thus
putting x = π2 in the above series, we get
 

4k  ππ 1  3π  1  5π  
f sin =
+ sin + sin + .....
2 π 2 3 2 5 2
4k  1 1 1 
k = 1 − + − + ....
π 3 5 7
1 1 1 π
i.e., 1 − + − + .... = .
3 5 7 4

Example 11.2.3. Find the Fourier series to represent the function f (x) given by Expand

0, if − π < x < 0




f (x) = 

 πx

, if0 < x < π



4

Transform Techniques B.Sc.(Mathematics)-IDE(UNOM)-II Year-IV Sem


11.2. Worked out Problems 215
π2
Also deduce that 1 + 1
32
+ 1
52
+ 1
72
+ .... = 8
..

Solution. The Fourier series of f (x) in the interval (0, 2π) is given by

∞ ∞
a0 X X
f (x) = + an cos nx + bn sin nx
2 n=1 n=1

Where

1 π
Z
a0 = f (x)dx
π −π
Z π
1h 0
Z
= f (x)dx + f (x)dx
π −π 0
Z π
1h 0 πx
Z
= 0.dx + .dx
π −π 0 4
π  x2 π
=
4π 2 0
1 π2
= .
4 2
π2
a0 = .
8

1 π
Z
an = f (x) cos nxdx
π −π
Z π
1h 0
Z
= f (x) cos nxdx + f (x) cos nxdx
π −π 0
Z π
1h 0 πx
Z
= 0. cos nxdx + cos nxdx
π −π 0 4
1 π h  sin nx   − cos nx iπ
= . x − 1.
π 4 n n2 0
1 h cos nπ   1 i
= 0+ − 0+ 2
4 n2 n
1
= [(−1)n − 1]
4n2

 2n2 ,
 −1
when n is odd



an = 



0,

 when n is even

B.Sc.(Mathematics)-IDE(UNOM)-II Year-IV Sem Transform Techniques


216 11.2. Worked out Problems

1 π
Z
bn = f (x) sin nxdx
π −π
Z π
1h 0
Z
= f (x) sin nxdx + f (x) sin nxdx
π −π 0
Z π
1h 0 πx
Z
= 0. sin nxdx + sin nxdx
π −π 0 4
1 π h  − cos nx   − sin nx iπ
bn = . x − 1.
π 4 n n2 0

1 h −π cos nπ  i
bn = + 0 − (0 + 0)
4 n
π(−1) n+1
bn =
4n

Substituting these values of a0 , an and bn in (11.2.1), we get

∞ ∞
π2 X  (−1)n − 1  X (−1)n π
f (x) = + cos nx − sin nx
16 n=1 4n2 n=1
4n
π2 1  1 1  π 1 1 
= − cos x + 2 cos 3x + 2 cos 5x + ... + sin x − sin 2x + sin 3x + ..... .
16 2 3 5 4 2 3

Deduction. At x = 0, f (x) is continuous. Hence the Fourier series converges to f (0) at x = 0.


Thus putting x = 0 in the above series, we get

π2 1 1 1 
f (0) = − 1+ + + ...
16 2 32 52
π2 1 1 1 
0 = − 1+ + + ...
16 2 32 52
1 1 1 π2
i.e., 1 + 2 + 2 + 2 + .... = .
3 5 7 8

Example 11.2.4. Find the Fourier series of the periodic function defined as

−π, if − π < x < 0




f (x) = 


if 0 < x < π

 x,

Transform Techniques B.Sc.(Mathematics)-IDE(UNOM)-II Year-IV Sem


11.2. Worked out Problems 217
π2
Also deduce that 1 + 1
32
+ 1
52
+ 1
72
+ .... = 8
..

Solution. The Fourier series of f (x) in the interval (0, 2π) is given by

∞ ∞
a0 X X
f (x) = + an cos nx + bn sin nx
2 n=1 n=1

Where

1 π
Z
a0 = f (x)dx
π −π
Z π
1h 0
Z
= f (x)dx + f (x)dx
π −π 0
Z π
1h 0
Z
a0 = (−π)dx + xdx
π −π 0

1h  x2 π i
a0 = (−π)(x)0−π +
π 2 0
1h π i
2
= − π2 +
π 2
1  π 2
a0 = −
π 2
π
a0 = − .
2

1 π
Z
an = f (x) cos nxdx
π −π
Z π
1h 0
Z
= f (x) cos nxdx + f (x) cos nxdx
π −π 0
Z π
1h 0
Z
= (−π) cos nxdx + x cos nxdx
π −π 0
1h  sin nx 0  sin nx cos nx π i
= (−π) + 1. x. +
π n −π n n2 0

B.Sc.(Mathematics)-IDE(UNOM)-II Year-IV Sem Transform Techniques


218 11.2. Worked out Problems

1 1 1
= 0 + 2 cos nπ − 2
π n n
1
= 2 (cos nπ − 1)

1
= [(−1)n − 1].
πn 2


 n2 π , when n is odd


 −2

an = 



0,

 when n is even

1 π
Z
bn = f (x) sin nxdx
π −π
Z π
1h 0
Z
= f (x) sin nxdx + f (x) sin nxdx
π −π 0
Z π
1h 0
Z
= (−π) sin nxdx + x sin nxdx
π −π 0
1h  − cos nx 0  cos nx sin nx π i
= −π + −x +
π n −π n n2 0
1 π
h π i
= (1 − cos nπ) − cos nπ
π n n
1
= (1 − 2 cos nπ)
n

Substituting these values of a0 , an and bn in (11.2.1), we get

∞ ∞
π 2 X 1  X 1
f (x) = − ((−1)n
− 1) cos nx + (1 − 2 cos nπ) sin nx
4 π n=1 n2 π n=1
n
π 2 1 1   1 1 
= − − cos x + 2 cos 3x + 2 cos 5x + ... + 3 sin x − sin 2x + 3 sin 3x − ..... .
4 π 3 5 2 3

Deduction. Putting x = 0 in the above series, we get

π 2 1 1 
f (0) = − 1 + 2 + 2 + ... (11.2.2)
4 π 3 5

Now f (x) is discontinuous at x = 0.

Transform Techniques B.Sc.(Mathematics)-IDE(UNOM)-II Year-IV Sem


11.2. Worked out Problems 219

f (0 − 0) = −π f (0 + 0) = 0.
1 pi
∴ f (0) = [ f (0 − 0) + f (0 + 0)] = − .
2 2
Now (11.2.2) becomes

π π 2 1 1 
= − − 1 + 2 + 2 + ...

2 4 π 3 5
1 1 1 π 2
i.e., 1 + 2 + 2 + 2 + .... = .
3 5 7 8

Example 11.2.5. Find the Fourier series to represent the function f (x) given by

if − π < x < 0

0,



f (x) = 


 x2 , if 0 < x < π


Solution. The Fourier series of f (x) in the interval (0, 2π) is given by

∞ ∞
a0 X X
f (x) = + an cos nx + bn sin nx
2 n=1 n=1

Where

1 π
Z
a0 = f (x)dx
π −π
Z π
1h 0
Z
= 0.dx + x2 dx
π −π 0
1  x3 π
a0 =
π 3 0

1 h π3 i
a0 =
π 3
π2
a0 = .
3

B.Sc.(Mathematics)-IDE(UNOM)-II Year-IV Sem Transform Techniques


220 11.2. Worked out Problems

1 π
Z
an = f (x) cos nxdx
π −π
Z π
1h 0
Z
= f (x) cos nxdx + f (x) cos nxdx
π −π 0
Z π
1h 0
Z
= 0. cos nxdx + x2 cos nxdx
π −π 0
1 h 2  sin nx   − cos nx   − sin nx iπ
= x. − 2x + 2
π n n2 n3 0
1 2π
 
= cos nπ
π n2
2
an = (−1)n n , 0
n2

1 π
Z
bn = f (x) sin nxdx
π −π
Z π
1h 0
Z
= f (x) sin nxdx + f (x) sin nxdx
π −π 0
Z π
1h 0
Z
= 0. sin nxdx + x2 sin nxdx
π −π 0
1 h 2  − cos nx   − sin nx  cos nx iπ
= x − 2x + 2
π n n2 n3 0
2
1  −π 2 
= cos nπ + 3 (cos nπ − 1)
π n n
−π 2
bn = (−1)n + 3 [(−1)n − 1] n , 0
n πn

Substituting these values of a0 , an and bn in (11.2.1), we get

∞ ∞ h
π2 X (−1)n X π 2
f (x) = +2 cos nx + (−1)n+1 + 3 [(−1)n − 1] sin nx
6 n=1
n 2
n=1
n πn

Example 11.2.6. Find the Fourier series to represent the function f (x) given by

if − π < x < 0

0,



f (x) = 


if 0 < x < π

sin x,

Transform Techniques B.Sc.(Mathematics)-IDE(UNOM)-II Year-IV Sem


11.2. Worked out Problems 221

Solution.
∞ ∞
a0 X X
f (x) = + an cos nx + bn sin nx (11.2.3)
2 n=1 n=1

1 π
Z
a0 = f (x)dx
π −π
Z π
1h 0
Z i
= 0.dx + sin xdx
π −π 0
1
= (− cos x)π0
π
1
= [1 + 1]
π
2
a0 = .
π

1 π
Z
an = f (x) cos nxdx
π −π
Z π
1 0
Z
= 0. cos nxdx + sin x cos nxdx
π −π 0
1 π sin(n + 1)x − sin(n − 1)x
Z
=
π 0 2
Z π
1
= [sin(n + 1)x + sin(1 − n)x]
2π 0
1 h cos(n + 1)x  cos(n − 1)x iπ
= − + (n , 1)
2π n+1 (n − 1) 0
1 h cos(n + 1)π cos(n − 1)π 1 1 i
= − + + − , n,1
2π n+1 n−1 n+1 n−1
1 h −(−1)n+1 (−1)n−1 1 1 i
= + + − n,1
2π n + 1 n−1 n+1 n−1
1h 1 1
an = (1 − (−1)n+1 ) + ((−1)n−1 − 1), n , 1
2π n + 1 n−1
∴ an = 0, when n is odd
1  −4 
and an =
2π n2 − 1
−2
an = when n is even and n , 1.
π(n2 − 1)

B.Sc.(Mathematics)-IDE(UNOM)-II Year-IV Sem Transform Techniques


222 11.2. Worked out Problems

If n = 1, we have

1 π
Z
a1 = sin x cos xdx
π 0
Z π
1
= sin 2xdx
2π 0
1  cos 2x π
= −
2π 2 0
−1
= (cos 2π − cos 0)

a1 = 0.

1 π
Z
bn = f (x) sin xdx
π 0
1 π
Z
= sin x sin nxdx
π 0
Z π
1 cos(n − 1)x − cos(n + 1)x
=
π 0 2
1 h sin(n − 1)x sin(n + 1)x iπ
= − (n , 1)
2π n−1 n+1 0
1
= (0), n , 1

bn = 0, n , 1.

If n = 1, then

1 π 2
Z
b1 = sin xdx
π 0
Z π
1
= (1 − cos 2x)dx
2π 0
1 sin 2x π
= x−
2π 2 0
1 h i
= π − 0 − (0 − 0)

1
b1 = .
2

Transform Techniques B.Sc.(Mathematics)-IDE(UNOM)-II Year-IV Sem


11.2. Worked out Problems 223

Substituting theses values of a0 , an and bn in (11.2.3), we get the required Fourier series of f (x)


1 2 X cos 2nx 1
f (x) = − + + sin x
π π n=1 (2n) − 1 2
2

π
Deduction. Putting x = 2
in the above series, we get

π ∞ π
1 2 X cos nπ 1
f = − + sin
2 π π n=1 (2n)2 − 1 2 2
π ∞
1 2X (−1)n 1
sin = − +
2 π π n=1 (2n + 1)(2n − 1 2

1 1 2X (−1)n+1
1− − =
2 π π n=1 (2n + 1)(2n − 1)

π−2 π X (−1)n+1
. =
2π 2 n=1
(2n + 1)(2n − 1)
1 1 1 π−2
− + − ... = .
1.3 3.5 5.7 4

Example 11.2.7. Given 


1 + , if − π < x < 0


 2x
π

f (x) = 


2x
, if 0 < x < π

1 −


π

π2
 
Show that f (x) = 8 cos x
π2 12
+ cos 3x
32
+ cos 5x
52
+ .... . Deduce that 1
12
+ 1
32
+ 1
52
+ ... = 8
.

Solution. The Fourier series of f (x) in the interval (0, 2π) is given by

∞ ∞
a0 X X
f (x) = + an cos nx + bn sin nx
2 n=1 n=1

B.Sc.(Mathematics)-IDE(UNOM)-II Year-IV Sem Transform Techniques


224 11.2. Worked out Problems

Where

1 π
Z
a0 = f (x)dx
π −π
Z π
1h 0 
Z
2x  2x
= 1+ dx + 1 − dx
π −π π 0 π
2 0 2 π i
1 h x  x
= x+ + x+
π π −π π 0
1
= [0 + 0]
π
a0 = 0.

1 π
Z
an = f (x) cos nxdx
π −π
Z π
1h 0
Z
= f (x) cos nxdx + f (x) cos nxdx
π −π 0
Z π
1h 0 
Z
2x  2x 
= 1+ cos nxdx + 1− cos nxdx
π −π π 0 π
1 h sin nx 0  2  sin nx   − cos nx 0  sin nx π 2 h sin nx   − cos nx iπ i
= + x − 1. + − x − 1
π n −π π n n2 −π n 0 π n n2 0
1 h 2 1 cos nπ
   2 cos nπ 1
 i
= 0+ − + 0− − 2
π π n2 n2 π n2 n
2 h 1 cos nπ cos nπ 1 i
= − − + 2
π2 n2 n2 n2 n
2 h 2 2 cos nπ i
= −
π2 n2 n2
4
= (1 − (−1)n )
n2 π2


0, when n is even



∴ an = 


 n28π2 , when n is odd


Transform Techniques B.Sc.(Mathematics)-IDE(UNOM)-II Year-IV Sem


11.2. Worked out Problems 225

1 π
Z
bn = f (x) sin nxdx
π −π
Z π
1h 0
Z
= f (x) sin nxdx + f (x) sin nxdx
π −π 0
Z π
1h 0 
Z
2x  2x 
= 1+ sin nxdx + 1− sin nxdx
π −π π 0 π
1 h − cos nx 0  2  − cos nx   − sin nx 0
= + x − 1.
π n −π π n n2 −π
 − cos nx π 2 h − cos nx  − sin nx π i i
+ − x − 1.
n 0 π n n2 0
1 −1 cos nπ 2 −π cos nπ
h  i h − cos nπ 1 2  −π cos nπ i
= + + + + −
π n n π n n n π n
= 0.

Substituting these values of a0 , an and bn in (11.2.3), we get

8 h cos x cos 3x cos 5x i


f (x) = + + + ... .
π2 12 32 52

Deduction. Putting x = 0 in the above series, we get

8h1 1 1 i
f (0) =+ + + ...
π2 12 32 52
8h1 1 1 i
1 = 2 2 + 2 + 2 + ...
π 1 3 5
1 1 1 π2

2
+ 2 + 2 + ... = .
1 3 5 8

Example 11.2.8. Find the Fourier series in [−π, π] for the function

 2 (π + x), if − π ≤ x < 0


 −1

f (x) = 


 12 (π − x), if 0 < x ≤ π.


B.Sc.(Mathematics)-IDE(UNOM)-II Year-IV Sem Transform Techniques


226 11.2. Worked out Problems

Solution. The Fourier series of f (x) in the interval (0, 2π) is given by

∞ ∞
a0 X X
f (x) = + an cos nx + bn sin nx
2 n=1 n=1

Where

1 π
Z
a0 = f (x)dx
π −π
Z π
1 h 0 −1
Z
1
= (π + x)dx + (π − x)dx
π −π 2 0 2
1h  x2 0  x2 π i
= − πx + + πx −
2π 2 −π 2 0
1 h π 2
π2i
= − π2 + + π2 −
2π 2 2
a0 = 0.

1 π
Z
an = f (x) cos nxdx
π −π
Z π
1h 0
Z i
= f (x) cos nxdx + f (x) cos nxdx
π −π 0
Z π
1h 0 1
Z
1 i
= − (π + x) cos nxdx + (π − x) cos nxdx
π −π 2 0 2
Z 0 Z 0 Z π Z π
−1 h i
= π cos nxdx + x cos nxdx − π cos nxdx + x cos nxdx
2π −π −π 0 0
−1 h  sin nx 0   sin nx  cos nx 0  sin nx π   sin nx  cos nx π i
an = π + x + −π + x +
2π n −π n n2 −π n 0 n n2 0

−1 h 1 cos nπ cos nπ 1 i
an = − + − 2
2π n2 n2 n2 n
= 0.

Transform Techniques B.Sc.(Mathematics)-IDE(UNOM)-II Year-IV Sem


11.2. Worked out Problems 227

1 π
Z
bn = f (x) sin nxdx
π −π
Z π
1h 0
Z i
= f (x) sin nxdx + f (x) sin nxdx
π −π 0
Z π
1h 0 1
Z
1 i
= − (π + x) sin nxdx + (π − x) sin nxdx
π −π 2 0 2
Z 0 Z 0 Z π Z π
−1 h i
= π sin nxdx + x sin nxdx − π sin nxdx + x sin nxdx
2π −π −π 0 0
−1 h  − cos nx 0   − cos nx  sin nx 0  cos nx π   − cos nx  sin nx π i
= π + x + +π + x +
2π n −π n n2 −π n 0 n n2 0
−1 h −π π cos nπ π  −π cos nπ i
= (1 − cos nπ) + + (cos nπ − 1) +
2π n n n n
1
= − (cos nπ − 1)
π
1
= − ((−1)n − 1).
n


0, when n is even



bn = 


 n2 ,


 if n is odd.

Substituting these values of a0 , an and bn in (11.2.3), we get


X 2
f (x) = sin nx
n=1,3,5...
n
h sin 3x sin 5x i
f (x) = 2 sin x + + + ... .
3 5

Example 11.2.9. Find the Fourier series for the following function

− cos x, if − π ≤ x < 0




f (x) = 


if 0 < x ≤ π.

cos x,

Solution. The Fourier series of f (x) in the interval (0, 2π) is given by

∞ ∞
a0 X X
f (x) = + an cos nx + bn sin nx
2 n=1 n=1

B.Sc.(Mathematics)-IDE(UNOM)-II Year-IV Sem Transform Techniques


228 11.2. Worked out Problems

Where

1 π
Z
a0 = f (x)dx
π −π
Z π
1h 0
Z
= − cos xdx + cos xdx
π −π 0
1 h 0  π i
= − sin x + sin x
π −π 0
1h i
= 0+0
π
a0 = 0.

1 π
Z
an = f (x) cos nxdx
π −π
Z π
1h 0
Z i
= f (x) cos nxdx + f (x) cos nxdx
π −π 0
Z 0 Z π
1 h i
= − cos x cos nxdx + cos x cos nxdx
π −π 0
Z 0 Z π
1 h i
= − cos x cos nxdx + cos x cos nxdx
π −π 0
Z 0 Z π
1h i
= − cos(−u) cos(−nu)(−du) + cos x cos nxdx ,
π π 0

putting -u for x in the first integral


Z π
1h 0
Z i
an = cos u cos nudu + cos x cos nxdx
π π 0
Z π Z π
1h i
= − cos x cos nxdx + cos x cos nxdx
π 0 0

an = 0.

Transform Techniques B.Sc.(Mathematics)-IDE(UNOM)-II Year-IV Sem


11.2. Worked out Problems 229

1 π
Z
bn = f (x) sin nxdx
π −π
Z π
1h 0
Z i
= − cos x sin nxdx + cos x sin nxdx
π −π 0
Z 0 Z π
1h i
= − cos(−u) sin(−nu)(−du) + cos x sin nxdx
π π 0

putting -u for x in the first integral


Z 0 Z π
1h i
bn = − cos u sin nudu + cos x sin nxdx
π
Z π Z π0
1h π i
= cos u sin nudu + cos x sin nxdx
π 0
Z π Z0 π
1 h i
= cos x sin nxdx + cos x sin nxdx
π 0 0
1h π
Z
= 2 cos x sin nxdx
π 0
1h π
Z
= [sin(n + 1)x + sin(n − 1)x]dx
π 0
1 h cos(n + 1)x cos(n − 1)x iπ
= − − , n,1
π n+1 n−1 0
1 h cos(n + 1)π cos(n − 1)π 1 1 iπ
= − − + + , n,1
π n+1 n−1 n+1 n−1 0
1 h cos nπ cos nπ 2n i
= + + 2
π n+1 n−1 n −1
1 h 2n 2n i
= cos nπ + 2
π n2 − 1 n −1
2n
= (1 + (−1)n ), n , 1.
π(n2 − 1)
bn = 0 when n is odd
4n
bn = when n is even
π(n2 − 1)

B.Sc.(Mathematics)-IDE(UNOM)-II Year-IV Sem Transform Techniques


230 11.2. Worked out Problems

When n = 1, we have

1 π
Z
b1 = f (x) sin xdx
π −π
Z π
1h 0
Z i
= − cos x sin xdx + cos x sin xdx
π −π 0
Z 0 Z π
1 h i
= − 2 sin x cos xdx + 2 sin x cos xdx
2π −π 0
Z 0 Z π
1h i
= − sin 2xdx + sin 2xdx
2π −π 0
1 h  − cos 2x 0  − cos 2x π i
= − +
2π 2 −π 2 0
1 1h 1 i
b1 = (1 − 1) + (1 − 1) = 0.
2π 2 2

Substituting these values of a0 , an and bn in (11.2.3), we get

4h 2 4 6 i
f (x) = sin 2x + sin 4x + sin 6x + ... .
π 1.3 3.5 5.7

Example 11.2.10. Find the Fourier series for the function


if − π ≤ x < 0





 x,


f (x) = 

if0 < x < π2


0,



 x − π , if π < x ≤ π.



2 2

Solution. The Fourier series of f (x) in the interval (0, 2π) is given by

∞ ∞
a0 X X
f (x) = + an cos nx + bn sin nx
2 n=1 n=1

Transform Techniques B.Sc.(Mathematics)-IDE(UNOM)-II Year-IV Sem


11.2. Worked out Problems 231

Where

1 π
Z
a0 = f (x)dx
π −π
Z π2 Z π
1h 0 π i
Z
= xdx + 0. dx + x − dx
π −π 0 π
2
2
2
 x − π2 π i

1 h x2 0
= +
π 2 −π 2 π
2

1 h π2  1  π2 i π2 i
= 0− + − 0 + π2 −
π 2 2 4 2
1 h π 2
π 2i
a0 = − +
π 2 8

a0 = − .
8

1 π
Z
an = f (x) cos nxdx
π −π
Z π2 Z π
1h 0 π
Z i
= x cos nxdx + 0. cos nxdx + x − cos nx
π −π 0 π
2
2
1 sin nx
h    − cos nx 0
  π sin nx
   − cos nx π i
= x − 1. + x − − 1
π n n2 −π 2 n n2 π
2

1 h x sin nx cos nx 0  π  sin nx cos nx π i


an = + + x− +
π n n2 −π 2 n n2 π2

 

1 h 1 cos nπ   cos nπ cos 2 i
an = − + −
π n2 n2 n2 n2
1 h  nπ i
an = 2 1 − cos .
nπ 2

B.Sc.(Mathematics)-IDE(UNOM)-II Year-IV Sem Transform Techniques


232 11.3. Exercises

1 π
Z
bn = f (x) cos nxdx
π −π
Z π2 Z π
1h 0 π
Z i
= x sin nxdx + 0. sin nxdx + x − sin nx
π −π 0 π
2
2
1 h  − cos nx   − sin nx 0  π  − cos nx   − sin nx π i
= x − 1. + x − − 1
π n n2 −π 2 n n2 π
2

1 h x cos nx sin nx 0   π  cos nx sin nx π i


= − + + − x− +
π n n2 −π 2 n n2 π2
 

1 h  π cos nπ   π cos nπ sin 2 i
= − + − −
π n 2 n n2

1 h 3π cos nπ sin 2 i
= − . −
π 2 n n2
1 3π
h 1  nπ i
bn = (−1)n + 2 sin
π 2n n 2

Substituting these values of a0 , an and bn in (11.2.3), we get

∞ ∞
3π X 1h  nπ i 1 X 1 h 3π 1  nπ i
f (x) = − + 1 − cos cos nx − (−1) n
+ sin sin nx
16 π n=1 n2 2 π n=1 π 2n n2 2

11.3 Exercises

1.If 
if − π < x ≤ 0

0,



f (x) = 


if 0 ≤ x ≤ π.

 x,

prove that

π 2 h cos x cos 3x cos 5x i h sin 2x sin 3x i


f (x) = − + + + ... + sin x − + − ... .
4 π 12 32 52 2 3

Hence show that


1 1 π2
1+ + + ... = .
32 52 8

Transform Techniques B.Sc.(Mathematics)-IDE(UNOM)-II Year-IV Sem


11.3. Exercises 233

2. If 
π
 2 + x, if − π < x < 0




f (x) = 

 π2 − x,

if 0 < x < π.


show that
4 h cos x cos 3x cos 5x i
f (x) = + + + ... .
π 12 32 52
3. Find the Fourier series of the following function:

−1 + x, if − π < x < 0




f (x) = 


1 + x, if 0 < x < π.


Hence prove that


1 1 1 π
1− + − + ... = .
3 5 7 4
4. Find the Fourier series expansion for f (x), if

if 0 ≤ x ≤ π

 x,



f (x) = 


if π ≤ x ≤ 2π.

2π − x,

Hence prove that


1 1 1 π2
1+ + + + ... = .
32 52 72 8

5. Find the Fourier series to represent the function f (x) given by



−x , if 0 ≤ x < 0


 2

f (x) = 


 x2 , if 0 ≤ x ≤ π.


6. Express f (x) = x + |x| as Fourier series in (−π, π).

B.Sc.(Mathematics)-IDE(UNOM)-II Year-IV Sem Transform Techniques


234 11.3. Exercises

Hint : 
 x − x = 0, when − π ≤ x ≤ 0




f (x) = 


 x + x = 2x, when 0 ≤ x ≤ π.


7. If 
when 0 < x < π

 x(π − x),



f (x) = 


when 0 < x < 2π.

−π(π − x),

Prove that
8 1 1 
f (x) = sin x + 2 sin 3x + 2 sin 5x + ... .
π 3 5

Answers.
3. f (x) = π2 (π + 2) sin x − 22 sin 2x + 2(π+2)

sin 3x − 42 sin 4x + 2(π+2)

sin 5x + ..
4. f (x) = π2 − π4 cos x + cos323x + cos525x + ...
 

sin 3x − π2 sin 4x + ...


   
5. f (x) = 2 π − π4 sin x − π sin 2x + 32 π − 9π 4
   
6. f (x) = π − π4 cos x + cos323x + cos525x + ... + 2 sin x − sin22x + sin33x − ... .

Transform Techniques B.Sc.(Mathematics)-IDE(UNOM)-II Year-IV Sem


UNIT-III

Lesson 12

FOURIER SERIES OF
EVEN AND ODD FUNCTIONS
Learning Objectives
Upon completion of this lesson, students will be able to

• find Fourier series of even and odd functions in [−π, π]

• find Fourier series of functions in [−π, π].

12.1 Introduction

I
n this Lesson, we study the Fourier Series of even and odd functions.

12.2 Even and Odd Functions

Definition 12.2.1. A function f (x) is said to be even (or symmetric) function if f (−x) = f (x)
and odd (or skew - symmetric) function if f (−x) = − f (x).

For example, x2 , x4 + x+ 1, e x + e−x , cos x, sec x are all even functions of x, while x, x3 ,
x5 + 2x3 + 3, sin x, csc x, tan x are all odd functions.
Graphically an even function is symmetrical about y− axis (which is a mirror for the reflection of

B.Sc.(Mathematics)-IDE(UNOM)-II Year-IV Sem 235 Transform Techniques


236 12.3. Fourier Series for Even and Odd Functions

the curve) and an odd function is symmetrical about the origin.


The product of two even or two odd functions will be an even function while the product of an
even function and an odd function will be an odd function.
Result. Most functions are neither even nor odd.
But any function f (x) can be written as the arithmetic mean of even and odd functions as

1 1
f (x) = [ f (x) + f (−x)] + [ f (x) − f (−x)].
2 2

We shall be frequently using the following property of definite integrals:




0, when f(x) is an odd function
Z a



f (x)dx = 

 Ra
−a 
2 0 f (x)dx,

 when f(x) is an even function

Note. It is desirable to consider the even or odd nature of a function f (x) when we are dealing
with domain of definition in the form (−l, l), (−π, π) etc.

12.3 Fourier Series for Even and Odd Functions

We know that a function f (x) defined in (−π, π) can be represented by the Fourier series

∞ ∞
a0 X X
f (x) = + an cos nx + bn sin nx,
2 n=1 n=1

where

1 π
Z
a0 = f (x)dx
π −π
1 π
Z
an = f (x) cos nxdx
π −π
1 π
Z
bn = f (x) sin nxdx
π −π

Transform Techniques B.Sc.(Mathematics)-IDE(UNOM)-II Year-IV Sem


12.3. Fourier Series for Even and Odd Functions 237

Case. I When f (x) is an even function

Z π Z π
1 2
a0 = f (x)dx = f (x)dx.
π −π π 0

Since cos nx is an even function, f (x) cos nx is also an even function. Hence
Z π Z π
1 2
an = f (x) cos nxdx = f (x) cos nxdx.
π −π π 0

Again since sin nx is an odd function, f (x) sin nx is an odd function.


Z π
1
∴ bn = f (x) sin nxdx = 0.
π −π

Hence if f (x) is defined in (−π, π) and f (x) is an even function, f (x) can be expanded as Fourier
series contains only cosine terms of the form


a0 X
f (x) = + an cos nx,
2 n=1

where

2 π
Z
a0 = f (x)dx
π 0
Z π
2
an = f (x) cos nxdx
π 0

Case. II When f (x) is an odd function


Since cos nx is an even function, f (x) cos nx is an odd function. Hence
Z π
1
an = f (x) cos nxdx = 0.
π −π

Again since sin nx is an odd function, f (x) sin nx is an even function.


Z π Z π
1 2
∴ bn = f (x) sin nxdx = f (x) sin nxdx.
π −π π 0

B.Sc.(Mathematics)-IDE(UNOM)-II Year-IV Sem Transform Techniques


238 12.4. Worked out Problems

Hence if f (x) is defined in (−π, π) and f (x) is an odd function, f (x) can be expanded as Fourier
series contains only sine terms of the form


X
f (x) = an sin nx,
n=1

where
Z π
2
an = f (x) sin nxdx.
π 0

Note. The function which is neither even nor odd functions contains both sine and cosine terms.

12.4 Worked out Problems

π
Example 12.4.1. Find Fourier series of f (x) = x in −π < x < π. Deduce that 4
= 1− 13 + 51 − 17 +...

Solution. Since f (−x) = −x = − f (x). Therefore f (x) is an odd function in (−π, π).


X
∴x= bn sin nx,
n=1

where

2 π
Z
bn = x sin nxdx
π 0
2 h  − cos nx   − sin nx iπ
= x −
π n n2 0
2
= − π cos nπ

2
bn = (−1)n+1 .
n

X (−1)n+1  sin 2x sin 3x 
∴x=2 sin nx = 2 sin x − + − ... (12.4.1)
n=1
n 2 3

which is the required Fourier series.

Transform Techniques B.Sc.(Mathematics)-IDE(UNOM)-II Year-IV Sem


12.4. Worked out Problems 239

Deduction. Since the function f (x) = x is continuous, bounded and monotonic on the interval
−π < x < π, Dirichlet’s conditions apply. Therefore the series will be converges to f (x) in the
interval −π < x < π.

π  1 1 
Hence (12.4.1) ⇒ = 2 1 − + − ..
2 3 5
π 1 1
= 1 − + − ....
4 3 5

This series is called Gregory series.

Example 12.4.2. Find Fourier series of f (x) = x2 in −π < x < π. Hence deduce that
π2
(i) 1 − 1
22
+ 1
32
− 1
42
+ .... = 12
.
π2
(ii) 1 + 1
22
+ 1
32
+ 1
42
+ .... = 6
.
π2
(iii) 1 + 1
32
+ 1
32
+ 1
52
+ .... = 8
.

Solution. Since f (−x) = (−x)2 = x2 = f (x). Therefore f (x) is an even function in (−π, π).


a0 X
∴ x2 = + an cos nx (12.4.2)
2 n=1

where

2 π 2
Z
a0 = x dx
π 0
2
a0 = π2 .
3

B.Sc.(Mathematics)-IDE(UNOM)-II Year-IV Sem Transform Techniques


240 12.4. Worked out Problems

2 π 2
Z
an = x cos nxdx
π 0
2 h 2  sin nx   − cos nx   − sin nx iπ
= x − 2x + 2
π n n2 n3 0
2h cos nx i
= 0 + 2π 2 + 2.0
π n
4 cos nπ
=
n2
4
an = 2
(−1)n .
n

Substituting these values of a0 and an in (12.4.2), we get


π2 X 4
x =
2
+ (−1)n cos nx
3 n=1 n2

π2 X (−1)n+1
= −4 2
cos nx
3 n=1
n

π2  cos 2x cos 3x 
∴ x2 = − 4 cos x − + − ... (12.4.3)
3 22 32
Deduction.
(i) Putting x = 0 in (12.4.3), we get

π2  1 1 1 
0 = − 4 1 − 2 + 2 − 2 + ...
3 2 3 4

1 1 1 π2
1 − 2 + 2 − 2 + ... = . (12.4.4)
2 3 4 12
(ii) Putting x = π in (12.4.3), we get

π2  cos 2π cos 3π 
π2 = − 4 cos π − 2
+ 2
− ...
3 2 3
π 2  1 1 1 
π2 = − 4 − 1 − 2 − 2 − 2 − ...
3 2 3 4

Transform Techniques B.Sc.(Mathematics)-IDE(UNOM)-II Year-IV Sem


12.4. Worked out Problems 241

π2  1 1 1 
π2 = + 4 1 + 2 + 2 + 2 + ...
3 2 3 4

1 1 1 π2
1+ + + + ... = . (12.4.5)
22 32 42 6
(iii) Adding (12.4.4) and (12.4.5) and dividing it by 2, we get the required result as

1 1 1 π2
1+ + + + ... = .
32 52 72 8

Example 12.4.3. Expand the function f (x) = x3 as a Fourier series of in −π < x < π.

Solution. Since f (−x) = (−x)3 = −x3 = − f (x). Therefore f (x) is an odd function in (−π, π).


X
∴x =
3
bn sin nx,
n=1

where

2 π 3
Z
bn = x sin nxdx
π 0
2 h 3  − cos nx   − sin nx   cos nx   sin nx iπ
= x − 3x2 + 6x − 6
π n n2 n3 n4 0
2h π 3
6π i
= − cos nπ + 3 cos nπ
π n n
h π2 6 i
bn = 2(−1)n+1 − .
n n3
∞ h π2
X 6i
∴x =2
3
(−1)n+1 − sin nx.
n=1
n n3

Example 12.4.4. Find Fourier series of f (x) = |x| in −π < x < π. Hence deduce that
π2
1+ 1
32
+ 1
52
+ 1
72
+ .... = 8
.

Solution. Since f (−x) = | − x| = |x| = f (x). Therefore f (x) is an even function in (−π, π).


a0 X
∴ |x| = + an cos nx
2 n=1

B.Sc.(Mathematics)-IDE(UNOM)-II Year-IV Sem Transform Techniques


242 12.4. Worked out Problems

where

2 π
Z
a0 = |x|dx
π 0
2 π
Z
a0 = xdx
π 0

2  x2 π
a0 =
π 2 0
a0 = π.

2 π
Z
an = |x| cos nxdx
π 0
2 π
Z
= x cos nxdx
π 0
2 h  sin nx   − cos nx iπ
= x − 1.
π n n2 0
2 h cos nπ  1 i
= 0+ − 0+ 2
π n2 n
2
an = [(−1)n − 1].
n2 π


0, when n is even



an = 


− n42 π , when n is odd.



π 4 X 1
|x| = − cos nx.
2 π n=1,3,5,.. n2

Deduction. Put x = 0 in the above series, we get

π 4 1 1 1 
0= − + + + ...
2 π 12 32 52
1 1 1 π2
+ + + ... = .
12 32 52 8

Transform Techniques B.Sc.(Mathematics)-IDE(UNOM)-II Year-IV Sem


12.4. Worked out Problems 243
h i
Example 12.4.5. Show that for −π < x < π, sin ax = 2 sin aπ sin ax
π 12 −a2 2 −a2 + 32 −a2 + ...
− 22sin 2x 3 sin 3x
( a is not
an integer).

Solution. As sin ax is an odd function, its Fourier series expansion will consist of sine terms
only.

X
∴ sin ax = bn sin nx,
n=1

where

2 π
Z
bn = sin ax sin nxdx
π 0
1 π
Z
bn = [cos(a − n)x − cos(a + n)x]dx
π 0

1 h sin(a − n)π sin(a + n)π i


bn = −
π a−n a+n
1 h sin aπ cos nπ − cos π sin nπ sin aπ cos nπ + cos aπ sin nπ i
= −
π a−n a+n
1 h sin aπ cos nπ sin aπ cos nπ i
= −
π a−n a+n
1  1 1 
= sin aπ cos nπ −
π a−n a+n
(−1)n 2n
bn = sin aπ.
π(a2 − n2 )


2 sin aπ X (−1)n n
∴ sin ax = sin nx
π n=1
a 2 − n2


2 sin aπ X (−1)n+1 n
= sin nx
π n=1
a2 − n2
2 sin aπ h sin ax 2 sin 2x 3 sin 3x i
sin ax = − + + .....
π 12 − a2 22 − a2 32 − a2

Example 12.4.6. Expand the function f (x) = x sin x as a Fourier series in the interval −π < x <
π. Hence deduce that 1
1.3
− 1
1.5
+ 1
5.7
− .... = 41 (π − 2).

B.Sc.(Mathematics)-IDE(UNOM)-II Year-IV Sem Transform Techniques


244 12.4. Worked out Problems

Solution. Since f (x) = x sin x is an even function, bn = 0.


a0 X
x sin x = + an cos nx
2 n=1

2 π
Z
a0 = f (x)dx
π 0
Z π
2
= x sin xdx
π 0
2 π
= x(− cos x) − 1.(− sin x)
π 0
2
= [−π(−1) + 0 − (0 + 0)]
π
a0 = 2.

2 π
Z
an = f (x) cos nxdx
π 0
2 π
Z
an = x sin x cos nxdx
π 0

1 π sin(n + 1)x − sin(n − 1)x


Z
an = x
π 0 2
Z π
1
= x[sin(n + 1)x − sin(n − 1)x]
π 0
1 h  cos(n + 1)x cos(n − 1)x   sin(n + 1)x sin(n − 1)x iπ
= x − + − 1. − + (n , 1)
π n+1 n−1 (n + 1)2 (n − 1)2 0
1 h  cos(n + 1)π cos(n − 1)π i
= 2π − + , n,1
π n+1 n−1
h (−1)n+1 (−1)n−1 (−1)2 i
= − − n,1
n+1 n−1

Transform Techniques B.Sc.(Mathematics)-IDE(UNOM)-II Year-IV Sem


12.4. Worked out Problems 245
h (−1)n+1 (−1)n+1 i
= − − , n,1
n+1 n−1
hn − 1 − n − 1
= (−1)(−1)n+1 n,1
n2 − 1
2(−1)n+1
an = , n , 1.
n2 − 1

If n = 1, we have

1 π
Z
a1 = x sin 2x
π 0
1 h  − cos 2x   − sin 2x iπ
= x − 1.
π 2 4 0
1
= (−π)

1
a1 = − .
2

X 2(−1)n+1 ∞
1
∴ x sin x = 1 − cos x + 2−1
cos nx
2 n=2
n

1 X 2(−1)n+1
x sin x = 1 − cos x + cos nx
2 n=2
(n − 1)(n + 1)
1  −2 2 2 
x sin x = 1 − cos x + cos 2x + cos 3x − cos 4x + ... .
2 1.3 2.4 3.5

π
Deduction. Putting x = 2
in the above series, we get

π π 2 2 −2
sin = 1−0− (−1) + (0) + (1) + ....
2 2 1.3 2.4 3.5
π 2 2 2 2
= 1+ − + − + ....
2 1.3 3.5 5.7 7.9

π 2 2 2 2
−1 = − + − + ....
2 1.3 3.5 5.7 7.9
1 1 1 1 1
− + − + .... = (π − 2).
1.3 3.5 5.7 7.9 4

B.Sc.(Mathematics)-IDE(UNOM)-II Year-IV Sem Transform Techniques


246 12.4. Worked out Problems

Example 12.4.7. Obtain a Fourier series expansion for 1 − cos x in the interval −π ≤ x ≤ π.

Solution. Let f (x) = 1 − cos x. Since


f (−x) = 1 − cos(−x) = 1 − cos x = f (x).
p

Therefore f (x) is an even function.


√ a0 X
1 − cos x = + cos nx
2 n=1

2 π
Z
a0 = f (x)dx
π 0
2 π√
Z
= 1 − cos xdx
π 0
√ Z
2 2 π  x  θ  1 − cos θ
= sin dx, sin2 =
π 0 2 2 2
√  
x
2 2  cos 2 π
= −
π 1
2
0

4 2 π 
= − cos( ) − cos 0
√π 2
4 2
a0 = .
π

2 π
Z
an = f (x) cos nxdx
π 0
2 π√
Z
= 1 − cos x cos nxdx
π 0
√ Z
2 2 π  x
= sin cos nxdx
π 0 2
√ Z π
2 h 1  1  i
an = sin + n x + sin − n x dx
π 0 2 2

Transform Techniques B.Sc.(Mathematics)-IDE(UNOM)-II Year-IV Sem


12.4. Worked out Problems 247
√    
2 h− cos 2 + n x cos 2 − n x iπ
1 1

an = −
π 1
2
+ n 1
2
−n 0

√    
2 h− cos 2 + n π cos 2 − n π
1 1
1 1 i
= − + +
π 1
2
+n 1
2
−n 1
2
+ n 12 − n

2h 1 1 i  π  π
= + since cos nπ + = cos nπ − =0
π 12 + n 12 − n 2 2

2h 1 i
=
π 14 − n2

2h 4 i
an = .
π 1 − 4n2

√ ∞ √
√ 2 2 X 2 4 
∴ 1 − cos x = + cos nx
π n=1
π 1 − 4n 2
√ √ ∞
√ 2 2 4 2X 1
1 − cos x = − cos nx.
π π n=1 4n2 − 1

Example 12.4.8. Find the Fourier series to represent the function f (x) = sin x, −π < x < π.

Solution. Since sin x is an odd function a0 = an = 0.


X
sin x = bn sin nx,
n=1

where

2 π
Z
bn = sin x sin nxdx
π 0
Z π
1
= 2 sin x sin nxdx
π 0
1 π
Z
= [cos(1 − n)x − cos(1 + n)x]dx
π 0
1 h sin(1 − n)x sin(1 + n)x iπ
= −
π 1−n 1+n 0

B.Sc.(Mathematics)-IDE(UNOM)-II Year-IV Sem Transform Techniques


248 12.4. Worked out Problems

1 h sin(1 − n)π sin(1 + n)π i


= −
π 1−n 1+n
bn = 0, n , 1.

If n = 1, then

2 π 2
Z
b1 = sin xdx
π 0
1 π
Z
= (1 − cos 2x)dx
π 0
1 sin 2x π
= x−
π 2 0
b1 = 1.

∴ sin x = b1 sin x = sin x in (−π, π).

Example 12.4.9. Expand the function f (x) = | sin x| as a Fourier series in the interval −π < x <
π.

Solution. Since f (x) = | sin x| is an even function, bn = 0, for all n.


a0 X
| sin x| = + an cos nx
2 n=1

2 π
Z
a0 = f (x)dx
π 0
2 π
Z
= | sin x|dx
π 0
2
= (− cos x)π0
π
2
= − [−1 − 1]
π
4
a0 = .
π

Transform Techniques B.Sc.(Mathematics)-IDE(UNOM)-II Year-IV Sem


12.4. Worked out Problems 249

2 π
Z
an = f (x) cos nxdx
π 0
Z π
2
= | sin x| cos nxdx
π 0
2 π sin(n + 1)x − sin(n − 1)x
Z
= dx
π 0 2
1 π
Z
an = [sin(n + 1)x + sin(1 − n)x]dx
π 0

1 h cos(n + 1)x cos(1 − n)x iπ


an = − − (n , 1)
π n+1 1−n 0
−1 h cos(n + 1)π cos(1 − n)π  1 1 i
= + − − , n,1
π n+1 1−n n+1 n−1
1 h (−1)n+1 − 1 (−1)n+1 − 1 i
= − +
π n+1 1−n
−1 h  1 1   1 1 i
= (−1) n+1
+ − +
π 1+n 1−n 1+n 1−n
−1 h 2 2 i
= (−1)n+1 . 2 −
π n − 1 1 − n2
2
= [(−1)n+1 − 1]
π(n − 1)
2

−2
an = [1 + (−1)n ], n , 1
π(n − 1)
2



0, when n is odd



an = 


4
, when n is odd.

− (n2 −1)π

If n = 1, we have

2 π
Z
a1 = sin 2x
π 0
1  − cos 2x π
=
π 2 0
−1
= (cos 2π − 1)

a1 = 0.

B.Sc.(Mathematics)-IDE(UNOM)-II Year-IV Sem Transform Techniques


250 12.4. Worked out Problems


2 X −4
∴ | sin x| = + cos nx
π n=2,4,6,.. π(n2 − 1)

2 4 X cos nx
= − cos nx
π π n=2,4,6,.. n2 − 1

2 4 X cos 2nx
= − Replacing n by 2n
π π n=1 n2 − 1
2 4  cos 2x cos 4x 
Hence | sin x| = − + + ..... .
π π 3 15

π2 x2
Example 12.4.10. Express f (x) = 12
− 4
as a Fourier series in the interval −π < x < π.

Solution. Since
π2 (−x)2 π2 x2
f (−x) = − = − = f (x)
12 4 12 4
Therefore f (x) is an even function and hence


a0 X
f (x) = + an cos nx
2 n=1

where

2 π
Z
a0 = f (x)dx
π 0
2 π h π2 x2 i
Z
= − dx
π 0 12 4
2 h π2 x x 3 iπ
= −
π 12 12 0
a0 = 0.

and

2 π
Z
an = f (x) cos nxdx
π 0
2 π h π2 x 2 i
Z
= − cos nxdx
π 0 12 4

Transform Techniques B.Sc.(Mathematics)-IDE(UNOM)-II Year-IV Sem


12.4. Worked out Problems 251

2 h π2 x2  sin nx  −2x  − cos nx   −1  − sin nx iπ


= − − +
π 12 4 n 4 n2 2 n3 0
n
2 h 2π (−1) i
= 0− + 0 − (0 − 0 + 0)
π 4 n2
(−1)n+1
an = .
n2

Hence

X (−1)n+1 cos 2x cos 3x cos 4x
f (x) = cos nx = cos x − + − + ....
n=1
n2 22 32 42

Example 12.4.11. Prove that in the interval −π < x < π,


2 X n(−1)n
sinh ax = sinh aπ sin nx.
π n=1
n 2 + a2

eax −e−ax
Solution. Let f (x) = sinh ax = 2

Now
e−ax − eax  eax − e−ax 
f (−x) = =− = − sinh ax = − f (x).
2 2
Therefore f (x) is an odd function.


X
f (x) = bn sin nx
n=1

where

2 π
Z
bn = f (x) sin nxdx
π 0
2 π
Z
= sinh ax sin nxdx
π 0
Z π ax
2 e − e−ax
= sin nxdx
π 0 2

B.Sc.(Mathematics)-IDE(UNOM)-II Year-IV Sem Transform Techniques


252 12.4. Worked out Problems

1 h π ax
Z Z π i
= e sin nxdx − e−ax sin nxdx
π 0 0
1 h eax π  e−ax π i
= (a sin nx − n cos nx) − (−a sin nx − n cos nx)
π a2 + n2 0 a2 + n2 0
aπ −aπ
1 h e 1   e 1 i
= (−n cos nπ) − (−n) − (−n cos nπ) − (−n)
π a2 + n2 a2 + n2 a2 + n2 a2 + n2
n
= [−eaπ (−1)n + 1 + (−1)n e−aπ − 1]
π(a + n )
2 2

n
= (−1)n [e−aπ − eaπ ]
π(n + a2 )
2

n
= (−1)n+1 [eaπ − e−aπ ]
π(n + a2 )
2

n(−1)n+1
bn = 2 sinh aπ
π(n2 + a2 )


2 X n(−1)n+1
∴ sinh x = sinh aπ sinh aπ sin nx.
π n=1
n 2 + a2

Example 12.4.12. If f (x) = cosh ax, expand f (x) as a Fourier series in (−π, π).

eax +e−ax
Solution. Let f (x) = cosh ax = 2

Now
e−ax + eax
f (−x) = = f (x).
2
Therefore f (x) is an even function.


a0 X
f (x) = + an cos nx
2 n=1

where

2 π
Z
a0 = f (x)dx
π 0
2 π
Z
= cosh axdx
π 0
2  sinh ax π
=
π a 0

Transform Techniques B.Sc.(Mathematics)-IDE(UNOM)-II Year-IV Sem


12.4. Worked out Problems 253
2
= (sinh aπ − sinh 0)

2
= sinh aπ.

2 π
Z
an = f (x) cos nxdx
π 0
2 π
Z
= cosh ax sin nxdx
π 0
2 π eax + e−ax
Z
= sin nxdx
π 0 2
1 h π ax
Z Z π i
= e sin nxdx + e−ax sin nxdx
π 0 0
1 h eax π  e−ax π i
= (a cos nx + n sin nx) + (−a cos nx + n sin nx)
π a2 + n2 0 a2 + n2 0
aπ −aπ
1 h e 1   e 1 i
= (a cos nπ + 0) − (a + 0) − (−a cos nπ + 0) − (−a + 0)
π a2 + n2 a2 + n2 a2 + n2 a2 + n2
a h i
= eaπ (−1)n − 1 + [−e−aπ (−1)n + 1]
π(a2 + n2 )
a
= (−1)n [eaπ − e−aπ ]
π(n + a2 )
2

2a(−1)n
= sinh aπ.
π(n2 + a2 )


2a X 2a(−1)n
∴ cosh ax = sinh aπ sinh aπ cos nx.
π n=1
n2 + a2

Example 12.4.13. Obtain Fourier series for the function f (x) given by

− 2 (π + x), for − π < x ≤ 0


 1

f (x) = 


 12 (π − x), for 0 ≤< π.


Solution. Since
−1
f (−x) = (π − x) in (−π, 0) = − f (x) in (0, π)
2
and
1
f (−x) = (π + x) in (0, π) = − f (x) in (−π, 0)
2

B.Sc.(Mathematics)-IDE(UNOM)-II Year-IV Sem Transform Techniques


254 12.4. Worked out Problems

Therefore f (x) is an odd function in (−π, π). Here a0 = an = 0.


X
∴ f (x) = bn sin nx
n=1

where

2 π
Z
bn = f (x) sin nxdx
π 0
2 π1
Z
= (π − x) sin nxdx
π 0 2
1h  − cos nx   − sin nx iπ
= (π − x) − (−1)
π n n2 0
1 h  π i
= − 0− +0
π n
1
= .
n

X 1 1 1
∴ f (x) = sin nx = sin x + sin 2x + sin 3x + ....
n=1
n 2 3

Example 12.4.14. (a) Is 


1 + , for − π < x ≤ 0


 2x
π

f (x) = 


2x
, for 0 ≤< π.

1 −


π

(b) If so, find the Fourier series for the function.


π2
(c) Deduce that 1
12
+ 1
32
+ 1
52
+ 1
72
+ .... = 8
.

Solution. (a) Since

2x
f (−x) = 1 − in (−π, 0) = − f (x) in (0, π)
π

and
2x
f (−x) = 1 + in (0, π) = f (x) in (−π, 0)
π
Therefore f (x) is an even function in (−π, π).

Transform Techniques B.Sc.(Mathematics)-IDE(UNOM)-II Year-IV Sem


12.4. Worked out Problems 255

(b)

a0 X
∴ f (x) = + an cos nx
2 n=1

where

2 π
Z
a0 = f (x)dx
π 0
2 π
Z
2x 
= 1− dx
π 0 π
a0 = 0.

2 π
Z
an = f (x) cos nxdx
π 0
2 π
Z
2x 
= 1− cos nxdx
π 0 π
2h 2x  sin nx  2  − cos nx iπ
= Big(1 − − (− )
π π n π n2 0
2 h  cos nπ 2 i
= − 0 − 2 2 − (0 − 2
π nπ nπ
4
= [1 − (−1)n ].
n2 π2

 n2 π2 ,
 8
if n is odd



an = 



0,

 if n is even.

8 X 1 8h cos 3x cos 5x i
∴ f (x) = 2 cos nx = cos x + + + .... .
π n=1,3,5,.. n2 π2 32 52

(c) Put x = 0 in the above series, we get

81 1 1 
f (0) =
+ + + ...
π2 12 32 52
81 1 1 
1 = 2 2 + 2 + 2 + ...
π 1 3 5
1 1 1 π2
+ + + ...... = .
12 32 52 8

B.Sc.(Mathematics)-IDE(UNOM)-II Year-IV Sem Transform Techniques


256 12.5. Exercises

12.5 Exercises

x2
1. If (i) f (x) = 2x ; (ii) f (x) = 4
expand f (x) as a Fourier series in the interval (−π, π).
2. Find the Fourier series expansion of the function f (x) = x cos x, −π < x < π.
3. Find the Fourier series to represent the function f (x) = | cos x|, −π < x < π.
4. If x lies between −π and π and a is neither zero nor an integer, prove that

2a sin aπ  1 cos x cos 2x cos 3x


cos ax = + 2 − 2 + 2 − ...
π 2a 2 1 −a 2 2 −a 2 3 − a2

Hence prove that


1 2a 2a 2a
π cot aπ = + 2 + 2 − + ....
a a − 1 a − 22 a2 − 32
π cosh ax
5. Find Fourier series of 2 sinh aπ
in (−π, π).
6. Obtain Fourier series for the function f (x) given by:

for − π < x ≤ 0

−x



f (x) = 


for 0 < x < π.

 x,

and deduce the value of 1


12
+ 1
22
+ 1
32
+ ...

7. A function f (x) is defined as follows:



for − π ≤ x ≤ 0
 2
−x



f (x) = 


 x2 , for 0 ≤ x ≤ π.



 
Show that f (x) = − π8 1
13
sin x + 1
33
sin 3x + 1
53
sin 5x + ..... .

8. Given 
π + x for − π < x ≤ 0




f (x) = 


π − x, for 0 < x < π.


Is the function f (x) even? If so, find the Fourier series for f (x).

Transform Techniques B.Sc.(Mathematics)-IDE(UNOM)-II Year-IV Sem


12.5. Exercises 257

9. A periodic function f (x) is defined as follows:



2 for − π < x ≤ 0




f (x) = 


1, for0 ≤ x <≤ π.


and f (x + 2π) = f (x). Find the Fourier series for f (x).

Answers.
1.(i) x
2
= sin x. 12 sin 2x + 31 sin 3x. 14 sin 4x + ....
x2 π2
(ii). 2
= 12
− 1
12
cos x + 1
22
1
cos 2x −
32
cos 3x + ...
P∞ (−1)n n sin nx
2. x cos x = sin x +
−1
2
2 n=2 n2 −1 .
 
3. | cos x| = π2 + π4 31 cos 2x − 151 cos 4x + ...
π cosh ax P a(−1)n
5. 2 sinh aπ
= 2a1 + ∞ n=1 a2 +n2 cos nx.

f (x) = π2 − π4 112 cos x + 312 cos 3x + 512 cos 5x + ... ; π2


 
6. 8
.
f (x) = π2 + π4 112 cos x + 312 cos 3x + 512 cos 5x + ... .
 
8.
 
9. f (x) = 32 − π2 sin x + 13 sin 3x + 51 sin 5x + .... .

B.Sc.(Mathematics)-IDE(UNOM)-II Year-IV Sem Transform Techniques


258
UNIT-III

Lesson 13

HALF RANGE FOURIER SERIES


Learning Objectives
Upon completion of this lesson, students will be able to

• identify half range Fourier series in the interval [0, π]

• find half range Fourier sine series of functions in [0, π]

• find half range Fourier cosine series of functions in [0, π].

13.1 Introduction

I
t is often required to obtain Fourier series of a function f (x) in the interval (0, π).
The Sine Series. If it be required to express f (x) as a sine series in (0, π), we define an odd
function f1 (x) in (−π, π), identical with f (x) in (0, π). That is, we extend the function reflecting
it with respect to the origin, so that f (−x) = − f (x).
Hence the half range sine series in (0, π) is given by


X
f (x) = bn sin nx
n=1
Z π
2
where bn = f (x) sin nxdx.
π 0

B.Sc.(Mathematics)-IDE(UNOM)-II Year-IV Sem 259 Transform Techniques


260 13.2. Worked out Problems

The Cosine Series. If it be required to express f (x) as a cosine series in (0, π), we define an odd
function f2 (x) in (−π, π), identical with f (x) in (0, π). That is, we extend the function reflecting
it with respect to the y - axis, so that f (−x) = − f (x).
Hence the half range cosine series in (0, π) is given by


a0 X
f (x) = + an cos nx
2 n=1
2 π
Z
where a0 = f (x)dx
π 0
2 π
Z
and an = f (x) cos nxdx.
π 0

Note.1 Suppose f (x) = x in [0, π]. It can have Fourier cosine series expansion as well as Fourier
sine series expansion in [0, π].
2. Similarly f (x) = x2 in [0, π] can have Fourier cosine series expansion as well as Fourier sine
series expansion in [0, π].
We note that, if f (x) satisfies Dirichlet’s conditions in (0, π) we can have valid expansion of f (x)
in terms of sines only or cosines only in that interval.

13.2 Worked out Problems

Example 13.2.1. Find the half range cosine and sine series of f (x) = x, 0 < x < π. Hence
π2
deduce that 1
12
+ 1
32
+ 1
52
+ ... = 8
.

Solution. The half range cosine series expansion of f (x) in [0, π] is given by


a0 X
f (x) = + an cos nx
2 n=1

Transform Techniques B.Sc.(Mathematics)-IDE(UNOM)-II Year-IV Sem


13.2. Worked out Problems 261

where

2 π
Z
a0 = f (x)dx
π 0
2 π
Z
= xdx
π 0
2  x2 π
=
π 2 0
a0 = π.

2 π
Z
an = f (x) cos nxdx
π 0
2 π
Z
= x cos nxdx
π 0
2 h  sin nx   cos nx iπ
= x − 1. −
π n n2 0
n
2 h (−1) 1 i
= 0 + 2 − (0 + 2 )
π n n
2
an = [(−1)n − 1]
πn2


0, for n even



an = 


2 π , for n odd
 n−4



π 4 X 1
∴x = − cos nx
2 π n=1,3,5,.. n2
π 4  cos x cos 3x 
i.e., x = − + + ..... .
2 π 12 32

Deduction. When x = 0, f (x) = 0, i.e., f (0) = 0. Putting x = 0 in the above series, we have

π 4 1 1 1 
0 =
− + + .....
2 π 12 32 52
1 1 1 π2
+ + ..... = .
12 32 52 8

B.Sc.(Mathematics)-IDE(UNOM)-II Year-IV Sem Transform Techniques


262 13.2. Worked out Problems

The half range sine series expansion of f (x) in [0, π] is given by


X
f (x) = bn sin nx
n=1

where

2 π
Z
bn = f (x) sin nxdx
π 0
2 π
Z
= x sin nxdx
π 0
2 h  − cos nx   sin nx iπ
= x − 1. −
π n n2 0
n
2 h (−1) i 2(−1)n+1
bn = −π + 0 − (0 + 0) = .
π n n


X (−1)n+1 2
∴x = sin nx
n=1
n
 sin 2x sin 3x 
i.e., x = 2 sin x − + − ..... .
2 3

Example 13.2.2. Expand πx − x2 in half range sine series in the interval [0, π] up to three terms.
π3
Deduce that 1
13
− 1
33
+ 1
53
− 1
73
+ ..... = 32
.

Solution. The half range sine series expansion of f (x) in [0, π] is given by


X
f (x) = bn sin nx
n=1

Transform Techniques B.Sc.(Mathematics)-IDE(UNOM)-II Year-IV Sem


13.2. Worked out Problems 263

where

2 π
Z
bn = f (x) sin nxdx
π 0
2 π
Z
= (πx − x2 ) sin nxdx
π 0
2h  − cos nx   sin nx   cos nx iπ
= (πx − x2 ) − (π − 2x). − + (−2)
π n n2 n3 0
n
2 h (−1) 2 i
= 0 + 0 − 2 3 − (0 + 0 − 3 )
π n n
2h 2
= (1 − (−1)n )
π n3
4
bn = [1 − (−1)n ].
n3 π


0, for n even



bn = 


 n83 π , for n odd



X 8
∴ πx − x = 2
sin nx
n=1,3,5,..
πn3
8 sin 3x sin 5x 
i.e., πx − x2 = sin x + + + ..... .
π 33 53

π
Deduction. Putting x = 2
in the above series, we have

π π 8 π 1 3π 1 5π 
x− = sin + 3 sin + 3 sin + .....
2 2 π 2 3 2 5 2
π2 8h 1 1 1 i
= 1 − 3 + 3 − 3 + ....
4 π 3 5 7

1 1 1 π2
1− + − + .... = .
33 53 73 32

Example 13.2.3. Find cosine and sine series for f (x) = π − x in [0, π].

B.Sc.(Mathematics)-IDE(UNOM)-II Year-IV Sem Transform Techniques


264 13.2. Worked out Problems

Solution. The half range cosine series expansion of f (x) in [0, π] is given by


a0 X
f (x) = + an cos nx
2 n=1

where

2 π
Z
a0 = f (x)dx
π 0
2 π
Z
= (π − x)dx
π 0
2 x2 π
= πx −
π 2 0
2  2 π2  
= π − − (0 − 0)
π 2
a0 = π.

2 π
Z
an = f (x) cos nxdx
π 0
2 π
Z
= (π − x) cos nxdx
π 0
2h  sin nx   cos nx iπ
= (π − x) − (−1). −
π n n2 0
n
2 h (−1) 1 i
= 0 − 2 − (0 − 2 )
π n n
2h 1
= (1 − (−1)n )
π n2
2
bn = [1 − (−1)n ].
n2 π


0, for n even



an = 


 n42 π , for n odd



π 4 X 1
∴π−x = + cos nx
2 π n=1,3,5,.. n2

Transform Techniques B.Sc.(Mathematics)-IDE(UNOM)-II Year-IV Sem


13.2. Worked out Problems 265
π 4 1 cos 3x cos 5x 
i.e., π − x = + cos x + + + ..... .
2 π 12 32 52

The half - range sine series of f (x) = π − x in [0, π] is given by


X
π−x= bn sin nx
n=1

2 π
Z
bn = f (x) sin nxdx
π 0
Z π
2
= (π − x) sin nxdx
π 0
2h  − cos nx   sin nx iπ
= (π − x) − (−1). −
π n n2 0
2h 1 i
= − 0 − 0 − (−π )
π n
2
bn =
n

X 1  1 1 
∴π−x=2 sin nx = 2 sin x + sin 2x + sin 3x + ... .
n=1
n 2 3
πx
Example 13.2.4. Obtain the half range cosine and sine series for the function f (x) = 8
(π − x) in
[0, π].

Solution. The half range cosine series expansion of f (x) in [0, π] is given by


a0 X
f (x) = + an cos nx
2 n=1

B.Sc.(Mathematics)-IDE(UNOM)-II Year-IV Sem Transform Techniques


266 13.2. Worked out Problems

where

2 π
Z
a0 = f (x)dx
π 0
2 π πx
Z
= (π − x)dx
π 0 8
1 π
Z
= (π − x)dx
4 0
1  x2 x3 π
= π −
4 2 3 0
1 π
 3
π 3
π3
a0 = − = .
4 2 3 24

2 π
Z
an = f (x) cos nxdx
π 0
2 π πx
Z
= (π − x) cos nxdx
π 0 8
1 π
Z
= (πx − x2 ) cos nxdx
4 0
1h  sin nx   cos nx   − sin nx iπ
= (πx − x2 ) − (π − 2x). − + (−2)
4 n n2 n3 0
n
1 h (−1) 1 i
= 0 − π 2 + 0 − (0 + π 2 − 0)
4 n n
1 h −1
= (1 + (−1) ) n
4 n2
π
an = − 2 [1 + (−1)n ].
4n


0, when n is odd



an = 


2 , when n is even


 −π
 2n


πx π3 π X 1
∴ (π − x) = − cos nx
8 48 2 n=2,4,6,.. n2
πx π3 π  1 cos 4x cos 6x 
i.e., (π − x) = − cos 2x + + + ..... .
8 48 2 22 42 62

Transform Techniques B.Sc.(Mathematics)-IDE(UNOM)-II Year-IV Sem


13.2. Worked out Problems 267
πx
The half - range sine series of f (x) = 8
(π − x) in [0, π] is given by


X
π−x= bn sin nx
n=1

2 π
Z
bn = f (x) sin nxdx
π 0
2 π πx
Z
= (π − x) sin nxdx
π 0 8
1 π
Z
= (πx − x2 ) sin nxdx
4 0
1h  − cos nx   sin nx   cos nx iπ
= (πx − x2 ) − (π − 2x). − + (−2)
4 n n2 n3 0
n
1 h (−1) 1 i
= 0 − 0 − 2 3 − (0 − 0 − 2 3 )
4 n n
1
bn = [1 − (−1) ] n
2n3


0, when n is even



bn = 


 n13 , when n is odd



πx X 1 1 1 1
∴ (π − x) = 3
sin nx = 3 sin x + 3 sin 3x + 3 sin 5x + ....
8 n=1
n 1 3 5

Example 13.2.5. Express the function f (x) = sin x as Fourier cosine series in the interval (0, π).
Hence show that ∞n=1 4n2 −1 = 2 .
P 1 1

Solution. Since f (x) = | sin x| is an even function, bn = 0, for all n.


a0 X
sin x = + an cos nx
2 n=1

B.Sc.(Mathematics)-IDE(UNOM)-II Year-IV Sem Transform Techniques


268 13.2. Worked out Problems

2 π
Z
a0 = f (x)dx
π 0
Z π
2
= sin xdx
π 0
2
= (− cos x)π0
π
2
= − [−1 − 1]
π
4
a0 = .
π

2 π
Z
an = f (x) cos nxdx
π 0
2 π
Z
= sin x cos nxdx
π 0
Z π
2 sin(1 + n)x + sin(1 − n)x
= dx
π 0 2
1 π
Z
= [sin(1 + n)x + sin(1 − n)x]dx
π 0
1 h cos(n + 1)x cos(1 − n)x iπ
= − − (n , 1)
π n+1 1−n 0
−1 h cos(n + 1)π cos(1 − n)π  1 1 i
= + − − , n,1
π n+1 1−n n+1 n−1
1 h (−1)n+1 − 1 (−1)n+1 − 1 i
an = − +
π n+1 1−n

−1 h  1 1   1 1 i
an = (−1)n+1 + − +
π 1+n 1−n 1+n 1−n
−1 h 2 2 i
= (−1)n+1 . 2 −
π n − 1 1 − n2
2
= [(−1)n+1 − 1]
π(n − 1)
2

−2
an = [1 + (−1)n ], n , 1
π(n2 − 1)


0, when n is odd



an = 


4
, when n is odd.

− (n2 −1)π

Transform Techniques B.Sc.(Mathematics)-IDE(UNOM)-II Year-IV Sem


13.2. Worked out Problems 269

If n = 1, we have

2 π
Z
a1 = sin 2x
π 0
1  − cos 2x π
=
π 2 0
−1
= (cos 2π − 1)

a1 = 0.


2 X −4
∴ sin x = + cos nx
π n=2,4,6,.. π(n2 − 1)

2 4 X cos nx
= − cos nx
π π n=2,4,6,.. n2 − 1

2 4 X cos 2nx
sin x = − Replacing n by 2n
π π n=1 4n2 − 1

Deduction. Putting x = 0 in the above series, we have

∞ ∞
2 4X 1 X 1 1
0= − ⇒ = .
π π n=1 4n − 1
2
n=1
2
4n − 1 2

Example 13.2.6. Find the half - range sine and cosine series of f (x) = e x , 0 < x < π.

Solution. The half range sine series expansion of f (x) = e x in (0, π) is given by


X
e =x
bn sin nx
n=1

B.Sc.(Mathematics)-IDE(UNOM)-II Year-IV Sem Transform Techniques


270 13.2. Worked out Problems

where

2 π
Z
bn = f (x) sin nxdx
π 0
2 π x
Z
= e sin nxdx
π 0
2  ex π
= (sin nx − n cos nx)
π 1 + n2 0
π
2 e
 1 
= (0 − n cos nπ) − (0 − n)
π 1 + n2 1 + n2
n+1 π
2 h (−1) ne n i
= +
π 1 + n2 1 + n2
2n
bn = [1 + (−1)n+1 eπ ]
π(1 + n ) 2


2 X n[1 + (−1)n+1 eπ
∴e =
x
sin nx.
π n=1 n2 + 1

To find the half - range cosine series:


a0 X
f (x) = + an cos nx
2 n=1

where

2 π
Z
a0 = f (x)dx
π 0
Z π
2
= e x dx
π 0
2  x π
= e
π 0
2 π
= (e − 1)
π

Transform Techniques B.Sc.(Mathematics)-IDE(UNOM)-II Year-IV Sem


13.2. Worked out Problems 271

2 π
Z
an = f (x) cos nxdx
π 0
Z π
2
= e x cos nxdx
π 0
2  ex π
= (cos nx + n sin nx)
π 1 + n2 0
2  eπ 1 
an = (cos nπ + 0) − (1 + 0)
π 1 + n2 1 + n2

2 h eπ cos nπ − 1 i
an =
π 1 + n2
π
2 e (−1)n − 1 

an =
π 1 + n2

eπ − 1 2 X eπ (−1)n − 1
∴ ex = + cos nx.
π π n=1 n2 + 1

Example 13.2.7. Obtain Fourier sine series for f (x) = x sin x, 0 < x < π and show that
π−2
1
1.3
− 1
1.5
+ 1
5.7
− .... = 4
.

Solution.

a0 X
x sin x = + an cos nx
2 n=1

2 π
Z
a0 = f (x)dx
π 0
Z π
2
= x sin xdx
π 0
2 π
= x(− cos x) − 1.(− sin x)
π 0
2
= [−π(−1) + 0 − (0 + 0)]
π
a0 = 2.

B.Sc.(Mathematics)-IDE(UNOM)-II Year-IV Sem Transform Techniques


272 13.2. Worked out Problems

2 π
Z
an = f (x) cos nxdx
π 0
Z π
2
= x sin x cos nxdx
π 0
1 π sin(n + 1)x − sin(n − 1)x
Z
= x dx
π 0 2
1 π
Z
= x[sin(n + 1)x − sin(n − 1)x]dx
π 0
1 h  cos(n + 1)x cos(n − 1)x   sin(n + 1)x sin(n − 1)x iπ
= x − + − 1. − + (n , 1)
π n+1 n−1 (n + 1)2 (n − 1)2 0
1 h  cos(n + 1)π cos(n − 1)π i
= 2π − + , n,1
π n+1 n−1
h (−1)n+1 (−1)n−1 (−1)2 i
an = − − n,1
n+1 n−1

h (−1)n+1 (−1)n+1 i
an = − − , n,1
n+1 n−1
hn − 1 − n − 1i
= (−1)(−1)n+1 n,1
n2 − 1
2(−1)n+1
an = , n , 1.
n2 − 1

If n = 1, we have
Z π
a1 = x sin 2x
π 0
1 h  − cos 2x   − sin 2x iπ
= x − 1.
π 2 4 0
1
= (−π)

1
a1 = − .
2

Transform Techniques B.Sc.(Mathematics)-IDE(UNOM)-II Year-IV Sem


13.2. Worked out Problems 273
X 2(−1)n+1 ∞
1
∴ x sin x = 1 − cos x + 2−1
cos nx
2 n=2
n

1 X 2(−1)n+1
x sin x = 1 − cos x + cos nx
2 n=2
(n − 1)(n + 1)
1  −2 2 2 
= 1 − cos x + cos 2x + cos 3x − cos 4x + ... .
2 1.3 2.4 3.5

π
Deduction. Putting x = 2
in the above series, we get

π π 2 2 −2
sin = 1−0− (−1) + (0) + (1) + ....
2 2 1.3 2.4 3.5
π 2 2 2 2
= 1+ − + − + ....
2 1.3 3.5 5.7 7.9
π 2 2 2 2
−1 = − + − + ....
2 1.3 3.5 5.7 7.9
1 1 1 1 1
− + − + .... = (π − 2).
1.3 3.5 5.7 7.9 4

eax −e−ax
Example 13.2.8. Find the half - range sine series for the function f (x) = eaπ −e−aπ
in (0, π).

Solution.

X
f (x) = bn sin nx
n=1

where

2 π
Z
bn = f (x) sin nxdx
π 0
2 π eax − e−ax
Z
= sin nxdx
π 0 eaπ − e−aπ
2 hZ π Z π i
= e ax
sin nxdx − e−ax
sin nxdx
π(eaπ − e−aπ ) 0 0
2 h e ax π  e−ax π i
= (a sin nx − n cos nx) − (−a sin nx − n cos nx)
π(eaπ − e−aπ ) a2 + n2 0 a2 + n2 0
aπ −aπ
2 h e 1   e 1 i
= (−n cos nπ) − (−n) − (−n cos nπ) − (−n)
π(eaπ − e−aπ ) a2 + n2 a2 + n2 a2 + n2 a2 + n2

B.Sc.(Mathematics)-IDE(UNOM)-II Year-IV Sem Transform Techniques


274 13.2. Worked out Problems

2 1
= [−eaπ (−1)n + 1 + (−1)n e−aπ − 1]
π(e − e ) a + n
aπ −aπ 2 2

2
= (−1)n [e−aπ − eaπ ]
π(e − e )(n2 + a2 )
aπ −aπ

2
= [eaπ − e−aπ ]
π(e − e )(n + a )
aπ −aπ 2 2

2n(−1)n+1
= .
π(n2 + a2 )


eax − e−ax 2 X n(−1)n+1
∴ aπ = sin nx.
e − e−aπ π n=1 n2 + a2

Example 13.2.9. If 
π
in 0 < x <

1,


 2
f (x) = 

π

< x < π.

−1,

 in 2

expand f (x) in a series of cosines.

Solution.

a0 X
f (x) = + an cos nx
2 n=1

where

2 π
Z
a0 = f (x)dx
π 0
Z π Z π
2h 2
= 1.dx + (−1)dx
π 0 π
2

2 h π2
(x)0 − (x)ππ
i
a0 =
π 2

2 h π   π i
a0 = −0 − π−
π 2 2
a0 = 0.

Transform Techniques B.Sc.(Mathematics)-IDE(UNOM)-II Year-IV Sem


13.2. Worked out Problems 275

and

2 π
Z
an = f (x) cos nxdx
π 0
Z π Z π
2h 2
= 1. cos nxdx + (−1) cos nxdx
π 0 π
2

2 h sin nx  π2  sin nx π i
= −
π n 0 n π2
2h1 nπ  1 nπ i
= sin − 0 − 0 − sin
π n 2 n 2
2 2 nπ
= . sin
π n 2
4 nπ
an = sin
nπ 2

∴ an = 0, when n is even.

Hence


X 4 nπ
f (x) = sin cos nx
1,3,5,...
nπ 2
4 π 1 3π 1 5π 
=Big(sin cos x + sin cos 3x + sin cos 5x + ....
π 2 3 2 5 2
4 1 1 
f (x) = cos x − cos 3x + cos 5x − ...
π 3 5

Example 13.2.10. Represent the following function by Fourier sine series



π π
2, for 0 < x <



 2
f (x) = 

π

π − x, < x < π.


 in 2

expand f (x) in a series of cosines.

Solution.

X
f (x) = bn sin nx
n=1

B.Sc.(Mathematics)-IDE(UNOM)-II Year-IV Sem Transform Techniques


276 13.2. Worked out Problems

where
Z π
2
bn = f (x) sin nxdx
π 0

Z π Z π
2h 2 π
bn = . sin nxdx + (π − x) sin nxdx
π 0 2 π
2

2 h π  − cos nx  π2   − cos nx   − sin nx π i


= + (π − x) − (−1)
π 2 n 0 n n2 π
2

2 h −π nπ π π nπ 1 nπ i
= cos + + cos + 2 sin
π 2n 2 2n 2n 2 n 2
2h π 1 nπ i
bn = + sin
π 2n n2 2

Hence


X 2h π 1 nπ i
f (x) = + 2 sin sin nx
1
π 2n n 2

2 Xh π 1 nπ i
f (x) = + 2 sin
π n=1 2n n 2

Example 13.2.11. Find the Fourier cosine series of



π
cos x, in 0 < x < 2




f (x) = 

in π2 < x < π.


0,

expand f (x) in a series of cosines.

Solution.

a0 X
f (x) = + an cos nx
2 n=1

Transform Techniques B.Sc.(Mathematics)-IDE(UNOM)-II Year-IV Sem


13.2. Worked out Problems 277

where

2 π
Z
a0 = f (x)dx
π 0
Z π Z π
2h 2
= cos x.dx + 0.dx
π 0 π
2

2h πi
= (sin x)02
π
2
= (1 − 0)
π
2
a0 = .
π

and
Z π
2
an = f (x) cos nxdx
π 0

Z π Z π
2h 2
an = cos x cos nxdx + 0. cos nxdx
π 0 π
2
Z π
1 2
= [cos(n + 1)x + cos(n − 1)x]dx
π 0
1 h sin(n + 1)x sin(n − 1)x i π2
= + n,1
π n+1 n−1 0
π π
1 h sin(n + 1) 2 sin(n − 1) 2 i
= + , n,1
π n+1 n−1
1h 1 nπ π 1 nπ πi
= cos sin − cos sin , n,1
π n+1 2 2 n−1 2 2
1 nπ h 1 1 i
= cos − , n,1
π 2 n+1 n−1
2 nπ
an = − 2 cos , n , 1.
π(n − 1) 2

B.Sc.(Mathematics)-IDE(UNOM)-II Year-IV Sem Transform Techniques


278 13.2. Worked out Problems

If n = 1, then
Z π
1 2
a1 = 2 cos2 xdx
π 0
Z π
1 2
= (1 + cos 2x)dx
π 0
1 sin 2x  π2
= x+
π 2 0
1 π
h  i
= + 0 − (0 + 0)
π 2
1
a1 = .
2

Hence


1 1 2X 1 nπ
f (x) = + cos x − cos cos nx
π 2 π n=1 n2 − 1 2
1 1 2h cos π 1 3π i
= + cos x − cos 2x + cos cos 3x + ....
π 2 π (2 − 1)(2 + 1) (3 − 1)(3 + 1) 2
1 1 2 h cos 2x cos 4x cos 6x i
= + cos x − − + − + ...
π 2 π 1.3 3.5 5.7
1 1 2 h cos 2x cos 4x cos 6x i
f (x) = + cos x + + + + ....
π 2 π 1.3 3.5 5.7
4 1 1 
f (x) = cos x − cos 3x + cos 5x − ...
π 3 5

Example 13.2.12. Find a Fourier sine series of



π
for 0 < x <

 x,


 2
f (x) = 

π

π − x, < x < π.


 in 2

Solution.

X
f (x) = bn sin nx
n=1

Transform Techniques B.Sc.(Mathematics)-IDE(UNOM)-II Year-IV Sem


13.2. Worked out Problems 279

where

2 π
Z
bn = f (x) sin nxdx
π 0
Z π Z π
2h 2
= x sin nxdx + (π − x) sin nxdx
π 0 π
2

2 h  − cos nx   − sin nx  π2   − cos nx   − sin nx π i


= x − 1. + (π − x) − (−1)
π n n2 0 n n2 π
2

2 h −π nπ 1 nπ   π nπ 1 nπ i
= cos + 2 sin −0 + 0+ cos + 2 sin
π 2n 2 n 2 2n 2 n 2
4 nπ
bn = sin
πn2 2

bn = 0, when n is even

and
4 nπ
bn = sin , when n is odd.

2 2
Hence


X 4 nπ
f (x) = 2π
sin sin nx
1
n 2
4h 1 nπ 1 3π 1 5π i
= sin sin x + sin sin 3x + sin sin 5x + ...
π 12 2 32 2 52 2
4 h 1 1 i
= sin x − 2 sin 3x + 2 sin 5x − .... .
π 3 5

Example 13.2.13. Find a Fourier cosine series of



for 0 < x < π2

 x,



f (x) = 

π − x, in π2 < x < π.



Solution.

a0 X
f (x) = + an cos nx
2 n=1

B.Sc.(Mathematics)-IDE(UNOM)-II Year-IV Sem Transform Techniques


280 13.2. Worked out Problems

where

2 π
Z
a0 = f (x)dx
π 0
Z π Z π
2h 2 i
= xdx + (π − x)dx
π 0 π
2
2 π
2 h x  2  x2 π i
= + πx −
π 2 0 2 π2
2 h π2  2 π2   π2 π2 i
= + π − − −
π 8 2 2 8
2
2 2π

=
π 8
π
a0 = .
2

and

2 π
Z
an = f (x) cos nxdx
π 0
Z π Z π
2h 2
= x cos nxdx + (π − x) cos nxdx
π 0 π
2

2 h  sin nx   − cos nx  π2   sin nx   − cos nx π i


= x − 1. + (π − x) − (−1)
π n n2 0 n n2 π
2

2 h π nπ 1 nπ 1   cos nπ π nπ 1 nπ i
= sin + 2 cos − 2 + − − sin + cos
π 2n 2 n 2 n n2 2n 2 n2 2
2 h nπ i
an = 2 cos − 1 − (−1) n
πn2 2

an = 0, when n is odd

and
2 h nπ i
an = 2 cos − 2 , when n is even.
n2 π 2

Transform Techniques B.Sc.(Mathematics)-IDE(UNOM)-II Year-IV Sem


13.2. Worked out Problems 281

Hence


π X 2 h nπ i
f (x) = + 2 cos − 2 cos nx
4 n=2,4,6,.. n2 π 2

π X 2
= + [2 cos nπ − 2] cos 2nx, Replacing n with 2n
4 n=1 π(4n)2

π 1 X 1
f (x) = + 2
[2(−1)n − 2] cos 2nx
4 2π n=1 n


π 1 X  −2 
f (x) = + cos 2nx
4 π n=1,3,5,.. n2
π 2h 1 1 1 i
f (x) = − 1 2 cos 2x + 2 cos 6x + 2 cos 10x + ...... .
4 π 1 3 5

Example 13.2.14. Find the Fourier sine series of f (x) = cos x in 0 < x < π.

Solution.

X
f (x) = bn sin nx
n=1

where

2 π
Z
bn = f (x) sin nxdx
π 0
2 π
Z
= cos x cos nxdx
π 0
Z π
1 2
= [sin(n + 1)x + sin(n − 1)x]dx
π 0
1 h − cos(n + 1)x cos(n − 1)x iπ
= − n,1
π n+1 n−1 0

B.Sc.(Mathematics)-IDE(UNOM)-II Year-IV Sem Transform Techniques


282 13.2. Worked out Problems

1 h (−1)n+1 (−1)n 1 1 i
= − + + + , n,1
π n+1 n−1 n+1 n−1
1h  1 1  1 1 i
= (−1)n + + + , n,1
π n+1 n−1 n+1 n−1
1h  1 1 i
= [(−1)n + 1] + + , n,1
π n+1 n−1
2n h 1 + (−1)n i
bn = , n , 1.
π n2 − 1


0, when n is odd



bn = 


2 −1) ,
 π(n4n


 when n is even.

If n = 1, then

2 π
Z
b1 = cos x sin xdx
π 0
Z π
1
= sin 2xdx
π 0
1  − cos 2x π
=
π 2 0
1h i
b1 = − cos 2π − cos 0 = 0.

Hence


X 4n
f (x) = sin nx
n=2,4,6,...
π(n2 − 1)

4 X 2n
= sin 2nx, Replacing n by 2n
π n=1 4n2 − 1
81 2 3 
= sin 2x + sin 4x + sin 6x + ....
π 3 15 35
4 2 4 6 
f (x) = sin 2x + sin 4x + sin 6x + ...... .
π 1.3 3.5 5.7

Transform Techniques B.Sc.(Mathematics)-IDE(UNOM)-II Year-IV Sem


13.3. Exercises 283

13.3 Exercises

1. Express f (x) = 1 as Fourier sine series in (0, π).


2. Find cosine and sine series for f (x) = π − x, in [0, π].
3. Obtain cosine and sine series for the function f (x) = x2 in [0, π] and hence find the sum of
the series 1
12
− 1
22
+ 1
32
− 1
42
+ ....
4. Express f (x) = x3 as Fourier sine series in (0, π).
5. Find the Fourier sine series of eax in (0, π).
πx
6. Obtain the half range sine series for the function f (x) = 8
(π − x) in the range 0 ≤ x ≤ π.
7. Find the half range cosine series for the function f (x) = x sin x in (0, π). Hence deduce that
1
2
+ 1
1.3
− 1
3.5
+ 1
5.7
− .... = π4 .
8. Obtain the Fourier expansion of x cos x as a sine series in the interval 0 < x < π.
9. Express f (x) = 1 + 2 cos x + 3 cos2 x + 4 cos3 x as a Fourier cosine series in (0, π).
10. Express f (x) = 2 sin 2x cos x as Fourier sine series in (0, π).
11. If 
πx
4, for 0 < x < π2




f (x) = 

 π4 (π − x), for π2 < x < π.





then show that (i) f (x) = sin x − 312 sin 3x + 512 sin 5x − 1
72
sin 7x + ....
π2
 
(ii) f (x) = 16 − 12 112 cos x + 312 cos 6x + ....
12. Find the Fourier sine and cosine series in 0 < x < π for the function

π
, when 0 < x < π3





 3


f (x) = 

when π3 < x < 2π


0, 3



− π , when 2π < x < π.



3 3

B.Sc.(Mathematics)-IDE(UNOM)-II Year-IV Sem Transform Techniques


284 13.3. Exercises

Answers.
h i
1. 1 = π4 sin x + sin 3x
3
+ sin 5x
5
+ ..... .
π
h i
2. π − x = 2
+ 4 cos x
π 12
+ cos 3x
32
+ cos 5x
52
+ .... .
h i
π − x = 2 sin x + sin22x + sin33x + .... .
3. x2 = π3 − 4 cos
2
h i
12
x
− cos 2x
22
+ cos 3x
32
− cos 4x
42
+ ....
h   i
x = π (π − 4) sin x − 2 π sin 2x + 13 π2 − 342 sin 3x − 14 π2 sin 4x + ......
2 2 2 1 2

π2
 
4. f (x) = 2 ∞ n 6
P
n=1 (−1) n3 − n
sin nx.
h aπ aπ aπ ) i
5. eax = π2 1+e
a2 +1
sin x + 2(1−ea2 +22
)
sin 2x + 3(1+e
a2 +32
sin 3x + ...
6. f (x) = sin x
13
+ sin 3x
33
+ sin 5x
53
+ ...
7. x sin x = 1 − cos 2x
2
− 2 cos 2x
1.3
+ 2 cos 3x
2.4
− 2 cos 4x
3.5
+ ....
h i
8. x cos x = − 12 sin x + 2 a sin 2x
1.3
− 3 sin 3x
2.4
+ 4 sin 4x
3.5
− ....
9. f (x) = 5
2
+ 5 cos x + 23 cos 2x + cos 3x + ....
10. 2 sin 2x cos x = sin x + sin 3x.
12. f (x) = sin 2x + 21 sin 4x + 41 sin 8x + 15 sin 10x + 71 sin 4x + .....
h i
f (x) = √23 cos x − 15 cos 5x + 17 cos 7x − 11 1
cos 11x + ...... .

Transform Techniques B.Sc.(Mathematics)-IDE(UNOM)-II Year-IV Sem


UNIT-III

Lesson 14

OTHER FORMS OF FOURIER SERIES


Learning Objectives
Upon completion of this lesson, students will be able to

• identify Fourier series in the interval [c, c + 2l] and [−l, l]

• find the Fourier series of functions in [c, c + 2l] and [−l, l].

14.1 Introduction

S
o far, we have expanded a given function in a Fourier series over the intervals (−π, π)
and (0, 2π) of length 2π. In most engineering problems, the period of the function to
be expanded is not 2π but some other quantity say 2l. In order to apply earlier discussions to
functions of period 2l, this interval must be converted to the length 2π. This involves only a
proportional change in the scale.

Let f (x) be a periodic function with period 2l defined in the interval c < x < c + 2l. We must
introduce a new variable z such that the period becomes 2π.

2l 2π l πx
Put = i.e., x = z or z = .
x z π l
πc
When x = c, z = = d (say)
l

B.Sc.(Mathematics)-IDE(UNOM)-II Year-IV Sem 285 Transform Techniques


286 14.1. Introduction

π πc
when x = c + 2l, z = (c + 2l) = + 2π = d + 2π.
l l
Hence the function f (x) of period 2l in (c, c + 2l) is transferred to the new function

 lz 
f = F(z) (say)
π

of period 2π in the interval d < z < d + 2π.


Thus F(z) being of period 2π can be expanded in a Fourier series.

∞ ∞
a0 X X
F(z) = + an cos nx + bn sin nx (14.1.1)
2 n=1 n=1

where

1 d+2π
Z
a0 = F(z)dz,
π d
1 d+2π
Z
an = F(z) cos nzdz,
π d
1 d+2π
Z
and bn = F(z) sin nzdz
π d

πx
To change the variable z to x, put z = l
so that dz = πl dx.
When z = d, x = c and when z = d + 2π, x = c + 2l. Also

 lz 
F(z) = f = f (x)
π

Then

1 c+2l π 1 c+2l
Z Z
a0 = f (x). dx = f (x)dx,
π c l l c
1 c+2l nπx π 1 c+2l
Z Z
nπx
an = f (x) cos . dx = f (x) cos dx
π c l l l c l
1 c+2l nπx π 1 c+2l
Z Z
nπx
bn = f (x) sin . dx = f (x) sin dx
π c l l l c l

Transform Techniques B.Sc.(Mathematics)-IDE(UNOM)-II Year-IV Sem


14.1. Introduction 287

Using these a0 , an , and bn in (14.1.1), the Fourier expansion for f (x) in the interval c < x <
c + 2l is given by
∞ ∞
a0 X nπx X nπx
f (x) = + an cos + bn sin
2 n=1 l n=1
l

where

1 c+2l
Z
a0 = f (x)dx,
l c
1 c+2l
Z
nπx
an = f (x) cos dx (14.1.2)
l c l
1 c+2l
Z
nπx
bn = f (x) sin dx.
l c l

Fourier series of f (x) defined in [0, 2l].


Case (i) Putting c = 0, the interval c < x < c + 2l becomes 0 < x < 2l and the formulae (14.1.2)
reduces to

1 2l
Z
a0 = f (x)dx,
l 0
1 2l
Z
nπx
an = f (x) cos dx
l 0 l
1 2l
Z
nπx
bn = f (x) sin dx.
l 0 l

Fourier series of f (x) defined in [−l, l].


Case (ii) Putting c = −l, the interval c < x < c + 2l becomes −l < x < l and the formulae
(14.1.2) reduces to

1 l
Z
a0 = f (x)dx,
l −l
1 l
Z
nπx
an = f (x) cos dx
l −l l
1 l
Z
nπx
bn = f (x) sin dx.
l −l l

Note. It can be verified directly that, when m, n are integers

B.Sc.(Mathematics)-IDE(UNOM)-II Year-IV Sem Transform Techniques


288 14.2. Fourier Series for Even and Odd Functions in (−l, l)

Z c+2l  nπx   nπx 


sin cos dx = 0
c l l







0, if m , n,
Z c+2l  nπx   nπx  

dx = 

sin sin l, if m = n , 0,

c l l 




0, if m = n = 0








0, if m , n,
Z c+2l  nπx   nπx  

dx = 

cos cos if m = n , 0,

l,
c l l 




if m = n = 0


2l,

14.2 Fourier Series for Even and Odd Functions in (−l, l)

Case.(i) Fourier Cosine Series


Let f (x) be even function in (−l, l). Then
Z l Z l
1 2
a0 = f (x)dx = f (x)dx.
l −l l 0

Since f (x) cos nπx


l
is also an even function,

Z l Z l
1 nπx 2 nπx
∴ an = f (x) cos dx = f (x) cos dx.
l −l l l 0 l

Again since f (x) sin nπx


l
is also an odd function,

Z l
1 nπx
∴ bn = f (x) sin dx = 0.
l −l l

Transform Techniques B.Sc.(Mathematics)-IDE(UNOM)-II Year-IV Sem


14.2. Fourier Series for Even and Odd Functions in (−l, l) 289

Hence if f (x) is even in the interval (−l, l) then the Fourier cosine series is


a0 X nπx
f (x) = + an cos
2 n=1 l
Z l
2
where a0 = f (x)dx,
l 0
2 l
Z
nπx
and an = f (x) cos dx
l 0 l

Case.(ii) Fourier Sine Series


Let f (x) be odd function in (−l, l). Then
Z l
1
a0 = f (x)dx = 0.
l −l

Since f (x) cos nπx


l
is also an odd function,

Z l
1 nπx
∴ an = f (x) cos dx = 0.
l −l l

Again since f (x) sin nπx


l
is an even function,

Z l Z l
1 nπx 2 nπx
∴ bn = f (x) sin dx = f (x) sin dx.
l −l l l 0 l

Hence if f (x) is an odd in the interval (−l, l) then the Fourier sine series is


X nπx
f (x) = bn sin
n=1
l
Z l
2 nπx
where bn = f (x) sin dx
l 0 l

Remark. The even function or odd function is to be considered only if the function defined in the
interval (−l, l).
Note.
1. In the above discussion, if we put 2l = 2π, i.e., l = π, we get the discussions regarding the

B.Sc.(Mathematics)-IDE(UNOM)-II Year-IV Sem Transform Techniques


290 14.3. Worked out Problems

intervals (0, 2π) and (−π, π) as special cases.


2. If f (x) is the sum of an odd function and an even function then the Fourier series of f (x) is
obtained by adding the Fourier series of odd and even functions.

14.3 Worked out Problems

Example 14.3.1. Express f (x) = x2 as Fourier series in [−l, l].

Solution. Since f (x) = x2 is an even function, the corresponding Fourier series is given by


a0 X nπx
f (x) = + an cos . (14.3.1)
2 n=1 l

where

2 l
Z
a0 = f (x)dx
l 0
2 l 2
Z
= x dx
l 0
2  x3 l
=
l 3 0
22
a0 = l .
3

and

2 l
Z
nπx
an = f (x) cos dx
l 0 l
2 l 2
Z
nπx
= x cos dx
l 0 l
nπx  − cos nπx  sin nπx
2 h 2  sin l   il
= x nπ − 2x n π
2 2
l
+ 2 − n π
3 3
l
l l 2 3
0
l l

Transform Techniques B.Sc.(Mathematics)-IDE(UNOM)-II Year-IV Sem


14.3. Worked out Problems 291
nπx
2 h  cos l il
= 2x n2 π2
l 2
0
l
4l2
an = 2 2 (−1)n .

Substituting the values of a0 and an in (14.3.3), we get


l2 X 4l2 nπx
x =
2
+ (−1)n cos
3 n=1 n π 2 2 l

l2 4l2 X (−1)n nπx
= + 2
cos
3 3 n=1 n l
l2 4l2 h 1 πx 1 2πx 1 3πx i
= + − 2 cos + 2 cos − 2 cos + ...
3 3 1 l 2 l 3 l
l 2
4l 2h
1 πx 1 2πx 1 3πx i
x2 = − cos − cos + cos − ...
3 3 12 l 22 l 32 l

Deduction. Obtain the Fourier series for f (x) = x2 in −1 < x < 1.


Solution. Taking l = 1 in the above problem, we get

1 4h 1 1 1 i
x2 = − cos πx − cos 2πx + cos 3πx − ... .
3 3 12 22 32

Example 14.3.2. Obtain Fourier series expansion for sin ax in the interval −l < x < l.

Solution. Since sin ax is an odd function in (−l, l) , therefore, the required series is of the form


X nπx
sin ax = bn sin (14.3.2)
n=1
l

where

2 l
Z
nπx
bn = f (x) sin dx
l 0 l
2 l
Z
nπx
= sin ax cos dx
l 0 l

B.Sc.(Mathematics)-IDE(UNOM)-II Year-IV Sem Transform Techniques


292 14.3. Worked out Problems
Z lh
1  nπ   nπ  i
= cos a − x − cos a + x dx
l 0 l l
   
1 hsin a − nπ
l
x sin a + nπ
l
x il
= −
l a − nπl a + nπl 0

1 h sin al cos nπ sin al cos nπ i


= −
l a − nπl a − nπl
(−1)n sin al h 1 1 i
= −
l a − nπl a + nπl
(−1)n+1 2nπ sin al
bn = .
n2 π2 − a2 l2


X (−1)n+1 2nπ sin al nπx
∴ (14.3.2) ⇒ sin ax = 2π sin al sin
n=1
n2 π2 − a2 l 2 l
h 1 πx 2 2πx 3 3πx i
sin ax = 2π sin al sin − 2 sin + 2 sin − ... .
π2 2
−a l 2 l 2
2π − a l 2 l 2
3π − a l 2 l

Example 14.3.3. Find the Fourier series to represent f (x) = 1 − x2 in the interval −l ≤ x ≤ l.

Solution. Since f (x) = 1 − x2 is an even function, the corresponding Fourier series is given by


a0 X nπx
f (x) = + an cos . (14.3.3)
2 n=1 l

Here l = 1

a0 X
∴ 1 − x2 = + an cos nπx
2 n=1

where

2 l
Z
a0 = f (x)dx
1 0
2 l
Z
= (1 − x2 )dx
1 0
 x3 1  1 4
= 2 x− =2 1− = .
3 0 3 3

Transform Techniques B.Sc.(Mathematics)-IDE(UNOM)-II Year-IV Sem


14.3. Worked out Problems 293

and
Z 1
2
an = f (x) cos nπxdx
1 0
Z 1
= 2 (1 − x2 ) cos nπxdx
0
h  sin nπx   − cos nπx   sin nπx i1
= 2 (1 − x2 ) − (−2x) + (−2) − 3 3
nπ n2 π2 nπ 0
4
= − 2 2 cos nπ

4
an = 2 2 (−1)n+1 .


2 4 X (−1)n+1
1−x = + 2
cos nπx
3 π2 n=1 n2
2 4h1 1 1 i
= + 2 2 cos πx − 2 cos 2πx + 2 cos 3πx − ...
3 π 1 2 3

Example 14.3.4. Find the Fourier series of



π
− 2 , 0 < x < 2


 1

f (x) = 

 π2 ,

1
< x < 2.



2

Solution. Let
∞ ∞
a0 X nπx X nπx
f (x) = + an cos + bn sin
2 n=1 l n=1
l

Here 2l = 1 ⇒ l = 12 . Thus

∞ ∞
a0 X X
f (x) = + an cos 2nπx + bn sin 2nπx (14.3.4)
2 n=1 n=1

B.Sc.(Mathematics)-IDE(UNOM)-II Year-IV Sem Transform Techniques


294 14.3. Worked out Problems

where
Z 2l
1
a0 = f (x)dx
l 0
Z 1
= 2 f (x)dx
0
1
π π i
hZ 2
Z 1
= 2 − dx + dx
0 2 1
2
2
h π 1 π i
= 2 − (x)0 + (x)11
2

2 2 2
h π πi
= 2 − +
4 4
a0 = 0.

Z 2l
1 nπx
an = f (x) cos dx
l 0 l
Z 1
= 2 f (x) cos 2nπxdx
0
1
π π
hZ 2
Z 1 i
= 2 − cos 2nπxdx + cos 2nπxdx
0 2 1
2
2
h π  sin 2nπx  21 π  sin 2nπx 1 i
= 2 − +
2 2nπ 0 2 2nπ 21
h π π i
= 2 − (0 − 0) + (0 − 0)
2 2
an = 0.

Z 2l
1 nπx
bn = f (x) sin dx
l 0 l
Z 1
bn = 2 f (x) sin 2nπxdx
0

Transform Techniques B.Sc.(Mathematics)-IDE(UNOM)-II Year-IV Sem


14.3. Worked out Problems 295
1
π π
hZ 2
Z 1 i
bn = 2 − sin 2nπxdx + sin 2nπxdx
0 2 1
2
2
h π  − cos 2nπx  21 π  cos 2nπx 1 i
= 2 − + − 1
2 2nπ 0 2 2nπ 2

1 1
= (cos 2nπ − 1) − (cos 2nπ − cos nπ)
2n 2n
1 h   i
bn = (−1)n − 1 − 1 − (−1)n
2n



0, when n is even



bn = 


− 2n , when n is odd.



X −2 h 1 1 i
∴ (14.3.4) ⇒ f (x) = sin 2nπx = (−2) sin 2πx + sin 6πx + sin 10πx + .... .
n=1,3,5,...
n 3 5

Example 14.3.5. Develop f (x) as Fourier series in (−2, 2) if


0, −2 < x < −1








f (x) = 

k, −1 < x < 1






0, 1 < x < 1


in the interval 0 < x < 1.

Solution. The Fourier series of f (x) in (−l, l) is given by

∞ ∞
a0 X nπx X nπx
f (x) = + an cos + bn sin
2 n=1 l n=1
l

Here l = 2. Thus
∞ ∞
a0 X nπx X nπx
f (x) = + an cos + bn sin (14.3.5)
2 n=1 2 n=1
2

B.Sc.(Mathematics)-IDE(UNOM)-II Year-IV Sem Transform Techniques


296 14.3. Worked out Problems

where

1 2
Z
a0 = f (x)dx
2 −2
1 h −1
Z Z 1 Z 2 i
= 0.dx + k.dx + 0.dx
2 −2 −1 1
k 1
= (x)−1 = k.
2

1 l
Z
nπx
an = f (x) cos dx
l −l l
1 2
Z
2nπx
= f (x) cos dx
2 −2 2
1 1
Z
nπx
= k. cos dx
2 −1 2
Z 1
nπx
= k cos dx
0 2
 sin nπx 1 2k nπ
= k nπ 2 = sin .
2
0 nπ 2



0, when n is even



an = 


2k
sin nπ ,



 nπ 2
when n is odd.

1 l
Z
nπx
bn = f (x) sin dx
l −l l
1 2
Z
2nπx
= f (x) sin dx
2 −2 2
1 1
Z
nπx
= k. sin dx
2 −1 2
nπx
= 0, since sin is odd.
2

k 2k h πx 1 3πx 1 5πx i
∴ (14.3.5) ⇒ f (x) = + cos − cos + cos − .... .
2 π 2 3 2 5 2

Transform Techniques B.Sc.(Mathematics)-IDE(UNOM)-II Year-IV Sem


14.3. Worked out Problems 297
π−x
Example 14.3.6. Find the Fourier series of f (x) = 2
in 0 < x < 2.

Solution. Let
∞ ∞
a0 X nπx X nπx
f (x) = + an cos + bn sin
2 n=1 l n=1
l

Here 2l = 2 ⇒ l = 1. Thus

∞ ∞
a0 X X
f (x) = + an cos nπx + bn sin nπx (14.3.6)
2 n=1 n=1

where
Z 2l
1
a0 = f (x)dx
l 0
2
π−x
Z
= dx
0 2
1 x2 2
= πx −
2 2 0
a0 = π − 1

1 2l
Z
nπx
an = f (x) cos dx
l 0 l
Z 2
= f (x) cos nπxdx
0
π−x
Z 2
= cos nπxdx
0 2
1h  sin nπx   cos nπx i2
= (π − x) − (−1) − 2 2
2 nπ nπ 0
1 h cos 2nπ 1 i
= − 2 2 + 2 2
2 nπ nπ
1
= (1 − cos 2nπ)
2n2 π2
1
an = (1 − 1)
2n2 π
an = 0.

B.Sc.(Mathematics)-IDE(UNOM)-II Year-IV Sem Transform Techniques


298 14.3. Worked out Problems
Z 2l
1 nπx
bn = f (x) sin dx
l 0 l
Z 2
= f (x) sin nπxdx
0
1 2
Z
= (π − x) sin nπxdx
2 0
1h  cos nπx   sin nπx i2
= (π − x) − − (−1) − 2 2
2 nπ nπ 0
1 h cos 2nπ 1 i
= − 2 2 + 2 2
2 nπ nπ
1
bn = − (π − 2 − π)
2nπ

1
bn =


π−1 X 1
∴ (14.3.6) ⇒ f (x) = + sin nπx.
2 n=1

Example 14.3.7. Find the Fourier series with period 3 to represent f (x) = x + x2 in (0, 3).

Solution. Let
∞ ∞
a0 X nπx X nπx
f (x) = + an cos + bn sin
2 n=1 l n=1
l

Here 2l = 3 ⇒ l = 32 . Thus

∞ ∞
a0 X 2nπx X 2nπx
f (x) = + an cos + bn sin (14.3.7)
2 n=1 3 n=1
3

where

1 2l
Z
a0 = f (x)dx
l 0
2 3
Z
= (x + x2 )dx
3 0
2 h x 2 x 3 i3
= + = 9.
3 2 3 0

Transform Techniques B.Sc.(Mathematics)-IDE(UNOM)-II Year-IV Sem


14.3. Worked out Problems 299

1 2l
Z
nπx
an = f (x) cos dx
l 0 l
2 3
Z
2nπx
= (x + x2 ) cos dx
3 0 3
2h  sin 2nπx   − cos 2nπx   − sin 2nπx i3
= (x + x ) 2nπ
2 3
− (1 + 2x) 4n2 π2
3
+ (2) 8n3 π3
3
3 3
0
9 27
2h 3 9 i
= −
3 4n2 π2 4n2 π2
9
an = .
n π2
2

1 2l
Z
nπx
bn = f (x) sin dx
l 0 l
2 3
Z
2nπx
= (x + x2 ) sin dx
3 0 3
2h  − cos 2nπx   − sin 2nπx   cos 2nπx i3
= (x + x )
2
2nπ
3
− (1 + 2x) 4n π
2 2
3
+ (2) 8n π
3 3
3
3 3
0
9 27
12
bn = − .

∞ ∞
9 9 X 1 2nπx 12 X 1 2nπx
∴ (14.3.13) ⇒ f (x) = + 2 cos − sin .
2 π n=1 n2 3 π n=1 n 3

Example 14.3.8. Expand f (x) = e−x as a Fourier series in the interval (−1, 1).

Solution. Let
∞ ∞
a0 X nπx X nπx
f (x) = + an cos + bn sin
2 n=1 l n=1
l

Here l = 1. Thus
∞ ∞
a0 X X
f (x) = + an cos nπx + bn sin nπx (14.3.8)
2 n=1 n=1

B.Sc.(Mathematics)-IDE(UNOM)-II Year-IV Sem Transform Techniques


300 14.3. Worked out Problems

where

1 l
Z
a0 = f (x)dx
l −l
Z 1
= e−x dx
−1

= −(e−x )1−1
 e1 − e−1 
= 2
2
a0 = 2 sinh 1.

1 l
Z
nπx
an = f (x) cos dx
l −l l
Z 1
= f (x) cos nπxdx
−1
Z 1
= e−x cos nπxdx
−1
h e−x i1
= (− cos nπx + nπ sin nπx)
1 + n2 π2 −1
1 h i
an = 2 e−1
(− cos nπ + nπ sin nπ) − e(− cos nπ + nπ sin nπ)
n + π2

1 h i
an = − e−1
(−1) n
+ e(−1)n
n2 + π2
(−1)n
= (e − e−1 )
1 + n2 π2
(−1)n 2 sinh 1
an = .
1 + n2 π2

Transform Techniques B.Sc.(Mathematics)-IDE(UNOM)-II Year-IV Sem


14.3. Worked out Problems 301

1 l
Z
nπx
bn = f (x) cos dx
l −l l
Z 1
= e−x sin nπxdx
−1
Z 1
= e−x cos nπxdx
−1
h e−x i1
= (− sin nπx − nπ cos nπx)
1 + n2 π2 −1
1 h −1 i
= e (− sin nπ − nπ cos nπ) − e(sin nπ − nπ cos nπ)
n2 + π2
1 h i
= − nπe−1
(−1)n
+ e(−1) n

n2 + π2
nπ(−1)n
= (e − e−1 )
1+n π 2 2
2nπ(−1)n sinh 1
bn = .
1 + n2 π2
∞ ∞
X (−1)n 2 sinh 1 X 2nπ(−1)n sinh 1
∴ (14.3.8) ⇒ f (x) = sinh 1 + cos nπx + sin nπx.
n=1
1 + n2 π2 n=1
1 + n2 π2

Example 14.3.9. Express f (x) = x2 − 2 as Fourier series in [−2, 2].

Solution. Since f (x) = x2 − 2 is an even function, the corresponding Fourier series is given by


a0 X nπx
f (x) = + an cos . (14.3.9)
2 n=1 l

Here l = 2.

a0 X nπx
∴ f (x) = + an cos .
2 n=1 2

where
Z l
2
a0 = f (x)dx
l 0

B.Sc.(Mathematics)-IDE(UNOM)-II Year-IV Sem Transform Techniques


302 14.3. Worked out Problems

2 2 2
Z
a0 = (x − 2)dx
2 0
 x3 l
= − 2x
3 0
4
a0 = − .
3

and

2 l
Z
nπx
an = f (x) cos dx
l 0 l
2 2 2
Z
nπx
= x cos dx
2 0 2
h  sin nπx   − cos nπx   sin nπx i2
= (x2 − 2) nπ 2 − 2x n π
2 2
2
+ 2 − n π
3 3
2
0
2 4 8
h 16  i
= 0 + 2 2 cos nπ + 0 − (0 + 0 − 0)

16
an = 2 2 (−1)n .

Substituting the values of a0 and an in (14.3.9), we get


2 16 X (−1)n nπx
x2 − 2 = − + 2 cos
3 π n=1 n2 2
2 16 h 1 πx 1 2πx 1 3πx i
= − + 2 − 2 cos + 2 cos − 2 cos + ...
3 π 1 2 2 2 3 2
2 16 h1 πx 1 2πx 1 3πx i
x2 = − − 2 2 cos − 2 cos + 2 cos − ...
3 π 1 2 2 2 3 2

Example 14.3.10. Express f (x) = 3x2 − 2 as Fourier series in the interval (−3, 3).

Solution. Since f (x) = 3x2 − 2 is an even function, the corresponding Fourier series is given by


a0 X nπx
f (x) = + an cos . (14.3.10)
2 n=1 l

Transform Techniques B.Sc.(Mathematics)-IDE(UNOM)-II Year-IV Sem


14.3. Worked out Problems 303

Here l = 3.

a0 X nπx
∴ f (x) = + an cos .
2 n=1 3

where

2 l
Z
a0 = f (x)dx
l 0
2 3 2
Z
= (3x − 2)dx
3 0
2  3x3 3
= − 2x
3 3 0

a0 = 14.

and

2 l
Z
nπx
an = f (x) cos dx
l 0 l
2 3 2
Z
nπx
= x cos dx
3 0 3
2h 2  sin nπx   − cos nπx   sin nπx i2
= (3x − 2) nπ 3
− 6x n π
2 2
3
+ 6 − n π
3 3
3
3 3
0
9 27
108
an = 2 2 (−1)n .

Substituting the values of a0 and an in (14.3.10), we get


108 X (−1)n nπx
3x2 − 2 = 7 + cos
π2 n=1 n2 3
108 h 1 πx 1 2πx 1 3πx i
= 7 + 2 − 2 cos + 2 cos − 2 cos + ...
π 1 3 2 3 3 3
108 h1 πx 1 2πx 1 3πx i
x2 = 7 − 2 2 cos − 2 cos + 2 cos − ...
π 1 3 2 3 3 3

Example 14.3.11. If f (x) = |x|, expand f (x) as a Fourier series in the interval (−2, 2).

B.Sc.(Mathematics)-IDE(UNOM)-II Year-IV Sem Transform Techniques


304 14.3. Worked out Problems

Solution. Since f (x) = |x| is an even function, the corresponding Fourier series is given by


a0 X nπx
f (x) = + an cos . (14.3.11)
2 n=1 l

Here l = 2.

a0 X nπx
∴ f (x) = + an cos .
2 n=1 2

where

2 l
Z
a0 = f (x)dx
l 0
2 2
Z
= |x|dx
2 0
Z 2
= xdx
0
 x2 2
=
2 0

a0 = 2.

and

2 l
Z
nπx
an = f (x) cos dx
l 0 l
2 2
Z
nπx
= |x| cos dx
2 0 2
Z 2
nπx
= x cos dx
0 2
h  sin nπx   − cos nπx i2
= x nπ 2
−1 n2 π2
2
0
2 4
 cos nπ   1 
= 0 + n2 π2 − 0 + n2 π2
4 4
4
= 2 2 [(−1)n − 1].

Transform Techniques B.Sc.(Mathematics)-IDE(UNOM)-II Year-IV Sem


14.3. Worked out Problems 305


0, when n is even



an = 


− n28π2


 when n is odd

Substituting the values of a0 and an in (14.3.11), we get


8 1 X nπx
|x| = 1 − cos
π2
n=1,3,5,...
n2 2
8h1 πx 1 3πx 1 5πx i
|x| = 1 − 2 2 cos + 2 cos + 2 cos + ..... .
π 1 2 3 2 5 2

Example 14.3.12. Express f (x) = 1 + sin x as a Fourier series in the interval (−1, 1).

Solution. Here l = 1 The required series is of the form

∞ ∞
a0 X X
f (x) = + an cos nπx + bn sin nπx (14.3.12)
2 n=1 n=1

where
Z l
a0 = f (x)dx
−l
Z 1
= (1 + sin x)dx
−1
Z 1 Z 1
= dx + sin xdx
−1 −1

= (x)1−1 +0

a0 = 2.

1 l
Z
nπx
an = f (x) cos dx
l −l l
Z 1
= (1 + sin x) cos nπxdx
−1

B.Sc.(Mathematics)-IDE(UNOM)-II Year-IV Sem Transform Techniques


306 14.3. Worked out Problems
Z 1 Z 1
= cos nπxdx + sin x cos nπxdx
−1 −1
Z 1
= 2 cos nπxdx
0
 sin nπx 1
= 2
nπ 0
2
= (sin nπ − 0)

an = 0.

1 l
Z
nπx
bn = f (x) sin dx
l −l l
Z 1
= (1 + sin x) sin nπxdx
−1
Z 1 Z 1
= sin nπxdx + sin x sin nπxdx
−1 −1
Z 1
bn = 0 + 2 sin x sin nπxdx
0

Z 1
bn = [cos(nπ − 1)x − cos(nπ + 1)x]dx
0
h sin(nπ − 1)x sin(nπ + 1)x i1
= −
nπ − 1 nπ + 1 0
sin(nπ − 1) sin(nπ + 1)
= −
nπ − 1 nπ + 1
1 1
= [sin nπ. cos 1 − cos nπ. sin 1] − [sin nπ. cos 1 + cos nπ. sin 1]
nπ − 1 nπ + 1
1 1
= (0 − (−1)n sin 1) − ((−1)n sin 1)
nπ − 1 nπ + 1
 1 1 
= −(−1)n sin 1 +
nπ − 1 nπ + 1
2nπ sin 1
bn = (−1)n+1 .
n2 π2 − 1

X (−1)n+1
∴ (14.3.12) ⇒ 1 + sin x = 1 + 2π sin 1 sin nπx.
n=1
n2 π2 − 1

Transform Techniques B.Sc.(Mathematics)-IDE(UNOM)-II Year-IV Sem


14.3. Worked out Problems 307

Example 14.3.13. Find the Fourier series with period 3 to represent f (x) = 2x− x2 in 0 < x < 3.

Solution. Let
∞ ∞
a0 X nπx X nπx
f (x) = + an cos + bn sin
2 n=1 l n=1
l

Here 2l = 3 ⇒ l = 32 . Thus

∞ ∞
a0 X 2nπx X 2nπx
f (x) = + an cos + bn sin (14.3.13)
2 n=1 3 n=1
3

where

1 2l
Z
a0 = f (x)dx
l 0
2 3
Z
= (2x − x2 )dx
3 0
2 h 2x2 x3 i3
= −
3 2 3 0
2
a0 = (9 − 9)
3
a0 = 0.

1 2l
Z
nπx
an = f (x) cos dx
l 0 l
2 3
Z
2nπx
= (2x − x2 ) cos dx
3 0 3
2 h −36 18 i
= −
3 4n2 π2 4n2 π2
9
an = − 2 2.

B.Sc.(Mathematics)-IDE(UNOM)-II Year-IV Sem Transform Techniques


308 14.3. Worked out Problems

1 2l
Z
nπx
bn = f (x) sin dx
l 0 l
2 3
Z
2nπx
= (2x − x2 ) sin dx
3 0 3
2h  − cos 2nπx   − sin 2nπx   cos 2nπx i3
= 2
(2x − x ) 2nπ
3
− (2 − 2x) 4n2 π2
3
+ (−2) 3
8n3 π3
3 3
0
9 27
3
= − .

∞ ∞
9 X 1 2nπx 3 X 1 2nπx
∴ (14.3.13) ⇒ f (x) = − cos + sin .
π n=1 n
2 2 3 π n=1 n 3

Example 14.3.14. Find the Fourier series for f (x) = 2lx − x2 in 0 < x < 2l and hence deduce
π2
that 1 − 1
22
+ 1
32
− 1
42
+ .... = 2
.

Solution. The Fourier series for f (x) = 2lx − x2 in the interval (0, 2l) is given by

∞ ∞
a0 X nπx X nπx
f (x) = + an cos + bn sin
2 n=1 l n=1
l

where

1 2l
Z
a0 = f (x)dx
l 0
1 2l
Z
= (2lx − x2 )dx
l 0
1 h 2lx2 x3 i2l
= −
l 2 3 0
1  3 8 3
a0 = 4l − l
l 3
a0 = 4l2 .

Transform Techniques B.Sc.(Mathematics)-IDE(UNOM)-II Year-IV Sem


14.3. Worked out Problems 309

1 2l
Z
nπx
an = f (x) cos dx
l 0 l
1 2l
Z
nπx
= (2lx − x2 ) cos dx
l 0 l
1h  sin nπx   − cos nπx   − sin nπx i2l
= (2lx − x2 ) nπ l − (2l − 2x) n π
2 2
l
+ (−2) n π
3 3
l
l l
0
l2 l3
1 h −2l3 2l3 i
= −
l n2 π2 n2 π2
4l2
an = − 2 2.

1 2l
Z
nπx
bn = f (x) sin dx
l 0 l
1 2l
Z
nπx
= (2lx − x2 ) sin dx
l 0 2
1h  − cos nπx   − sin nπx   cos nπx i2l
= (2lx − x2 ) nπ
l
− (2l − 2x) n π
2 2
l
+ (−2) n π
3 3
l
l l
0
l2 l3
1 h 2l3   2l3 i
= 0+0− 3 3 − 0+0− 3 3
l nπ nπ
= 0.


4l2 X 1 nπx
∴ f (x) = 2l − 2
2
cos .
π n=1 n 2 l

Deduction. Putting x = 0 in the above series, we obtain


4l2 X 1
0 = 2l2 −
π2 n=1 n2

X 1 π2
= .
n=1
n2 2

B.Sc.(Mathematics)-IDE(UNOM)-II Year-IV Sem Transform Techniques


310 14.3. Worked out Problems

Example 14.3.15. Find the Fourier series of the function



when − l < x < 0

−a,



f (x) = 


when 0 < x < l

a,

Solution. Since f (x) is an odd function,


X nπx
f (x) = bn sin ,
n=1
l

where

2 l
Z
nπx
bn = f (x) sin dx
l 0 l
2 l
Z
nπx
= a sin dx
l 0 l
nπx
2a  cos l l
= − nπ
l l
0

2a
= − (cos nπ − cos 0)

2a
= [1 − (−1)n ].



0, when n is even



bn = 


4a
, when n is odd



 nπ
.

4a X 1 nπx 4a  πx 1 3πx 1 5πx 
∴ f (x) = sin = sin + sin + sin + ..... .
π n=1,3,5,.. n l π l 3 l 5 l

Example 14.3.16. Obtain the Fourier series expansion of



0<x<1

1,



f (x) = 


1<x<3

2,

and f (x) = 3
2
when x = 0, 1, 3 and f (x + 3) = f (x) for all x.

Transform Techniques B.Sc.(Mathematics)-IDE(UNOM)-II Year-IV Sem


14.3. Worked out Problems 311

Solution. Let
∞ ∞
a0 X nπx X nπx
f (x) = + an cos + bn sin
2 n=1 l n=1
l

Here 2l = 3 ⇒ l = 32 . Thus

∞ ∞
a0 X 2nπx X 2nπx
f (x) = + an cos + bn sin (14.3.14)
2 n=1 3 n=1
3

where

1 2l
Z
a0 = f (x)dx
l 0
2 3
Z
= f (x)dx
3 0
2h 1
Z Z 3 i
= 1.dx + 2.dx
3 0 1
2h 1 i
= (x)0 + 2(x)31
3
2
= (1 + 4)
3
10
a0 = .
3

1 2l
Z
nπx
an = f (x) cos dx
l 0 l
2h 1
Z Z 3
2nπx 2nπx
= 1. cos dx + 2. cos dx
3 0 3 1 3
2nπx  sin 2nπx
2 h sin 3 1 3 i
= 2nπ
+ 2 2nπ
3
3 3
0
3
1

1 2nπ 2 2nπ 
= sin + 0 − sin
nπ 3 nπ 3
1 2nπ
an = − sin .
nπ 3

B.Sc.(Mathematics)-IDE(UNOM)-II Year-IV Sem Transform Techniques


312 14.3. Worked out Problems

1 2l
Z
nπx
bn = f (x) sin dx
l 0 l
2h 1
Z Z 3
2nπx 2nπx
= 1. sin dx + 2. sin dx
3 0 3 1 3
2nπx  − cos 2nπx
2 h − cos 3 1 3 i
= 2nπ
+ 2 2nπ
3
3 3
0
3
1

2h 3  2nπ  3 2nπ i
= − cos −1 − cos 2nπ − cos
nπ 2nπ 3 nπ 3
1 2nπ 1 2 2 2nπ
= − cos + − + cos
nπ 3 nπ nπ nπ 3
1 2nπ 2  2nπ 
bn = sin + 0 − sin
nπ 3 nπ 3

1 2nπ 
bn = cos −1 .
nπ 3

∞ ∞
5 X −1 2nπ 2nπx X 1  2nπ  2nπx
∴ (14.3.14) ⇒ f (x) = + sin cos + cos − 1 sin
3 n=1 nπ 3 3 n=1
nπ 3 3

5 1 X 1 h 2nπ 2nπx 2nπ 2nπx  2nπx i
= − sin cos + cos sin + sin
3 π n=1 n 3 3 3 3 3

5 1 X 1h  2nπx 2nπ  2nπx i
= − sin − + sin
3 π n=1 n 3 3 3
∞  2nπ(1 − x)
5 1 X 1h 2nπx i
= − sin + sin .
3 π n=1 n 3 3

which is the required Fourier series.

Example 14.3.17. Find the Fourier series expansion for f (x), if




2, if − 2 ≤ x ≤ 0



f (x) = 


 x if 0 < x < 2


Transform Techniques B.Sc.(Mathematics)-IDE(UNOM)-II Year-IV Sem


14.3. Worked out Problems 313

Solution. Here l = 2.

∞ ∞
a0 X nπx X nπx
∴ f (x) = + an cos + bn sin .
2 n=1 2 n=1
2

where

1 2
Z
a0 = f (x)dx
2 −2
1h 0
Z Z 2 i
= 2dx + xdx
2 −2 0
1h  x2 2 i
= 2(x)0−2 +
2 2 0
1
= (4 + 2)
2
a0 = 3.

and
Z 2
1 nπx
an = f (x) cos dx
2 −2 2

1h 0
Z Z 2
nπx nπx i
an = 2. cos dx + x. cos
2 −2 2 0 2
nπx nπx  cos nπx i2 i
1 h  sin 2 0 h  sin 
= 2 nπ + x nπ 2 − 1. n2 π22
2 2
−2
2
0
4
1h 4 4 i
= cos nπ −
2 n2 π2 n62π2
2
an = 2 2 [(−1)n − 1].



0, when n is even



an = 


− n24π2


 when n is odd

B.Sc.(Mathematics)-IDE(UNOM)-II Year-IV Sem Transform Techniques


314 14.3. Worked out Problems

1 2
Z
nπx
bn = f (x) sin dx
2 −2 2
1h 0
Z Z 2
nπx nπx i
= 2. sin dx + x. sin
2 −2 2 0 2
nπx nπx  − sin nπx
1 h  − cos 2 0 h  cos 2  i2 i
= 2 nπ + x − nπ − 1. n π
2 2
2
2 2
−2
2
0
4
1 h −4 4 i 1h 4 4 i
= + cos nπ + − cos nπ + 2 2 sin nπ
2 nπ nπ 2 nπ nπ
1 4 
= −
2 nπ
2
bn = − .

∞ ∞
3 4 X 1 nπx 2 X 1 nπx
f (x) = − cos − sin .
2 π2 n=1,3,5,...
n 2 2 π n=1
n 2

Example 14.3.18. Find the Fourier series of the function f (x), if



 2 + x,
 1
if − 1 ≤ x ≤ 0



f (x) = 


 12 − x


 if 0 ≤ x ≤ 1

Solution. Since f (−x) = 1


2
− x in (−1, 0) = f (x) in (0, 1).
and f (−x) = 1
2
+ x in (0, 1) = f (x) in (−1, 0).
∴ f (x) is an even function. Here l = 1 The required series is of the form

∞ ∞
a0 X nπx a0 X
f (x) = + an cos = + an cos nπx
2 n=1 l 2 n=1

where
Z l
2
a0 = f (x)dx
l 0
Z 1
= 2 f (x)dx
0

Transform Techniques B.Sc.(Mathematics)-IDE(UNOM)-II Year-IV Sem


14.3. Worked out Problems 315
Z 1
1 
= 2 − x dx
0 2
 x x2 1
= 2 −
2 2 0
a0 = 0.

Z l
2 nπx
an = f (x) cos dx
l 0 l
Z 1
= 2 f (x) cos nπxdx
0
Z 1
1 
= 2 − x cos nπxdx
0 2
h 1  sin nπx   cos nπx i1
= 2 −x − (−1) − 2 2
2 nπ nπ 0
h cos nπ   1 i
= 2 0− 2 2 − 0− 2 2
nπ nπ
2
= (1 − cos nπ)
n2 π2
2
an = [1 − (−1)n ]
n π2
2



0, if n is even



an = 


 n24π2 ,


 if n is odd
∞ ∞
4 X 1 4 X 1
∴ f (x) = cos nπx = cos(2n − 1)πx.
π2 n=1,3,5,.. n2 π2 n=1 (2n − 1)2

Example 14.3.19. Find a Fourier series for the function f (x), if


if − l < x < 0





−1,


f (x) = 

if x = 0


0



if 0 < x < 1


1,

π
Hence deduce that 4
= 1 − 13 + 51 − 17 + 19 − ....

B.Sc.(Mathematics)-IDE(UNOM)-II Year-IV Sem Transform Techniques


316 14.3. Worked out Problems

Solution. The Fourier series of f (x) in the interval (−l, l) is given by

∞ ∞
a0 X nπx X nπx
f (x) = + an cos + bn sin
2 n=1 l n=1
l

where

1 l
Z
a0 = f (x)dx
l −l
1h 0
Z Z 1 i
= (−1) f (x)dx + 1.dx
l −l 0
1h i
= (−x)0−l + (x)l0
l
1
= [0 + l + l − 0]
l
a0 = 0.

1 l
Z
nπx
an = f (x) cos dx
l −l l
1h 0
Z Z l
nπx nπx i
= (−1) cos dx + (1) cos dx
l −l l 0 l
nπx nπx
1 h  sin l 0  sin l l i
= − nπ + nπ
l l
−l
l
0

1 1
= − (0 − 0) + (0 − 0)
nπ nπ
an = 0.

1 l
Z
nπx
bn = f (x) sin dx
l −l l
1h 0
Z Z l
nπx nπx i
= (−1) sin dx + (1) sin dx
l −l l 0 l

Transform Techniques B.Sc.(Mathematics)-IDE(UNOM)-II Year-IV Sem


14.4. Exercises 317
nπx nπx
1 h cos l 0  − cos l l i
= nπ + nπ
l l
−l
l
0

1 1
= − (1 − cos nπ) + (cos nπ − 1)
nπ nπ
2
bn = [1 − (−1)n ].



0, if n is even



bn = 


4
,



 nπ if n is odd

4 X 1 nπx
∴ f (x) = sin .
π n=1,3,5,.. n l

Deduction. Putting x = l
2
in the above series, we get


4 X 1 l nπ
f =
sin
2 π n=1,3,5,... n 2
4 π 1 3π 1 5π 
1 = sin + sin + sin + ...
π 2 3 2 5 2
4 1 1 1 
1 = 1 − + − + ....
π 3 5 7
1 1 1 π
∴ 1 − + − + .... = .
3 5 7 4

14.4 Exercises

1. Find the Fourier series expansion for the function f (x) = x − x2 in (−1, 1).
2. Find the Fourier series of the following function:

−2 < x < 0

0,



f (x) = 


0<x<2

1,

3. Obtain the Fourier series for the function f (x) given by

B.Sc.(Mathematics)-IDE(UNOM)-II Year-IV Sem Transform Techniques


318 14.4. Exercises

(i) 
0<x<l

0,



f (x) = 


l < x < 2l

a,

(ii) 
−5 < x < 0

0,



f (x) = 


0<x<5

3,

4. Obtain the Fourier series for the function



πx, 0 ≤ x <≤ 1




f (x) = 


π(2 − x), 1 ≤ x ≤ 2


5. Find Fourier series expansion for f (x) = x2 in −1 < x < 1.


6. Expand f (x) = e−x as a Fourier series in the interval (−l, l).
7. Find Fourier series to represent eax in (−l, l).
8. Find the Fourier series of the function

(i)

0, −2 < x < −1








f (x) = 

k, −1 < x < 1






0, 1 < x < 2


(ii) 
−2 ≤ x <≤ −1





0,



1 + x, −1 ≤ x ≤ 0




f (x) = 







1 − x, 0 ≤ x ≤ 1




0, 1≤x≤2

Transform Techniques B.Sc.(Mathematics)-IDE(UNOM)-II Year-IV Sem


14.4. Exercises 319

Answers.
1. x − x2 = − 31 + π42 cos πx
   
12 − cos 2πx
22 + cos 3πx
32 − .... + 2
π
sin πx − 1
2
sin 2πx + 1
3
sin 3πx − .....
πx
 
2. f (x) = 2 + π sin 2 + 3 sin 2 + 5 sin 2 + ..... .
1 2 1 3πx 1 5πx

πx
3. (i) f (x) = a2 − 2a
P∞ 1
π n=1 2n−1 sin(2n − 1) l

(ii) f (x) = 23 + π6 ∞ 1 nπx


P
1,3,5,.. n sin 5

4. f (x) = π2 − π4 cos12πx + cos33πx


 
2 + cos 5πx 52 + .... .
πx
   
5. x2 = 3 + 18 π
− 72
π 3 sin 3
− 18

sin 2πx
3
+ 18
π
− 72
π3 sin 3πx
3
+ .... .
h n  i
6. e−x = sinh 1 1l + ∞ n=1 n2 π2 +l2 2l cos l + 2nπ sin l .
(−1) nπx nπx
P

8. (i) f (x) = 2k + 2kπ ∞ n=1 n sin 2 cos 2 .


P 1 nπ nπx
 
(ii) f (x) = 14 + π42 ∞ 1 nπ
cos nπx .
P
n=1 n2 1 − cos 2 2

B.Sc.(Mathematics)-IDE(UNOM)-II Year-IV Sem Transform Techniques


320
UNIT-III

Lesson 15

FOURIER COSINE AND SINE SERIES


Learning Objectives
Upon completion of this lesson, students will be able to

• idnetify half range series in the interval [0, l]

• find the half range Fourier sine series of functions in [0, l]

• find the half range Fourier cosine series of functions in [0, l].

15.1 Introduction

S
o far we have considered the Fourier series expansion of a function defined in an interval
(c, c + 2l) of length 2l. In various engineering problems it is required to obtain a Fourier
series expansion of a function defined in half of the above interval say (0, l) of length l. Such
expansions are known as half - range expansion or half - range Fourier Series. As it is
immaterial whatever the function may be outside the range 0 < x < l, we extend the function to
cover the range −l < x < l so that the new function may be even or odd. The Fourier expansion of
such a function consists of cosine or sine terms only.The expansion containing only cosine terms
is known as half - range Fourier cosine series and the expansion containing only sine terms is
known as half - range Fourier sine series.

Cosine Series. If it is required to expand f (x) in the interval (0, l) then we extend the function

B.Sc.(Mathematics)-IDE(UNOM)-II Year-IV Sem 321 Transform Techniques


322 15.1. Introduction

reflecting in the y - axis, so that f (−x) = f (x). We can define a new function g(x) such that

g(x) = f (x) f or 0<x<l

and g(x) = f (−x) f or −l< x<0

Clearly g(x) is an even function in (−l, l) and it is identical with f (x) in (0, l). We can now
construct the Fourier series for this even function g(x) in (−l, l). We know that the expansion will
be of the form

a0 X nπx
g(x) = + an cos
2 n=1 l

where

2 l
Z
a0 = g(x)dx
l 0
2 l
Z
nπx
an = g(x) cos dx.
l 0 l

But g(x) is identical with f (x) in (0, l). ∴ in the interval (0, l), the Fourier cosine series is


a0 X nπx
f (x) = + an cos
2 n=1 l

where

2 l
Z
a0 = f (x)dx
l 0
2 l
Z
nπx
an = f (x) cos dx.
l 0 l

Sine Series. If it is required to expand f (x) in the interval (0, l) then we extend the function
reflecting it in the origin, so that f (−x) = f (x). We can define a new function g(x) such that

g(x) = f (x) f or 0<x<l

Transform Techniques B.Sc.(Mathematics)-IDE(UNOM)-II Year-IV Sem


15.2. Worked out Problems 323

and g(x) = − f (−x) f or −l< x<0

Clearly g(x) is an odd function in (−l, l) and it is identical with f (x) in (0, l). We can now
construct the Fourier series for this odd function g(x) in (−l, l). We know that the expansion will
be of the form

X nπx
g(x) = an sin
n=1
l

where
Z l
2 nπx
bn = g(x) sin dx.
l 0 l

But g(x) is identical with f (x) in (0, l).


X nπx
f (x) = an sin
n=1
l

where
Z l
2 nπx
bn = f (x) sin dx.
l 0 l

15.2 Worked out Problems

Example 15.2.1. Find the half - range sine series of f (x) = 1 in [0, l].

Solution. The Fourier sine series of f (x) in [0, l] is given by


X nπx
f (x) = bn sin
n=1
l

B.Sc.(Mathematics)-IDE(UNOM)-II Year-IV Sem Transform Techniques


324 15.2. Worked out Problems

where

2 l
Z
nπx
bn = f (x) sin dx
l 0 l
2 l
Z
nπx
= 1. sin dx
l 0 l
nπx
2  cos l l
= − nπ
l l
0

2 l  
= . − cos nπ + 1
l nπ
2
= [(−1)n+1 + 1]



0, when n is even



∴ bn = 


4
, when n is odd



 nπ

Hence the required Fourier series is


X 4 nπx
f (x) = sin
n=1,3,5,..
nπ l
4 πx 1 3πx 1 5πx 
1 = sin + sin + sin + ... .
π l 3 l 5 l

Example 15.2.2. Obtain the half - range cosine and sine series for f (x) = x in [0, l].

Solution. The Fourier cosine series of f (x) in [0, l] is given by


a0 X nπx
f (x) = + an cos
2 n=1 l

where
l l
2  x 2 l
Z Z
2 2
a0 = f (x)dx = xdx = = l.
l 0 l 0 l 2 0

Transform Techniques B.Sc.(Mathematics)-IDE(UNOM)-II Year-IV Sem


15.2. Worked out Problems 325

and

2 l
Z
nπx
an = f (x) cos dx
l 0 l
2 l
Z
nπx
= x cos dx
l 0 l
nπx  cos nπx
2 h  sin l  il
= x nπ − 1 n2 π2 l
l l
0
l2
2 h l2   l2 i
= 0 + 2 2 (−1)n − 0 + 2 2
l nπ nπ
2l
an = [(−1)n − 1]
n2 π2


0, when n is even



∴ an = 


− n24lπ2 ,


 when n is odd

Hence the required Fourier series is


l 4l X 1 nπx
f (x) = − 2 cos
2 π n=1,3,5,.. n 2 l
l 4l  πx 1 3πx 1 5πx 
x = − 2 cos + 2 cos + 2 cos + ... .
2 π l 3 l 5 l

Note. Expand f (x) = x as a half - range cosine series in 0 < x < 2. Putting l = 2 in the above
series, we get
8 πx 1 3πx 1 5πx 
x=1− cos + cos + cos + ... .
π2 2 32 2 52 2
Sine Series.

X nπx
f (x) = bn sin
n=1
l

B.Sc.(Mathematics)-IDE(UNOM)-II Year-IV Sem Transform Techniques


326 15.2. Worked out Problems

where

2 l
Z
nπx
bn = f (x) sin dx
l 0 l
2 l
Z
nπx
= x sin dx
l 0 l
nπx  − sin nπx
2 h  − cos l  il
= x nπ − 1 n2 π2
l
l l
0
l2
2 h l2  i
= − (−1)n + 0 − (0 + 0)
l nπ
2l
bn = −
nπ(−1)n
2l
bn = (−1)n+1 .


2l X (−1)n+1 nπx
f (x) = sin
nπ n=1 n l
2l  πx 1 2πx 1 3πx 
x = sin − sin + cos − ... .
π l 2 l 3 l

Note. Express f (x) = x as a half - range sine series in 0 < x < 2.


Putting l = 2 in the above sine series, we get

4 πx 1 2πx 1 3πx 
x= sin − sin + cos − ... .
π 2 2 2 3 2

Example 15.2.3. Find the half range sine series expansion of f (x) = x2 in [0, 4].

Solution. The half - range sine series is given by


X nπx
f (x) = bn sin
n=1
l

where Here l = 4.

X nπx
∴ f (x) = bn sin
n=1
4

Transform Techniques B.Sc.(Mathematics)-IDE(UNOM)-II Year-IV Sem


15.2. Worked out Problems 327

where

2 l
Z
nπx
bn = f (x) sin dx
l 0 l
2 4 2
Z
nπx
= x sin dx
4 0 4
nπx  − sin nπx  cos nπx
1 h 2  − cos 4   i4
= x nπ − 2x n2 π2
4
+ 2 n3 π3
4
2 4
0
16 64
1h 4 32 2.64 128 i
= − (16) cos nπ + 2 2 (4) sin nπ + cos nπ −
2 nπ nπ n63π3 n3 π3
n+1 n
1 (−1) 64 (−1) 128 128
h i
= + − 3 3
2 nπ n3 π3 nπ
h 2 n+1 i
(−1)
bn = 32 3 3 [(−1)n − 1] +
nπ nπ


− nπ ,
 32
when n is even



∴ bn = 

  
4
+ 1
,

32 −


n3 π2 nπ
when n is odd

∞ ∞
32 X 1 nπx X  4 1 nπx
x2 = − sin + 32 − 3 3+ sin .
π n=2,4,6,.. n 4 n=1.3.5,..
n π nπ 4

Example 15.2.4. Find the half - range cosine series expansion of f (x) = x − x2 in 0 < x < l.

Solution. The required Fourier series is


a0 X nπx
f (x) = + an cos
2 n=1 l

Here l = 1.

a0 X
∴ f (x) = + an cos nπx.
2 n=1

B.Sc.(Mathematics)-IDE(UNOM)-II Year-IV Sem Transform Techniques


328 15.2. Worked out Problems

where

2 1
Z
a0 = f (x)dx
1 0
2 1
Z
= (x − x2 )dx
1 0
2  x2 x3 1
= −
1 2 3 0
1 1
= 2 −
2 3
1
a0 = .
3

and

2 l
Z
nπx
an = f (x) cos dx
l 0 l
2 1
Z
= (x − x2 ) cos nπxdx
1 0
h  sin nπx   − cos nπx   − sin nπx i1
= 2 (x − x2 ) − (1 − 2x) − 2
nπ n2 π2 n3 π3 0
h 1 1 i
= 2 − 2 2 cos nπ − 2 2
nπ nπ
2
= − 2 2 (1 + cos nπ)

2
an = − 2 2 [1 + (−1)n ]



0, when n is odd



∴ an = 


− n24π2 ,


 when n is even


1 4 X 1
x − x2 = − 2 cos nπx,
6 π n=2,4,6,.. n2

which is the required Fourier series.

Example 15.2.5. Find half - range Fourier sine series for f (x) = ax + b, in 0 < x < l.

Transform Techniques B.Sc.(Mathematics)-IDE(UNOM)-II Year-IV Sem


15.2. Worked out Problems 329

Solution. The half- range sine series is given by


X nπx
f (x) = bn sin .
n=1
l

Here l = 1

X
∴ f (x) = bn sin πx
n=1

where

2 l
Z
nπx
bn = f (x) sin dx
l 0 l
2 1
Z
= (ax + b) sin nπxdx
1 0
h  − cos nπx   − sin nπx i1
= 2 (ax + b) −a
nπ n2 π2 0
h 1 a bi
= 2 − (a + b) cos nπ + 2 2 sin nπ +
nπ nπ nπ
2
= [(−1)n+1 (a + b) + b]


− nπ ,
 2a
when n is even



∴ bn = 


2
(a + 2b),



 nπ when n is odd

2 2a 2 2a
ax + b = (a + 2b) sin πx − sin 2πx + (a + 2b) sin 3πx − sin 4πx + .....
π 2π 3π 4π

Example 15.2.6. Find the half - range cosine series for f (x) = x(2 − x) in 0 ≤ x ≤ 2 and hence
find sum of the series 1
12
− 1
22
+ 1
32
− 1
42
+ ...

Solution. The required Fourier series is


a0 X nπx
f (x) = + an cos
2 n=1 l

B.Sc.(Mathematics)-IDE(UNOM)-II Year-IV Sem Transform Techniques


330 15.2. Worked out Problems

Here l = 2.

a0 X nπx
∴ f (x) = + an cos .
2 n=1 2

where

2 1
Z
a0 = f (x)dx
1 0
2 2
Z
a0 = (2x − x2 )dx
2 0

 2x2 x3 2
a0 = −
2 3 0
4
a0 = .
3

and

2 l
Z
nπx
an = f (x) cos dx
l 0 l
2 2
Z
nπx
= (2x − x2 ) cos dx
2 0 2
h  sin nπx   − cos nπx   − sin nπx i2
= (2x − x ) nπ
2 2
− (2 − 2x) n2 π2
2
− 2 n3 π3
2
0
2 4 8
h 1 1 i
= 2 − 2 2 cos nπ − 2 2
nπ nπ
8
= − 2 2 (1 + cos nπ)

8
an = − 2 2 [1 + (−1)n ]



0, when n is odd



∴ an = 


2 π2 ,
− n16


 when n is even

Transform Techniques B.Sc.(Mathematics)-IDE(UNOM)-II Year-IV Sem


15.2. Worked out Problems 331

2 16 X 1 nπx
2x − x2 = − 2 cos
3 π n=2,4,6,.. n2 2
2 16  1 1 1 
= − 2 2 cos πx + 2 cos 2πx + 2 cos 3πx + ......
3 π 2 4 6
2 16 1  1 1 
= − 2 . 2 cos πx + 2 cos 2πx + 2 cos 3πx + ......
3 π 2 2 3
2 4  1 1 
2x − x2 = − 2 cos πx + 2 cos 2πx + 2 cos 3πx + ...... .
3 π 2 3

which is the required Fourier series.


Deduction. Put x = 1 in the above series, we get

2 4 1 1 
2−1 = − 2 cos π + 2 cos 2π + 2 cos 3π + ......
3 π 2 3
2 4 1 1 
1− = − 2 − 1 + 2 − 2 + ......
3 π 2 3
1 4 1 1 
= 2 1 − 2 + 2 − ......
3 π 2 3

1 1 1 π2
1− + − + ...... = .
22 32 52 12

πx
 
Example 15.2.7. Find the half - range cosine series expansion of f (x) = sin l
in the interval
0 < x < l.

Solution. The required Fourier series is


a0 X nπx
f (x) = + an cos
2 n=1 l

where

2 1
Z
a0 = f (x)dx
1 0
2 l
Z  πx 
= sin dx
l 0 l

B.Sc.(Mathematics)-IDE(UNOM)-II Year-IV Sem Transform Techniques


332 15.2. Worked out Problems
πx
2 h cos l il
= − π
l l
0

2
a0 = − (cos π − 1)
π
4
a0 = .
π

and

2 l
Z
nπx
an = f (x) cos dx
l 0 l
2 2 πx
Z
nπx
= sin cos dx
l 0 l 2
1 lh πx πx
Z
= sin(n + 1) − sin(n − 1) Big]dx
l 0 l l
πx πx i
1 h cos(n + 1) cos(n − 1) l l
= − π
l
+ π
l (n + 1) l (n − 1) l 0
1 h cos(n + 1)π cos(n − 1)π i  1 1 i
= − + − − +
π n+1 n−1 n+1 n−1
1 h (−1)n+1 (−1)n−1 1 1 i
an = − + + − , n,1
π n+1 n−1 n+1 n−1


0, when n is odd, n , 1



∴ an = 


4
,

− π(n+1)(n−1)

 when n is even, n , 1

If n = 1,

1 l πx πx
Z
a1 = 2 sin cos dx
l 0 l l
Z l
1 2πx
= sin dx
l 0 l
1 l h 2πx il
= . − cos
l 2π l 0
1
= − (cos 2π − cos 0)

a1 = 0.

Transform Techniques B.Sc.(Mathematics)-IDE(UNOM)-II Year-IV Sem


15.2. Worked out Problems 333

πx 2 4 X 1 nπx
sin = − cos
l π π n=2,4,6,.. (n + 1)(n − 1) l

which is the required Fourier series.

Example 15.2.8. Find the half range sine series of e x in the interval 0 < x < 1.

Solution. Here l = 1. The half - range sine series is given by


X
ex = bn sin nπx.
n=1

where
Z 1
bn = 2 e x sin nπxdx
0
h ex i1
= 2 2 2 (sin nπx − nπ cos nπx)
n π +1 0
h e 1 i
= 2 2 2 (0 − nπ cos nπ) − 2 2 (0 − nπ)
n π +1 n π +1
h 1 i
= 2 2 2 (nπ − nπ cos nπ)
n π +1
2nπ
= 2 2 [1 − e(−1)n ].
n π +1

h 2(1 + e) 2(1 − e) 3(1 + e) i


∴ ex = π 2 sin πx + 2 sin 2πx + 2 sin 3πx + ...... .
π +1 4π + 1 9π + 1
Example 15.2.9. Expand cos πx in (0, 1) as Fourier sine series.

Solution. Here l = 1. The half - range sine series is given by


X
cos πx = bn sin nπx.
n=1

B.Sc.(Mathematics)-IDE(UNOM)-II Year-IV Sem Transform Techniques


334 15.2. Worked out Problems

where
Z 1
bn = 2 cos πx sin nπxdx
0
Z 1
= [sin(n + 1)πx + sin(n − 1)πx]dx
0
h cos(n + 1)πx cos(n − 1)πx i1
= − − , n,1
(n + 1)π (n − 1)π 0
1 h cos(n + 1)π cos(n − 1)π   1 1 i
= − + − + , n,1
π n+1 n−1 n+1 n−1
1h  1 1   1 1 i
= − (−1)n+1 + − + , n,1
π n+1 n−1 n+1 n−1
1 1 1 
= − + [(−1)n+1 − 1], n , 1
π n+1 n−1
1 2n
= − . 2 [(−1)n+1 − 1], n , 1.
π n −1


0, when n is odd



∴ bn = 


2 −1) ,
 π(n4n


 when n is even

If n = 1, then
Z 1
b1 = 2 sin πx cos πxdx
0
Z 1
= sin 2πxdx
0
1
= − (cos 2π − cos 0)

b1 = 0.

∞ ∞ ∞
X 4n 4 X 2n 8X n
∴ cos πx = sin nπx = sin 2nπx = sin 2πx.
2,4,6,..
π(n2 − 1) π n=1 4n2 − 1 π n=1 4n2 − 1

Example 15.2.10. Find the half - range cosine series for the function f (x) = (x − 1)2 in the
π2
interval 0 < x < 1. Hence show that 1
12
+ 1
22
+ 1
32
+ ... = 6
.

Transform Techniques B.Sc.(Mathematics)-IDE(UNOM)-II Year-IV Sem


15.2. Worked out Problems 335

Solution. Let

a0 X
f (x) = + an cos nπx,
2 n=1

where
Z 1
a0 = 2 f (x)dx
0
Z 1
= 2 (x − 1)2 dx
0
h (x − 1)3 i1
= 2
3 0
2
a0 = .
3

and
Z 1
an = 2 f (x) cos nπxdx
0
Z 1
= 2 (x − 1)2 cos nπxdx
0
h  sin nπx   cos nπx   − sin nπx i1
= 2 (x − 1)2 − 2(x − 1) − 2 2 + 2
nπ nπ n3 π3 0
h  2 i
= 2 (0 + 0 − 0) − 0 − 2 2 − 0

4
= 2 2.


1 4 X 1
∴ (x − 1)2 = + 2 cos nπx.
3 π n=1 n2

Example 15.2.11. Find the half - range cosine series for




kx, 0 ≤ x ≤ L2



f (x) = 


k(L − x), L2 ≤ x ≤ L


π2
Deduce that 1
12
+ 1
32
+ 1
52
+ ..... = 8
.

B.Sc.(Mathematics)-IDE(UNOM)-II Year-IV Sem Transform Techniques


336 15.2. Worked out Problems

Solution. Let

a0 X nπx
f (x) = + an cos ,
2 n=1 L

where
Z L
2
a0 = f (x)dx
L 0
L
Z Z L
2h 2
= kxdx + k(L − x)dx
L 0 L
2

2k h x2  L2  x2  L i
= + Lx −
L 2 0 2 L2
2k h L2  2 L2   L2 L2 i
= + L − − −
L 8 2 2 8
2
2k 2L
 
= + L2 − L2
L 8
kL
=
2

and
Z L Z L
2h 2 nπx nπx i
an = kx cos dx + k(L − x) cos dx
L 0 L L
2
L
2kL h nπ i
= 2 2 2 cos − 1 − (−1)n
nπ 2


0, when n is odd



∴ an = 

  
2 π2 cos 2 − 1 ,
 n4kL
 nπ

 when n is even


kL 4kL X 1  nπ  nπx
f (x) = + 2 cos − 1 cos
4 π 2,4,6,.. n 2 2 L
kL 8kL h 1 2πx 1 6πx 1 10πx i
f (x) = − 2 2 cos + 2 cos + 2 cos + .... .
4 π 2 L 6 L 10 L

Deduction.

Transform Techniques B.Sc.(Mathematics)-IDE(UNOM)-II Year-IV Sem


15.2. Worked out Problems 337

Putting x = 0 in the above series, we get

kL 8kL  1 1 1 
f (0) =
− 2 2 + 2 + 2 + ....
4 π 2 6 10
kL 8kL 1  1 1 1 
0 = − 2 . 2 2 + 2 + 2 + ....
4 π 2 1 3 5
kL 2kL 1 1 1 
= + 2 + 2 + ....
4 π 1
2 2 3 5
1 1 1 π2
+ + + .... = .
12 32 52 8

Example 15.2.12. Find the Fourier sine series for f (x) = 2x − x2 , in 0 < x < 3 and
f (x + 3) = f (x).

Solution. Here l = 3. The required Fourier sine series is given by


X nπx
f (x) = bn sin ,
n=1
3

where

2 3
Z
nπx
bn = f (x) sin dx
3 0 3
2 3
Z
nπx
= (2x − x2 ) sin dx
3 0 3
2h  cos nπx   sin nπx   cos nπx i3
= (2x − x2 ) − nπ 3 − (2 − 2x) − n2 π23 + (−2) n3 π23
3 3
0
9 27
2h 3 54  54 i
= − 3. cos nπ + 0 − 3 3 cos nπ − 0 − 0 − 3 3
3 nπ nπ nπ
2h 9 54 i
= (−1)n + 3 3 [1 − (−1)n ]
3 nπ nπ
h (−1)n 6 i
= 6 + 3 3 [1 − (−1)n ]
nπ nπ


 nπ ,
 6
if n is even



∴ bn = 

  
1
+ 12
,

6 −


nπ n3 π3
if n is odd

B.Sc.(Mathematics)-IDE(UNOM)-II Year-IV Sem Transform Techniques


338 15.3. Exercises

 1 12  πx 3 2πx  1 12  3πx 3 4πx


∴ f (x) = 6 − + 3 sin + sin +6 − + sin + sin + ......
π π 3 π 3 3π 27π3 3 2π 3

15.3 Exercises

1. Find the half - range cosine series for the function f (x) = x in 0, l.
2. Obtain the half - range cosine and sine series for x in (0, 2).
3. Find the half range cosine series for the function f (x) = (x − 1)2 in the interval 0 < x < 1.
Hence deduce that
(i)1 + 212 + 312 + .... = π6 ;
2

π2
(ii) ∞n=1 (2n−1)2 = 12 + 32 + + ... = .
P 1 1 1 1
52 8

4. Obtain the half - range sine series for x − x2 in (0, 1).


5. Express f (x) = lx− x2 as a half - range sine series in (0, l). Hence show that 1
13
− 313 + 513 −..... =
π2
32
.
6. Find the Fourier sine series to represent the function f (x) = sin x in 0 < x < π.
7. Express f (x) = x3 as half - range cosine series in (0, L).
8. Represent f (x) = x2 in 0 < x < L by Fourier sine series.
9. Represent f (x) = 2x − 1 in 0 < x < 1 by Fourier cosine series.
10. Obtain the half - range sine series for the function

when, 0 < x <
 1 1
 4 − x,


 2
f (x) = 


 x − 34 , when, 12 < x < 1.


Answers.
h i
1. x = l
2
− 4l
π2
cos 4πl + 1
32
cos 3πx
cos 5πxl
l
+
+ · ·1
·
52
πx
h i
2. x = l − π2 cos 2 + 32 cos 2 + 52 cos 2 + · · ·
8 1 3πx 1 5πx

and x = π4 sin πx
h i
2
− 12 sin 2πx
2
+ 13 cos 3πx
2
+ ···
h i
3. (x − 1)2 = 13 + π42 cos πx + 212 cos 2πx + 312 cos 3πx + · · ·
h i
4. x − x2 = π83 113 sin πx + 313 sin 3πx + 513 sin 5πx + · · ·

Transform Techniques B.Sc.(Mathematics)-IDE(UNOM)-II Year-IV Sem


15.3. Exercises 339

sin πxl + 312 sin 3πx


h i
8l2
5. lx − x2 = π3 l
+ 1
53 cos 5πx
l
+ · · · .
3
 
3L3 12L3 3L3 P∞
7. x3 = L4 + ∞ 1,3,5,.. − 2n2 π2 + n4 π4 cos L + 2π2
nπx
n=2,4,6,... n2 cos L .
1 nπx
P

(−1)n
2 P
h i
8. x2 = 2Lπ ∞ 2 n
n=1 n3 π2 [(−1) − 1] − n sin nπx
L
.
9. 2x − 1 = − π8 cos12πx + cos33πx
h i
2 + cos55πx
2 + ··· .
     
10. f (x) = π1 − π42 sin πx + 3π 1
+ 324π2 sin 3πx + 5π 1
− 524π2 sin 5πx + · · ·

B.Sc.(Mathematics)-IDE(UNOM)-II Year-IV Sem Transform Techniques


340
UNIT-IV

Lesson 16

FOURIER TRANSFORMS
Learning Objectives
Upon completion of this lesson, students will be able to

• apply Fourier integral theorem and Dirichlet’s conditions

• state basic definition of Fourier transform

• use the properties of Fourier transform

• find the Fourier transform of given functions by using the definition of Fourier transform
and inverse Fourier transform.

16.1 Introduction

A
transformation is a mathematical device which converts or changes one function into
another function. For example, differentiation and integration are transformations.
Fourier transforms are used in solving partial differential equations with boundary conditions.
In this chapter, some of the well known integral transforms will be introduced and their
properties will be studied which will be used in the solution of partial differential equations. Also
we discuss the application of finite and infinite Fourier integral transforms which are mathematical
devices from which we obtain the solutions of boundary value problems related to engineering.
For example, conduction of heat, free and forced vibrations of membrane, transverse vibrations of

B.Sc.(Mathematics)-IDE(UNOM)-II Year-IV Sem 341 Transform Techniques


342 16.1. Introduction

a string, transverse oscillations of an elastic beam etc.


A periodic function f (x) defined in an interval (−l, l) can be expressed in Fourier series. By
extending this concept, non - periodic functions defined in the interval (−∞, ∞) can be expressed
as a Fourier integral.

Definition 16.1.1. The integral transform of a function f (t) is defined by


Z b
f (p) = I[ f (t)] = f (t)k(p, t)dt
a

where k(p, t) is called the kernel of the integral transform and is known function of p and t. The
function f (t) is called the inverse transform of f (p).

Examples of different kernels are as follows:


(i) When k(p, t) = e−pt , a = 0 and b → ∞, it gives the Laplace transform of f (t).

Z ∞
∴ f (t) = L [ f (t)] = e−pt f (t)dt.
0

(ii) If k(p, t) = eipt , a → ∞ and b → ∞ then we have the Infinite Fourier transform of f (t)
Z ∞
∴ f (p) = F [ f (t)] = eipt f (t)dt
−∞

(iii) If k(p, t) = t p−1 , a → 0 and b → ∞, we have the Melin transform of f (t).


Z ∞
∴ f (p) = M [ f (t)] = t p−1 f (t)dt.
0

Other special transforms arise when the kernel is a sine or cosine function or a Bessel’s function.
These lead to Fourier sine or cosine transforms and the Hankel transform respectively.

First we shall prove the Fourier integral theorem. Before proving this theorem we must know the
well known Dirichlet’s conditions which play an important role in the study of Fourier transforms
and covers many engineering applications.

Transform Techniques B.Sc.(Mathematics)-IDE(UNOM)-II Year-IV Sem


16.2. Dirichlet’s Conditions 343

16.2 Dirichlet’s Conditions

A function f (x) is said to satisfy Dirichlet’s conditions in the interval (a, b), if
(i) f (x) is defined and is single valued function except possibly at a finite number of points in the
interval (a, b) and
(ii) f (x) and f 0 (x) are piecewise continuous in the interval (a, b).

Theorem 16.2.1. Fourier Integral Theorem. If f (x) is piece-wise continuously differentiable


and absolutely integrable function in (−∞, ∞), then
Z ∞ Z ∞
1
f (x) = f (t) cos p(t − x)dtd p.
π 0 −∞

Proof. Let f (x) be a function satisfying the Dirichlet’s conditions in every interval (−l, l) and
defined as
1h i
f (x) = f (x + 0) + f (x − 0)
2
at every point of discontinuity.
We know that the Fourier series of f (x) in (−l, l) is given by


a0 X h nπx nπx i
f (x) = + an cos + bn sin (16.2.1)
2 n=1 l l

where

1 l
Z
a0 = f (t)dt
l −l
1 l
Z
nπt
an = f (t) cos dt
l −l l
1 l
Z
nπt
bn = f (t) sin dt
l −l l

B.Sc.(Mathematics)-IDE(UNOM)-II Year-IV Sem Transform Techniques


344 16.2. Dirichlet’s Conditions

Substituting these values of a0 , an and bn in (16.2.1), we get

Z l ∞ Z l ∞ Z l
1 1X nπt nπx 1X nπt nπx
f (x) = f (t)dt + f (t) cos cos dt + f (t) sin sin dt
2l −l l n=1 −l l l l n=1 −l l l

Z l ∞ Z l
1 1X nπ(t − x)
f (x) = f (t)dt + f (t) cos dt. (16.2.2)
2l −l l n=1 −l l
R∞
Let −∞
| f (x)|dx converges, i,e., it has a finite value. Now assuming that the limit as t → ∞ in
(16.2.2), we have the first term
h1 Z l i
lim f (t)dt = 0,
t→∞ 2l −l

since
1 Z l 1 Z l
f (t)dt ≤ | f (t)|dt.
2l −l 2l −l
π
Now we put l
= δp in the second term of (16.2.2), then it takes the form

∞ Z l
1 Xh i
f (t) cos[nδp(t − x)]dt δp (16.2.3)
π n=1 −l

Now as l → ∞ or δp → 0, (16.2.3) takes the form


Z ∞ hZ ∞
1 i
f (t) cos[p(t − x)]dt δp,
π 0 −∞

since

X Z ∞
lim φ(nδp)δp = φ(p)d p.
δp→0 0
n=1

Thus, as l → ∞, (16.2.2)takes the form


Z ∞ Z ∞
1
f (x) = f (t) cos p(t − x)dtd p. (16.2.4)
π 0 −∞

The representation of (16.2.4) of f (x) is known as Fourier integral of f (x). 

Transform Techniques B.Sc.(Mathematics)-IDE(UNOM)-II Year-IV Sem


16.3. Fourier sine and cosine integrals 345

16.3 Fourier sine and cosine integrals

Remembering cos(A − B) expansion, expanding the cos p(t − x), the Fourier integral of f (x)
given by (16.2.4) may be written as
Z ∞ Z ∞
1
f (x) = f (t)[cos(pt) cos(px) + sin(pt) sin(px)]dtd p.
π 0 −∞

Z ∞ Z ∞ Z ∞ Z ∞
1 1
f (x) = cos(px) f (t) cos(pt)dtd p + sin(px) f (t) sin(pt)dtd p (16.3.1)
π 0 −∞ π 0 −∞

When f (t) is an odd function, f (t). cos pt is an odd function and f (t) sin pt is an even function.
So, the first integral in the right side of (16.3.1) becomes zero. Therefore, we get
Z ∞ Z ∞
2
f (x) = sin px f (t) sin ptdtd p (16.3.2)
π 0 0

which is known as Fourier sine integral. In the same way, when f (t) is an even function, the
second integral in the right side of (16.3.1) becomes zero. Therefore, we get
Z ∞ Z ∞
2
f (x) = cos px f (t) cos ptdtd p (16.3.3)
π 0 0

This is known as Fourier cosine integral.

16.4 Fourier Integral Theorem in Complex Form


R∞ R∞
Theorem 16.4.1. The complex form of the Fourier integral is f (x) = 1
2π −∞ −∞
eip(t−x) f (t)dtd p

Proof. Fourier integral theorem states that


Z ∞ Z ∞
1
f (x) = f (t) cos p(t − x)dtd p.
π 0 −∞

B.Sc.(Mathematics)-IDE(UNOM)-II Year-IV Sem Transform Techniques


346 16.5. Fourier Integral Representation of any Function

Since cos p(t − x) is an even function of p, we have


Z ∞ Z ∞
1
f (x) = f (t) cos p(t − x)dtd p. (16.4.1)
2π −∞ −∞

Also,since sin p(t − x) is an odd function, we have


Z ∞ Z ∞
1
f (x) = f (t) sin p(t − x)dtd p. (16.4.2)
2π −∞ −∞

Now, multiply (19.4.2) by i and adding to (19.4.1), we get


Z ∞ Z ∞
1
f (x) = eip(t−x) f (t)dtd p.
2π −∞ −∞

This relation is called Fourier’s Complex Integral. 

16.5 Fourier Integral Representation of any Function

Let f (x) be any function, then from (16.3.1), we have


Z ∞ Z ∞ Z ∞ Z ∞
1 1
f (x) = cos px cos pt f (t)dtd p + sin px sin pt f (t)dtd p
π 0 −∞ π 0 −∞

Now write Z ∞
θ1 (p) = cos pt f (t)dt
−∞

and Z ∞
θ2 (p) = sin pt f (t)dt
−∞

Therefore

1 ∞ 1 ∞
Z Z
f (x) = θ1 (p) cos pxd p + θ2 (p) sin pxd p
π 0 π 0
1 ∞
Z i
= [θ1 (p) cos px + θ2 (p) sin px d p
π 0

Transform Techniques B.Sc.(Mathematics)-IDE(UNOM)-II Year-IV Sem


16.6. Finite and Infinite Fourier Transforms and Inverse Transforms 347

Further, if f (x) is an odd function, then


Z ∞
1
f (x) = θ2 (p) sin pxd p,
π 0

where Z ∞
θ2 (p) = 2 sin pt f (t)dt.
0

and, if f (x) is an even function, then


Z ∞
1
f (x) = θ1 (p) cos pxd p,
π 0

where Z ∞
θ1 (p) = 2 cos pt f (t)dt.
0

16.6 Finite and Infinite Fourier Transforms and Inverse


Transforms

The complex form of Fourier integral of any function f (x) is in the form
Z ∞ Z ∞ Z ∞ Z ∞
1 1
f (x) = eip(t−x)
f (t)dtd p = e −ipx
dp eipt f (t)dt
2π −∞ −∞ 2π −∞ −∞

Now, we write Z ∞
F(p) = eipt f (t)dt
−∞

Therefore Z ∞
1
f (x) = F(p)e−ipx d p.
2π −∞

Here F(p) is called the Fourier transform of f (x) and the function f (x) is called the inverse
Fourier transform of F(p). Thus we obtain the definition of Fourier transform as follows:
1. The Infinite Fourier Transform of f (x) .

B.Sc.(Mathematics)-IDE(UNOM)-II Year-IV Sem Transform Techniques


348 16.6. Finite and Infinite Fourier Transforms and Inverse Transforms

Definition 16.6.1. Let f (x) be a function defined on (−∞, ∞) and be piecewise continuous in
each finite partial interval and absolutely integrable in (−∞, ∞). Then the Fourier transform of
f (x) denoted by F [ f (x)] = f (p) is defined as
Z ∞
1
F [ f (x)] = f (p) = √ eipx f (x)dx.
2π −∞

The function f (x) is called the inverse Fourier transform of f (p).

i.e., f (x) = F −1 [ f (p)].

The inverse Fourier transform of f (p) is given by


Z ∞
1
f (x) = √ f (p)e−ipx d p.
2π −∞

Definition 16.6.2. Some authors also define the Fourier transform as


(i) Z ∞
1
F [ f (x)] = f (p) = √ e−ipx f (x)dx.
2π −∞

(ii) Z ∞
1
f (x) = √ f (p)eipx d p.
2π −∞

Definition 16.6.3. Some authors also define the Fourier transform as follows
(i) Z ∞
F [ f (x)] = f (p) = e−ipx f (x)dx.
−∞

(ii) Z ∞
1
f (x) = f (p)eipx d p.
2π −∞

2. Fourier Sine Transform

(i) Let f (x) be a function defined in (0, ∞). Thus we obtain the definition of Fourier sine

Transform Techniques B.Sc.(Mathematics)-IDE(UNOM)-II Year-IV Sem


16.6. Finite and Infinite Fourier Transforms and Inverse Transforms 349

transform as r Z ∞
2
f s (p) = F s [ f (x)] = f (x) sin pxdx
π 0

The function f (x) is called the inverse Fourier sine transform of f s (p)
r Z ∞
2
i.e., f (x) = F [ f s (p)] =
−1
f (p) sin pxd p
π 0 s

Note. Some authors also define


Z ∞
f s (p) = F s [ f (x)] = f (x) sin pxdx
0

(ii) The Finite Fourier Sine Transform of f (x) when 0 < x < l, is defined as
Z l
pπx
f s (p) = F s [ f (x)] = f (x) sin dx,
0 l

where p is an integer. The function f (x) is given by


2X pπx
f (x) = f s (p) sin
l p=1 l

is called the Inverse finite Fourier sine transform of f s (p).

2. Fourier Cosine Transform.

(i) Let f (x) be a function defined in (0, ∞). Thus we obtain the definition of Fourier cosine
transform as r Z ∞
2
f s (p) = Fc [ f (x)] = f (x) cos pxdx
π 0

The function f (x) is called the inverse Fourier cosine transform of f c (p)
r Z ∞
2
i.e., f (x) = F [ f c (p)] =
−1
f (p) cos pxd p
π 0 c

B.Sc.(Mathematics)-IDE(UNOM)-II Year-IV Sem Transform Techniques


350 16.7. Relation Between Fourier Transform and Laplace Transform

Note. Some authors also define


Z ∞
f c (p) = Fc [ f (x)] = f (x) cos pxdx
0

(ii) The Finite Fourier Cosine Transform of f (x) when 0 < x < l, is defined as
Z l
pπx
f c (p) = Fc [ f (x)] = f (x) cos dx,
0 l

where p is an integer. The function f (x) is given by


1 2X pπx
f (x) = f c (0) + f c (p) cos
l l p=1 l

is called the Inverse finite Fourier cosine transform of f c (p).

16.7 Relation Between Fourier Transform and Laplace


Transform

Consider the function f (t) defined by



e φ(t), t>0


 −xt

f (t) = 


t<0

0,

Z ∞
∴ F [ f (t)] = f (t)eipt dt
−∞
Z 0 Z ∞
= f (t)e dt + ipt
f (t)eipt dt
−∞ 0
Z 0 Z ∞
F [ f (t)] = 0.e dt +
ipt
e−xt φ(t)eipt dt
−∞ 0

Transform Techniques B.Sc.(Mathematics)-IDE(UNOM)-II Year-IV Sem


16.8. Properties of Fourier Transform 351
Z ∞
F [ f (t)] = φ(t)e−(x−ip)t dt
Z0 ∞
= φ(t)e−st dt, where s = x − ip
0

∴ F [ f (t)] = L [φ(t)]

This is the required relation between Fourier and Laplace transform.

16.8 Properties of Fourier Transform

1. Linear Property. If f (p) and g(p) are Fourier transforms of f (x) and g(x) respectively, then

F [a f (x) + bg(x)] = a f (p) + bg(p)

where a and b are constants.

Proof. We know that


Z ∞ Z ∞
1 1
f (p) = √ e ipx
f (x)dx and g(p) = √ eipx g(x)dx
2π −∞ 2π −∞

Z ∞
1
F [a f (x) + bg(x)] = √ eipx [a f (x) + bg(x)]dx
2π −∞
Z ∞ Z ∞
1 h i
= √ a e f (x)dx + b
ipx
eipx g(x)dx
2π −∞ −∞

= a f (p) + bg(p).

B.Sc.(Mathematics)-IDE(UNOM)-II Year-IV Sem Transform Techniques


352 16.8. Properties of Fourier Transform

Note.
Similarly, (i)
F s [a f (x) + bg(x)] = a f s (p) + bg s (p)

(ii)
Fc [a f (x) + bg(x)] = a f c (p) + bgc (p)

2. Change of Scale Property. If f (p) is the complex Fourier transforms of f (x), then

1  p
F [ f (ax)] = f
a a

where a > 0.

Proof. We have Z ∞
1
f (p) = √ eipx f (x)dx
2π −∞
Z ∞
1
∴ F [ f (ax)] = √ eipx f (ax)dx.
2π −∞

Put ax = t so that dx = dt
a
.
If x → −∞ then t → −∞ and if x → ∞ then t → ∞.

Z ∞ t
1 ip dt
F [ f (ax)] = √ e a f (t)
2π −∞ a
Z ∞  p
1 1 i t
= √ e a . f (t)dt
2π a −∞
1  p
F [ f (ax)] = f
a a

Transform Techniques B.Sc.(Mathematics)-IDE(UNOM)-II Year-IV Sem


16.8. Properties of Fourier Transform 353

Note. If f s (p) and f c (p) are the Fourier sine and cosine transform of f (x) respectively, then

1  p h  x i
F s [ f (ax)] = fs (or) F s f = a f s (ap).
a a a

and
1  p h  x i
Fc [ f (ax)] = fc (or) Fc f = a f c (ap).
a a a
3. Shifting Property. If f (p) is the complex Fourier transforms of f (x), then the complex
Fourier transform f (x − a) is eipa f (p). i.e., F [ f (x − a)] = eipa f (p).

Proof. We have Z ∞
1
f (p) = √ eipx f (x)dx
2π −∞
Z ∞
1
∴ F [ f (x − a)] = √ eipx f (x − a)dx.
2π −∞

Put x − a = t so that dx = dt.


If x → −∞ then t → −∞ and if x → ∞ then t → ∞.

Z ∞
1
F [ f (x − a)] = √ eip(a+t) f (t)dt
2π −∞Z
1 ipa ∞ ipt
= √ e e . f (t)dt
2π −∞

F [ f (x − a)] = eipa f (p).

4. Modulation Theorem. If f (p) is the complex Fourier transforms of f (x), then


F [ f (x) cos ax] = 12 [ f (p + a) + f (p − a)].

Proof. We have Z ∞
1
f (p) = √ eipx f (x)dx
2π −∞

B.Sc.(Mathematics)-IDE(UNOM)-II Year-IV Sem Transform Techniques


354 16.8. Properties of Fourier Transform
Z ∞
1
∴ F [ f (x) cos ax] = √ eipx cos axdx
2π −∞
1
Z ∞  eiax + e−iax 
= √ eipx f (x) dx
2π −∞Z 2
1 h 1  ∞ i(p+a)x
Z ∞ i
= √ e f (x)dx + ei(p−a)x f (x)dx
2 2π −∞ −∞
1
∴ F [ f (x) cos ax] = [ f (p + a) + f (p − a)].
2

5. If f s (p) and f c (p) are the Fourier sine and cosine transforms of f (x) respectively, then

(i) F s [ f (x) cos ax] = 21 [ f s (p + a) + f s (p − a)].


(ii) Fc [ f (x) sin ax] = 21 [ f s (p + a) − f s (p − a)].
(iii) F s [ f (x) sin ax] = 12 [ f c (p − a) − f c (p − a)].

Proof. (i) We have


r Z ∞
2
F s [ f (x) cos ax] = eipx f (x) cos ax sin pxdx
π −∞

r Z ∞
2 h sin(p + a)x + sin(p − a)x i
F s [ f (x) cos ax] = eipx f (x) dx
π −∞ 2
r Z
1 h 2  ∞ ipx
Z ∞ i
= e f (x) sin(p + a)xdx + eipx f (x) sin(p − a)xdx
2 π −∞ −∞
1
F s [ f (x) cos ax] = [ f s (p + a) + f s (p − a)].
2

Transform Techniques B.Sc.(Mathematics)-IDE(UNOM)-II Year-IV Sem


16.8. Properties of Fourier Transform 355

(ii)
r Z ∞
2
Fc [ f (x) sin ax] = eipx f (x) sin ax cos pxdx
π −∞
r Z ∞
2 h sin(p + a)x − sin(p − a)x i
= eipx f (x) dx
π −∞ 2
r Z
1 h 2  ∞ ipx
Z ∞ i
= e f (x) sin(p + a)xdx − eipx f (x) sin(p − a)xdx
2 π −∞ −∞
1
Fc [ f (x) sin ax] = [ f (p + a) − f s (p − a)].
2 s

(iii) Similarly, we can prove

1
F s [ f (x) sin ax] = [ f c (p − a) − f c (p − a)].
2

n
6. F [xn f (x)] = (−i)n ddpn [ f (p)].

Proof.

f (p) = F [ f (x)]
Z ∞
1
= √ f (x)eipx dx
2π −∞Z

d d h 1 i
[ f (p)] = √ f (x)eipx dx
dp d p 2π −∞

Z ∞
1
= √ f (x). (eipx )dx
2π Z−∞ ∂p

1
= √ f (x)(ix)eipx dx
2π −∞
1 d
i.e., F [x f (x)] = [ f (p)]
i dp

B.Sc.(Mathematics)-IDE(UNOM)-II Year-IV Sem Transform Techniques


356 16.8. Properties of Fourier Transform

d
F [x f (x)] = (−i) [ f (p)]
dp
Z ∞
d2 d h 1 i
[ f (p)] = √ f (x)(ix)eipx dx
d p2 d p 2π −∞

Z ∞
1
= √ f (x) [eipx ](ix)dx
2π Z−∞ ∂p

1
= √ f (x)(ix)2 eipx dx
2π −∞Z

1
= (i)2 √ f (x)x2 eipx dx
2π −∞
1 d2
i.e., F [x2 f (x)] = [ f (p)]
(i)2 d p2
(−i)2 d2
= [ f (p)]
(i)2 (−i)2 d p2
d2
F [x2 f (x)] = (−i)2 2 [ f (p)]
dp
dn
In general F [xn f (x)] = (−i)n n [ f (p)].
dp

h
dn
7. F dxn
[ f (x)] = (−ip)n f (p) if f (x) → 0 as x → ±∞

Proof.

hd i
F [ f 0 (x)] = F f (x)
dxZ

1
= √ f 0 (x)eipx dx
2π Z−∞

1
= √ eipx d[ f (x)]
2π −∞ Z ∞
1 h ipx ∞ i
= √ e f (x) − f (x)(ip)eipx dx by integration by parts
−∞
2π −∞

Transform Techniques B.Sc.(Mathematics)-IDE(UNOM)-II Year-IV Sem


16.8. Properties of Fourier Transform 357
Z ∞
1 h i
= √ − ip f (x)eipx dx
2π −∞

= −ipF [ f (x)]

F [ f 0 (x)] = −ip f (p).

h d2 i
Now F [ f 00 (x)] = F f (x)
dxZ2

1
= √ f 00 (x)eipx dx
2π Z−∞

1
= √ eipx d[ f 0 (x)]
2π −∞ Z ∞
1 h ipx 0 ∞ i
= √ e f (x) − f 0 (x)(ip)eipx dx
−∞
2π −∞
Z ∞
1 h i
= √ − ip f 0 (x)eipx dx
2π −∞
1 h i
= √ − ip(−ipF [ f (x)]) since F [ f 0 (x)] = −ipF [ f (x)]

F [ f 00 (x)] = (−ip)2 F [ f (x)]

In general F [ f n (x)] = (−ip)n F [ f (x)]

8. (i) F s [ f 0 (x)] = −p f c (p); (ii) Fc [ f 0 (x)] = p f s (p) − f (0)

Proof.
r Z ∞
2
F s [ f (x)] =
0
f 0 (x) sin pxdx
π
r Z0 ∞
2
= sin pxd[ f (x)]
π 0
r Z ∞
2h i
= ∞
(sin px. f (x))0 − f (x).p. cos pxdx
π 0

B.Sc.(Mathematics)-IDE(UNOM)-II Year-IV Sem Transform Techniques


358 16.9. Worked out Problems
r Z ∞
2h i
= −p f (x) cos pxdx , assuming f (x) → 0 as x → ∞
π 0

F s [ f 0 (x)] = −p f c (p).

Similarly, we can prove (ii)


Fc [ f 0 (x)] = p f s (p) − f (0).

16.9 Worked out Problems

Example 16.9.1. Find the Fourier transform of f (x) defined by



1, |x| < a




f (x) = 


0, |x| > a


R∞ sin p
R∞ sin ap. cos px
and hence evaluate 0 p
dp and −∞ p
d p.

Solution. Part (i):


By the definition of Fourier transform, we have
Z ∞
1
F [ f (x)] = √ f (x)eipx dx
2π −∞
1 h −a
Z Z a Z ∞ i
= √ f (x)e dx +
ipx
f (x)e dx +
ipx
f (x)eipx dx
2π Z−∞ −a a
a
1 h
= √ 1.eipx dx
2π −a
1 h eipx ia
= √
2π ip −a
1 h eipa − e−ipa i
= √
2π ip

Transform Techniques B.Sc.(Mathematics)-IDE(UNOM)-II Year-IV Sem


16.9. Worked out Problems 359
1 2
= √ . sinh(ipa)
2π ip
1 2 sin pa
= √ . (p , 0) (since sinh ix = i sin x)
2π p
1 2 sin pa
∴ F [ f (x)] = √ .
2π p
1 2 sin pa
i.e., f (p) = √ .
2π p
2a
For p = 0, f (0) = √ .

Part (ii):
By the definition of inverse Fourier transform, we have
Z ∞
1
f (x) = √ f (p)e−ipx d p
2π Z−∞

1 1 2 sin pa −ipx
f (x) = √ √ . e dp
2π −∞ 2π p

1 h ∞ sin pa
Z Z ∞
sin pa
f (x) = cos pxd p − i sin pxd p
π −∞ p −∞ p
1 ∞ sin pa
Z
f (x) = cos pxd p
π −∞ p

π, |x| < a


Z 
sin pa


cos pxd p = 

p 
−∞
|x| > a

0,

Part (iii) Putting x = 0 in the above equation, we get

1 ∞ sin pa
Z
f (0) = dp
π −∞ p
2 ∞ sin pa
Z
1 = dp
π 0 p

B.Sc.(Mathematics)-IDE(UNOM)-II Year-IV Sem Transform Techniques


360 16.9. Worked out Problems

π
Z
sin pa
∴ dp =
p 2
Z0 ∞
sin ax π
i.e., dx = .
0 x 2

Put a = 1 in the above equation, we get


π
Z
sin x
dx = .
0 x 2
Example 16.9.2. Find the Fourier transform of f (x) defined by

1 − x ,
 2
|x| ≤ 1



f (x) = 


|x| > 1

0,

R∞ x cos x−sin x
R∞ x cos x−sin x
and hence evaluate (i) 0 x3
cos 2x dx and (ii) −∞ x3
dx.

Solution. Part (i):


By the definition of Fourier transform, we have
Z ∞
1
F [ f (x)] = √ f (x)eipx dx
2π −∞
1 h −1
Z Z 1 Z ∞ i
= √ f (x)e dx +
ipx
f (x)e dx +
ipx
f (x)eipx dx
2π −∞ −1 1
Z 1
1
= √ (1 − x2 ).eipx dx
2π −1
1 h 1
Z Z 1 i
F [ f (x)] = √ (1 − x ) cos pxdx + i
2
(1 − x2 ) sin pxdx
2π −1 −1

Z 1
2
F [ f (x)] = √ (1 − x2 ) cos pxdx
2π 0
2 h  sin px   − cos px   − sin px i1
= √ (1 − x2 ) − (−2x) + (−2)
2π p p2 p3 0

2 h cos p sin p i
= √ 0 − 2 2 + 2 3 − (0 − 0 + 0)
2π p p

Transform Techniques B.Sc.(Mathematics)-IDE(UNOM)-II Year-IV Sem


16.9. Worked out Problems 361
4 h sin p − p cos p i
F [ f (x)] = f (p) = √
2π p3

Part (ii)
By the definition of inverse Fourier transform, we have
Z ∞
1
f (x) = √ f (p)e−ipx d p
2π Z−∞

1 4 h sin p − p cos p i −ipx
= √ √ e dp
2π −∞ 2π p3
2 ∞ h sin p − p cos p i −ipx
Z
f (x) = e dp
π −∞ p3
2 h ∞  sin p − p cos p 
Z Z ∞
sin p − p cos p  i
= cos pxd p − i sin pxdx
π −∞ p3 −∞ p3
Z ∞
2 sin p − p cos p 
= cos pxd p
π −∞ p3
Z ∞  sin p − p cos p 
4
f (x) = cos pxd p. (16.9.1)
π 0 p3
Put x = 1
2
in (16.9.1), we get

4 ∞  sin p − p cos p 
1 Z
p
f = cos dp
2 π 0 p3 2
4 ∞  sin p − p cos p 
Z
1 p
1− = cos dp
4 π 0 p3 2
4 ∞  sin p − p cos p 
Z
3 p
= cos dp
4 π 0 p3 2
Z ∞
sin p − p cos p  p 3π
3
cos d p =
p 2 16
Z0 ∞ 
p cos p − sin p  p 3π
∴ cos dp = −
0 p3 2 16

B.Sc.(Mathematics)-IDE(UNOM)-II Year-IV Sem Transform Techniques


362 16.9. Worked out Problems

(iii) Put x = 0 in (16.9.1), we get

4 ∞  sin p − p cos p 
Z
f (0) = dp
π 0 p3
4 ∞  sin p − p cos p 
Z
1 = dp
π 0 p3
π
Z ∞
p cos p − sin p 
∴ 3
dp = − .
0 p 4

Example 16.9.3. Find the Fourier transform of f (x) defined by



e , α<x<β


 iqx

f (x) = 


x < α and x > β

0,

Solution. By the definition of Fourier transform, we have


Z ∞
1
F [ f (x)] = √ f (x)eipx dx
2π −∞
Z β
1
= √ eiqx eipx dx
2π α
Z β
1
= √ ei(p+q)x dx
2π α
1 h ei(p+q)x iβ
= √
2π i(p + q) α
1 ei(p+q)β − ei(p+q)α
F [ f (x)] = √ . .
2π i(p + q)

Note. If the Fourier transform of f (x) is equal to f (p) then f (x) is called self - reciprocal.

x2
Example 16.9.4. Find the Fourier transform of f (x) defined by f (x) = e− 2 , −∞ < x < ∞.
x2
(or) Show that the Fourier transform of e− 2 is self - reciprocal.

Transform Techniques B.Sc.(Mathematics)-IDE(UNOM)-II Year-IV Sem


16.9. Worked out Problems 363

Solution. By the definition of Fourier transform, we have


Z ∞
1
F [ f (x)] = √ f (x)eipx dx
2π Z−∞

1 x2
= √ e− 2 eipx dx
2π Z−∞

1 x2 +2ipx
F [ f (x)] = √ e− 2 dx
2π −∞

−x2 + i2px = −[(x − ip)2 − (ip)2 ] = −p2 − (x − ip)2

Z ∞
1 1 2 2
F [ f (x)] = √ e 2 (−p −(x−ip) ) dx
2π −∞ Z
1 − p22 ∞ − 12 ((x−ip)2 )
= √ .e e dx
2π −∞


Put x−ip

2
= t. Then dx = 2dt.

1 − p22 ∞ −t2 √
Z
∴ F [ f (x)] = √ .e e 2dt
2π −∞

2 − p22 ∞ −t2
Z
= √ .e e dt
2π −∞
Z ∞
1 − p22 √ 2 √
= √ e . π, since e−x dx = π
π −∞
p2
F [ f (x)] = e− 2 .

x2 p2 x2
Since Fourier transform of e− 2 is e− 2 . Therefore e− 2 is self reciprocal.

Example 16.9.5. Find Fourier transform of f (x) defined by


0, −∞ < x < α








f (x) = 

x, α ≤ x ≤ β






0, x > β


B.Sc.(Mathematics)-IDE(UNOM)-II Year-IV Sem Transform Techniques


364 16.9. Worked out Problems

Solution. By the definition of Fourier transform, we have


Z ∞
1
F [ f (x)] = √ f (x)eipx dx
2π −∞
1 h α
Z Z β Z ∞ i
= √ 0.e dx +
ipx
xe dx +
ipx
0.eipx dx
2π −∞ α β
Z β
1
= √ xeipx dx
2π α
1 h  eipx   eipx i
= √ x − 1.
2π ip (ip)2
1 h βeipβ eipβ i 1 h αeipα eipα i
= √ + 2 − √ + 2
2π ip p 2π ip p
1 h i  ipα  1 i
F [ f (x)] = √ αe − βeipβ + 2 eipβ − eipα .
2π p p

Example 16.9.6. Find the inverse Fourier transform f (x) of F (p) = e−|p|y .

Solution. We have 
ifp < 0





 −p,


|p| = 

ifp > 0


 p,



ifp = 0


0,

From the inverse Fourier transform, we have


Z ∞
1
f (x) = √ f (p)e−ipx d p
2π Z−∞

1
= √ e−|p|y e−ipx d p
2π −∞
Z ∞
1 h 0 py −ipx
Z i
= √ e e dp + e−py e−ipx d p
2π −∞ 0
Z 0 Z ∞
1 h
= √ e(y−ix)p d p + e−(y+ix)p d p
2π −∞ 0

Transform Techniques B.Sc.(Mathematics)-IDE(UNOM)-II Year-IV Sem


16.9. Worked out Problems 365

1 h e(y−ix)p i0 1 h e−(y+ix)p i∞
= √ + √
2π y − ix −∞ 2π −(y + ix) 0
1 h 1 i 1 h 1 i
= √ + √
2π y − ix 2π y + ix
1 2y
= √ . 2
2π y + x2
r
2 y
f (x) = .
π y2 + x2

Example 16.9.7. Show that the Fourier transform of



a − |x|, for|x| < a




f (x) = 


for|x| > a > 0

0,

q  R ∞  2
. Hence deduce that 0 sint t dt = π2 .

2 1−cos as
is π s2

Solution. By the definition of Fourier transform, we have


Z ∞
1
F [ f (x)] = √ f (x)eipx dx
2π −∞
1 h −a
Z Z a Z ∞ i
= √ f (x)e dx +
ipx
f (x)e dx +
ipx
f (x)eipx dx
2π Z −∞ −a a
a
1
F [ f (x)] = √ (a − |x|).eipx dx
2π −a

1 h a
Z Z a i
F [ f (x)] = √ (a − |x|) cos pxdx + i (a − |x|) sin pxdx
2π Z −a −a
a
2
= √ (a − |x|) cos pxdx
2π Z0
a
2
= √ (a − x) cos pxdx since a − |x| = a − x in (0, a)
2π 0

B.Sc.(Mathematics)-IDE(UNOM)-II Year-IV Sem Transform Techniques


366 16.9. Worked out Problems

2 h  sin px   − cos px ia


= √ (a − x) − (−1)
2π p p2 0

2 h cos ap  1 i
= √ 0− − 0 −
2π p2 p2
2 h 1 − cos ap i
= √
2π p2
r
2 h 1 − cos ap i
F [ f (x)] = f (p) = .
π p2

Deduction. By the definition of inverse Fourier transform, we have


Z ∞
1
f (x) = √ f (p)e−ipx d p
2π −∞
Z ∞r h
1 2 1 − cos ap i −ipx
= √ e dp
2π −∞ π p2
1 ∞ 1 − cos ap −ipx
Z
f (x) = e dp
π −∞ p2

Put x = 0, we have

1 ∞ 1 − cos ap
Z
f (0) = dp
π −∞ p2
1 ∞ 1 − cos ap
Z
a = dp
π −∞ p2

Since f (x) = a − |x| in |x| < a. Therefore f (0) = a

1 ∞
Z  ap  1
a = 2 sin2 . dp
π −∞ 2 p2
2 ∞ 2  ap  1
Z
a = sin . dp
π −∞ 2 p2
4 ∞ 2  ap  1
Z
a = sin . dp
π 0 2 p2

Transform Techniques B.Sc.(Mathematics)-IDE(UNOM)-II Year-IV Sem


16.9. Worked out Problems 367

Put ap
2
= t. Then a2 d p = dt ⇒ d p = 2dt
a

Z ∞
4 1 2dt
a = sin2 t. 4t2
π 0 2
a
a
Z ∞ 2
4 a sin t
a = . dt
π 2 0 t2

sin2 t π
Z
2
dt = .
0 t 2

Example 16.9.8. Find the Fourier transform of the function


1 + ax , for − a < x < 0








f (x) = 

1 − ax , for 0 < x < a








0,
 otherwise

Solution. By the definition of Fourier transform, we have


Z ∞
1
F [ f (x)] = √ f (x)eipx dx
2π −∞
1 h −a
Z Z 0 Z a Z ∞ i
= √ f (x)e dx +
ipx
f (x)e dx +
ipx
f (x)e dx +
ipx
f (x)eipx dx
2π −∞ −a 0 a
1 h 0
Z Z a
x  ipx x  ipx i
= √ 1 + e dx + 1 − e dx
2π −a a 0 a
ipx  ipx 0
1 h x e 1 e
  x  eipx  1  eipx 0 i
= √ 1+ − + 1 − +
2π a ip a (ip)2 −a a ip a (ip)2 −a
1 h 1 1 1 e−ipa   1 eipa 1 1 i
= √ + 2− + − − +
2π ip ap a p2 a p2 ip ap2
1 h 2 1  ipa i
= √ − e + e −ipa
2π ap2 ap2
1 h 2 1 i
= √ − 2 cos pa
2π ap2 ap2
2
= √ (1 − cos pa)
ap2 2π
2  pa 
= √ 2 sin2
ap2 2π 2

B.Sc.(Mathematics)-IDE(UNOM)-II Year-IV Sem Transform Techniques


368 16.9. Worked out Problems
r
2 1 2 ap
 
F [ f (x)] = 2 sin .
π ap2 2

Example 16.9.9. Find the Fourier transform of √1 .


|x|

Solution. By the definition of Fourier transform, we have


Z ∞
1
F [ f (x)] = √ f (x)eipx dx
2π Z−∞

1 1
= √ √ eipx dx
2π −∞ |x|
Z ∞
1 h 0 1 ipx
Z
1
= √ √ e dx + √ eipx dx
2π −∞ −x 0 x

Put −x = t in the first integral. Then dx = −dt.


When x = 0 → t = 0 and when x → −∞, t → ∞.
Z ∞
1 h 0 1 −ipt
Z
1
F [ f (x)] = √ √ e (−dt) + √ eipx dx
2π Z∞ t x
Z ∞0
1 h ∞ 1 −ipt 1 ipx
= √ √ e dt + √ e dx
2π Z0 t 0 x
1 h ∞ 1 −ipx
Z ∞
1
= √ √ e dx + √ eipx dx
2π 0 x 0 x
e +e
Z ∞ ipx −ipx
1
= √ √ dx
2π Z0 x

1 2 cos px
= √ √ dx
2π 0 x
Z ∞ −ipx
1 e d(ipx)
= 2 × R.P o f √ √
2π 0 x ip
Z ∞ −ipx
1 e d(ipx)
= 2 × R.P o f √ √ √
2π Z 0 ipx ip
∞ −t
1 2 e
= R.P o f √ . √ √ dt, putting ipx = t
2π ip 0 t

Transform Techniques B.Sc.(Mathematics)-IDE(UNOM)-II Year-IV Sem


16.9. Worked out Problems 369
√ Z ∞
2 1
= R.P o f √ √ e−t t 2 −1 dt
π ip 0

2 1
1
= R.P o f √ √ i− 2 Γ , by the definition of gamma function
π p 2

2  π π − 21 √
= R.P o f √ √ cos + i sin . π
π p 2 2

2 π π
F [ f (x)] = R.P o f √ cos − i sin
p 4 4


2 π
F [ f (x)] = √ cos
p 4

2 1
= √ .√
p 2
h 1 i 1
F √ = √ .
|x| p

Example 16.9.10. Find the Fourier transform of




 x, if |x| ≤ a



f (x) = 


0, if |x| > a


Solution. Z ∞
1
F [ f (x)] = √ f (x)eipx dx
2π −∞

Given f (x) = x in −a ≤ x ≤ a and f (x) = 0 in −∞ < x < −a and a < x < ∞

1 h −a
Z Z a Z ∞ i
F [ f (x)] = √ f (x)e dx +
ipx
f (x)e dx +
ipx
f (x)eipx dx
2π −∞ −a a
Z −a Z a Z ∞
1 h i
= √ 0.eipx dx + x.eipx dx + 0.eipx dx
2π Z −∞ −a a
a
1
= √ x.eipx dx
2π −a

B.Sc.(Mathematics)-IDE(UNOM)-II Year-IV Sem Transform Techniques


370 16.9. Worked out Problems

1 h  eipx   eipx ia


= √ x − 1.
2π ip (ip)2 −a
1 h aeipa eipa   −ae−ipa e−ipa i
= √ + 2 − + 2
2π ip p ip p
1 a ipa
h 1 i
= √ (e + e−ipa ) + 2 (eipa − e−ipa )
2π ip p
1 a h 1 i
= √ (2 cos pa) + 2 (2i sin pa)
2π ip p
r
2  a cos pa i sin pa 
= +
π ip p2
r
2  sin ap − ap cos ap 
F [ f (x)] = i .
π p2

Example 16.9.11. Find the Fourier transform of




cos x, if |x| ≤ a



f (x) = 


if |x| > a > 0

0,

Solution. Z ∞
1
F [ f (x)] = √ f (x)eipx dx
2π −∞
1 h −a
Z Z a Z ∞ i
F [ f (x)] = √ f (x)e dx +
ipx
f (x)e dx +
ipx
f (x)eipx dx
2π −∞ −a a
Z −a Z a Z ∞
1 h i
= √ 0.eipx dx + cos x.eipx dx + 0.eipx dx
2π Z −∞ −a a
a
1
= √ cos x.eipx dx
2π −a
1 h eipx ia
= √ (ip cos x + sin x)
2π i2 p2 + 1 −a

1 h eipa e−ipa i
= √ (ip cos a + sin a) − (ip cos a − sin a)
2π 1 − p2 1 − p2
1 1 h ipa i
= √ (e − e−ipa
)ip cos a + (eipa
− e −ipa
) sin a
2π 1 − p2
1 1 h i
= √ (2i sin pa)ip cos a + (2 cos pa) sin a
2π 1 − p2

Transform Techniques B.Sc.(Mathematics)-IDE(UNOM)-II Year-IV Sem


16.9. Worked out Problems 371
1 2 h i
F [ f (x)] = √ sin a cos pa − p cos a sin pa .
2π 1 − p2

Example 16.9.12. Find the Fourier transform of



if 0 < x < π

sin x,



f (x) = 



0,

 otherwise

Solution. Z ∞
1
F [ f (x)] = √ f (x)eipx dx
2π −∞
Z π Z ∞
1 h 0
Z i
F [ f (x)] = √ f (x)e dx +
ipx
f (x)e dx +
ipx
f (x)eipx dx
2π −∞ 0 π
Z 0 Z π Z ∞
1 h i
= √ 0.eipx dx + sin x.eipx dx + 0.eipx dx
2π Z −∞ 0 π
π
1
F [ f (x)] = √ sin x.eipx dx
2π 0

1 h eipx iπ
F [ f (x)] = √ (ip sin x − cos x)
2π i2 p2 + 1 0

1 h eipπ 1 i
= √ (0 − (−1)) − (0 − 1)
2π 1 − p2 1 − p2
1 h eipπ 1 i
= √ +
2π 1 − p2 1 − p2
1 1 + eipπ
F [ f (x)] = √ .
2π 1 − p2

Example 16.9.13. Find the Fourier transform of f (x) = xe−x , 0 ≤ x < ∞.

Solution. Z ∞
1
F [ f (x)] = √ f (x)eipx dx
2π Z−∞

1
= √ xe−x eipx dx
2π 0

B.Sc.(Mathematics)-IDE(UNOM)-II Year-IV Sem Transform Techniques


372 16.9. Worked out Problems
Z ∞
1
= √ xe−(1−ip)x dx
2π 0
1 h  e−(1−ip)x  e−(1−ip)x i∞
= √ x − 1.
2π 1 − ip (1 − ip)2 0
1 h  1 i
= √ 0−0− 0−
2π (1 − ip)2
1 1
= √
2π (1 − ip)2
1 (1 + ip)2
F [ f (x)] = √ .
2π (1 + p2 )2

x2 p2
Example 16.9.14. Show that the Fourier transform of e− 2 is e− 2 by finding the Fourier
transform of e−a x .
2 2

Solution. Z ∞
1
F [ f (x)] = √ f (x)eipx dx
2π Z−∞

1 2 2
= √ e−x a eipx dx
2π Z−∞

1
e−x a +ipx dx
2 2
F [ f (x)] = √
2π −∞

 ip 
−a2 x2 + ipx = −a2 x2 − 2 x
a
h ip 2  ip 2 i
= −a2 x − 2 −
2a 2a2
h ip 2 p2 i
= −a x − 2 + 4
2
2a 4a
2
 ip 2 p
−a2 x2 + ipx = −a2 x − 2 − 2
2a 4a
Z ∞  2 2
1 −a2 x− ip2 − p 2
F [ f (x)] = √ e 2a 4a
dx
2π −∞
 2
1 − p22 ∞ −a2 x− 2aip2
Z
= √ e 4a e dx
2π −∞

Transform Techniques B.Sc.(Mathematics)-IDE(UNOM)-II Year-IV Sem


16.9. Worked out Problems 373
 
Put a x − ip
2a2
= t. Then dx = dt
a
.
When x → −∞ ⇒ t → ∞ and x → ∞ ⇒ t → −∞

1 − p22 ∞ −t2 dt
Z
∴ F [ f (x)] = √ e 4a e
2π −∞ a
Z ∞
1 − p2 √
2
2 √
= √ e 4a . π, since e−x dx = π
2π −∞
1 − p22
F [ f (x)] = √ e 4a
a 2

Put a = √1
2
in the above equation, we get

x2 p2
F [e− 2 ] = e− 2 .

Example 16.9.15. Find the Fourier transform of e−a|x| and hence show that
R ∞ px √
π −a|x|
(i) 0 acos
2 +p2 d p = 2a
e and (ii) F [xe−a|x|
] = √
2 2pai
π(p2 +a2 )2
.

Solution. Z ∞
1
F [ f (x)] = √ f (x)eipx dx
2π Z−∞

1
F [e−a|x| ] = √ e−a|x| eipx dx
2π Z −∞

1
F [e−a|x| ] = √ e−a|x| [cos px + i sin px]dx
2π −∞

1 h ∞ −a|x|
Z Z ∞ i
F [e −a|x|
] = √ e cos pxdx + i e−a|x| sin pxdx
2π Z−∞ −∞

1
= √ 2. e−a|x| cos pxdx
2π Z0

1
= √ 2. e−ax cos pxdx
2π 0
r
2 a
= .
π a2 + p2

B.Sc.(Mathematics)-IDE(UNOM)-II Year-IV Sem Transform Techniques


374 16.9. Worked out Problems

(i) By the definition of inverse Fourier transform, we have


Z ∞
1
f (x) = √ F [ f (x)]e−ipx d p
2π −∞
Z ∞r
1 2 a
e−a|x| = √ e−ipx d p
2π −∞ π a 2 + p2

a ∞
Z
1
= (cos px − i sin px)dx
π −∞ a + p2
2

ah ∞
Z Z ∞
1 1 i
= cos pxdx − i sin pxdx
π −∞ a2 + p2 −∞ a + p
2 2
Z ∞
a 1
e−a|x| = 2. cos pxdx
π 0 a + p
2 2

π −a|x|
Z
1
cos pxdx = e .
0 a2 + p2 2a

(ii) We know that


dn
F [xn f (x)] = (−i)n F [ f (x)]
d pn
For n = 1, we have
d
F [x f (x)] = −i F [ f (x)]
dp

r
d h 2 a i
F [xe −a|x|
] = −i
dp π a2 + p2
r
2h 1 i i
= −ai − 2 (2p)
π (a + p2 )2

2 2pai
F [xe−a|x| ] = √ 2 .
π(p + a2 )2

Example 16.9.16. Find the Fourier transform of e−|x| and hence show that
R ∞ xt √
π −|x|
(i) 0 cos
1+t2
d p = 2
e and (ii) F [xe−|x|
] = √
2 2pi
π(p2 +1)2
.

Solution. Put a = 1 in the above problem.

Transform Techniques B.Sc.(Mathematics)-IDE(UNOM)-II Year-IV Sem


16.9. Worked out Problems 375

Example 16.9.17. Find the Fourier transform of f (x) defined by



a − x , |x| < a


 2 2

f (x) = 


|x| > a

0,

R∞
and hence show that 0
sin x−cos x
x3
dx = π4 .

Solution. By the definition of Fourier transform, we have


Z ∞
1
F [ f (x)] = √ f (x)eipx dx
2π −∞
1 h −a
Z Z a Z ∞ i
= √ f (x)e dx +
ipx
f (x)e dx +
ipx
f (x)eipx dx
2π Z −∞ −a a
a
1
= √ (a2 − x2 ).eipx dx
2π Z−a
1 h a 2
Z a i
= √ (a − x ) cos pxdx + i
2
(a2 − x2 ) sin pxdx
2π Z −a −a
a
2
= √ (a2 − x2 ) cos pxdx
2π 0
2 h  sin px   − cos px   − sin px ia
= √ (a2 − x2 ) − (−2x) + (−2)
2π p p2 p3 0

2 h cos ap sin ap i
= √ 0 − 2a 2 + 2 3 − (0 − 0 + 0)
2π p p
4 sin ap − ap cos ap
h i
F [ f (x)] = f (p) = √
2π p3

Deduction. By the definition of inverse Fourier transform, we have


Z ∞
1
f (x) = √ f (p)e−ipx d p
2π Z−∞

1 4 h sin ap − ap cos ap i −ipx
= √ √ e dp
2π −∞ 2π p3
2 ∞ h sin ap − ap cos ap i −ipx
Z
f (x) = e dp
π −∞ p3

B.Sc.(Mathematics)-IDE(UNOM)-II Year-IV Sem Transform Techniques


376 16.10. Exercises

Put x = 0 in the above equation, we get

2 ∞ h sin ap − ap cos ap i
Z
f (0) = dp
π −∞ p3
2 ∞ h sin ap − ap cos ap i
Z
a =
2
dp
π −∞ p3

Put ap = t. Then d p = dt
a
.
When p → −∞ ⇒ t → −∞ and p → ∞ ⇒ t → ∞.
Z ∞ h sin t − t cos t i dt
2
a =
2
π −∞
t3 a
a3
∞h
2a2
Z
sin t − t cos t i
= dt
π −∞ t3
π ∞h
Z
sin t − t cos t i
a2 . = 2. dt
2a2 0 t3
π
Z ∞h
sin t − t cos t i
∴ 3
dt = .
0 t 4

16.10 Exercises

1. Find the Fourier transform of

(i) 
x , |x| < x0


 2

f (x) = 


|x| > x0

0,

(ii) 
 2a , |x| ≤ a


 1

f (x) = 


0, |x| > a


Transform Techniques B.Sc.(Mathematics)-IDE(UNOM)-II Year-IV Sem


16.10. Exercises 377

2. Show that the Fourier transform of the function


√
 2a ,
 2π
|x| ≤ a



f (x) = 


|x| > a

0,

is sin pa
pa
.
x2
3. Find the Fourier transform of e− 2 , −∞ < x < ∞.
4. Show that the Fourier transform of

0, forx < α








f (x) = 

1, forα < x < β






0, forx > β.


1 iβp
is ip
(e − eiαp ).

Answers.
√ h i
1. (i) √ 2 3 (x2 p2
πp 0 − 2) sin px0 = 2px0 cos px0 .
(ii) π √1 2 sinaαaα .

p2
3. e− 2 .

B.Sc.(Mathematics)-IDE(UNOM)-II Year-IV Sem Transform Techniques


378
UNIT-IV

Lesson 17

FOURIER SINE AND


COSINE TRANSFORMS
Learning Objectives
Upon completion of this lesson, students will be able to

• solve problems by using infinite Fourier sine and cosine transform

• solve integral equations by using infinite Fourier sine and cosine transform.

17.1 Introduction

I
n this Lesson, we present problems related to (infinite) Fourier Sine and Cosine Transforms.

17.2 Worked out Problems

Example 17.2.1. Find the Fourier sine and cosine transform of f (x) = x.

Solution. By the definition of Fourier sine and cosine transform, we have


r Z ∞ r Z ∞
2 2
f s (p) = f (x) sin pxdx = x sin pxdx
π 0 π 0

B.Sc.(Mathematics)-IDE(UNOM)-II Year-IV Sem 379 Transform Techniques


380 17.2. Worked out Problems

and r Z ∞ r Z ∞
2 2
f c (p) = f (x) cos pxdx = x cos pxdx
π 0 π 0
Let r Z ∞
2
f (p) = xe−ipx dx
π 0
Put ipx = t. Then ipdx = dt
r Z ∞
2 t −t dt
f (p) = e
π 0 ip ip
r Z ∞
1 2
= − 2 e−t t2−1 dt
p π 0
r Z ∞
1 2
= − 2 Γ(2), since Γ(n) = e−x xn−1 dx, n > 0
p π 0
r
1 2
= − 2 , Γ(n) = (n − 1)! i f n is a positive integer
p π
r Z ∞ r
2 1 2
∴ xe−ipx dx = − 2
π 0 p π
Z ∞
1
xe−ipx dx = − 2.
0 p

Equating real and imaginary parts on both sides, we get


Z ∞ Z ∞
1
x sin pxdx = 0 x cos pxdx = − .
0 0 p2

1
i.e., f s (p) = 0 and f c (p) = − .
p2
Example 17.2.2. Find the Fourier cosine transform of the function f (x) defined by

0<x<a

cos x,



f (x) = 



0,

 x≥a

Transform Techniques B.Sc.(Mathematics)-IDE(UNOM)-II Year-IV Sem


17.2. Worked out Problems 381

Solution. By the definition of Fourier cosine transform, we have


r Z ∞
2
f c (p) = f (x) cos pxdx
π 0
r Z a
2
= cos x cos pxdx
π 0
r Z a
2 h cos(1 + p)x + cos(1 − p)x i
f c (p) = dx
π 0 2

Z a
1
f c (p) = √ (cos(1 + p)x + cos(1 − p)x)dx
2π 0
1 h sin(1 + p)x sin(1 − p)x ia
= √ +
2π 1+ p 1− p 0

1 h sin(1 + p)a sin(1 − p)a i


f c (p) = √ + .
2π 1+ p 1− p

Example 17.2.3. Find the Fourier cosine transform of the function f (x) defined by

0<x<a

sin x,



f (x) = 



0,

 x≥a

Solution. By the definition of Fourier cosine transform, we have


r Z ∞
2
f s (p) = f (x) sin pxdx
π 0
r Z a
2
= sin x cos pxdx
π 0
r Z a
2 h cos(1 − p)x − cos(1 + p)x
= dx
π 0 2
Z a
1
= √ (cos(1 − p)x − cos(1 + p)x)dx
2π 0
1 h sin(1 − p)x sin(1 + p)x ia
= √ −
2π 1− p 1+ p 0

1 h sin(1 − p)a sin(1 + p)a i


= √ − .
2π 1− p 1+ p

B.Sc.(Mathematics)-IDE(UNOM)-II Year-IV Sem Transform Techniques


382 17.2. Worked out Problems

Example 17.2.4. Find the Fourier sine transform of e−x

Solution. r Z ∞
2
f s (p) = f (x) sin pxdx
π 0
r Z ∞
2
= e−x sin pxdx
π 0
r
2 h e−x i∞
= (− sin px − p cos px)
π p2 + 1 0
r
2 p
∴ f s (e−x ) = .
π p2 + 1

Note. r Z ∞ r
2 2 1
f c (e−x ) = e−x cos pxdx = .
π 0 π p2 + 1
−ax
Example 17.2.5. Find the Fourier sine and cosine transforms of f (x) = e x , deduce that
R ∞ −ax −bx    
(i) 0 e −e x
sin pxdx = tan −1 p
a
− tan −1 p
b
R ∞ h e−ax −e−bx i  q p2 +b2 
and (ii) 0 x
cos pxdx = log p2 +a2
.

Solution. Fourier Sine Transform.


r Z ∞
2
f s (p) = f (x) sin pxdx
π 0
r Z ∞
2 e−ax
= sin pxdx
π 0 x

Differentiating with respect to p, we get


r Z ∞
d 2 e−ax
[F s [ f (x)]] = x cos pxdx
dp π 0 x
r Z ∞
2
= e−ax cos pxdx
π 0
r
d 2 a
[F s [ f (x)]] =
dp π p2 + a2

Transform Techniques B.Sc.(Mathematics)-IDE(UNOM)-II Year-IV Sem


17.2. Worked out Problems 383

Integrating with respect to p, we get


r Z r
2 a 2 −1 p
 
F s [ f (x)] = d p = tan .
π p2 + a2 π a
r
e−ax 2  p
Fs[ ] = tan−1 .
x π a

We know that Fourier sine transform of f (x) is given by


r Z ∞
2
F s [ f (x)] = f s (p) = f (x) sin pxdx
π 0
r Z ∞
2 h e−ax − e−bx i h e−ax − e−bx i
sin pxdx = Fs
π 0 x x
r Z ∞
2 h e−ax − e−bx i −ax
e   e−bx 
sin pxdx = Fs − Fs
π 0 x x x
r Z ∞ r
2 h e−ax − e−bx i 2 h  p   p i
sin pxdx = tan−1 − tan−1
π 0 x π a b

Fourier Cosine Transform.


r Z ∞
2
f c (p) = f (x) cos pxdx
π 0
r Z ∞
2 e−ax
= cos pxdx
π 0 x

Differentiating with respect to p, we get


r Z ∞
d 2 e−ax
[F s [ f (x)]] = (−x sin px)dx
dp π 0 x
r Z ∞
2
= − e−ax sin pxdx
π 0
r
d 2 p
[F s [ f (x)]] = −
dp π p2 + a2

B.Sc.(Mathematics)-IDE(UNOM)-II Year-IV Sem Transform Techniques


384 17.2. Worked out Problems

Integrating with respect to p, we get


r Z ∞ r Z
2 p 2 2p 1
Fc [ f (x)] = − dp = − d p = − √ log(p2 + a2 ).
π 0 p +a
2 2 π p +a
2 2

e−ax 1
Fc [ ] = − √ log(p2 + a2 )
x 2π
We know that Fourier cosine transform of f (x) is given by
r Z ∞
2
Fc [ f (x)] = f c (p) = f (x) cos pxdx
π 0

r Z ∞
2 h e−ax − e−bx i h e−ax − e−bx i
cos pxdx = Fc
π x x
r Z0 ∞
2 h e−ax − e−bx i  e−ax   e−bx 
cos pxdx = Fc − Fc
π 0 x x x

r Z ∞
2 h e−ax − e−bx i 1 1
cos pxdx = − √ log(p2 + a2 ) + √ log(p2 + b2 )
π 0 x 2π 2π
r Z ∞
2 h e−ax − e−bx i 1  p2 + b2 
cos pxdx = √ log 2
π 0 x 2π p + a2
1  p2 + b2 
= log 2
2 p + a2
s
Z ∞ h −ax
e − e−bx i  p2 + b2 
cos pxdx = log .
0 x p2 + a2

e−2x
Note. Find the Fourier sine transform of x
.
h i q  p
−2x
Putting a = 2 in the above problem, we get Fs e x = 2
π
tan−1 2
.

Transform Techniques B.Sc.(Mathematics)-IDE(UNOM)-II Year-IV Sem


17.2. Worked out Problems 385

Example 17.2.6. Find the Fourier sine and cosine transform of



0<x<1

1,



f (x) = 


x>1

0,

Solution. Fourier Sine Transform. By definition,


r Z ∞
2
F s [ f (x)] = f (x) sin pxdx
π 0
r Z 1 Z ∞
2
= f (x) sin px + f (x) sin pxdx
π 0 1
r Z 1 Z ∞
2
= 1. sin px + 0. sin pxdx
π 0 1
r
2  cos px 1
= − +0
π p 0
r
21
= − (cos p − 1)
πp
1 − cos p
F s [ f (x)] = .
p

Fourier Cosine Transform. By definition,


r Z ∞
2
Fc [ f (x)] = f (x) cos pxdx
π 0
r Z 1 Z ∞
2
Fc [ f (x)] = f (x) cos px + f (x) cos pxdx
π 0 1
r Z 1 Z ∞
2
Fc [ f (x)] = 1. cos px + 0. cos pxdx
π 0 1
r
2  sin px 1
= +0
π p 0
r
21
= (sin p − 0)
πp
r
2 sin p
F s [ f (x)] = .
π p

B.Sc.(Mathematics)-IDE(UNOM)-II Year-IV Sem Transform Techniques


386 17.2. Worked out Problems

Example 17.2.7. Find the Fourier cosine transform of f (x) defined by f (x) = 1
1+x2
and hence
find the Fourier sine transform of f (x) = x
1+x2
.

Solution. r Z ∞
2
Let I = Fc [ f (x)] = f (x) cos pxdx
π 0
r Z ∞
2 1
= cos pxdx
π 0 1 + x2

Differentiating both sides with respect to p, we get


r Z ∞
d 2 x sin px
Fc [ f (x)] = − dx
dp π 0 1 + x2
r Z ∞
2 x2 sin px
= − dx
π 0 x(1 + x2 )
r Z ∞
2 1 1 
= − − sin pxdx
π 0 x x(1 + x2 )
r Z ∞ Z ∞
2h sin px x sin px
= − dx + dx
π 0 x 0 x(1 + x2 )
r
2h π
Z ∞
sin px i
= − + dx
π 2 0 x(1 + x2 )

Differentiating with respect to p, we get


r Z ∞
d2 2 cos px
[Fc [ f (x)]] = dx = Fc [ f (x)]
d p2 π 0 1 + p2

which gives (D2 − 1)Fc [ f (x)] = 0, whose general solution is Fc [ f (x)] = c1 e p + c2 e−p . When
p = 0, r Z ∞ r r
2 π π
r
2 1 2
Fc [ f (x)] = dx = (tan (x))0 =
−1 ∞
. =
π 0 1+x 2 π π 2 2

π
r
c1 + c2 = (17.2.1)
2
Also r
d 2
[Fc [ f (x)] = − , (when p = 0)
dp π

Transform Techniques B.Sc.(Mathematics)-IDE(UNOM)-II Year-IV Sem


17.2. Worked out Problems 387
r
2
c1 + c2 = − (17.2.2)
π
q
From (20.2.1) and (17.2.2), we have c1 = 0 and c2 = 2
π

r
2 −p
∴ I = Fc [ f (x)] = e .
π

Now, r
dI 2 −p
=− e
dp π
Since r Z ∞
dI 2 x sin px h x i
=− dx = −F s
dp π 0 1 + x2 1 + x2
r r
2 −p h x i h x i 2 −p
∴− e = −F s ⇒ Fs = e .
π 1+x 2 1+x 2 π
Example 17.2.8. Find the Fourier sine transform of f (x) defined by f (x) = 1
x(x2 +a2 )
and hence
deduce Fourier cosine transform of f (x) = 1
x2 +a2
.

Solution. r Z ∞
2
Let F s [ f (x)] = f (x) sin pxdx
π 0
r Z ∞
2 1
= sin pxdx
π 0 x(x + a2 )
2

Differentiating both sides with respect to p, we get


r Z ∞
d 2 cos px
[F s [ f (x)]] = dx (17.2.3)
dp π 0 x 2 + a2
Z ∞
1
2ye−(x +a )y dy =
2 2 2
S ince
0 x2 + a2
Now multiplying both sides by cos px and then integrating the resulting function from 0 to ∞,
with respect to x, we get

B.Sc.(Mathematics)-IDE(UNOM)-II Year-IV Sem Transform Techniques


388 17.2. Worked out Problems

r Z ∞ r Z ∞Z ∞
2 cos px 2
cos px2ye−(x +a )y dydx
2 2 2
dx =
π 0 x +a
2 2 π
r Z0 ∞ 0
2 hZ ∞ 2 2
−a2 y2
i
= 2ye e−x y cos pxdx dy
π 0 0

Now, let Z ∞
2 2
S = e−x y cos pxdx
0

Differentiating with respect to p, we get


Z ∞
dS 2 2
= e−x y (−x sin px)dx
dp 0
Z ∞
1 2 2
= 2
2y2 e−x y (−x sin px)dx
2y 0
Z ∞
1 2 −x2 y2
= −2xy e sin pxdx
2y2 0
1 h −x2 y2 ∞ Z ∞ 2 2 i
= e sin px − e−x y
p cos pxdx
2y2 0
0
1 h
= 0 − pS ]
2y2
dS pS
= − 2
dp 2y
dS dp
= −p 2
S 2y
p2
Integrating log S = − 2 + log c1
4y
p2

= log e 4y2 + log c1
p2

∴ S = c1 e 4y2

When p = 0, √

π
Z
−x2 y2
S = e dx =
0 2y

π
∴ c1 = .
2y

Transform Techniques B.Sc.(Mathematics)-IDE(UNOM)-II Year-IV Sem


17.2. Worked out Problems 389

Hence √

π p2
Z
−x2 y2
S = e cos pxdx = . − 4y2 .
0 2y e

r Z ∞ r Z ∞ √
2 cos px 2 −a2 y2 π p2
∴ dx = 2ye . − dy
π 0 x 2 + a2 π 0 2y e4y2
 
√ Z ∞ −a2 y2 + p22 2
= 2 e 4a y
dy
0

√ π −2 2ap a2
= 2. e
r Z ∞ 2a
πe
r −pa
2 cos px
dx = .
π 0 x +a
2 2 2 a

π e−pa
r
d
∴ F s [ f (x)] = .
ds 2 a
Integrating both sides with respect to p, we get
√ Z √  −pa  √
π π e π
F s [ f (x)] = √ e dp = √ .
−pa
+ c2 = − √ e−pa + c2
a 2 a 2 −a a2 2

When p = 0, we have r Z ∞
2
F s [ f (0)] = f (x).0dx = 0
π 0

π
∴ c2 = √
a2 2
Hence √ √
π −pa π π
r
F s [ f (x)] = − 2 √ e + √ = [1 − e−pa ].
a a 2
a 2 2

If f (x) = 1
x2 +a2
, then

r Z ∞ r
2 π −pa π −pa
r
2 cos px
Fc [ f (x)] = dx = . e = e
π 0 x +a
2 2 π 2a 2

Example 17.2.9. Find the Fourier cosine transform of e−x .


2

B.Sc.(Mathematics)-IDE(UNOM)-II Year-IV Sem Transform Techniques


390 17.2. Worked out Problems

Solution. Let r Z ∞
2 2
I = Fc [ f (x)] = e−x cos pxdx.
π 0
Differentiating both sides with respect to p, we get
r Z ∞
dI 2 2
= e−x (−x sin px)dx
dp π 0

r
2 1 ∞
Z
dI 2
= . (−2xe−x ) sin pxdx
dp π 2 0
Z ∞
1 h −x2 2
= √ (e sin px)0 − ∞
pe−x cos pxdx
2π Z 0

p 2
= −√ e−x cos pxdx
2π 0
dI p
= − √ .I
dp 2π
dI p
= − √ dp
I 2π
p
Integrating, log I = − √ + log c
2 2π
p2

I = ce .

2 2π

If p = o, then √

π
Z
−x2
I= e dx = .
0 2


π
∴ =c
2 √
π − √p2
Hence I = e 2 2π
2

π − √p2
i, e., Fc [ f (x)] = e 2 2π .
2

Example 17.2.10. Find the Fourier cosine and sine transform of e−ax , a > 0 and hence evaluate
R ∞ px R ∞ p sin px
(i) 0 acos
2 +p2 d p and (ii) 0 a2 +p2
dp

Transform Techniques B.Sc.(Mathematics)-IDE(UNOM)-II Year-IV Sem


17.2. Worked out Problems 391

Solution. By definition,
r Z ∞
2
Fc [ f (x)] = cos pxdx
π 0
r Z ∞
2
Fc [e )] =
−ax
e−ax cos pxdx
π 0
r
2 a
Fc [e−ax )] = .
π a2 + p2

r Z ∞
2
F s [ f (x)] = sin pxdx
π 0
r Z ∞
2
F s [e−ax )] = e−ax sin pxdx
π 0
r
2 p
F s [e−ax )] = .
π a2 + p2

(i)Now by the inverse Fourier cosine transform, we have


r Z ∞
2
f (x) = Fc [ f (x)] cos pxdx
π 0
r Z ∞r
2 2 a
f (x) = cos pxdx
π 0 π a2 + p2
2a ∞ cos px
Z
e−ax = dx
π 0 a2 + p2

π −ax
Z
cos px
i.e., dx = e .
0 a2 + p2 2a

(ii)Now by the inverse Fourier sine transform, we have


r Z ∞
2
f (x) = F s [ f (x)] sin pxdx
π 0
r Z ∞r
2 2 p
f (x) = sin pxdx
π 0 π a2 + p2

B.Sc.(Mathematics)-IDE(UNOM)-II Year-IV Sem Transform Techniques


392 17.2. Worked out Problems

2 ∞ p sin px
Z
e = −ax
dx
π 0 a2 + p2

π −ax
Z
p sin px
i.e., dx = e .
0 a2 + p2 2

Note. Find the Fourier cosine and sine transform of f (x) = e−x . Put a = 1 in the above problems,
we get Fc [ f (x)] = 1
1+p2
and F s [ f (x)] = p
1+p2

Example 17.2.11. Find the Fourier sine transform of x


x2 +a2
and Fourier cosine transform of 1
a2 +x2
.

Solution. We have r
2 p
F s [e −ax
)] = .
π a2 + p2
The inverse Fourier sine transform of e−ax is
r Z ∞
2
f (x) = F s [ f (x)] sin pxd p
π 0

r Z ∞
2
f (x) = F s [e−ax ] sin pxd p
π 0
r Z ∞r
2 2 p
f (x) = sin pxd p
π 0 π a2 + p2
2 ∞ p sin px
Z
e−ax = dp
π 0 a2 + p2

π −ax
Z
p sin px
i.e., dp = e .
0 a2 + p2 2

Changing p to x and x to p, we get


π −ap
Z
x sin xp
dx = e .
0 a2 + x 2 2

Hence
h x i π −ap
Fs = e .
x 2 + a2 2

Transform Techniques B.Sc.(Mathematics)-IDE(UNOM)-II Year-IV Sem


17.2. Worked out Problems 393

Also we have r
2 a
Fc [e−ax )] = .
π a2 + p2
The inverse Fourier sine transform of e−ax is
r Z ∞
2
f (x) = Fc [ f (x)] cos pxd p
π 0
r Z ∞
2
f (x) = Fc [e−ax ] cos pxd p
π 0
r Z ∞r
2 2 1
f (x) = cos pxd p
π 0 π a2 + p2
2a ∞ cos px
Z
e−ax = dp
π 0 a2 + p2

π −ax
Z
cos px
i.e., dp = e .
0 a2 + p2 2a

Changing p to x and x to p, we get


π
Z
cos xp
dx = e−ap .
0 a +x
2 2 2a

Hence
h 1 i π
Fc = e−ap .
x +a
2 2 2a
Example 17.2.12. Find the Fourier sine transform of f (x) = xm−1 , 0 < m < 1.

Solution. r Z ∞
2
F s [ f (x)] = sin pxdx
π 0
r Z ∞
2
F s [xm−1 ] = xm−1 sin pxdx
π 0

B.Sc.(Mathematics)-IDE(UNOM)-II Year-IV Sem Transform Techniques


394 17.2. Worked out Problems

We know that
Z ∞ h e−st i∞
e−xt sin pxdx =(−t sin px − p cos px)
0 t 2 + p2 0
p
= 2
t + p2
Z ∞
1h i
e−xt sin pxdx = (t − ip)−1 − (t + ip)−1
0 2i

Differentiating both sides with respect to t, m − 1 times, we get


r Z ∞
2 1 h i
(−1)m−1 xm−1 e−xt sin pxdx = (−1)m−1 (m − 1)! (t − ip)−m − (t + ip)−m
π 0 2i

Put t = r cos θ and p = r sin θ we get


r Z ∞
2
(−1)m−1 xm−1 e−xt sin pxdx = (−1)m−1 (m − 1)!r−m sin mθ
π 0
1 
−1 p
 
= (−1)m−1 (m − 1)! sin m tan
(t2 + p2 ) 2
m
t

1
Since r = (t2 + p2 ) 2 and p
t
= tan θ.


π (m − 1)!
Z r
−1 p
  
∴ x m−1 −xt
e sin pxdx = sin m tan .
2 (t2 + p2 ) 2
m
0 t

Now taking t = 0, we get


π (m − 1)!
Z r
 mπ 
xm−1 sin pxdx = sin
0 2 pm 2

. Thus
π Γ(m)
r
 mπ 
F s [ f (x)] = sin
2 pm 2
.

Example 17.2.13. Find the Fourier sine and cosine transform of 2e−5x + 5e−2x .

Transform Techniques B.Sc.(Mathematics)-IDE(UNOM)-II Year-IV Sem


17.2. Worked out Problems 395

Solution. (i) We have r


2 p
F s [e
−ax
]= .
π a2 + p2

∴ F s [2e−5x + 5e−2x ] = F s [2e−5x ] + F s [5e−2x ]


r r
2 2p 2 5p
= +
π 25 + p 2 π 4 + p2
r
2 h 2p 5p i
∴ F s [2e−5x + 5e−2x ] = +
π 25 + p2 4 + p2

(i) We have r
2 a
Fc [e−ax ] = .
π a2 + p2

∴ Fc [2e−5x + 5e−2x ] = Fc [2e−5x ] + Fc [5e−2x ]


r r
2 10 2 10
= +
π 25 + p2 π 4 + p2
r
2h 1 1 i
∴ Fc [2e−5x + 5e−2x ] = 10 +
π 25 + p2 4 + p2

Example 17.2.14. Find the Fourier sine and cosine transform of (i) xn−1 , (ii) √1 ,
x
and (iii) x.

Solution. r Z ∞
2
F s [xn−1 ] = xn−1 sin pxdx
π 0
r Z ∞
2
Fc [x ] =
n−1
xn−1 cos pxdx
π 0

r Z ∞
2
Fc [x n−1
] + iF s [x n−1
] = xn−1 (cos px + i sin px)dx
π 0
r Z ∞
2
= xn−1 eipx dx
π 0
r Z ∞
2  −t n−1  dt 
= e−t − , Puttingipx = −t
π 0 ip ip

B.Sc.(Mathematics)-IDE(UNOM)-II Year-IV Sem Transform Techniques


396 17.2. Worked out Problems
r ∞
2 (−1)n
Z
Fc [x n−1
] + iF s [x n−1
] = e−t tn−1 dt
π (ip)n 0
r
2
2 (i) n
= Γ(n)
π in pn
r
2 in
= Γ(n)
π pn
r
2 1 nπ nπ 
= cos + i sin
π pn 2 2

Equating real and imaginary parts on both sides, we get


r r
2 Γ(n) nπ 2 Γ(n) nπ
Fc [xn−1 ] = cos and F s [xn−1 ] = sin
π p n 2 π p n 2

(ii) Put n = 1
2
in the above result, we have


2 Γ( 2 )
r 1
r
h 1 i π 2 π 1
Fc √ = cos = √ = √ .
x π p2 1
4 π p p


2 Γ( 2 )
r 1
r
h 1 i π 2 π 1
Fs √ = sin = √ = √ .
x π p2 1
4 π p p
(iii) Putting n = 2 in the above result
r r
2 Γ(2) 2 1
Fc [x] = cos π = − .
π p2 π p2

and r
2 Γ(2)
F s [x] = sin π = 0.
π p2
Example 17.2.15. Show that (i) F s [x f (x)] = − ddp [Fc (p)] and (ii) Fc [x f (x)] = d
dp
[F s (p)].

Solution. (i) We have r Z ∞


2
Fc (p) = f (x) cos pxdx
π 0

Transform Techniques B.Sc.(Mathematics)-IDE(UNOM)-II Year-IV Sem


17.2. Worked out Problems 397

Differentiating with respect to p, we get


r Z ∞ r Z ∞
d 2 2
[Fc (p)] = f (x)(−x sin px)dx = − [x f (x)] sin pxdx = −F s [x f (x)]
dp π 0 π 0

d
∴ F s [x f (x)] = − [Fc (p)].
dp
(ii) We have r Z ∞
2
F s (p) = f (x) sin pxdx
π 0
Differentiating with respect to p, we get
r Z ∞ r Z ∞
d 2 2
[F s (p)] = f (x)(x cos px)dx = [x f (x)] cos pxdx = −Fc [x f (x)]
dp π 0 π 0

d
∴ Fc [x f (x)] = [F s (p)].
dp
2 x2
Example 17.2.16. Find the Fourier cosine transform of e−a and hence evaluate Fourier sine
transform of xe−a x .
2 2

Solution. r Z ∞
−a2 x2 2 2 2
Fc [e ] = e−a x cos pxdx
π 0
r Z ∞
2 2 2
= R.P o f e−a x eipx dx
π
r Z0 ∞
2
e−a x +ipx dx
2 2
= R.P o f
π 0
r √
2 π − p22
= . e 4a
π 2a
2 2 1 − p2
Fc [e−a x ] = √ e 4a2 .
a 2

We know that
d h i
F s [x f (x)] = − Fc [ f (x)]
dp

B.Sc.(Mathematics)-IDE(UNOM)-II Year-IV Sem Transform Techniques


398 17.2. Worked out Problems

2 2 d h 2 2
∴ F s [xe−a x ] = − Fc [e−a x ]
dp
d h 1 − p22 i
= − √ e 4a
dp a 2
1 p2  p 
= − √ e− 4a2 − 2
a 2 2a
2 2 p −p
2
∴ F s [xe−a x ] = √ e 4a2 .
a3 2 2
R∞ x∼mx
Example 17.2.17. Find the Fourier sine transform of e−|x| and hence evaluate 0 1+x2
dx.

Solution. r Z ∞
2
F s [e−|x| ] = e−|x| sin pxdx
π 0
r Z ∞
2
= e−x sin pxdx
π 0
r
2 h e−x i∞
= (− sin px − p cos px)
π 1 + p2 0
r
2 p
= .
π 1 + p2

Taking the inverse sine transform,


r Z ∞
2
f (x) = F s [ f (x)] sin pxd p
π 0
r Z ∞r
2 2 p
e−x = sin pxd p
π 0 π 1 + p2
2 ∞ p
Z
e−x = sin pxd p
π 0 1 + p2

π −x
Z
p
sin pxd p = e
0 1 + p2 2
R∞ R∞
Changing x to m, 0
p
1+p2
sin pmd p = π2 e−m . Hence 0
p
1+p2
sin xmdx = π2 e−m by changing the
dummy variable p by x.

Transform Techniques B.Sc.(Mathematics)-IDE(UNOM)-II Year-IV Sem


17.2. Worked out Problems 399

Example 17.2.18. Show that the Fourier sine transform of


for, 0 < x < 1





 x,


f (x) = 

for, 1 < x < 2


2 − x,



for, x > 2


0,

q R
2 ∞
is π 0
2 sin p(1−cos p)
p2
.

Solution. By definition,
r Z ∞
2
F s [ f (x)] = f (x) sin pxdx
π 0

r Z ∞
2h 1
Z Z 2 i
F s [ f (x)] = f (x) sin pxdx + f (x) sin pxdx + f (x) sin pxdx
π
r Z0 1 Z 2
1 2

2 h i
= x sin pxdx + (2 − x) sin pxdx
π 0 1
r
2 hh  cos px   sin px i1 h  cos px   sin px i2 i
= x − − 1. − + (2 − x) − − (−1). −
π p p2 0 p p2 1
r
2 h cos p sin p sin 2p cos p sin p i
= − + 2 − + + 2
π p p p2 p p
r
2 h 2 sin p sin 2p i
= −
π p2 p2
r
2 h 2 sin p 2 sin p cos p i
= −
π p2 p2
r
2 sin p
F s [ f (x)] = 2 (1 − cos p).
π p2

B.Sc.(Mathematics)-IDE(UNOM)-II Year-IV Sem Transform Techniques


400 17.2. Worked out Problems

Example 17.2.19. Find the Fourier cosine transform of


for, 0 < x < 1





 x,


f (x) = 

for, 1 < x < 2


2 − x,



for, x > 2


0,

Solution. By definition,
r Z ∞
2
Fc [ f (x)] = f (x) cos pxdx
π
r Z0 1 Z 2 Z ∞
2h i
= f (x) cos pxdx + f (x) cos pxdx + f (x) cos pxdx
π
r Z0 1 Z 2
1 2

2 h i
= x cos pxdx + (2 − x) cos pxdx
π 0 1
r
2 hh  sin px   cos px i1 h  sin px   cos px i2 i
= x − 1. − + (2 − x) − (−1). −
π p p2 0 p p2 1
r
2 h sin p cos p 1 cos 2p sin p cos i
= + 2 − 2− − + 2
π p p p p2 p p
r
2 2 cos p cos 2p
h 1 i
= − − 2
π p2 p 2 p
r
2 h 2 cos p − cos 2p − 1 i
Fc [ f (x)] =
π p2

r
2 h 2 cos p − (2 cos2 p − 1) − 1 i
Fc [ f (x)] =
π p2
r
2 h 2 cos p − 2 cos2 p i
=
π p2
r
2 cos p
Fc [ f (x)] = 2 (1 − cos p).
π p2

Example 17.2.20. Find the Fourier cosine transform of e−ax cos ax, a > 0 and Fourier sine
transform of e−ax sin ax.

Transform Techniques B.Sc.(Mathematics)-IDE(UNOM)-II Year-IV Sem


17.2. Worked out Problems 401

Solution. (i)
r Z ∞
2
Fc [ f (x)] = f (x) cos pxdx
π 0
r Z ∞
2
= e−ax cos ax cos pxdx
π
r Z0 ∞
2 h cos(a + p)x + cos(a − p)x i
= e−ax dx
π 0 2
r Z ∞
2 1h a a i a
= . + 2 , since eax cos bxdx = 2
π 2 a + (p + a)
2 2 a + (p − a) 2
0 a + b2
r
2 a(p2 + 2a2 )
= .
π (p2 + 2ap + 2a2 )(p2 − 2ap + 2a2 )

(i)
r Z ∞
2
F s [ f (x)] = f (x) sin pxdx
π 0
r Z ∞
2
= e−ax sin ax sin pxdx
π
r Z0 ∞
2 h cos(a − p)x − cos(a + p)x i
= e−ax dx
π 0 2
r
2 1h a a i
= . −
π 2 a2 + (p − a)2 a2 + (p + a)2
1 4a2 p
= √ .
2π (p− 2ap + 2a2 )(p2 + 2ap + 2a2 )

Example 17.2.21. Find the Fourier sine and cosine transform of



k, if 0 < x < a




f (x) = 


0, if x > a


B.Sc.(Mathematics)-IDE(UNOM)-II Year-IV Sem Transform Techniques


402 17.2. Worked out Problems

Solution. r Z ∞
2
F s [ f (x)] = f (x) sin pxdx
π 0
r Z a Z ∞
2h i
= k sin pxdx + 0. sin pxdx
π 0 a
r
2  cos px a
= k −
π p 0
r
2
= k (− cos ap + 1)
π
r
2 1 − cos ap
F s [ f (x)] = k
π p

r Z ∞
2
Fc [ f (x)] = f (x) cos pxdx
π 0
r Z a Z ∞
2h i
= k cos pxdx + 0. cos pxdx
π 0 a
r
2 sin px a
 
= k
π p 0
r
2 sin ap − 0
= k
π p
r
2 sin ap
Fc [ f (x)] = k .
π p

Example 17.2.22. Find Fourier sine and cosine transform of xe−ax .

Solution. Fourier sine transform. We know that

d
F s [x f (x)] = − [Fc [ f (x)]]
dp
d
F s [xe−ax ] = − [Fc [e−ax ]]
dp
r
d h 2 a i
= −
dp π a2 + p2
r
2  1 
F s [x f (x)] = − a − 2 .2p
π (a + p )
2 2

Transform Techniques B.Sc.(Mathematics)-IDE(UNOM)-II Year-IV Sem


17.2. Worked out Problems 403
r
2 2ap
F s [xe −ax
] = . 2 .
π (a + p2 )2

Fourier cosine transform. We know that

d
Fc [x f (x)] = [F s [ f (x)]]
dp
d
Fc [xe−ax ] = [F s [e−ax ]]
dp
r
d h 2 p i
=
dp π a2 + p2
r
2  (a2 + p2 ).1 − p.2p 
=
π (a2 + p2 )2
r
2 a2 − p2
Fc [xe−ax ] = . .
π (a2 + p2 )2

Example 17.2.23. Find the inverse Fourier sine transform f (x) of f s (p) = pn e−ap .

Solution. From the inverse Fourier sine transform, we have


r Z ∞ r Z ∞
2 2
f (x) = f s (p) sin pxd p = pn e−ap sin pxd p
π 0 π 0

We know that Z ∞
x
e−ap sin pxdx = .
0 a2 + x2
Differentiating both sides with respect to a, n times, we get


dn  x 
Z
(−1) n
e−ap pn sin pxd p =
0 dan a2 + x2
dn h 1  1 1 i
= −
dan 2i a − ix a + ix
1h i
= (−1)n n!(a − ix)−n−1 − (−1)n n!(a + ix)−n−1
2i
(−1)n n! h i
= (a − ix)−n−1 − (a + ix)−n−1
2i
 
Put x = r sin θ and a = r cos θ. Then r2 = x2 + a2 , and θ = tan−1 x
a

B.Sc.(Mathematics)-IDE(UNOM)-II Year-IV Sem Transform Techniques


404 17.2. Worked out Problems

Now a − ix = r(cos θ − i sin θ).

∴ (a − ix)−(n+1) = r−(n+1) [cos(n + 1)θ + i sin(n + 1)θ]

Similarly,
(a + ix)−(n+1) = r−(n+1) [cos(n + 1)θ − i sin(n + 1)θ]

∴ (a − ix)−(n+1) − (a + ix)−(n+1) = 2ir−(n+1) sin(n + 1)θ.


(−1)n n!  −(n+1)
Z 
(−1)n
e−ap pn sin pxd p = 2ir sin(n + 1)θ
0 2i
= (−1)n n!r−(n+1) sin(n + 1)θ
Z ∞
1
(−1)n
e−ap pn sin pxd p = (−1)n n! n+1
sin(n + 1)θ
0 (r2 ) 2

n! sin(n + 1)θ
Z
∴ e−ap pn sin pxd p = .
(x2 + a2 )
n+1
0 2

Hence r
2 n! sin(n + 1)θ
f (x) = .
π (x2 + a2 ) n+1
2

Example 17.2.24. Find the inverse Fourier cosine transform f (x) of Fc [ f (x)] = pn e−ap .

Solution. From the inverse Fourier cosine transform, we have


r Z ∞ r Z ∞
2 2
f (x) = f c (p) cos pxd p = pn e−ap cos pxd p
π 0 π 0

We know that Z ∞
a
e−ap cos pxdx = .
0 a2 + x2

Transform Techniques B.Sc.(Mathematics)-IDE(UNOM)-II Year-IV Sem


17.2. Worked out Problems 405

Differentiating both sides with respect to a, n times, we get


dn  a 
Z
(−1) n
e−ap pn cos pxd p =
0 dan a2 + x2
dn h 1  1 1 i
= +
dan 2 a − ix a + ix
1h i
= (−1)n n!(a − ix)−n−1 + (−1)n n!(a + ix)−n−1
2

(−1)n n!
Z
(−1)n e−ap pn cos pxd p = cos(n + 1)θ Re f er Example 2.1.23
rn+1
Z0 ∞
n!
∴ e−ap pn cos pxd p = cos(n + 1)θ
rn+1
Z0 ∞
n!
i.e., e−ap pn cos pxd p = cos(n + 1)θ
(a2 + x2 ) 2
n+1
0

r
2 n!
∴ f (x) = cos(n + 1)θ.
π (a2 + x2 ) n+1
2

Example 17.2.25. Find the inverse Fourier cosine transform f (x) of



 
p
, when p < 2a
 1
a −



 2a 2
f c (p) = 



0,

 when p ≥ a

Solution. r Z ∞
2
f (x) = f (p) cos pxd p
π 0 c
r Z 2a Z ∞
2h 1 p i
= a− cos pxd p + 0. cos pxd p
π 0 2a 2 2a
r Z 2a 
2 1 p 
= . a− cos pxdxp
π 2a 0 2
1 h p  sin px   1  cos px i2a
= √ a− − − −
a 2π 2 x 2 x2 0

1 h 1 cos 2ax 1 i
= √ 0− +
a 2π 2 x2 2x2

B.Sc.(Mathematics)-IDE(UNOM)-II Year-IV Sem Transform Techniques


406 17.2. Worked out Problems

1 1 − cos 2ax
= √
a 2π 2x2
1 2 sin2 ax
= √
a 2π 2x2
1 sin2 ax
f (x) = √ .
a 2π x2

e−ap
Example 17.2.26. Find the inverse Fourier cosine transform f (x) of F s (p) = p
and hence
 
deduce F s−1 1p

Solution. From the inverse Fourier sine transform, we have


r Z ∞
2
f (x) = f (p) sin pxd p
π 0 s
r Z ∞
2 e−ap
f (x) = sin pxd p (17.2.4)
π 0 p
Differentiating with respect to x, we get
r Z ∞
df 2 e−ap
= (p cos px)d p
dx π 0 p

r Z ∞
df 2
= e−ap cos pxd p
dx π 0
r
2 h e−ap i∞
= (−a cos px + x sin px)
π a2 + x2 0
r
df 2 a
=
dx π a2 + x 2

Integrating,
r Z
12
f (x) = a dx
π
+ x2 a2
r
2 1 −1  x 
= a. tan +c
π a a

Transform Techniques B.Sc.(Mathematics)-IDE(UNOM)-II Year-IV Sem


17.2. Worked out Problems 407
r
2  x
f (x) = tan− +c (17.2.5)
π a
From (20.2.4), at x = 0, f (0) = 0

(20.2.6) ⇒ f (0) = 0 + c ⇒ c = 0.
r
2  x
f (x) = tan−1 .
π a
Deduction. Putting a = 0 in the above equation, we get
r
2 π π
r
1
f (x) = F s−1 = . = .
p π 2 2

Example 17.2.27. Find the inverse Fourier cosine transform of sin ap


p
.

Solution. From the inverse Fourier cosine transform, we have


r Z ∞
2
f (x) = f (p) cos pxd p
π 0 c
r Z ∞
2 sin ap
= cos pxd p
π 0 p
r
2 1 ∞ 2 sin ap
Z
= . cos pxd p
π 2 0 p
Z ∞
1 1
f (x) = √ [sin(a + x)p + sin(a − x)p]d p
2π 0 p


sin(a + x)p ∞
Z Z
1 h sin(a − x)p i
f (x) = √ dp + dp
2π 0 p 0 p

π π
 
 √2π . 2 + 2 , if a − x > 0


 1

f (x) = 

 √12π . π2 − π2 ,
  
if a − x > 0



R∞
Since 0
sin ax
x
dx = π2 , when a > 0

B.Sc.(Mathematics)-IDE(UNOM)-II Year-IV Sem Transform Techniques


408 17.3. Problems Related to Integral Equations (Fourier Transforms)

Hence 
 √2π , if x > a


 1

f (x) = 


if x < a

0,

Example 17.2.28. Find f (x) if its Fourier sine transform is e−ap

Solution. From the inverse Fourier sine transform, we have


r Z ∞
2
f (x) = f (p) sin pxd p
π 0 s
r Z ∞
2
= e−ap sin pxd p
π 0
r
2 h e−ap i∞
= (−a sin px − p cos px)
π a2 + p2 0
r
2 p
f (x) = . 2 .
π p + a2

17.3 Problems Related to Integral Equations (Fourier


Transforms)

17.4 Worked out Problems


R∞
Example 17.4.1. Solve the integral equation 0
f (x) cos(λx)dx = e−λ

Solution. Z ∞
Let f (x) cos(λx)dx = e−λ (17.4.1)
0

r Z ∞ r
2 2 −λ
∴ f (x) cos(λx)dx = e .
π 0 π
r
2 −λ
∴ f c (λ) = e .
π

Transform Techniques B.Sc.(Mathematics)-IDE(UNOM)-II Year-IV Sem


17.4. Worked out Problems 409

By Fourier cosine inversion formula, we have


r Z ∞
2
f (x) = f (λ) cos(λx)dλ
π 0 c
r Z ∞ r
2 2 −λ
= (λ) e cos(λx)dλ
π π
Z ∞0
2
= e−λ cos(λx)dx
π 0
2 h e−λ i∞
= (− cos λx + x sin λx)
π 1 + x2 0
2 1
f (x) = . .
π 1 + x2
R∞ cos λx
Example 17.4.2. Show that 0 1+λ2
dλ = π2 e−x , x > 0.

Solution. We know that


Z ∞
1
e−x cos λxdx =
1 + λ2
r Z0 ∞ r
2 2 1
e−x cos λxdx =
π 0 π 1 + λ2
r
2 1
⇒ Fc [e−x ] =
π 1 + λ2

By Fourier cosine inversion formula, we have


r Z ∞r
2 2 1 
e−x = cos λxdλ
π 0 π 1 + λ2
2 ∞ cos λx
Z
= dλ
π 0 1 + λ2

cos λx π
Z
dλ = e−x .
0 1+λ 2 2

B.Sc.(Mathematics)-IDE(UNOM)-II Year-IV Sem Transform Techniques


410 17.4. Worked out Problems

Example 17.4.3. Solve for F(x) the integral equation






 1, 0≤t≤1
Z ∞ 


F(x) sin xtdx = 


 2, 1≤t≤2
0 





0,
 t≥2

Solution. Let Z in f ty
F(x) sin pxdx = f (p).
0







1, 0≤t≤1


∴ f (t) = 



2, 1≤t≤2





0,
 t≥2

r Z ∞ r
2 2
F(x) sin xtdx = f (t) = F s (t).
π 0 π
By Fourier sine inversion formula, we have
r Z ∞
2
f (x) = F s (p) sin xtdt
π 0
r Z ∞r
2 2
= f (t) sin xtdt
π 0 π
2 ∞
Z
= f (t) sin xtdt
π 0
Z ∞
2h 1
Z Z 2 i
= 1. sin xtdt + 2. sin xtdt + 0. sin xtdt
π 0 1 2
2 h  cos xt 1  cos xt 2 i
= − −2
π x 0 x 1
2 h cos x 1 2 cos 2x 2 cos x i
= − + − +
π x x x x
2  1 + cos x − 2 cos 2x 
= .
π x

Transform Techniques B.Sc.(Mathematics)-IDE(UNOM)-II Year-IV Sem


17.5. Exercises 411

17.5 Exercises

1. Find Fourier sine transform of f (x) = 1x .


2. Find the Fourier cosine transform of the function f (x), if
 
0<x<1
 
cos x, sin x, 0≤x≤a


 


(i) f (x) =  (ii) f (x) = 
 

 

0,

 x≥1 0,

 x≥a

3. Find the Fourier sine and cosine transform of the function f (x) = e−2x + 4e−3x .
eax +e−ax
4. Find the Fourier sine and cosine transform of the function f (x) = eπx −e−πx
.
5. Find the sine transform of f (x) defined by f (x) = 1
eπx −e−πx
and hence deduce that F s (csc hπx) =
 p
1
2
tanh 2
.
6. Find the cosine transform of a function of x which is unity for 0 < x < a and zero for x > a.
q
What is the function whose cosine transform is 2 sin ap
π p
.

Answers.
1. π2 .
p
h i h cos(1+p) cos(p−1) i
2. (i) √12π sin(1+p)
1+p
+ sin(1−p)
1−p
; (ii) √1

− 1+p + p−1 + p+1 1
− 1
p−1
.
q   q  
3. 2
π p2 +4
p
+ p4p2 +9 ;
2 2
− 12 .
q π pp +4− p p +9  
2 2

p −p e 2 +e 2
4. √12π e p +ee−p−e+2 cos a ; 2
π 2 cos a+e p +e−p
cos a2 .
q
2 e p −1
5. .
π 4(e p +1)
.
q
6. π
. p ; f (x) = 1 when x < a and f (x) = 0, when x > a.
2 sin ap

B.Sc.(Mathematics)-IDE(UNOM)-II Year-IV Sem Transform Techniques


412
UNIT-IV

Lesson 18

CONVOLUTION
Learning Objectives
Upon completion of this lesson, students will be able to

• use convolution theorem and Parseval’s identity in Fourier transform

• solve problems using Parseval’s identity

• solve problems related to finite Fourier transform.

18.1 Introduction

I
n this Lesson, we discuss convolution concept and related results with illustrated worked out
examples.
R∞
Definition 18.1.1. The function H(x) = F ∗ G = √1
2π −∞
F(u)G(x − u)du is defined as the
Convolution or Falting of two integrable functions F and G over the interval (−∞, ∞).

Theorem 18.1.1. If F [ f (x)] and G [ f (x)] are the Fourier transforms of the functions f (x) and
g(x) respectively, then the Fourier transform of the convolution of f (x) and g(x) is the product
of their Fourier transform.

i.e., F [ f (x) ∗ g(x)] = F [ f (x)].F [g(x)].

B.Sc.(Mathematics)-IDE(UNOM)-II Year-IV Sem 413 Transform Techniques


414 18.2. Parseval’s Identity for Fourier Transforms

Proof. We have

h 1 Z ∞ i
F [ f (x) ∗ g(x)] = F √ f (u)g(x − u)du
2π −∞

Z ∞h Z ∞
1 1 i
F [ f (x) ∗ g(x)] = √ √ f (u)g(x − u)du eipx dx

Z ∞−∞ h 2π Z ∞ −∞
1 i
= f (u) g(x − u)eipx dx du by changing the order of integration
2π −∞ −∞
Z ∞
1
= f (u)F [g(x − u)]du
2π −∞
Z ∞
1
= f (u)eiup F [g(x)]du, by using shifting theorem
2π −∞
Z ∞
1
= F [g(x)]. f (u)eiup du
2π −∞
Z ∞
1
= F [g(x)]. f (x)eixp dx, by changing the dummy variable u by x
2π −∞
= F [g(x)].F [ f (x)]

F [ f (x) ∗ g(x)] = F [ f (x)].F [g(x)].

18.2 Parseval’s Identity for Fourier Transforms

Theorem 18.2.1. Parseval’s Theorem or Rayleigh’s Theorem. If f (p) and g(p) are Fourier
transform of f (x) and g(x) respectively. Then

R∞ R∞
(i) −∞
f (p)g(p)d p = −∞
f (x)g(x)dx

R∞ R∞
(ii) −∞
| f (x)|2 dx = −∞
| f (p)|2 d p. where bar denotes the complex conjugate.

Transform Techniques B.Sc.(Mathematics)-IDE(UNOM)-II Year-IV Sem


18.2. Parseval’s Identity for Fourier Transforms 415

Proof. By the inversion formula for Fourier transform, we have


Z ∞
1
g(x) = √ g(p)e−ipx d p
2π −∞

Taking conjugate complex on both sides, we get


Z ∞
1
g(x) = √ g(p)eipx d p
Z ∞ Z 2π

−∞
h 1 Z ∞ i
∴ f (x)g(x)dx = f (x) √ g(p)eipx d p dx
−∞ −∞ 2π −∞

Z ∞ Z ∞ h 1 Z ∞
1 i
f (x)g(x)dx = √ g(p) √ f (x)eipx dx d p
−∞ 2π −∞ 2π −∞

by changing the order of integration.


Z ∞ Z ∞
f (x)g(x)dx = g(p) f (p)d p, by definition Fourier transform
−∞ −∞

(ii) Putting g(x) = f (x) in the above equation, we get


Z ∞ Z ∞
f (p) f (p)d p = f (x) f (x)dx
−∞ −∞

or Z ∞ Z ∞
| f (p)| d p =
2
| f (x)|2 dx.
−∞ −∞

Note.1 Some authors define the theorem as follows:


Z ∞ Z ∞
1
| f (x)| dx =
2
| f (p)|2 d p.
−∞ 2π −∞

2. Similarly, we can prove the following Parseval’s identities for Fourier sine and cosine

B.Sc.(Mathematics)-IDE(UNOM)-II Year-IV Sem Transform Techniques


416 18.3. Worked out Problems

transforms
R ∞ as above. R∞ R∞ R∞
(i) 0 f c (p)gc (p)d p = 0 f (x)g(x)dx; (ii) 0
f s (p)g s (p)d p = 0
f (x)g(x)dx;
R∞ R∞ R∞ R∞
(iii) 0 | f c (p)|2 d p = 0 | f (x)|2 dx; (iv) 0 | f s (p)|2 d p = 0 | f (x)|2 dx;

18.3 Worked out Problems

Example 18.3.1. Find the Fourier transform of f (x) defined by



|x| < a

1,



f (x) = 


|x| > a

0,

R∞ sin2 ax πa
and hence prove that 0 x2
dx = 2
.

Solution. r
2 sin ap
F [ f (x)] = f (p) = . Refer to Example 1.9.1.
π p
Using Parseval’s identity for Fourier transform, we get
Z ∞ Z ∞
| f (x)| dx = 2
| f (p)|2 d p
−∞ −∞
Z ∞
2 sin2 ap
Z a
(1)2 dx = dp
−a −∞ π p2
2 ∞ sin2 ap
Z
2a = dp
π −∞ p2
4 ∞ sin2 ap
Z
2a = dp
π 0 p2
πa
Z ∞ 2
sin ap
2
dp = .
0 p 2

Example 18.3.2. Find the Fourier transform of f (x) defined by



1, |x| < 1




f (x) = 


0, |x| > 1


Transform Techniques B.Sc.(Mathematics)-IDE(UNOM)-II Year-IV Sem


18.3. Worked out Problems 417
R∞
and f (p) = 2
p
sin p, using Parseval’s identity, prove that 0
sin2 t
t2
dt = π2 .

Solution. Using Parseval’s identity for Fourier transform, we get


Z ∞ Z ∞
| f (x)| dx =
2
| f (p)|2 d p
−∞ −∞
Z 1 Z ∞
4
(1)2 dx = 2
sin2 pd p
−1 −∞ p
Z ∞ 2
sin p
2 = 4.2 dp
0 p2
Z ∞ 2
sin p 1
2
dp =
0 p 4
Z ∞ 2
sin t 1
or dt = .
0 t2 4

Example 18.3.3. Find the Fourier transform of



1 − |x|, for |x| < 1




f (x) = 


for |x| > 1

0,

R ∞  sin x 4
Hence deduce that 0 x
dt = π3 .

Solution. By the definition of Fourier transform, we have


Z ∞
1
F [ f (x)] = √ f (x)eipx dx
2π −∞
1 h −1
Z Z 1 Z ∞ i
= √ f (x)e dx +
ipx
f (x)e dx +
ipx
f (x)eipx dx
2π −∞ −1 1
Z 1
1
= √ (1 − |x|).eipx dx
2π −1
1 h 1
Z Z 1 i
= √ (1 − |x|) cos pxdx + i (1 − |x|) sin pxdx
2π −1 −1

B.Sc.(Mathematics)-IDE(UNOM)-II Year-IV Sem Transform Techniques


418 18.3. Worked out Problems
Z 1
2
= √ (1 − |x|) cos pxdx
2π 0
Z 1
2
= √ (1 − x) cos pxdx since 1 − |x| = 1 − x in (0, a)
2π 0
2 h  sin px   − cos px ia
= √ (1 − x) − (−1)
2π p p2 0

2 h cos p  1 i
= √ 0− 2 − 0− 2
2π p p
2 1 − cos p
h i
= √
2π p2
r
2 h 1 − cos p i
F [ f (x)] = f (p) = .
π p2

Using Parseval’s identity for Fourier transform, we get


Z ∞ Z ∞
| f (x)| dx =
2
| f (p)|2 d p
−∞ −∞
1 ∞
2 (1 − cos p)2
Z Z
(1 − |x|)2 dx = dp
−1 −∞ π p4
1
4 ∞ 1
Z Z
2 p
 2
2 (1 − x)2 dx = 2 sin dp
0 π 0 p4 2
h (1 − x)3 i1 16 ∞ 1
Z
4 p
 
2 = sin dp
−3 0 π 0 p4 2
16 ∞ 1
Z
2 4 p
 
= sin dp
3 π 0 p4 2

π
Z
1 4 p
 
sin dp =
0 p4 2 24

Put p
2
= x. Then d p = 2dx


π
Z
1
4
sin4 x2dx =
0 (2x) 24
π
Z ∞ 4
1 sin x
dx =
8 0 x4 24
π
Z ∞ 4
sin x
4
dx = .
0 x 3

Transform Techniques B.Sc.(Mathematics)-IDE(UNOM)-II Year-IV Sem


18.3. Worked out Problems 419

Example 18.3.4. Evaluate the following using Parseval’s identity


R ∞ x2 R∞ 1
(i) 0 (x2 +a2 )2 dx, (a > 0) (ii) 0 (x2 +a2 )2
dx, (a > 0).

Solution. Let f (x) = e−ax , x > 0. Then


q q
F s [ f (x)] = F(p) = π2 p2 +a
p
2 and F c [ f (x)] = F(p) = 2 a
π p2 +a2

(i) Using Parseval’s identity for Fourier transform, we get


Z ∞ Z ∞
| f (x)| dx =
2
|F s (p)|2 d p
Z0 ∞ Z0 ∞
2 p2
|e | dx =
−ax 2
dp
0 π (p + a )
2 2 2
0

2 ∞ p2
Z Z
e−2ax dx = dp
0 π 0 (p2 + a2 )2
 e−2ax ∞ 2 ∞ p2
Z
= dp
−2a 0 π 0 (p2 + a2 )2

p2 π
Z
dp = .
0 (p2 + a2 )2 4a

(ii) Using Parseval’s identity for Fourier transform, we get


Z ∞ Z ∞
| f (x)| dx =
2
|Fc (p)|2 d p
Z0 ∞ Z0 ∞
2 a2
|e−ax |2 dx = dp
π (p2 + a2 )2
Z0 ∞
0
Z ∞
2a2 1
e−2ax dx = dp
0 π 0 (p + a )
2 2 2

 e−2ax ∞ Z ∞
2a2 1
= dp
−2a 0 π 0 (p + a )
2 2 2
Z ∞
1 2a2 1
= dp
2a π 0 (p + a )
2 2 2

π
Z
1
dp = .
0 (p + a )
2 2 2 4a3

B.Sc.(Mathematics)-IDE(UNOM)-II Year-IV Sem Transform Techniques


420 18.3. Worked out Problems

Example 18.3.5. Show that the Fourier transform of



1 − x , |x| < 1


 2

f (x) = 


|x| > 1

0,

R ∞  sin x−x cos x 2 π


is √4

sin p−p cos p
p3
. Using Parseval’s identity prove that 0 x3
dx = 15
.

Solution.
4 sin p − p cos p
F [ f (x)] = f (p) = √ , Refer to Example 1.9.2.
2π p3
Using Parseval’s identity for Fourier transform, we get
Z ∞ Z ∞
| f (x)| dx =
2
| f (p)|2 d p
−∞ −∞
Z 1 Z ∞
16  sin p − p cos p 2
(1 − x2 )2 dx = dp
−1 −∞ 2π p3
1
8 ∞  sin p − p cos p 2
Z Z
2 (1 + x4 − 2x2 )dx = dp
0 π −∞ p3
x5 x 3 i1 16 ∞  sin p − p cos p 2
h Z
2 x+ −2 = dp
5 3 0 π 0 p3
16 ∞  sin p − p cos p 2
Z
16
= dp
15 π 0 p3
π
Z ∞
sin p − p cos p  2
dp =
p3 15
Z0 ∞ 
sin x − x cos x 2 π
i.e., dx = .
0 x3 15
R∞ π
Example 18.3.6. Using Parseval’s identity, show that 0 dx
(x2 +a2 )(x2 +b2 )
= 2ab(a+b)
.
R∞ π
(or) Evaluate 0 (x2 +a2dx
)(x2 +b2 )
= 2ab(a+b) using transform.

Solution. Let f (x) = e−ax and g(x) = e−bx . Then


r r
2 a 2 b
f c (p) = gc (p) =
π a2 + p2 π b2 + p2

Transform Techniques B.Sc.(Mathematics)-IDE(UNOM)-II Year-IV Sem


18.4. Exercises 421

Using Parseval’s identity for Fourier cosine transforms, we have


Z ∞ Z ∞
Fc (p)Gc (p)d p = f (x)g(x)dx
0 0
Z ∞r r Z ∞
2 a 2 b
. dp = e−ax e−bx dx
0 π a 2 + p2 π b 2 + p2
Z0 ∞
2ab ∞
Z
dp
dp = e−(a+b)x dx
π 0 (a2 + p2 )(b2 + p2 ) 0
h e−(a+b)x i∞
=
−(a + b) 0
Z ∞
2ab dp 1
dp =
π 0 (a + p )(b + p )
2 2 2 2 a+b
π
Z ∞
dp
dp = .
0 (a + p )(b + p ) 2ab(a + b)
2 2 2 2

18.4 Exercises

Using Parseval’s identity, show that


R∞ 2 π
R∞ π
(1) 0 (x2 +a2x)(x2 +b2 ) = 2(a+b) . (2) 0 =
sin ax 2
x(x2 +a2 ) 2a2
(1 − e−a ).

18.5 Worked out Problems Related to Finite Fourier


Transform

Example 18.5.1. Find the finite Fourier sine and cosine transform of f (x) = 1 in 0 < x < π.

Solution. (i)
Z l  pπx 
F s [ f (x)] = f (x) sin
dx
l
Z0 π  pπx 
f s (p) = 1. sin dx
0 π

B.Sc.(Mathematics)-IDE(UNOM)-II Year-IV Sem Transform Techniques


422 18.5. Worked out Problems Related to Finite Fourier Transform
Z π  cos px π cos pπ 1
= sin pxdx = − =− +
0 p 0 p p
1 − cos pπ
F s [ f (x)] = .
p

(ii)
Z l  pπx 
Fc [ f (x)] = f (x) cos
dx
0 l
Z π  pπx 
f c (p) = 1. cos dx
0 π
Z π
= cos pxdx
0
 sin px π sin pπ
= =
p 0 p
Fc [ f (x)] = 0

Example 18.5.2. Find the finite Fourier sine and cosine transform of f (x), defined by f (x) = x,
where 0 < x < 4.

Solution. (i)
Z l  pπx 
F s [ f (x)] = f (x) sin dx
0 l
Z 4  pπx 
f s (p) = x. sin dx
0 4
h  cos pπx   sin pπx i4
= x − pπ 4
− 1. − p2 π24
0
4 16
16 16
= − cos pπ + 2 2 sin pπ
pπ pπ
16
F s [ f (x)] = − cos pπ

Transform Techniques B.Sc.(Mathematics)-IDE(UNOM)-II Year-IV Sem


18.5. Worked out Problems Related to Finite Fourier Transform 423

(ii)
Z l  pπx 
Fc [ f (x)] = f (x) cos
dx
0 l
Z 4  pπx 
f c (p) = x. cos dx
0 4
h  sin pπx   cos pπx i4
Fc [ f (x)] = x pπ 4
− 1. − p2 π24
0
4 16

16 16 16
Fc [ f (x)] = sin pπ + 2 2 cos pπ − 2 2
pπ pπ pπ
16
Fc [ f (x)] = 2 2 [(−1) p − 1], whenp > 0

and if p = 0, Z 4
Fc [ f (x)] = xdx = 8.
0

Example 18.5.3. Find the finite Fourier sine and cosine transforms of f (x), defined by f (x) = 2x,
where 0 < x < 2π

Solution. (i)
Z l  pπx 
F s [ f (x)] = f (x) sin
dx
0 l
Z 2π  px 
f s (p) = 2x. sin dx
0 2
h  cos px   sin px i2π
= 2x − p 2
− 1. − p2 2
0
2 4
h 4π 4 i
= 2 − cos pπ + 2 sin pπ
p p

= (−1) p+1 , if p = 1, 2, 3, ...
p

B.Sc.(Mathematics)-IDE(UNOM)-II Year-IV Sem Transform Techniques


424 18.5. Worked out Problems Related to Finite Fourier Transform

(i)
Z l  pπx 
Fc [ f (x)] = f (x) cos
dx
0 l
Z 2π  px 
f c (p) = 2x. cos dx
0 2
h  sin px   cos px i2π
= 2x p 2
− 1. − p2 2
0
2 4
h 4π 4  4i
= 2 sin pπ + cos pπ − 0 +
p p2 p2
8
= 2 (cos pπ − 1)
p
8
= 2 ((−1)n − 1), if p = 1, 2, 3, ...
p

Example 18.5.4. Find the finite sine Fourier sine transform of f (x) = x3 in (0, π)

Solution. Z l  pπx 
F s [ f (x)] = f (x) sin dx
l
Z0 π
f s (p) = x3 . sin pxdx
0
h  cos px   sin px   cos px   sin px iπ
= x3 − − 3x2 − + 6x − 6
p p2 p3 p4 0

π 3
3π 2
6π 6
= − cos pπ + 2 sin pπ + 3 cos pπ − 4 sin pπ
p p p p
 π3 6π 
F s [ f (x)] = − + 3 (−1) p , i f p = 1, 2, 3, ..
p p

 2
Example 18.5.5. (i) Find the finite Fourier sine transform of f (x) defined by f (x) = 1 − πx ,
where 0 < x < π.
(ii) Show that the finite sine transform of 1 − x
π
is 1p .

Solution. (i)
Z l  pπx 
F s [ f (x)] = f (x) sin dx
0 l

Transform Techniques B.Sc.(Mathematics)-IDE(UNOM)-II Year-IV Sem


18.5. Worked out Problems Related to Finite Fourier Transform 425
Z π
 x 2
f s (p) = 1− . sin pxdx l = π
0 π
h x 2  cos px   x  1  sin px  2  cos px iπ
= 1− − −2 1− − − + 2
π p π π p2 π p3 0

2 cos pπ  1 2 1 
= − − + 2. 3
π p
2 3 p π p
2 1
= (cos π − 1) +

3 2 p
2 1
F s [ f (x)] = ((−1) n
− 1) +
p3 π2 p

Z l  pπx 
F s [ f (x)] = f (x) sin dx
l
Z0 π 
x
f s (p) = 1 − . sin pxdx l = π
0 π
h x  cos px   1  sin px iπ
= 1− − − − −
π p π p2 0
h 1 i
F s [ f (x)] = 0 − − − 0
p

1
F s [ f (x)] = .
p

Example 18.5.6. Find the finite Fourier sine and cosine transforms of f (x) = sin ax in (0, π) .

Solution. Finite Fourier Sine Transform.


Z l  pπx 
F s [ f (x)] = f (x) sin dx
l
Z0 π
f s (p) = sin ax. sin pxdx, l = π
Z0 π h
cos(a − p)x − cos(a + p)x i
= dx
0 2

B.Sc.(Mathematics)-IDE(UNOM)-II Year-IV Sem Transform Techniques


426 18.5. Worked out Problems Related to Finite Fourier Transform

1 h sin(a − p)x cos(a + p)x iπ


= −
2 a− p a+ p 0
Z πh
1 sin(a − p)π cos(a + p)π i
= −
2 0 a− p a+ p
1
= (0 − 0), if a , b and a, p are integers
2

If a = p, then
Z π
F s [ f (x)] = sin2 xdx, since a = p
0
1 π
Z
= (1 − cos 2px)dx
2 0
1h sin 2px π
= x−
2 2p 0
1
= (π − 0).
2


0, if a , n and a, p are integers



∴ F s [sin ax] = 

 π2 , if a = p



Finite Fourier cosine Transform.


Z l  pπx 
Fc [ f (x)] = f (x) cos dx
l
Z0 π
f c (p) = sin ax. cos pxdx, l = π
0
Z π h sin(a + p)x + sin(a − p)x i
f c (p) = dx
0 2

1 h − cos(a + p)x − cos(a − p)x iπ


f c (p) = +
2 a+ p a− p 0

1 cos(a + p)π cos(a − p)π


h  1 1 i
= − + − +
2 a+ p a− p a+ p a− p
1 h (−1)a+p (−1)a−p 1 1 i
f c (p) = − + − −
2 a+ p a− p a+ p a− p

Transform Techniques B.Sc.(Mathematics)-IDE(UNOM)-II Year-IV Sem


18.5. Worked out Problems Related to Finite Fourier Transform 427

− 2 (0) = 0, if a + p is even


 1

Fc [sin ax] = 

  
2
 12 a+p + a−p
2
= 2a
, if a + p is odd



a2 −p2

if a + p is even

0,



∴ Fc [sin ax] = 


 a22a , if a + p is odd



−p2

π x2
Example 18.5.7. Find the Fourier cosine transform of f (x) defined by f (x) = 3
−x+ 2π
, where
0 < x < π.

Solution. Z l  pπx 
Fc [ f (x)] = f (x) cos dx
0 l
Z π
π x2 
f c (p) = −x+ cos pxdx, l = π
0 3 2π
h π x2  sin px   x  cos px  1  sin px iπ
= −x+ − −1+ − + −
3 2π p π p2 π p3 0
 1 
= 0+0−0− − 2 +0
p
1
Fc [ f (x)] = , p > 0.
p2

If p = 0,
π π x2 
Z
Fc [ f (x)] = −x+ dx = 0.
0 3 2π
Example 18.5.8. Find the finite Fourier sine transform of f (x), defined by

π

 x, 0≤x≤


 2
f (x) = 

π

π − x, ≤x≤π



2

Solution. Z l  pπx 
F s [ f (x)] = f (x) sin dx
l
Z0 π
f s (p) = f (x) sin pxdx, l = π
0

B.Sc.(Mathematics)-IDE(UNOM)-II Year-IV Sem Transform Techniques


428 18.5. Worked out Problems Related to Finite Fourier Transform
π
Z 2
Z π
f s (p) = x sin pxdx + (π − x) sin pxdx
π
0 2
h  cos px   sin px i π2 h  cos px   sin px iπ
= x − −1 − + (π − x) − − (−1) − π
p p2 0 p p2 2

π pπ 1 pπ π pπ 1 pπ
= − cos + 2 sin + cos + 2 sin
2p 2 p 2 2p 2 p 2
2 pπ
F s [ f (x)] = 2 sin .
p 2

Example 18.5.9. Find the finite Fourier sine and cosine transform of (i) f (x) = x2 , 0 < x < l
(ii) 
π
0<x<

1,


 2
f (x) = 

π

<x<π

−1,


2

Solution. (i) Finite Fourier sine transform


Z l  pπx 
F s [ f (x)] = f (x) sin dx
0 l
Z l  pπx 
= x2 sin dx
0 l
h  cos pπx   sin pπx   cos pπx il
= x2 − pπ l − 2x − p2 π2l + 2 p3 π3 l
0
l l2 l3
l3 2l3 h l3 i
cos pπ + 0 + 3 3 cos pπ − 0 + 0 + 2 3 3
= −
pπ pπ pπ
3 3
l 2l
F s [ f (x)] = − (−1) p + 3 3 (1 − (−1) p ), p , 0.
pπ pπ

Finite Fourier cosine transform


Z l  pπx 
Fc [ f (x)] = f (x) cos dx
0 l
Z l  pπx 
= x2 cos dx
0 l
h  sin pπx   cos pπx   sin pπx il
= x 2

l
− 2x − p2 π2
l
+ 2 − p3 π3
l
0
l l2 l3

Transform Techniques B.Sc.(Mathematics)-IDE(UNOM)-II Year-IV Sem


18.5. Worked out Problems Related to Finite Fourier Transform 429

2l3
=
cos pπ
p2 π2
2l3
Fc [ f (x)] = 2 2 (−1) p , p , 0.

(ii) Finite Fourier sine transform


Z l  pπx 
F s [ f (x)] = f (x) sin
dx
0 l
Z π2 Z π
= (1) sin pxdx + (−1) sin pxdx
π
0 2
 cos px π π
cos px 
= +
2
− π
p 0 p 2

1 pπ  1 pπ 
= − cos −1 + cos pπ − cos
p 2 p 2
1  pπ 
F s [ f (x)] = − 2 cos − cos pπ − 1 , p , 0.
p 2

Finite Fourier cosine transform


Z l pπx 
Fc [ f (x)] = f (x) cos
dx
0 l
Z π2 Z π
= (1) cos pxdx + (−1) cos pxdx
π
0 2
 sin px  π2  sin px π
= − π
p 0 p 2

2 pπ
Fc [ f (x)] = sin .
p 2

(−1) p+1
Example 18.5.10. Show that the finite Fourier sine transform of x
π
is p
, 0<x<π

Solution. Z l  pπx 
F s [ f (x)] = f (x) sin dx
l
Z0 π
x
= sin pxdx
0 π

B.Sc.(Mathematics)-IDE(UNOM)-II Year-IV Sem Transform Techniques


430 18.5. Worked out Problems Related to Finite Fourier Transform

1 h  cos px   sin px iπ


=x − − 1. −
π p p2 0

1 h π i
= − cos pπ
π p
p+1
(−1)
F s [ f (x)] = .
p

Example 18.5.11. Find f (x) if its finite Fourier cosine transform is



sin pπ
 2p , if p = 1, 2, 3, ....


 2

(i) f c (p) = 

 π4 ,

if p = 0


and given 0 < x < 2π 


cos 2pπ
 (2p+1)2 , if p = 1, 2, 3, ....


 3

(ii) f c (p) = 


if p = 0

1,

and given 0 < x < l.

π
Solution. (i) Here f c (0) = 4
and l = 2π. From the inverse finite Fourier sine transform, we have


1 2X pπx
f (x) = f c (0) + f c (p) cos
l l p=1 l

1 π 2 X 1 pπ pπx
= . + sin cos
2π 4 2π p=1 2p 2 2π

1 1 X1 pπ pπx
f (x) = + + sin cos .
8 2π p=1 p 2 2π

(ii) Here f c (0) = 1 and l = 1.


1 2X 1 2pπ
f (x) = + cos cos pπx
l l p=1 (2p + 1) 2 3

X 1 2pπ
f (x) = 1 + 2 cos cos pπx.
p=1
(2p + 1) 2 3

Transform Techniques B.Sc.(Mathematics)-IDE(UNOM)-II Year-IV Sem


18.5. Worked out Problems Related to Finite Fourier Transform 431

Example 18.5.12. Find the finite Fourier cosine transform of f (x) = x2



− π6 , 0 ≤ x ≤ π.

Solution.
π
π  x2
Z
Fc [ f (x)] = cos pxdx −
0 2π 6
h x2 π  sin px  2x  − cos px  2  sin px iπ
= − ( )− −0 + −
2π 6 p 2π p2 2π p3 0
cos pπ
=
p2
(−1) p
Fc [ f (x)] = , p , 0.
p2

When p = 0,
π  x2 π
Z
Fc [ f (x)] = − dx = 0.
0 2π 6

Example 18.5.13. Find the inverse finite sine transform f (x) if f s (p) = 1−cos pπ
p2 π2
, where 0 < x <
π.

Solution. From the inverse Fourier sine transform, we have


2X pπx
f (x) = f s (p) sin
l p=1 l

2 X 1 − cos pπ
= sin px
π p=1 p2 π2

2 X 1 − cos pπ
= 3 sin px.
π p=1 p2

 2pπ 
Example 18.5.14. Find the inverse finite cosine transform f (x) if f c (p) = 1
(2p+1)2
cos 3
, where
0 < x < 4.

Solution. From the inverse finite cosine transform, we have


1 2X pπx
f (x) = f c (0) + f c (p) cos
l l p=1 l

B.Sc.(Mathematics)-IDE(UNOM)-II Year-IV Sem Transform Techniques


432 18.5. Worked out Problems Related to Finite Fourier Transform


1 2 2X 1  2pπ  pπx
= .1 + + cos cos
4 4 l p=1 (2p + 1) 2 3 4

1 1 2X 1  2pπ  pπx
f (x) = + + cos cos
4 2 l p=1 (2p + 1) 2 3 4

Example 18.5.15. Find the finite Fourier sine and cosine transforms of f (x) = eax , in (0, l).

Solution. Finite Fourier sine transform of a function f (x) is given by


Z l
pπx
F s [ f (x)] = f (x) sin
dx
0 l
Z l
pπx
= eax sin dx
0 l
h eax  pπx pπ pπx il
= a sin − cos
a2 + p 2π
2 2
l l l 0
l
eal  pπ  1 pπ
F s [ f (x)] = 0 − (−1) p
− .
a2 + p l2π a2 + p l2π l
2 2 2 2
l

1 pπ al
F s [ f (x)] = p2 π2
[e (−1) p+1 + 1]
a2 + l 2 l
pπl
= 22 [(−1) p+1 eal + 1].
a l + p2 π2

Finite Fourier cosine transform of a function f (x) is given by


Z l
pπx
Fc [ f (x)] = f (x) cos
dx
0 l
Z l
pπx
= eax cos dx
0 l
h eax  pπx pπ pπx il
= a cos + sin
a2 + p 2π
2 2
l l l 0
l

Transform Techniques B.Sc.(Mathematics)-IDE(UNOM)-II Year-IV Sem


18.5. Worked out Problems Related to Finite Fourier Transform 433

eal   1
= p2 π2
a cos pπ + 0 − p2 π2
(a + 0)
a2 + l2
a2 + l2
a
= p2 π2
[eal (−1) p − 1]
a2 + l2
2
al
= [(−1) p eal − 1].
a2 l2 + p2 π2

 2
Example 18.5.16. Find the finite Fourier cosine transform of f (x) = 1 − πx in (0, π).

Solution. Z l  pπx 
Fc [ f (x)] = f (x) cos dx
0 l
Z π
 x 2
f c (p) = 1− . cos pxdx l = π
0 π
h x 2  sin px   x  1  cos px  2  sin px iπ
= 1− −2 1− − − + 2 −
π p π π p2 π p3 0

2
Fc [ f (x)] = , if p > 0
πp2

When p = 0,
Z π
f c (0) = f (x)dx
0
Z π  x 2
= 1− dx
0 π
π
f c (0) = .
3

16(−1) p−1
Example 18.5.17. Find the inverse finite Fourier sine transform f (x) if f s (p) = p3
, where
p is a positive integer and 0 < x < 8.

Solution. The inverse Fourier sine transform is given by


2X pπx
f (x) = f s (p) sin
l p=1 l

B.Sc.(Mathematics)-IDE(UNOM)-II Year-IV Sem Transform Techniques


434 18.5. Worked out Problems Related to Finite Fourier Transform


2 X 16(−1) p−1 pπx
= 3
sin
8 p=1 p 8

X (−1) p−1 pπx
f (x) = 4 sin .
p=1
p3 8

Example 18.5.18. Find the finite Fourier sine and cosine transforms of

π
if 0 < x <

1,


 2
f (x) = 

π

<x<π

−1,

 if 2

Solution. Finite Fourier sine transform.


Z l  pπx 
F s [ f (x)] = f (x) sin
dx
0 l
Z π2 Z π
f s (p) = (1) sin pxdx + (−1) sin pxdx, l = π
π
0 2
 cos px π π
cos px 
= +
2
− π
p 0 p 2

1 pπ  1 pπ 
= − cos +1 + cos pπ − cos
p 2 p 2
1  pπ 
= 1 − 2 cos + (−1) p , p = 1, 2, 3, ...
p 2

If p = 0, then
F s [ f (x)] = 0.

Finite Fourier cosine transform.


Z l  pπx 
Fc [ f (x)] = f (x) cos
dx
0 l
Z π2 Z π
f c (p) = (1) cos pxdx + (−1) cos pxdx, l = π
π
0 2
 sin px  π2  sin px π
f c (p) = − π
p 0 p 2

Transform Techniques B.Sc.(Mathematics)-IDE(UNOM)-II Year-IV Sem


18.6. Exercises 435
1 pπ 1 pπ
f c (p) = sin + sin
p 2 p 2
2 pπ
Fc [ f (x)] = sin , p = 1, 2, 3, ...
p 2

If p = 0, then π
Z 2
Z π
Fc [ f (x)] = 1.dx + (−1)dx = 0.
π
0 2

18.6 Exercises

1. Find the finite Fourier cosine transform of 3x2 .


2. Find the Finite Fourier cosine transform of

π
if 0 < x <

 x,


 2
f (x) = 

π

π − x, <x<π


 if 2

3. Find the finite Fourier sine and cosine transform of f (x) defined by
(i) f (x) = x, 0 < x < π (ii) f (x) = 2x, 0 < x < 4 (iii) f (x) = x2 , 0 < x < 4.
4. Find the finite Fourier sine and cosine transform of f (x) = x(π − x), 0 < x < π.
5. (a) Find the finite Fourier sine and cosine transform of cos ax in (i) (0, l) and (ii) (0, π).
(b) Find the finite Fourier sine transform of cos kx.
2π(−1) p−1
6. Find inverse finite sine transform of f (x) of p3
, p = 1, 2, 3, .. where 0 < x < π.
 
7. Find inverse finite cosine transform of f (x) of 6
π(2p+1)
sin pπ
2
−cos pπ . for p = 1, 2, 3, .. where
0 < x < 4.
8. Find inverse finite cosine transform of f (x) of 1
(2p+1)2
cos 2pπ
3
, if 0 < x < 1, p = 1, 2, 3, ...

Answers
p
1. π3 when p = 0 and 6π(−1) p2
, p = 1, 2, 3, ....
2. πp + p22 cos pπ − p12 πp + p12 cos pπ.
   
2

B.Sc.(Mathematics)-IDE(UNOM)-II Year-IV Sem Transform Techniques


436 18.6. Exercises

π2 π(−1) p+1 (−) p−1


3. (i) 2
if p = 0 and p
, p2
, if p = 1, 2, 3, ...
(ii)16 , if p = 0 and − 32

cos π, 32

(cos pπ − 1) if p > 0.
(iii) − 64

cos π + 128
p3 π3
(cos pπ − 1), 128
p2 π2
cos pπ
4. 2
p3
[1 − (−1) p ], − pπ2 [1 + (−1) p ].
n
5. (a) (ii) n2 −a2
[1 − (−1)n cos aπ].
(−1) p−1
6. 4 ∞
P
n=1 p3
sin px.
 
7. 1

+ 3 P∞
π p=1
1
2p+1
sin pπ
2
− cos pπ cos pπx
4
.
8. 1 + 2 2pπ
P∞ 1
p=1 (2p+1)2 cos 3
cos pπx.

Transform Techniques B.Sc.(Mathematics)-IDE(UNOM)-II Year-IV Sem


UNIT-V

Lesson 19

APPLICATIONS TO ORDINARY
DIFFERENTIAL EQUATIONS
WITH CONSTANT COEFFICIENTS
Learning Objectives
Upon completion of this lesson, students will be able to

• use Laplace transform to solve ordinary differential equations with constant coefficients

• solve simultaneous linear differential equations by using Laplace transform.

19.1 Introduction

O
rdinary differential equations with constant coefficients can be easily solved by the Laplace
transform method, without the necessity of first finding the general solutions and then
evaluating the arbitrary constants. This method is, in general, shorter than our earlier methods
and is especially suitable to obtain the solution of linear non - homogeneous ordinary differential
equations with constant coefficients.

B.Sc.(Mathematics)-IDE(UNOM)-II Year-IV Sem 437 Transform Techniques


438 19.2. Worked out Problems

Let us consider a linear differential equation with constant coefficients

dn y dn−1 dn−2 dy
+ a1 + a2 + · · · + an−1 + an y = F(t), (19.1.1)
dtn dtn−1 dtn−2 dt

where F(t) is a function of independent variable t.

Let y(0) = c0 , y0 (0) = c1 , . . . , yn−1 (0) = cn−1 (19.1.2)

be the given initial or boundary conditions, where c0 , c1 , c2 , ..., cn−1 are constants.
If a1 , a2 , a3 , ...., an are constants, then we use

L [ f n (t)] = pn L [ f (t)] − pn−1 f (0) − pn−2 f 0 (0) − · · · − f n−1 (0) (19.1.3)

Taking Laplace transform of both sides of (19.1.1) and applying (19.1.3) and using conditions
(19.1.2) we obtain an algebraic equation known as subsidiary equation from which y(s) = L [y(t)]
is obtained. The required solution y(t) is obtained by taking inverse Laplace transform of y(s).
Working Rule to Solve Differential Equation by Laplace Transform Method.
1. We take the Laplace transforms of both sides of the given differential equation in y(t),
simultaneously using the given initial conditions. This gives an algebraic equation in y(s) =
L [y(t)].
2. We solve the algebraic equation to get y(s) as a function of s.
3. Finally we take L −1 [y(s)] to get y(t). The various methods we have discuss in UNIT -III will
enable us to find L −1 [y(s)].
The procedure is illustrated in the worked examples given below:

19.2 Worked out Problems

Example 19.2.1. Using Laplace transform, solve (D2 + 4D + 5)y = 5, given that y(0) = 0,
y0 (0) = 0.

Transform Techniques B.Sc.(Mathematics)-IDE(UNOM)-II Year-IV Sem


19.2. Worked out Problems 439

Solution. Given differential equation can be written as

y00 + 4y0 + 5y = 5.

Taking Laplace transform on both sides, we get

L [y00 ] + 4L [y0 ] + 5L [y] = 5.L [1].

i.e., p2 L [y] − p.y(0) − y0 (0) + 4[pL [y] − y(0)] + 5L [y] = 5.L [1].

Given
y(0) = 0 = y0 (0)

5
p2 L [y] + 4pL [y] + 5L [y] =
p
5
(p2 + 4p + 5)L [y] =
p
5
L [y] =
p(p2 + 4p + 5)
1 p+4
L [y] = − 2 (Resolving into partial f ractions)
p p + 4p + 5
h1 p+4 i
∴y = L −1 − 2
p p + 4p + 5
h1i h p+4 i
= L −1 − L −1 2
p p + 4p + 5
h p+2+2 i
= 1 − L −1
(p + 2)2 + 1
 h p+2 i h 2 i
= 1 − L −1 + L −1
(p + 2)2 + 1 (p + 2)2 + 1
 h p i h 1 i
= 1 − e−2t L −1 2 + 2e−2t L −1 2 ,
(p + 1 p +1

B.Sc.(Mathematics)-IDE(UNOM)-II Year-IV Sem Transform Techniques


440 19.2. Worked out Problems

by first shifting theorem.

 
= 1 − e−2t cost + 2e−2t sin t

y = 1 − e−2t (cos t + 2 sin t).

Example 19.2.2. Solve the following initial value problem by using Laplace transform
4y00 + π2 y = 0, y(0) = 2, y0 (0) = 0.

Solution. Given
4y00 + π2 y = 0,

Taking Laplace transform on both sides, we get

4L [y00 ] + π2 L [y] = 0.

i.e., 4(p2 L [y] − p.y(0) − y0 (0)) + π2 L [y] = 0.

Given
y(0) = 2 and y0 (0) = 0

4p2 L [y] − 8p + π2 L [y] = 0.

(4p2 + π2 )L [y] = 8p
8p
L [y] =
4p2 + π2
8p
=
π2
 
4 p2 + 4
h p i
∴ y = 2L −1 π2
p2 + 4
πt
y = 2 cos .
2

d3 y 2
Example 19.2.3. Solve by Laplace transform dt3
+2 ddt2y − dy
dt
−2y = 0, y(0) = 1, y0 (0) = y00 (0) = 2.

Transform Techniques B.Sc.(Mathematics)-IDE(UNOM)-II Year-IV Sem


19.2. Worked out Problems 441

Solution. The given differential equation is

y000 + 2y00 − y0 − 2y = 0

Taking Laplace transform on both sides, we get

L [y000 ] + 2L [y00 ] − L [y0 ] − 2L [y] = 0.

p3 L [y] − p2 y(0) − py0 (0) − y00 (0) + 2[p2 L [y] − py(0) − y0 (0)] − [pL [y] − y(0)] − 2L [y] = 0.

Given
y(0) = 1 and y0 (0) = y00 (0) = 2.

∴ p3 L [y] − p2 − 2p − 2 + 2[p2 L [y] − p − 2] − [pL [y] − 1] − 2L [y] = 0.

(p3 + 2p2 − p − 2)L [y] = p2 + 4p + 5


p2 + 4p + 5
L [y] =
p3 + 2p2 − p − 2
(p + 2)2 + 1
=
(p + 2)(p2 − 1)
p+2 1
= +
p − 1 (p + 2)(p2 − 1)
2

p 2 1
= + 2 +
p − 1 p − 1 (p + 2)(p − 1)(p + 1)
2

p 2 1 1 1 1
L [y] = + 2 + − +
p − 1 p − 1 3(p + 2) p − 1 6 p − 1
2
h p 2 1 1 1 1 i
∴y = L −1 2 + 2 + − +
p − 1 p − 1 3(p + 2) p − 1 6 p − 1
1 1 1
y = cos ht + 2 sinh t + e−2t − e−t + et .
3 2 6

d2 y
Example 19.2.4. Using Laplace transform, solve dt2
+ 2 dy
dt
− 3y = sin t, y = dy
dt
= 0, when t = 0.

B.Sc.(Mathematics)-IDE(UNOM)-II Year-IV Sem Transform Techniques


442 19.2. Worked out Problems

Solution. The given differential equation is

y00 + 2y0 − 3y = sin t

Taking Laplace transform on both sides, we get

L [y00 ] + 2L [y0 ] − 3L [y] = L [sin t].

1
p2 L [y] − py(0) − y0 (0) + 2[pL [y] − y(0)] − 3L [y] = .
p2 +1
Given
y(0) = 0 and y0 (0) = 0.
1
∴ p2 L [y] + 2pL [y] − 3L [y] = .
p2 +1

1
L [y](p2 + 2p − 3) =
p2 +1
1
L [y] =
(p2 + 1)(p2 + 2p − 3)
1
=
(p2 + 1)(p + 3)(p − 1)
−p
− 15 1 1
= 10
− + , resolving into partial f ractions
p + 1 40(p + 3) 8(p − 1)
2

1 p 1 1 1 1
L [y] = − − − +
10 p + 1 5 p + 1 40(p + 3) 8(p − 1)
2 2
h 1 p 1 1 1 1 i
∴y = L −1 − − − +
10 p + 1 5 p + 1 40(p + 3) 8(p − 1)
2 2

1 1 1 1
y = − cos t − sin t − e−3t + et .
10 5 40 8

Example 19.2.5. Using Laplace transform method, solve (D2 + 1)y = 6 cos 2t, t > 0, if y = 3,
Dy = 1 when t = 0.

Solution. Given
y00 + y = 6 cos 2t

Transform Techniques B.Sc.(Mathematics)-IDE(UNOM)-II Year-IV Sem


19.2. Worked out Problems 443

Taking Laplace transform on both sides, we get

L [y00 ] + L [y] = 6L [cos 2t].

6p
p2 L [y] − py(0) − y0 (0) + L [y] = .
p2+4
Given
y(0) = 3 and y0 (0) = 1.
6p
∴ p2 L [y] − 3p − 1 + L [y] = .
p2 + 4

1
L [y](p2 + 1) = + 1 + 3p
p2 +1
6p 1 3p
L [y] = + 2 + 2
(p + 4)(p + 1) p + 1 p + 1
2 2
h p p i 1 3p
= 2 2 − 2 + 2 + 2
p +1 p +4 p +1 p +1
5p 2p 1
L [y] = − +
p2 + 1 p2 + 4 p2 + 1
h 5p 2p 1 i
y = L −1 2 − 2 + 2
p +1 p +4 p +1
y = 5 cos t − 2 cos 2t + sin t.

d2 x
Example 19.2.6. Solve the differential equation dt2
− 4 dx
dt
− 12x = e3t given that x(0) = 1 and
x0 (0) = −2 using Laplace transform.

Solution. The given differential equation is

x00 − 4x0 − 12x = e3t .

Taking Laplace transform on both sides, we get

L [x00 ] − 4L [x0 ] − 12L [x] = L [e3t ].

B.Sc.(Mathematics)-IDE(UNOM)-II Year-IV Sem Transform Techniques


444 19.2. Worked out Problems

1
p2 L [x] − px(0) − x0 (0) − 4[pL [x] − x(0)] − 12L [x] = .
p−3
Given
x(0) = 1 and x0 (0) = −2.
1
∴ p2 L [x] − p + 2 − 4[L [x] − 1] − 12L [x] = .
p−3

1
L [x](p2 − 4p − 12) = + p−6
p−3
1 p−6
L [x] = 2
+ 2
(p − 2p − 12)(p − 3) p − 2p − 12
1 p−6
L [x] = +
(p + 2)(p − 6)(p − 3) (p + 2)(p − 6)

1 1 1 1 1 1 1
L [x] = − + + ,
40 p + 2 15 p − 3 24 p − 6 p + 2
by resolving into partial f raction
h1 1 1 1 1 1 1 i
x = L −1 − + +
40 p + 2 15 p − 3 24 p − 6 p + 2
1 −2t 1 3t 1 6t
x = e − e + e + e−2t
40 15 24
41 −2t 1 3t 1 6t
x = e − e + e .
40 15 24

d2 x
Example 19.2.7. Solve the differential equation using Laplace transform dt2
+ 3 dx
dt
+ 2x = e−t
given that x(0) = 0 and x0 (0) = 1.

Solution. The given differential equation is

x00 + 3x0 + 2x = e−t .

Taking Laplace transform on both sides, we get

L [x00 ] + 3L [x0 ] + 2L [x] = L [e−t ].

Transform Techniques B.Sc.(Mathematics)-IDE(UNOM)-II Year-IV Sem


19.2. Worked out Problems 445
1
p2 L [x] − px(0) − x0 (0) + 3[pL [x] − x(0)] + 2L [x] = .
p+1
Given
x(0) = 0 and x0 (0) = 1.
1
∴ p2 L [x] − 1 + 3[pL [x] − 0] + 2L [x] = .
p+1

1
L [x](p2 + 3p + 2) = +1
p+1
p+2
L [x](p2 + 3p + 2) =
p+1
p+2
L [x] =
(p + 1)(p2 + 3p + 2)
p+2
=
(p + 1)(p + 1)(p + 2)
1
L [x] =
(p + 1)2
h 1 i
x = L −1
(p + 1)2
x = te−t , by shi f ting theorem.

d2 x
Example 19.2.8. Solve the differential equation dt2
+ 9x = sin t using Laplace transform given
that x(0) = 1 and x π2 = 1.
 

Solution. The given differential equation is

x00 + 9x = sin t.

Taking Laplace transform on both sides, we get

L [x00 ] + 9L [x] = L [sin t].

1
p2 L [x] − px(0) − x0 (0) + 9L [x] = .
p2 +1

B.Sc.(Mathematics)-IDE(UNOM)-II Year-IV Sem Transform Techniques


446 19.2. Worked out Problems

Note that x0 (0) value is not given. Let us take x0 (0) = A.

π
x(0) = 1 and x = 1.
2

1
∴ p2 L [x] − p − A + 9L [x] = .
p2 +1

1
L [x](p2 + 9) = + p+A
p2 +1
1 p A
L [x] = + 2 + 2
(p + 1)(p + 9) p + 9 p + 9
2 2

1h 1 1 p A
= − + +
8 p2 + 1 p2 + 9 p2 + 9 p2 + 9
h1h 1 1 p A i
x = L −1 − + +
8 p2 + 1 p2 + 9 p2 + 9 p2 + 9
1h sin 3t i A
x = sin t − + cos 3t + sin 3t
8 3 3

π
 
Given x 2
= 1.

sin 3 π2 i
 
1h π π A π
1 = sin − + cos 3 + sin 3
8 2 3 2 3 2
1 1 A
1 = [1 + ]+0−
8 3 3
5
A = −
2

1h sin 3t i 5 1 7
∴x= sin t − + cos 3t − sin 3t = sin t + cos 3t − sin 3t.
8 3 6 8 6
Example 19.2.9. Solve the differential equation y00 + n2 y = a sin(nt + 2) given that y(0) = 0 and
y0 (0) = 0 using Laplace transform.

Solution. The given differential equation is

y00 + n2 y = a sin(nt + 2) = a[sin nt cos 2 + cos nt sin 2].

Transform Techniques B.Sc.(Mathematics)-IDE(UNOM)-II Year-IV Sem


19.2. Worked out Problems 447

Taking Laplace transform on both sides, we get

L [y00 ] + n2 L [y] = a cos 2L [sin nt] + a sin 2L [cos nt].

n p
p2 L [y] − py(0) − y0 (0) + n2 L [y] = a cos 2 + a sin 2 2 .
p2 +n2 p + n2
Given y(0) = 0 andy0 (0) = 0.
n p
∴ p2 L [y] + n2 L [y] = a cos 2 + a sin 2 2 .
p2 +n 2 p + n2

n p
L [y](p2 + n2 ) = a cos 2 + a sin 2 2 .
+n 2p2 p + n2
n p
L [x] = a cos 2 2 + a sin 2 2
(p + n )2 2 (p + n2 )2
h n i h p i
x = an cos 2L −1 2 + a sin 2L −1
(p + n2 )2 (p2 + n2 )2

We have
h p i t
L −1 = sin nt.
(p + n )
2 2 2 2n
Since
h f (p) i Z t
L −1
= F(t)dt
p 0
Z t
h 1 i
−1 1
h p i 1 1
∴L −1
= L = t sin ntdt = [−nt cos nt + sin nt]
(p + n )
2 2 2 p (p + n )
2 2 2 2n 0 2n3

1 t
x = an cos 2. 3
[−nt cos nt + sin nt] + a sin 2. sin nt
2n 2n
a
= [−nt cos 2 cos nt + cos 2 sin nt + nt sin 2 sin nt]
2n2
a
= [cos 2 sin nt − nt(cos nt cos 2 − sin nt sin 2]
2n2
a
x = [cos 2 sin nt − nt cos(nt + 2)]
2n2

Example 19.2.10. Using Laplace transform method solve the differential equation (D2 + 1)y =

B.Sc.(Mathematics)-IDE(UNOM)-II Year-IV Sem Transform Techniques


448 19.2. Worked out Problems

sin t sin 2t, (t > 0) given that y = 1 and Dy = 0 when t = 0.

Solution. The given differential equation is

cos t − cos 3t
y00 + y = .
2

Taking Laplace transform on both sides, we get

1 1
L [y00 ] + L [y] = L [cos t] − L [cos 3t].
2 2

1 p 1 p
p2 L [y] − py(0) − y0 (0) + L [y] = − .
2 p + 1 2 p2 + 9
2

Given y(0) = 1 andy0 (0) = 0.


1 p 1 p
∴ p2 L [y] − p + L [y] = − .
2 p2 + 1 2 p2 + 9

1 p 1 p
L [y](p2 + 1) = − +p
2 p2 + 1 2 p2 + 9
1 p 1 p p
L [y] = − + 2
2 (p + 1)
2 2 2 (p + 1)(p + 9) p + 1
2 2

1 p 1h p p i p
= − − + 2
2 (p + 1)
2 2 16 (p + 1) p + 9
2 2 p +1
h1 p 1 h p p i p i
y = L −1 − − +
2 (p2 + 1)2 16 (p2 + 1) p2 + 9 p2 + 1
1 t 1 h p i t
= . sin t − [cos t − cos 3t] + cos t, since L −1 2 = sin at
2 2 16 (p + a )
2 2 2a
t 15 1
y = sin t + cos t + cos 3t.
4 16 16

d2 y
Example 19.2.11. Solve the differential equation using Laplace transform dt2
+2 dy
dt
+5y = e−t sin t
given that y(0) = 0 and y0 (0) = 1.

Solution. The given differential equation is

y00 + 2y0 + 5y = e−t sin t.

Transform Techniques B.Sc.(Mathematics)-IDE(UNOM)-II Year-IV Sem


19.2. Worked out Problems 449

Taking Laplace transform on both sides, we get

L [y00 ] + 2L [y0 ] + 5L [y] = L [e−t sin t].

1
p2 L [y] − py(0) − y0 (0) + 2[pL [y] − y(0)] + 5L [y] = .
(p + 1)2 + 1
Given
y(0) = 0 and y0 (0) = 1.
1
∴ p2 L [y] − 1 + 2[pL [y] − 0] + 5L [y] = .
(p + 1)2 + 1

1
L [y](p2 + 2p + 5) = +1
p2 + 2p + 2
p2 + 2p + 3
L [y] =
(p2 + 2p + 2)(p2 + 2p + 5)
1 1 2 1
L [y] = + . 2
3 p + 2p + 2 3 p + 2p + 5
2
h1 1 2 1 i
y = L −1 + .
3 p2 + 2p + 2 3 p2 + 2p + 5
1 −1 h 1 i 2 h 1 i
= L + L −1
3 (p + 1)2 + 1 3 (p + 1)2 + 4
1 −t −1 1 h i 2 −t −1 1 ih
= e L + e L
3 p2 + 1 3 p2 + 4
1 −t 2 sin 2t
= e sin t + e−t
3 3 2
e−t
y = (sin t + sin 2t).
3

Example 19.2.12. Using Laplace transform method solve the differential equation (D2 + 1)x =
t cos 2t, given that x = 0 and dx
dt
= 0 at t = 0.

Solution. The given differential equation is

x00 + x = t cos 2t.

B.Sc.(Mathematics)-IDE(UNOM)-II Year-IV Sem Transform Techniques


450 19.2. Worked out Problems

Taking Laplace transform on both sides, we get

L [x00 ] + L [x] = [t cos 2t]

d p
p2 L [x] − px(0) − x0 (0) + L [x] = −
dp p + 4
2

Given x(0) = 0 and x0 (0) = 0.


p2 − 4
∴ p2 L [x] + L [x] = .
(p2 + 4)2

p2 − 4
L [x](p2 + 1) =
(p2 + 4)2

p2 − 4
L [x] =
(p2 + 1)(p2 + 4)2
5 1 8 1 5 1
= + − , by resolving into partial f ractions
9 p + 4 3 (p + 4)
2 2 2 9 p2 + 1
h5 1 8 1 5 1 i
x = L −1 + −
9 p2 + 4 3 (p2 + 4)2 9 p2 + 1
5 1 8 h1 p i 5
= . sin 2t + L −1 . 2 − sin t
9 2 3 p (p + 4)2 9
Z t
5 8 u 5 h p i t
= sin 2t + sin 2udu − sin t, L −1 2 = sin at
18 3 0 4 9 (p + a )
2 2 2a
5 2 h  − cos 2u   sin 2u it 5
= sin 2t + u − 1. − − sin t
18 3 2 4 0 9
5 2h t sin 2t i 5
= sin 2t + − cos 2t + − sin t
18 3 2 4 9
4 5 t
x = sin 2t − sin t − cos 2t.
9 9 3

d2 x
Example 19.2.13. Solve dt2
+ 2 dx
dt
+ x = 3te−t given that x(0) = 4 and dx
dt
= 0 at t = 0.

Solution. The given differential equation is

x00 + 2x0 + x = 3te−t .

Transform Techniques B.Sc.(Mathematics)-IDE(UNOM)-II Year-IV Sem


19.2. Worked out Problems 451

Taking Laplace transform on both sides, we get

L [x00 ] + 2L [x0 ] + L [x] = L [3te−t ].

3
p2 L [x] − px(0) − x0 (0) + 2[pL [x] − x(0)] + L [x] = .
(p + 1)2
Given
x(0) = 4 and x0 (0) = 0.
3
∴ p2 L [x] − 4p + 2[pL [x] − 4] + L [x] = .
(p + 1)2

3
L [x](p2 + 2p + 1) = + 4p + 8
(p + 1)2
3
L [x]((p + 1)2 ) = + 4p + 8
(p + 1)2
3 p 8
L [x] = +4 +
(p + 1) 4 (p + 1) 2 (p + 1)2
h 3 p+1−1 8 i
x = L −1 + 4 +
(p + 1)4 (p + 1)2 (p + 1)2

h1i h 1 1 i −1 1
h i
x = 3e−t L −1 + 4L −1
− + 8e−t
L
p4 p + 1 (p + 1)2 p2
t3
= 3e−t . + 4e−t (1 − t) + 8e−t t
3!
h t3 i
x = e−t + 4t + 4 .
2

Example 19.2.14. Solve (D + 1)y = 0, t > 0 if y = y0 when t = 0.

Solution. Given
y0 + y = 0

Taking Laplace transform of both sides, we get

L [y0 ] + L [y] = 0

B.Sc.(Mathematics)-IDE(UNOM)-II Year-IV Sem Transform Techniques


452 19.2. Worked out Problems

pL [y] − y0 + L [y] = 0

Given y(0) = y0

L [y](p + 1) = y0
y0
=
p+1
 1 
y = y0 L −1
p+1
y = y0 e−t .

Example 19.2.15. Solve (D2 − 3D + 2)y = 1 − e2t given that y = 1, Dy = 0 when t = 0.

Solution. The given differential equation is

y00 − 3y0 + 2y = 1 − e2t .

Taking Laplace transform on both sides, we get

L [y00 ] − 3L [y0 ] + 2L [y] = L [1 − e2t ].

1 1
p2 L [y00 ] − py(0) − y0 (0) − 3[pL [y] − y(0)] + 2L [y] = − .
p p−2
Given
y(0) = 1 and y0 (0) = 0.
1 1
∴ p2 L [y] − p − 3[pL [y] − 1] + 2L [y] = − .
p p−2

1 1
L [y](p2 − 3p + 2) = − + p−3
p p−2
1 1
L [y]((p − 1)(p − 2)) = − + p−3
p p−2

Transform Techniques B.Sc.(Mathematics)-IDE(UNOM)-II Year-IV Sem


19.2. Worked out Problems 453
−2
L [y]((p − 1)(p − 2)) = + p−3
p(p − 2)
p3 − 5p2 + 6p − 2
L [y] =
p(p − 2)2 (p − 1)
1 1 1
L [y] = + −
2p 2(p − 2) (p − 2)2
h1 1 1 i
y = L −1 + −
2p 2(p − 2) (p − 2)2
1 −1 h 1 1 −1 1 1i
= L + L − e2t L −1 2
2 p 2 p−2 p
1 1 2t
y = + e − e2t t
2 2

Example 19.2.16. Solve (D2 + D)y = t2 + 2t, y(0) = 4, y0 (0) = −2

Solution. The given differential equation is

y00 + y0 = t2 + 2t.

Taking Laplace transform on both sides, we get

L [y00 ] + L [y0 ] = L [t2 + 2t].

2 2
p2 L [y00 ] − py(0) − y0 (0) + pL [y] − y(0) = 3
+ 2.
p p
Given
y(0) = 4 and y0 (0) = −2.
2 2
∴ p2 L [y] − 4p + 2 + pL [y] − 4 = 3
+ 2.
p p

B.Sc.(Mathematics)-IDE(UNOM)-II Year-IV Sem Transform Techniques


454 19.2. Worked out Problems

2 2
L [y](p2 + p) = + + 4p + 2
p3 p2
2 + 2p + 4p4 + 2p3
L [y](p + p)) =
2
p3
2 + 2p + 4p4 + 2p3
L [y] =
p4 (p + 1)
2 2 2
L [y] = 4 + +
p p p+1

h2 2 2 i
y = L −1 + +
p4 p p + 1
2t3
y = + 2 + 2e−t
3!
t3
y = + 2 + 2e−t .
3

Example 19.2.17. Solve (D2 − 2D + 2)y = 0, y = Dy = 1, when t = 0.

Solution. The given differential equation is

y00 + 2y0 + 2y = 0.

Taking Laplace transform on both sides, we get

L [y00 ] − 2L [y0 ] + 2L [y] = 0.

p2 L [y00 ] − py(0) − y0 (0) − 2[pL [y] − y(0)] + 2L [y] = 0.

Given
y(0) = 1 and y0 (0) = 1.

∴ p2 L [y] − p − 1 − 2pL [y] + 2 + 2L [y] = 0.

Transform Techniques B.Sc.(Mathematics)-IDE(UNOM)-II Year-IV Sem


19.2. Worked out Problems 455

L [y](p2 − 2p + 2) = p − 1
p−1
L [y] =
p2− 2p + 2
p−1
L [y] =
(p − 1)2 + 1
h p−1 i
y = L −1
(p − 1)2 + 1
h p i
y = et L −1 2
p +1
y = et cos t, by using f irst shi f ting theorem.

Example 19.2.18. Solve (D + 1)2 y = t, y = −3, when t = 0 and y = −1 when t = 1.

Solution. The given differential equation is

y00 + 2y0 + y = t.

Taking Laplace transform on both sides, we get

L [y00 ] + 2L [y0 ] + L [y] = L [t].

1
p2 L [y00 ] − py(0) − y0 (0) + 2[pL [y] − y(0)] + L [y] = .
p2
Given
y(0) = −3 and y(1) = −1.

Let y0 (0) = A
1
∴ p2 L [y] + 3p − A + 2pL [y] + 6 + L [y] = .
p2

1
L [y](p2 + 2p + 1) = + A − 3p − 6
p2
1
L [y](p + 1)2 = 2 + A − 3p − 6
p

B.Sc.(Mathematics)-IDE(UNOM)-II Year-IV Sem Transform Techniques


456 19.2. Worked out Problems

1 A 3p + 6
L [y] = + −
p2 (p + 1)2 (p + 1)2 (p + 1)2
2 1 A 3 3
= − + 2+ − − ,
p p (p + 1) 2 p + 1 (p + 1)2

by resolving into partial fractions.

2 1 1 A−2
L [y] = − + 2 − +
p p p + 1 (p + 1)2
h 2 1 1 A−2 i
y = L −1 − + 2 − +
p p p + 1 (p + 1)2
y = −2 + t − e−t + (A − 2)e−t t

Given that y(1) = −1

−1 = −2 + 2 − e−1 + (A − 2)e−1 ⇒ 0 = (A − 3)e−1 ⇒ A = 3.

∴ y = −2 + t − e−t + te−t .

Example 19.2.19. Solve (D2 + 6D + 9)y = sin x, y(0) = 1, and y0 (0) = 0

Solution. The given differential equation is

y00 + 6y0 + 9y = sin x.

Taking Laplace transform on both sides, we get

L [y00 ] + 6L [y0 ] + 9L [y] = L [sin x].

1
p2 L [y00 ] − py(0) − y0 (0) + 6[pL [y] − y(0)] + 9L [y] = .
1 + p2
Given
y(0) = 1 and y0 (0) = 0.
1
∴ p2 L [y] − p + 6pL [y] − 6 + 9L [y] = .
p2 + 1

Transform Techniques B.Sc.(Mathematics)-IDE(UNOM)-II Year-IV Sem


19.2. Worked out Problems 457
1
L [y](p2 + 6p + 9) = + p+6
p2 + 1
1
L [y](p + 3)2 = 2 + p+6
p +1
1 p+6
L [y] = +
(p + 1)(p + 3)
2 2 (p + 3)2
p3 + 6p2 + p + 7
=
(p2 + 1)(p + 3)2
53 1 31 1 3 p 4 1
L [y] = + − +
50 p + 3 10 (p + 3) 2 50 p + 1 50 p2 + 1
2
h 53 1 31 1 3 p 4 1 i
y = L −1 + − +
50 p + 3 10 (p + 3)2 50 p2 + 1 50 p2 + 1
53 −3x 31 −3x 3 4
= e + e x− cos x + sin x.
50 10 50 50

Example 19.2.20. Solve (D2 − D − 2)y = 20 sin 2t, y = −1, Dy = 2, when t = 0

Solution. The given differential equation is

y00 − y0 − 2y = 20 sin 2t.

Taking Laplace transform on both sides, we get

L [y00 ] − L [y0 ] − 2L [y] = 20L [sin 2t].

2
p2 L [y00 ] − py(0) − y0 (0) − [pL [y] − y(0)] − 2L [y] = 20. .
p2 +4
Given
y(0) = −1 and y0 (0) = 2.
40
∴ p2 L [y] + p − 2 − pL [y] − 1 − 2L [y] = .
p2 +4

40
L [y](p2 − p − 2) = +3− p
+4
p2
40
L [y](p − 2)(p + 1) = 2 +3− p
p +4

B.Sc.(Mathematics)-IDE(UNOM)-II Year-IV Sem Transform Techniques


458 19.2. Worked out Problems

40 3− p
L [y] = +
(p − 2)(p + 1)(p2 + 4) (p − 2)(p + 1)
3 1 8 1 p−6 1 1 4 1
= − + 2 + . −
5p−2 3p+1 p +4 3 p−2 3p+1
by resolving into partial fractions
2 4 p 6
L [y] = − + 2 − 2
p−2 p+1 p +4 p +4
h 2 4 p 6 i
y = L −1 − + 2 − 2
p−2 p+1 p +4 p +4
y = 2e2t − 4e−t + cos 2t − 3 sin 2t

Example 19.2.21. Solve dy


dt
+ y = 1, y = 2, when t = 0

Solution. The given differential equation is

y0 + y = 1.

Taking Laplace transform on both sides, we get

1
L [y0 ] + L [y] = .
p

1
pL [y] − y(0) + L [y] = .
p
Given y(0) = 2
1
∴ pL [y] − 2 + L [y] = .
p

1
L [y](p + 1) = +2
p
1 2
L [y] = +
p(p + 1) p + 1

Transform Techniques B.Sc.(Mathematics)-IDE(UNOM)-II Year-IV Sem


19.2. Worked out Problems 459
1 1 2
= − +
p p+1 p+1
1 1
L [y] = +
p p+1
h1 1 i
y = L −1 +
p p+1
y = 1 + e−t .

dy
Example 19.2.22. Solve dt2
+ y = 0, under the conditions that y = 1, dy
dt
= 0, when t = 0

Solution. The given differential equation is

y00 + y = 0.

Taking Laplace transform on both sides, we get

L [y00 ] + L [y] = 0.

p2 L [y00 ] − py(0) − y0 (0) + L [y] = 0

Given
y(0) = 1 and y0 (0) = 0.

∴ p2 L [y] − p + L [y] = 0.

L [y](p2 + 1) = p
p
L [y] =
+1
p2
h p2 i
y = L −1 2
p +1
y = cos t

Example 19.2.23. Solve (D2 + 9)y = 18t if y(0) = 0, y π2 = 0.


 

B.Sc.(Mathematics)-IDE(UNOM)-II Year-IV Sem Transform Techniques


460 19.2. Worked out Problems

Solution. The given differential equation is

y00 + 9y = 18t.

Taking Laplace transform on both sides, we get

L [y00 ] + 9L [y] = 18L [t].

18
p2 L [y00 ] − py(0) − y0 (0) + 9L [y] =
p2
Given
y(0) = 0 and let y0 (0) = A.
18
∴ p2 L [y] − A + 9L [y] =
p2

18
L [y](p2 + 9) = +A
p2
18 A
L [y] = 2 2 + 2
p (p + 9) p + 9

1 2 A
L [y] = − 2 + 2
p 2 p +9 p +9
h1 2 A i
y = L −1 2 − 2 + 2
p p +9 p +9
1
y = 2t + (A − 2) sin 3t
3

Given y π2 = 0
 

π 1
0 = 2 + (A − 2) (−1) ⇒ A = 2 + 3π
2 3
∴ y = 2t + π sin 3t.

Example 19.2.24. Solve (D2 − D − 6)y = 2 if y = 1, Dy = 0, when t = 0.

Transform Techniques B.Sc.(Mathematics)-IDE(UNOM)-II Year-IV Sem


19.2. Worked out Problems 461

Solution. The given differential equation is

y00 − y0 − 6y = 2.

Taking Laplace transform on both sides, we get

L [y00 ] − L [y0 ] − 6L [y] = 2L [1].

2
p2 L [y00 ] − py(0) − y0 (0) − [pL [y] − y(0)] − 6L [y] = .
p
Given
y(0) = 1 and let y0 (0) = 0.
2
∴ p2 L [y] − p − pL [y] + 1 − 6L [y] = .
p

2
L [y](p2 − p − 6) = + p−1
p
1
L [y]((p − 3)(p + 2)) = + p−1
p
1 p−2
L [y] = +
p(p − 3)(p + 2) (p − 3)(p + 2)
p2 − p + 2
=
p(p − 3)(p + 2)
11 8 1 4 1
= − + +
3 p 15 p − 3 5 p + 2
h 11 8 1 4 1 i
y = L −1 − + +
3 p 15 p − 3 5 p + 2

1 8 4
y = − + e3t + e−2t .
3 15 5

d2 x
Example 19.2.25. Solve dt2
+ x = t, x0 = 1, x1 = 2.

B.Sc.(Mathematics)-IDE(UNOM)-II Year-IV Sem Transform Techniques


462 19.2. Worked out Problems

Solution. The given differential equation is

x00 + x = t.

Taking Laplace transform on both sides, we get

L [x00 ] + L [x] = L [t].

1
p2 L [x00 ] − px(0) − x0 (0) + L [x] = .
p2
Given
x(0) = 1, and x0 (0) = 2.
1
∴ p2 L [x] − p − 2 + L [x] = .
p2

1
L [x](p2 + 1) = + p+2
p2
1 p+2
L [x] = + 2
p (p + 1) p + 1
2 2

1 1 p+2
L [x] = − +
p2 p2 + 1 p2 + 1
1 1 p
= + 2 + 2
p 2 p +1 p +1
h1 1 p i
x = L −1 2 + 2 + 2
p p +1 p +1
x = t + sin t + cos t

Example 19.2.26. Solve (D2 + D)x = 2 if x0 = 3, x1 = 1

Solution. The given differential equation is

x00 + x0 = 2.

Transform Techniques B.Sc.(Mathematics)-IDE(UNOM)-II Year-IV Sem


19.2. Worked out Problems 463

Taking Laplace transform on both sides, we get

L [x00 ] + L [x0 ] = 2L [1].

2
p2 L [x00 ] − px(0) − x0 (0) + pL [x] − x(0) = .
p
Given
x(0) = 3 and let x0 (0) = 1.
2
∴ p2 L [x] − 3p − 1 + pL [x] − 3 = .
p

2
L [x](p2 + p) = + 3p + 4
p
2 3p 4
L [x] = + +
p (p + 1) p(p + 1) p(p + 1)
2

2 + 3p2 + 4p
L [x] =
p2 (p + 1)
2 1 1
= + 2+
p 2 p p+1
h2 1 1 i
x = L −1 2 + 2 +
p p p+1
x = 2 + 2t + e−t .

Example 19.2.27. Solve (D2 + 5D + 6)y = 5et if y = 2, Dy = 1 when t = 0.

Solution. The given differential equation is

y00 + 5y0 + 6y = 5et .

Taking Laplace transform on both sides, we get

L [y00 ] + 5L [y0 ] + 6L [y] = 5L [et ].

B.Sc.(Mathematics)-IDE(UNOM)-II Year-IV Sem Transform Techniques


464 19.2. Worked out Problems

5
p2 L [y00 ] − py(0) − y0 (0) + 5pL [y] − 5y(0) + 6L [y] = .
p−1
Given
y(0) = 2 and y0 (0) = 1.
5
∴ p2 L [y] − 2p − 1 + 5pL [y] − 10 + 6L [y] = .
p−1

5
L [y](p2 + 5p + 6) = + 2p + 11
p−1
5
L [y]((p + 3)(p + 2)) = + 2p + 11
p−1
5 2p + 11
L [y] = +
(p − 1)(p + 2)(p + 3) (p + 3)(p + 2)

5 1 1 1 5 1 5 7
L [y] = + + − +
12 p − 1 3 p + 2 4 p + 3 p + 3 p + 2
5 1 22 1 15 1
= + −
12 p − 1 3 p+2 4 p+3
h5 1 22 1 15 1 i
y = L −1 + −
12 p − 1 3 p+2 4 p+3
5 t 22 −2t 15 −3t
y = e + e − e .
12 3 4

Example 19.2.28. Solve (D + 2)2 )Y = 4e−2t if Y(0) = −1, Y 0 (0) = 4.

Solution. The given differential equation is

Y 00 + 4Y 0 + 4Y = 4e−2t .

Taking Laplace transform on both sides, we get

L [Y 00 ] + 4L [Y 0 ] + 4L [Y] = 4L [e−2t ].

4
p2 L [Y 00 ] − pY(0) − Y 0 (0) + 4pL [y] − 4Y(0) + 4L [Y] = .
p+2

Transform Techniques B.Sc.(Mathematics)-IDE(UNOM)-II Year-IV Sem


19.2. Worked out Problems 465

Given
Y(0) = −1 and Y 0 (0) = 4.
4
∴ p2 L [Y] + p − 4 + 5pL [y] + 4 + 4L [Y] = .
p+2

4
L [Y](p2 + 5p + 4) = −p
p+2
4
L [Y]((p + 2)2 ) = −p
p+2
4 −p
L [Y] = +
(p + 2) 3 (p + 2)2
4 h p+2 2 i
= − −
(p + 2)3 (p + 2)2 (p + 2)2
4 1 2
= − +
(p + 2) 3 (p + 2) (p + 2)2
h 4 1 2 i
Y = L −1 − +
(p + 2)3 (p + 2) (p + 2)2
h1i h1i
= 4e−2t L −1 3 − e−2t + 2e−2t L −1 2
p p
2
t
Y = 4e−2t − e−2t + 2e−2t t
2

Y = e−2t [−1 + 2t + 2t2 ].

π
 
d2 Y
Example 19.2.29. Solve dt2
+ 9Y = cos 2t, if Y(0) = 1; Y 2
= 1.

Solution. The given differential equation is

Y 00 + 9Y = cos 2t.

Taking Laplace transform on both sides, we get

L [Y 00 ] + 9L [Y] = L [cos 2t].

B.Sc.(Mathematics)-IDE(UNOM)-II Year-IV Sem Transform Techniques


466 19.2. Worked out Problems

p
p2 L [Y 00 ] − pY(0) − Y 0 (0) + 9L [Y] = .
p2 +4
Given
Y(0) = 1, and Y 0 (0) = A.
p
∴ p2 L [Y] − p − A + 9L [Y] = .
p2 +4

p
L [Y](p2 + 9) = + p+A
p2 + 4
p p A
L [Y] = + 2 + 2
(p + 9)(p + 4) p + 9 p + 9
2 2

1h p p i p A
L [y] = − + 2 + 2
5 p +4 p +9
2 2 p +9 p +9
4 p 1 p A
= + +
5 p2 + 9 5 p2 + 4 p2 + 9
h4 p 1 p A i
Y = L −1 + +
5 p2 + 9 5 p2 + 4 p2 + 9
4 1 A
Y = cos 3t + cos 2t + sin 3t
5 5 3

π
 
Given Y 2
= 1.
4 3π 1 2π A 3π 18
∴1= cos + cos + sin ⇒A=− .
5 2 5 2 3 2 5
Hence
4 1 6
Y= cos 3t + cos 2t − sin 3t.
5 5 5
d2 Y
Example 19.2.30. Solve dt2
+ Y = t cos 2t, if Y = 0 = dY
dt
= 0 t = 0.

Solution. The given differential equation is

Y 00 + Y = t cos 2t.

Taking Laplace transform on both sides, we get

L [Y 00 ] + L [Y] = L [t cos 2t].

Transform Techniques B.Sc.(Mathematics)-IDE(UNOM)-II Year-IV Sem


19.3. Solution of Simultaneous Ordinary Differential Equations 467
d
p2 L [Y 00 ] − pY(0) − Y 0 (0) + L [Y] = − L [cos 2t].
dp
Given
Y(0) = 0, and Y 0 (0) = 0.
p2 − 4
∴ p L [Y] + L [Y] = 2
2
.
(p + 4)2

1 8
L [Y](p2 + 1) = −
p2 + 4 (p2 + 4)2
1 8
L [Y] = − 2
(p + 4)(p + 1) (p + 4)2 (p2 + 1)
2 2

1 1 1 1 8h 1 1 3 i
L [y] = − − − −
3 p2 + 1 3 p2 + 4 9 p2 + 1 p2 + 4 (p2 + 4)2
5 1 5 1 8 1
= − 2 + +
9 p + 1 9 p2 + 4 3 (p2 + 4)2
h 5 1 5 1 8 1 i
Y = L −1 − + +
9 p2 + 1 9 p2 + 4 3 (p2 + 4)2
5 5 8h 1 i
= − sin t + sin 2t + (sin 2t − 2t cos 2t)
9 18 3 16
5 4 16
= − sin t + sin 2t − t cos 2t,
9 9 3
h i
since L −1 1
(p2 +a2 )2
= 1
2a3
(sin at − at cos at).

19.3 Solution of Simultaneous Ordinary Differential


Equations

In the solution of simultaneous linear differential equations, the Laplace transform method can
also be used in the same way as we have done earlier.

B.Sc.(Mathematics)-IDE(UNOM)-II Year-IV Sem Transform Techniques


468 19.4. Worked out Problems

19.4 Worked out Problems

Example 19.4.1. Using Laplace transform, solve dx


dt
− 2x + 3y = 0; dy
dt
+ 2x − y = 0 given x = 8,
y = 3 when t = 0.

Solution. Given
dx dy
− 2x + 3y = 0 and + 2x = 0.
dt dt
Taking Laplace transform on both sides, we get

pL [x] − x(0) − 2L [x] + 3L [y] = 0 and pL [y] − y(0) + 2L [x] − L [y] = 0

Given initial conditions are x(0) = 8, and y(0) = 3

pL [x] − 8 − 2L [x] + 3L [y] = 0 and pL [y] − 3 + 2L [x] − L [y] = 0

(p − 2)L [x] + 3L [y] = 8

2L [x] + (p − 1)L [y] = 3

We solve these equations for L [x] and L [y].



8 3

3 p − 1
8p − 7 8p − 17
L [x] = = 2 =
p − 2 p − 3p − 4 (p + 1)(p − 4)
3

2 p − 1

Transform Techniques B.Sc.(Mathematics)-IDE(UNOM)-II Year-IV Sem


19.4. Worked out Problems 469

and
p − 2 8

2 3 3p − 22 3p − 22
L [y] = = 2 =
p − 2 p − 3p − 4 (p + 1)(p − 4)
3

2 p − 1

5 3
∴ L [x] = +
p+1 p−4
h 5 3 i
x = L −1 +
p+1 p−4
x = 5e−t + 3e4t .

5 2
and L [y] = −
p+1 p−4

h 5 2 i
x = L −1 −
p+1 p−4
x = 5e−t − 2e4t .

Example 19.4.2. Solve the equations dx


dt
+ y = sin t; dy
dt
+ x = cos t given x = 2, y = 0 when
t = 0.

Solution. Given
dx dy
+ y = sin t and + x = cos t.
dt dt
Taking Laplace transform on both sides, we get

pL [x] − x(0) + L [y] = L [sin t] and pL [y] − y(0) + L [x] = L [cos t]

B.Sc.(Mathematics)-IDE(UNOM)-II Year-IV Sem Transform Techniques


470 19.4. Worked out Problems

Given initial conditions are x(0) = 2, and y(0) = 0

1 p
pL [x] − 2 + L [y] = and pL [y] + L [x] =
p2 + 1 p2 + 1

1
pL [x] + L [y] = 2 +
p2 +1
p
L [x] + pL [y] =
p2 +1

We solve these equations for L [x] and L [y].



2 + 1 1
p2 +1


p
p2 +1 p 2p + p2p+1 − p
p2 +1 2p
L [x] = = =
p2 − 1 p2−1

p 1

1 p

h 2p i
x = L −1 = 2 cosh t.
p2 − 1

p 2 + 1
p2 +1



p p2 1
1 p +1
2

p2 +1
−2− p2 +1 1 2
L [y] = = = − 2
p2 −1 p2 +1 p −1

p 1

1 p

h 1 2 i
y = L −1 − = sin t − 2 sinh t.
p2 + 1 p2 − 1

Note. While solving simultaneous equations using Laplace transform, one can find one of the
unknown functions using Laplace transform and the other one directly using given equations.

Transform Techniques B.Sc.(Mathematics)-IDE(UNOM)-II Year-IV Sem


19.4. Worked out Problems 471

Example 19.4.3. Solve by Laplace transform technique, (D+2)x−y = −6t; −2x+(D+1)y = −30t
if x(0) = 2 and y(0) = 3.

Solution. Given
x0 + 2x − y = −6t (19.4.1)

and
y0 − 2x + y = −30t (19.4.2)

Taking Laplace transform of (19.4.1), we have

L [x0 ] + 2L [x] − L [y] = −6L [t]

6
pL [x] − x(0) + 2L [x] − L [y] = −
p2
Given x(0) = 2.

6 6
pL [x] − 2 + 2L [x] − L [y] = − 2
⇒ (p + 2)L [x] − L [y] = − 2 + 2
p p

2p2 − 6
(p + 2)L [x] − L [y] = (19.4.3)
p2
Now taking Laplace transform of (19.4.2), we have

L [y0 ] − 2L [x] + L [y] = −30L [t]

1
pL [y] − y(0) − 2L [x] + L [y] = −30
p2
Given y(0) = 3.

30 30
pL [y] − 3 − 2L [x] + L [y] = − 2
⇒ −2L [x] + (p + 1)L [y] = − 2 + 3
p p

3p2 − 30
−2L [x] + (p + 1)L [y] = (19.4.4)
p2

B.Sc.(Mathematics)-IDE(UNOM)-II Year-IV Sem Transform Techniques


472 19.4. Worked out Problems

(19.4.3) ×(p + 1)+ (19.4.4) gives

(2p2 − 6)(p + 1) 3p2 − 30


(p + 1)(p + 2)L [x] − 2L [x] = +
p2 p2
2p + 5p − 6p − 36
3 2
(p2 + 3p)L [X] =
p2

2p3 + 5p2 − 6p − 36
L [x] =
p3 (p + 3)
1 2 12 1
= + 2− 3+
p p p p+3
h1 2 12 1 i
x = L −1 + 2 − 3 +
p p p p+3
2
t
= 1 + 2t − 12. + e−3t
2!
x = 1 + 2t − 6t2 + e−3t

dx
= 2 − 12t − 3e−3t .
dt
dx
Substituting the values of x and dt
in (19.4.1), we get

(2 − 12t − 3e−3t ) + 2(1 + 2t − 6t2 + e−3t ) − y = −6t

y = 12t − 12t2 + 4 − 3e−3t .

Hence the required solution is

x = 1 + 2t − 6t2 + e−3t and y = 12t − 12t2 + 4 − 3e−3t .

Example 19.4.4. Solve by Laplace transform method (D−2)x−(D−2)y = 1−2t; (D2 +1)x+2Dy =
0 if x(0) = 0, y(0) = 0 and x0 (0) = 0.

Solution. Given
x0 − y0 − 2x + 2y = 1 − 2t; x00 + 2y0 + x = 0.

Transform Techniques B.Sc.(Mathematics)-IDE(UNOM)-II Year-IV Sem


19.4. Worked out Problems 473

Taking Laplace transform on both sides, we get

pL [x] − x(0) − [pL [y] − y(0)] − 2L [x] + 2L [y] = L [1 − 2t]

and
p2 L [x] − px(0) − x0 (0) + 2[pL [y] − y(0)] + L [x] = 0.

Given initial conditions are x(0) = 0, y(0) = 0 and x0 (0) = 0.

1 2
pL [x] − pL [y] − 2L [x] + 2L [y] = − 2; p2 L [x] + 2pL [y] + L [x] = 0
p p

p−2
(p − 2)L [x] − (p − 2)L [y] =
p2
(p2 + 1)L [x] + 2pL [y] = 0

We solve these equations for L [x] and L [y].



p−2 −(p − 2)
p2
0 2(p−2) 2p(p−2)
2p p p 2
L [x] = = = = .
p − 2 −(p − 2) 2p(p − 2) + (p − 2)(p2 + 1) (p − 2)(p + 2p
2 + 1) p(p + 1)2

p2 + 1 2p

h1 1 1 i
L [x] = 2 −

p p + 1 (p + 1)2
h1 1 1 i
x = 2L −1 − −
p p + 1 (p + 1)2
x = 2(1 − e−t − te−t ).

B.Sc.(Mathematics)-IDE(UNOM)-II Year-IV Sem Transform Techniques


474 19.4. Worked out Problems


p − 2 p−2
p2




p + 1
2
0 p2 + 1
L [y] = = − 2
p − 2 −(p − 2) p (p + 1)2

p2 + 1 2p

h p2 + 1 i
y = −L −1
p2 (p + 1)2
y = −t(1 + 2e−t ) + 2(1 − e−t )

Hence the required solution is

x = 2(1 − e−t − te−t ); y = −t(1 + 2e−t ) + 2(1 − e−t ).

Example 19.4.5. Solve (D2 − 3)x − 4y = 0; x + (D2 + 1)y = 0 t > 0 when t = 0, x = y = Dy = 0


and Dx = 2.

Solution. Given
x00 − 3x − 4y = 0; x + y00 + y = 0.

Taking Laplace transform on both sides, we get

p2 L [x] − px(0) − x0 (0) − 3L [x] − 4L [y] = 0; L [x] + p2 L [y00 ] − py(0) − y0 (0) + L [y] = 0.

Given initial conditions are x(0) = 0, y(0) = 0, y0 (0) = 0 and x0 (0) = 2.

p2 L [x] − 2 − 3L [x] − L [y] = 0; L [x] + p2 L [y00 ] + L [y] = 0

Transform Techniques B.Sc.(Mathematics)-IDE(UNOM)-II Year-IV Sem


19.4. Worked out Problems 475

(p − 3)L [x] − 4L [y] = 2

L [x] + (p2 + 1)L [y] = 0

From these two equations, we get

2(p2 + 1)
L [x] =
(p2 − 1)2
1 1
= +
(p − 1) 2 (p + 1)2
h 1 1 i
x = L −1 +
(p − 1)2 (p + 1)2
h1i h1i
x = et L −1 2 + e−t L −1 2
p p
x = et t + e−t t

x = 2t cos ht

dx d2 x
= 2[t sinh t + cosh t] and = 2[t cosh t + sinh t + sinh t] = 2[t cosh t + 2 sinh t].
dt dt2
d2 x
Using the values of x, and dt2
in x00 − 3x − 4y = 0

2t cosh t + 4 sinh t − 6t cosh t − 4y = 0

−4y = 4t cosh t − 4 sinh t

y = sinh t − t cosh t.

Hence the required solution is

x = 2 cosh t; y = sinh t − t cosh t.

Example 19.4.6. Solve (D − 2)x − (D + 1)y = 6e3t ; (2D − 3)x + (D − 3)y = 6e3t if x = 3, y = 0
when t = 0.

B.Sc.(Mathematics)-IDE(UNOM)-II Year-IV Sem Transform Techniques


476 19.4. Worked out Problems

Solution. Given
x0 − 2x − y0 − y = 6e3t ; 2x0 − 3x + y0 − 3y = 6e3t .

Taking Laplace transform on both sides, we get

6
pL [x] − x(0) − 2L [x] − pL [y] + y(0) − L [y] =
p−3

and
6
2pL [x] − 2x(0) − 3L [x] + pL [y] − y(0) − 3L [y] = .
p−3
Given initial conditions are x(0) = 3, y(0) = 0,

6 6
pL [x] − 3 − 2L [x] − pL [y] − L [y] = ; 2pL [x] − 6 − 3L [x] + pL [y] − 3L [y] = .
p−3 p−3

6 6 6p − 12
(p − 3)L [x] − (p + 1)L [y] = 3 + ; (2p − 3)L [x] + (p − 3)L [y] = 6 + = .
p−3 p−3 p−3
3p − 3
(p − 3)L [x] − (p + 1)L [y] = (19.4.5)
p−3
6p − 12
(2p − 3)L [x] + (p − 3)L [y] = (19.4.6)
p−3
From (19.4.5) and (19.4.6), we get

3p2 − 6p − 1 −3p + 5
L [x] = ; L [y] = .
(p − 3)(p − 1)2 (p − 3)(p − 1)2

1 2 2
L [x] = + 2
+
p − 1 (p − 1) p−3
h 1 2 2 i
x = L −1 + +
p − 1 (p − 1)2 p − 3
x = et + 2tet + 2e3t

Transform Techniques B.Sc.(Mathematics)-IDE(UNOM)-II Year-IV Sem


19.4. Worked out Problems 477
1 1 1
L [y] = − −
p − 1 (p − 1)2 p − 3
h 1 1 1 i
y = L −1 − −
p − 1 (p − 1)2 p − 3
y = et − tet − e3t

The required solution is


x = et + 2tet + 2e3t ; y = et − tet − e3t .

Example 19.4.7. Solve (D2 + 2)x − Dy = 1; Dx + (D2 + 2)y = 0 if x = 0 = Dx = y = Dy, when


t = 0.

Solution. Given
x00 + 2x − y0 = 1; x0 + y00 + 2y = 0.

Taking Laplace transform on both sides, we get

1
p2 L [x]−px(0)−x0 (0)+2L [x]−pL [y]+y(0) = ; pL [x]−x(0)+[p2 L [y]−py(0)−y0 (0)]+2L [y] = 0.
p

Given initial conditions are x(0) = 3, y(0) = 0, x0 (0) = y0 (0)

1
p2 L [x] + 2L [x] − pL [y] = ; pL [x] + p2 L [y] + 2L [y] = 0.
p

1
(p2 + 2)L [x] − pL [y] = ; pL [x] + (p2 + 2)L [y] = 0.
p
From these equations, we have

p2 + 2 1
L [x] = ; L [y] =
p(p2 + 1)(p2 + 4) (p2 + 1)(p2 + 4)

B.Sc.(Mathematics)-IDE(UNOM)-II Year-IV Sem Transform Techniques


478 19.5. Exercises

1 p p
L [x] = − −
2p 3(p2 + 1) 6(p2 + 4)
h1 p p i
x = L −1 − −
2p 3(p2 + 1) 6(p2 + 4)
1 1 1
x = − cos t − cos 2t
2 3 6

−1 h 1 1 i
L [y] = −
3 p2 + 1 p2 + 4
1 h 1 1 i
y = − L −1 2 − 2
3 p +1 p +4
1 1
y = − sin t + sin 2t.
3 6

The required solution is

1 1 1 1 1
x= − cos t − cos 2t; y = − sin t + sin 2t.
2 3 6 3 6

19.5 Exercises

Solve the following differential equations by using Laplace transform:


1. y000 + 2y00 − y0 − 2y = 0 with y(0) = y0 (0) = y00 (0) = 6;
2. y000 − 2y00 + 5y0 = 0 given that y(0) = y0 (0) = 0 and y00 (0) = 1;
d2 y
3. dt2
+ y = t, y = 1, dy
dt
= −2 when t = 0;
4. (D2 + 6D + 9)y = sin t, y(0) = 1, y0 (0) = 0;
d2 x
5. (D2 − 2D + 1)x = et , x = 2, Dx = −1 when t = 0. (or) dt2
− 2 dx
dt
+ x = et , given that x(0) = 2,
x0 (0) = −1 when t = 0;
6. (D2 + 4D + 3)y = e−t , given y = 1, dy
dt
= 1 at t = 0;
d2 x
7. + 9x = sin t, given that x0 (0) = 0;
dt2

8. (D2 + 9)y = cos 2t, y(0) = 1, y π2 = −1;


 

9. (D2 + 9)y = 18t, y(0) = 0, y π2 = 0, y0 (0) = A;


 

Transform Techniques B.Sc.(Mathematics)-IDE(UNOM)-II Year-IV Sem


19.5. Exercises 479

10. y00 − 3y0 + 2y = 4t + e3t when y(0) = 1, and y0 (0) = −1;


11. (D2 + 2D + 1)y = 3te−t , y = 4, Dy = 2 at t = 0;
d3 y 2 d2 y
12. dt3
− 3 ddt2y + 3 dy
dt
− y = t2 et , given that y = 1, dy
dt
= 0, dt2
= −2 at t = 0;
13. (D3 + 1)y = 1, y = Dy = D2 y = 0, when t = 0;
14. (D4 − 1)y = 1, y = Dy = D2 y = D3 y = 0, when t = 0;
15. (D4 + 2D2 + 1)y = 0, y = 0, y0 = 0, y00 = 2, y000 = −3 at t = 0;
16. y00 + ty0 − y = 0, if y(0) = 0, y0 (0) = 1;
17. y00 − ty0 + y = 0, if y(0) = 1, y0 (0) = 2.

Answers h i
1. y = et − 3e−t + 2e−2t ; 2. y = 1
10
et (sin 2t − 2 cos 2t) + 2 ;
 
3. y = t + cos t − 3 sin t; 4. y = 50
1
53e−3t + 155e−3t + 4 sin t − 3 cos t ;
2
5. x = 2et − 3tet + t2 et ; 6. y = 74 e−t + 2t e−t − 43 e−3t ;
7. y = cos 3t + 1
24
(3 sin t − sin 3t; 8. y = 15 (cos 2t + 4 sin 3t + 4 cos 3t);
9. y = 2t + π sin 3t; 10. y = 2t + 3 + 21 (e3t−e ) − 2e2t ;
t

3   2

t5
11. y = e−t t2 + 6t + 4 ; 12. y = et 1 − t − t2 + 60 ;
√ 
e−t t
13. y = 1 + 3
− 23 e 2 cos 2
3
t; 14. y = −1 + 12 (cos t + cosh t);
15. y = t(sin t + cos t); 16. y = t;
17. y = 1 = 2t.

B.Sc.(Mathematics)-IDE(UNOM)-II Year-IV Sem Transform Techniques


480
UNIT-V

Lesson 20

APPLICATION OF LAPLACE
TRANSFORMS TO PARTIAL
DIFFERENTIAL EQUATIONS
Learning Objectives
Upon completion of this lesson, students will be able to

• solve partial differential equations by using Laplace transform

• solve initial and boundary value problem by using Laplace transform.

20.1 Introduction

T
he Laplace transform is especially well - suited for solving initial boundary value problem
for which some auxiliary conditions are prescribed at t = 0. Such problems arise naturally
in the solution of the heat equation or wave equation. We will now prove some theorems.

Theorem 20.1.1. If Y = Y(x, t) then


(i) L ∂Y ∂ Y
   2 
∂t
= py(x, p) − Y(x, 0); (ii) L ∂t2
= p2 y(x, p) − pY(x, 0) − Y 0 (x, 0);
 2  d2 y
(iii) L ∂Y ∂ Y
  dy
∂x
= dx
; (iv) L ∂x2
= dx2 .

B.Sc.(Mathematics)-IDE(UNOM)-II Year-IV Sem 481 Transform Techniques


482 20.1. Introduction

∂Y
 
where L [Y(x, t)] = y(x, p), Yt (x, 0) = ∂t t=0
.

Proof. (i)

 ∂Y  Z
∂Y ∞
L = e−pt dt
∂t 0 ∂t
∂Y
Z a
= lim e−pt dt
a→∞ 0 ∂t
Z a i
= lim [e −pt
Y(x, t)at=0 +p e−pt Y(x, t)dt
a→∞ 0
Z ∞
= −Y(x, 0) + p e−pt Ydt
0

= pL [Y(x, t)] − Y(x, 0)


 ∂Y 
L = py(x, p) − Y(x, 0).
∂t

∂Y
(ii) Let V = ∂t
= Y, then

 ∂2 Y   ∂V 
L = L
∂t2 ∂t
= pV(x, p) − V(x, 0), by (i)

= pL [V] − Yt (x, 0)
 ∂Y 
= pL − Yt (x, 0)
∂t
= p[py(x, p) − Y(x, 0)] − Yt (x, 0), by (i)
 ∂2 Y 
L = p2 y(x, p) − pY(x, 0) − Yt (x, 0).
∂t2

(iii)
 ∂Y 
−pt ∂Y
Z ∞ Z ∞
d d dy
L = e dt = e−pt Ydt = L [Y] = .
∂x 0 ∂t dx 0 dx dx

Transform Techniques B.Sc.(Mathematics)-IDE(UNOM)-II Year-IV Sem


20.2. Worked out Problems 483

(iv)

 ∂2 Y   ∂U  ∂Y
L = L , where U =
∂x2 ∂x ∂x
d
=
L [U]
dx
d  ∂Y 
= L
dx ∂x
d  dy 
= by (iii)
dx dx
 ∂2 Y  d2 y
L = .
∂x2 dx2

20.2 Worked out Problems

∂Y
Example 20.2.1. Solve ∂x
= 2 ∂Y
∂t
+ Y, Y(x, 0) = 6e−3x which is bounded for x > 0, t > 0.

Solution. Let L [Y] = y. Given

 ∂Y   ∂Y 
L = L 2 +Y
∂x ∂t
dy
= 2[py(x, p) − Y(x, 0)] + y
dx
= 2py(x, p) − 12e−3x + y
dy
− (1 + 2p)y = −12e−3x
dx

This is the linear differential equation.

Z   Z 
IntegratingFactor = exp Pdx = exp − (1 + 2p)dx = exp(−(1 + 2p)x).

i.e., I.F = e−(1+2p)x

B.Sc.(Mathematics)-IDE(UNOM)-II Year-IV Sem Transform Techniques


484 20.2. Worked out Problems

The solution is
Z
ye −(1+2p)x
= −12e−3x .e−(1+2p)x dx + c
Z
= −12 e−(2p+4)x dx + c
e−(2p+4)x
= −12 +c
−(2p + 4)
e−(2p+4)x
= 6 +c
p+2
6e−3x
y = + ce(1+2p)x
p+2

Since Y(x, t) is bounded ∀x and so y(x, p) is bounded ∀x and p > 0.


c = 0, for any y = finite quantity as x → ∞.

6e−3x
∴y=
p+2

6e−3x h 1 i
i.e., L [Y] = ⇒ Y = 6e−3x L −1
p+2 p+2
Y = 6e−3x e−2t = 6e−(2t+3x) .

∂2 U ∂2 U ∂U
Example 20.2.2. Solve ∂x2
− ∂t2
= xt, where U = 0 = ∂t
at t = 0.

Solution.
∂2 U ∂2 U
− 2 = xt
∂x2 ∂t

Taking Laplace transform on both sides, we get

h ∂2 U i h ∂2 U i
L −L = xL [t]
∂x2 ∂t2
d2 u x
2
− [p2 u − pU(x, 0) − Ut (x, 0)] = 2
dx p

Transform Techniques B.Sc.(Mathematics)-IDE(UNOM)-II Year-IV Sem


20.2. Worked out Problems 485
∂U
Given U = 0 = ∂t
at t = 0.
d2 u x
2
− p2 u = 2
dx p
The solution is u = C.F + P.I

C.F = Ae−xp + Be xp

u = Ae−xp + Be xp + P.I (20.2.1)

let us assume that P.I = u p = A + Bx

u0p = B u00p = 0.

d2 u x
2
− p2 u = 2
dx p
x
0 − p2 (A + Bx) = 2
p

Equating likewise coefficients, we get

1
A = 0 and B=− .
p4

x
∴ P.I = u p = −
p4

x
∴ u = Ae−px + Be px − (20.2.2)
p4

U = 0 for t = 0 ⇒ U = 0, ∀x, p
⇒u=0

∴ from (20.2.2),we have B = 0 as x → ∞

x
∴ u = Ae−xp −
p4

B.Sc.(Mathematics)-IDE(UNOM)-II Year-IV Sem Transform Techniques


486 20.2. Worked out Problems

Again applying u = 0 for x = 0, we get A = 0.

x −1 1
h i xt3
∴u=− ⇒ U = −xL = −
p4 p4 3!

xt3
U=− .
6
∂Y
= 2 ∂∂xY2 , where Y(0, t) = 0 = Y(5, t) and Y(x, 0) = 10 sin 4πx.
2
Example 20.2.3. Solve ∂t

Solution.
∂Y ∂2 Y
=2 2
∂t ∂x

Taking Laplace transform on both sides, we get

h ∂Y i ∂2 Y i
L = 2L [
∂t ∂x2
d2 y
py − Y(x, 0) = 2
dx2
d2 y
2 = py − 10 sin 4πx
dx2

d2 y p
− y = −5 sin 4πx (20.2.3)
dx2 2
The general solution is
y = C.F + P.I
√p √p
C.F = Ae x 2 + Be−x 2

To find Particular solution: Take the trail solution is

y p = A sin 4πx + B cos 4πx

Transform Techniques B.Sc.(Mathematics)-IDE(UNOM)-II Year-IV Sem


20.2. Worked out Problems 487

y0p = 4πA cos 4πx − 4πB sin 4πx

y00p = −16π2 A sin 4πx − 16π2 cos 4πx

Using these in (20.2.3), we get

p
−16π2 A sin 4πx − 16π2 cos 4πx − (A sin 4πx + B cos 4πx) = −5 sin 4πx
2

Equating likewise coefficients, we get

10
A= and B = 0.
32π2 + p

10
∴ yp = sin 4πx
32π2 + p
∴ the general solution is

√p √p 10
y = Ae x 2 + Be−x 2 + sin 4πx.
32π2 + p

Given y(0, p) = 0 and y(5, p) = 0

∴ A+B = 0
√p √p 10 sin 20π
Ae5 2 + Be−5 2 + = 0
32π2 + p

Solving these two equations, we get A = 0 = B.

10 h 10 i
∴y= sin 4πx ⇒ Y = L −1
sin 4πx
32π2 + p 32π2 + p

2
Y = 10e−32π t sin 4πx.

∂Y ∂2 Y
Example 20.2.4. Solve ∂t
= ∂x2
, x > 0, t > 0 where Y(0, t) = 1, Y(x, 0) = 0.

B.Sc.(Mathematics)-IDE(UNOM)-II Year-IV Sem Transform Techniques


488 20.2. Worked out Problems

Solution.
∂Y ∂2 Y
= 2
∂t ∂x

Taking Laplace transform on both sides, we get

h ∂Y i ∂2 Y i
L = 2L [
∂t ∂x2
d2 y
py − Y(x, 0) = 2
dx2
d2 y
= py, since Y(x, 0) = 0.
dx2

d2 y
− py = 0 (20.2.4)
dx2
The general solution is
√ √
y = Ae x p
+ Be−x p
(20.2.5)

Given Y(0, t) = 1 ⇒ L [Y(0, t)] = L [1] ⇒ y(0, p) = 1


p
and Y(x, 0) = 0 ⇒ y(x, 0) = 0.
Y is bounded ⇒ L [Y] = y is bounded
⇒ y is finite as x → ∞.
∴ (20.2.5) ⇒ A = 0.

− √xp
∴ y = Be .

Since y(0, p) = 1p , we have B = 1p .


1 −x √ p
y= e .
p
1 −x √ p h1 √ i  x 
L[Y] = e ⇒ Y = L −1 e−x p = er f c √ .
p p 2 t
Physical Interpretation. This represents the temperature at any point of a semi - infinite solid
x > 0 whose face x = 0 is kept at unit temperature and whose initial temperature is zero.

∂Y ∂Y
Example 20.2.5. Solve ∂x
− ∂t
= 1 − e−t , 0 < x < 1, t > 0 given that Y(x, 0) = x.

Transform Techniques B.Sc.(Mathematics)-IDE(UNOM)-II Year-IV Sem


20.2. Worked out Problems 489

Solution.
∂Y ∂Y
− = 1 − e−t
∂x ∂t

Taking Laplace transform on both sides, we get

h ∂Y
∂Y i
L = L [1 − e−t ].

∂x ∂t
dy 1 1
− [py − Y(x, 0)] = −
dx p p+1
dy 1 1
− py + x = − , since Y(x, 0) = x
dx p p+1
dy 1 1
− py = − −x
dx p p+1

This is the linear differential equation. The general solution is

Z  Z Z 
y exp Pdx = Q exp pdx dx + c.

Z  Z   
I.F = exp Pdx = exp −pdx = exp − px .

i.e., I.F = e−px .

Z h
1 i
ye −px
= − x e−px dx + c
p(p + 1)
hx 1 1i
= − 2 + 2 e−px + c
p p (p + 1) p
x 1
y = + + ce px .
p p(p + 1)

Y is bounded ⇒ L [Y] = y is bounded


⇒ y is finite as x → ∞ ⇒ c = 0.

x 1
∴y= + .
p p(p + 1)

B.Sc.(Mathematics)-IDE(UNOM)-II Year-IV Sem Transform Techniques


490 20.2. Worked out Problems

x 1 hx 1 1 i
L [Y] = + ⇒ Y = L −1 + − = x + 1 − e−t .
p p(p + 1) p p p+1
∂Y
= 2 ∂∂xY2 , where Y(0, t) = 0 = Y(5, t) and Y(x, 0) = 10 sin 4πx −
2
Example 20.2.6. Solve ∂t

5 sin 6πx.

Solution.
∂Y ∂2 Y
=2 2
∂t ∂x

Taking Laplace transform on both sides, we get

h ∂Y i ∂2 Y i
L = 2L [
∂t ∂x2
d2 y
py − Y(x, 0) = 2
dx2
d2 y
2 = py − 10 sin 4πx + 5 sin 6πx
dx2

d2 y p 5
2
− y = −5 sin 4πx + sin 6πx (20.2.6)
dx 2 2
The general solution is
y = C.F + P.I
√p √p
C.F = Ae x 2 + Be−x 2

To find Particular solution: Take the trail solution is

y p1 = A sin 4πx + B cos 4πx

y0p1 = 4πA cos 4πx − 4πB sin 4πx

y00p1 = −16π2 A sin 4πx − 16π2 cos 4πx

Transform Techniques B.Sc.(Mathematics)-IDE(UNOM)-II Year-IV Sem


20.2. Worked out Problems 491

Using these equations in (20.2.6)

p
−16π2 A sin 4πx − 16π2 cos 4πx − [A sin 4πx + B cos 4πx] = −5 sin 4πx
2

Equating likewise coefficients, we get

10 sin 4πx
A= .
32π2 + p

Take the trail solution is


y p2 = C sin 6πx + D cos 6πx

y0p2 = 6πC cos 6πx − 6πD sin 6πx

y00p2 = −36π2C sin 6πx − 36π2 D cos 6πx

Using these in (20.2.6), we get

p 5
−36π2C sin 6πx − 36π2 D cos 6πx − (C sin 6πx + D cos 6πx) = sin 6πx
2 2

Equating likewise coefficients, we get

−5
D= and C = 0.
72π2 + p

10 5
∴ y p = y p1 + y p2 = sin 4πx − sin 6πx
32π + p
2 72π + p
2

∴ the general solution is

√p √p 10 5
y = Ae x 2 + Be−x 2 + sin 4πx − sin 6πx.
32π + p
2 72π + p
2

B.Sc.(Mathematics)-IDE(UNOM)-II Year-IV Sem Transform Techniques


492 20.2. Worked out Problems

Given y(0, p) = 0 and y(5, p) = 0

∴ A+B = 0
√p √p 10 sin 20π 5
Ae5 2 + Be−5 2 + − sin 30π = 0
32π + p
2 72π + p
2

Solving these two equations, we get A = 0 = B.

10 5 h 10 5 i
∴y= sin 4πx − sin 6πx ⇒ Y = L −1
sin 4πx − sin 6πx
32π2 + p 72π2 + p 32π2 + p 72π2 + p

2 2
Y = 10e−32π t sin 4πx − 5e−72π t sin 6πx.
∂y ∂2 y
 ∂y 
π
 
Example 20.2.7. Solve ∂t
= ∂x 2 , where Y 2 , t = 0, ∂x
= 0 and y(x, 0) = cos 3x.
x=0

Solution.
∂y ∂2 y
=
∂t ∂x2

Taking Laplace transform on both sides, we get

h ∂y i h ∂2 y i
L = L
∂t ∂x2
d2 y
py(x, p) − y(x, 0) =
dx2
d2 y
= py − cos 3x
dx2

d2 y
− py = − cos 3x (20.2.7)
dx2
The general solution is
y = C.F + P.I
√ √
C.F = Ae x p
+ Be−x p

To find Particular solution: Take the trail solution is

y p = A sin 3x + B cos 3x

Transform Techniques B.Sc.(Mathematics)-IDE(UNOM)-II Year-IV Sem


20.2. Worked out Problems 493

y0p = 3A cos 3x − 3B sin 3x

y00p = −9A sin 3x − 9B cos 3x

Using these in (20.2.7), we get

−9A sin 3x − 9B cos 3x − p(A sin 3x + B cos 3x) = − cos 3x

Equating likewise coefficients, we get

1
A = 0. B=
9+ p

1
∴ yp = cos 3x
9+ p
∴ the general solution is
√ √ 1
y = Ae x p
+ Be−x p
+ cos 3x. (20.2.8)
9+ p

Given y π2 , t) = 0 ⇒ y π2 , p) = 0
  

From (20.2.8),

√ √
p π2 p π2
Ae + Be− = 0

Also,
 ∂y  dy
=0⇒ = 0,
∂x dx
when x = 0.

Differentiating (20.2.8) with respect to x, we have

dy √ √ √ √ 3
= A pe x p − pBe−x p + sin 3x
dx 9+ p
√ √ dy
0 = A p − B p, since = 0, when x = 0
dx

B.Sc.(Mathematics)-IDE(UNOM)-II Year-IV Sem Transform Techniques


494 20.2. Worked out Problems

Solving these two equations, we get A = 0 = B.

1 h 1 i
∴y= cos 3x ⇒ Y = L −1 cos 3x
9+ p 9+ p

Y = e−9t cos 3x.

∂y ∂2 y
Example 20.2.8. Solve ∂t
= ∂x2
, y(x, 0) = 3 sin 2πx, y(0, t) = 0 = y(1, t).

Solution.
∂y ∂2 y
=
∂t ∂x2

Taking Laplace transform on both sides, we get

h ∂y i h ∂2 y i
L = L
∂t ∂x2
2
d y
py(x, p) − y(x, 0) =
dx2
d2 y
= py − 3 sin 2πx
dx2

d2 y
− py = −3 sin 2πx (20.2.9)
dx2
The general solution is
y = C.F + P.I
√ √
C.F = Ae x p
+ Be−x p

To find Particular solution: Take the trail solution is

y p = A sin 2πx + B cos 2πx

y0p = 2πA cos 2πx − 2πB sin 2πx

y00p = −4π2 A sin 2πx − 4π2 B cos 2πx

Transform Techniques B.Sc.(Mathematics)-IDE(UNOM)-II Year-IV Sem


20.2. Worked out Problems 495

Using these in (20.2.9), we get

−4π2 A sin 2πx − 4π2 B cos 2πx − p(A sin 2πx + B cos 2πx) = −3 sin 2πx

Equating likewise coefficients, we get

3
A= , B=0
4π2 +p

3
∴ yp = sin 2πx
4π2 +p
∴ the general solution is

√ √ 3
y = Ae x p
+ Be−x p
+ sin 2πx. (20.2.10)
4π2 +p

Given y(0, t) = 0 ⇒ y(0, p) = 0

y(1, t) = 0 ⇒ y(1, p) = 0.

From (20.2.9),

∴ A+B = 0
√ √
Ae p
+ Be− p
= 0

Solving these two equations, we get A = 0 = B.

3 h 3 i
∴y= sin 2πx ⇒ Y = L −1
sin 2πx
4π2 + p 4π2 + p

2
Y = 3e−4π t sin 2πx.

B.Sc.(Mathematics)-IDE(UNOM)-II Year-IV Sem Transform Techniques


496 20.3. Application of Laplace Transform to Initial and Boundary Value Problems

20.3 Application of Laplace Transform to Initial and


Boundary Value Problems

Partial differential equations are frequently occurring in solving many problems in Physics and
Engineering. If the partial differential equation is satisfied within a certain region and certain
condition are satisfied on the boundary region, the problem is said to be a boundary value problem.
Laplace transform is used to solve such type of problems.
Heat Conduction Equation. The heat flow in a homogeneous body is given by the heat equation

∂u  ∂2 u ∂2 u ∂2 u 
= c2 + + ,
∂t ∂x2 ∂y2 ∂z2

c2 = k

, where u(x, y, z, t) is the temperature of the body, k is the thermal conductivity, σ is the
specific heat, ρ the density of the material of the body and c2 the constant is called the diffusivity
of the body.
If we consider the heat flow in along thin bar on wire of constant cross section and homogeneous
material which along x - axis (is insulated so that the heat flows in x - direction only u depends
only) on x and t. The heat equation becomes

∂u ∂2 u
= c2 2 .
∂t ∂x

This is known as one dimensional heat equation.

Wave Equation. The vibration of an elastic string are governed by the one dimensional wave
equation
∂2 u 2∂ u
2
= c ,
∂t2 ∂x2
where c2 = Tρ . Here u(x, t) is the deflection of the string stretched between the fixed points (0, 0)
and (l, 0).

Laplace Equation. When the temperatures in a homogeneous material is in steady state then the

Transform Techniques B.Sc.(Mathematics)-IDE(UNOM)-II Year-IV Sem


20.4. Worked out Problems 497

heat conduction equation becomes

∂2 u ∂2 u ∂2 u
+ + = 0.
∂x2 ∂y2 ∂z2

This is known as Laplace equation in cartesian coordinates.

20.4 Worked out Problems

∂u
Example 20.4.1. Solve ∂t
+ x ∂u
∂x
= x, x > 0, t > 0, u(x, 0) = 0 = u(0, t).

Solution. Given
∂u ∂u
+ x = x.
∂t ∂x
Taking Laplace transform on both sides, we get

h ∂u i h ∂u i
L + xL= xL [1]
∂t ∂x
du x
pu(x, p) − u(x, 0) + x =
dx p
du x
pu(x, p) + x = , since u(x, 0) = 0
dx p
du p 1
+ u(x, p) =
dx x p

Z p   
I.F = exp dx = exp p log x = exp(log x p ) = x p .
x
The general solution is
Z
1 p
ux p
= x dx + c
p
1 x p+1
= +c
pp+1

Here
u(0, t) = 0 ⇒ L [u(0, t)] = u(0, p) = 0.

B.Sc.(Mathematics)-IDE(UNOM)-II Year-IV Sem Transform Techniques


498 20.4. Worked out Problems

Using this in the above equation, we get c = 0.

x 1 1 
u= =x −
p(p + 1) p p+1

1 1 
∴ u = xL −1 − = x(1 − e−t ).
p p+1
Example 20.4.2. Solve x ∂u
∂t
+ ∂u
∂x
= x, u(x, 0) = 0 = u(0, t).

Solution. Given
∂u ∂u
x + = x.
∂t ∂x
Taking Laplace transform on both sides, we get

h ∂u i h ∂u i
xL +L = xL [1]
∂t ∂x
du x
x[pu(x, p) − u(x, 0)] + =
dx p
du x
xpu(x, p) + = , since u(x, 0) = 0
dx p
du x
+ xpu(x, p) =
dx p

Z   px2 
I.F = exp pxdx = exp .
2
px2
i.e., I.F = e 2 .

The general solution is


Z
px2 x px22
ue 2 = e dx + c
p
1 px22
= e +c
p2

Here
u(0, t) = 0 ⇒ L [u(0, t)] = u(0, p) = 0.

Transform Techniques B.Sc.(Mathematics)-IDE(UNOM)-II Year-IV Sem


20.4. Worked out Problems 499

Using this in the above equation, we get c = − p12 .

px2 1 px22 1 1 px2 


ue 2 = e − = 1 − e 2 .
p2 p2 p2

Taking inverse Laplace transform,

 x2   x2 
∴u=t− t− H t− .
2 2

∂2 u
= a2 ∂∂xu2 , (t > 0, x > 0) where
2
Example 20.4.3. Solve the boundary value problem ∂t2

u(x, 0) = 0, x ≥ 0, ut (x, 0) = 0, x > 0, u(0, t) = t, lim x→∞ u(x, t) = 0, t ≥ 0.

Solution. Given
∂2 u 2∂ u
2
= a .
∂t2 ∂x2
Taking Laplace transform on both sides, we get

h ∂2 u i h ∂2 u i
L = a2 L
∂t2 ∂x2
2
d u
p2 u(x, p) − pu(x, 0) − ut (x, 0) = a2
dx2
d2 u
p2 u(x, p) = a2 2 , since u(x, 0) = 0 and ut (x, 0) = 0
dx
d 2 u p2
− u=0
dx2 a2

The general solution is

px px
u(x, p) = Ae a + Be− a

Here
1
u(0, t) = t ⇒ L [u(0, t)] = L [t] ⇒ u(0, p) = .
p2
lim u(x, t) = 0 ⇒ lim L [u(x, t)] = 0 ⇒ lim u(x, p) = 0
x→∞ x→ x→∞

B.Sc.(Mathematics)-IDE(UNOM)-II Year-IV Sem Transform Techniques


500 20.4. Worked out Problems

As x → ∞, u(x, p) → 0, we have A = 0.

−px
u(x, p) = Be p

1
u(0, p) = B ⇒ = B.
p2
1 −px
∴ u(x, p) = e a .
p2
Taking inverse Laplace transform,

h 1 px i
∴ u(x, t) = L −1 e− a ,
p2

t − a , t>

 x x
x  x

 a

u(x, t) =  = t− H t−

 a a
0, t < x



a

This is the required solution.

∂2 u
= a2 ∂∂xu2 , (t > 0, x > 0) where
2
Example 20.4.4. Solve the boundary value problem ∂t2

u(x, 0) = 0, x ≥ 0, ut (x, 0) = 0, x > 0, u(0, t) = F(t), lim x→∞ u(x, t) = 0, t ≥ 0.

Solution. Given
∂2 u 2∂ u
2
= a .
∂t2 ∂x2
Taking Laplace transform on both sides, we get

h ∂2 u i h ∂2 u i
L = a2 L
∂t2 ∂x2
2
d u
p2 u(x, p) − pu(x, 0) − ut (x, 0) = a2
dx2
d2 u
p2 u(x, p) = a2 2 , since u(x, 0) = 0 and ut (x, 0) = 0
dx
d 2 u p2
− u=0
dx2 a2

Transform Techniques B.Sc.(Mathematics)-IDE(UNOM)-II Year-IV Sem


20.4. Worked out Problems 501

The general solution is

px px
u(x, p) = Ae a + Be− a

Here Z ∞
u(0, t) = F(t) ⇒ L [u(0, t)] = F(t)e−pt dt = F(p).
0

lim u(x, t) = 0 ⇒ lim L [u(x, t)] = 0 ⇒ lim u(x, p) = 0


x→∞ x→∞ x→∞

As x → ∞, u(x, p) → 0, we have A = 0.

−px
u(x, p) = Be p

u(0, p) = B ⇒ F(p) = B.
−px
∴ u(x, p) = F(p)e a .

Taking inverse Laplace transform,

h px i  x  x
∴ u(x, t) = L −1 F(p)e− a = F t − H t − .
a a

in terms of Havisides unit step function.


 
F t − a , t >


 x x
 a
u(x, t) = 


t< x

0,


a

This is the required solution.

∂2 u
= a2 ∂∂xu2 − g, (t > 0, x > 0) where
2
Example 20.4.5. Solve the boundary value problem ∂t2

u(x, 0) = 0 = ut (x, 0), x ≥ 0, u(0, t) = 0, lim x→∞ ut (x, t) = 0, t ≥ 0.

B.Sc.(Mathematics)-IDE(UNOM)-II Year-IV Sem Transform Techniques


502 20.4. Worked out Problems

Solution. Given partial differential equation is

∂2 u 2∂ u
2
= a − g.
∂t2 ∂x2

Taking Laplace transform on both sides, we get

h ∂2 u i h ∂2 u i
L = a2 L − gL [1]
∂t2 ∂x2
d2 u g
p2 u(x, p) − pu(x, 0) − ut (x, 0) = a2 2 −
dx p
2
d u
p2 u(x, p) = a2 2 , since u(x, 0) = 0 and ut (x, 0) = 0
dx
d 2 u p2 g
− u= 2 .
dx2 a2 a p

The general solution is

px px g
u(x, p) = Ae a + Be− a −
p3

Here
u(0, t) = 0 ⇒ L [u(0, t)] = 0 ⇒ u(0, p) = 0.
du
lim = 0.
p→∞ dx

du
∴ lim = 0 ⇒ A = 0.
p→∞ dx
−px g
u(x, p) = Be p − 3
p
g g
u(0, p) = 0 ⇒ B − 3
= 0 ⇒ B = 3.
p p
g −px g
∴ u(x, p) = e a − .
p3 p2
Taking inverse Laplace transform,

h g −px gi
∴ u(x, t) = L −1 e a −
p3 p2

Transform Techniques B.Sc.(Mathematics)-IDE(UNOM)-II Year-IV Sem


20.4. Worked out Problems 503

 2
,
 1 x x
t − t≥
2

gt


2 a a
u(x, t) = − + g

2 
t< x

0,


a


− 2ag2 (2axt − x2 ),





 x ≤ at


u(x, t) = 







 2
− gt 0,

 x ≥ at
2

This is the required solution.

∂u
= k ∂∂xu2 which tends to zero as x → ∞ and
2
Example 20.4.6. Find the solution of the equation ∂t

which satisfies the conditions u = f (t) when x = 0, t > 0, and u = 0, where x > 0, t = 0.

Solution. Given partial differential equation is

∂u ∂2 u
= k 2.
∂t ∂x

Taking Laplace transform on both sides, we get

h ∂u i h ∂2 u i
L = kL
∂t ∂x2
2
d u
pu(x, p) − u(x, 0) = k
dx2
d2 u
pu(x, p) = k 2 , since u(x, 0) = 0
dx
2
d u p
− u = 0.
dx2 k

The general solution is

√p √p
u(x, p) = Ae k x
+ Be− k x

As x → ∞ we have u → 0. As x → ∞, we have u → 0. ∴ A = 0.
Since u = f (t), we have f (p) = B
√p
∴ u = f (p)e− k x

B.Sc.(Mathematics)-IDE(UNOM)-II Year-IV Sem Transform Techniques


504 20.4. Worked out Problems

Taking inverse Laplace transform,

h √p i 1
Z r+∞ √p
∴ u(x, t) = L −1
f (p)e − kx = e pt f (p)e− k x
d p.
2πi r−i∞

Example 20.4.7. A string is stretched between two fixed points (0, 0) and (c, 0). If it is displaced
into the curve y = b sin πxc and released from rest in that position at time t = 0, find its
 

displacement at any time t > 0 and at any point 0 < x < c.

Solution. The displacement is governed by the equation

∂2 y 2∂ y
2
= a .
∂t2 ∂x2

πx
 
with boundary conditions y(0, t) = 0, y(c, t) = 0 and initial condition y(x, 0) = b sin c
and
yt (x, 0) = 0.

Taking Laplace transform on both sides, we get

h ∂2 y i h ∂2 y i
L = a2 L
∂t2 ∂x2
2
d y
p2 y(x, p) − py(x, 0) − yt (x, 0) = a2
dx2
 πx  d2 y
p2 y(x, p) − pb sin = a2 2 , since yt (x, 0) = 0
2 dx
2
d y p 2
bp  πx 
2
− 2 y = − 2 sin
dx a a c

The general solution is

px px b  πx 
u(x, p) = Ae a + Be− a + π2 a2
p sin
c2
+ p2 c

Here
y(0, t) = 0 ⇒ L [y(0, t)] = 0 ⇒ y(0, p) = 0.

y(c, t) = 0 ⇒ L [y(c, t)] = 0 ⇒ y(c, p) = 0.

Transform Techniques B.Sc.(Mathematics)-IDE(UNOM)-II Year-IV Sem


20.4. Worked out Problems 505

Also
y(0, p) = 0 ⇒ A + B = 0.
pc −pc
y(c, p) = 0 ⇒ Ae a + Be a

Solving these two equations, we get


A = B = 0.
b  πx 
∴ y(x, p) = π2 a2
p sin .
c2
+ p2 c

Taking inverse Laplace transform,

h p i  πx   πx   πat 
∴ y(x, t) = bL −1 π2 a2
sin = b sin cos .
c2
+ p2 c c c

This is the required solution.

Example 20.4.8. An infinite long string having one end at x = 0 is initially at rest on the x−
axis. The end x = 0 undergoes a periodic transverse displacement given by A0 sin(nt), t > 0.
Find the displacement of any point on the string at t > 0.

Solution. The displacement at any point of the string is given by the partial differential equation
is
∂2 u 2∂ u
2
= a
∂t2 ∂x2
with boundary and initial condition u(0, t) = A0 sin(nt), t > 0 u(x, 0) = 0, and ut (x, 0) = 0,
x ≥ 0. The displacement is finite.

B.Sc.(Mathematics)-IDE(UNOM)-II Year-IV Sem Transform Techniques


506 20.4. Worked out Problems

h ∂2 u i h ∂2 u i
L = a2 L
∂t2 ∂x2
2
d u
p2 u(x, p) − pu(x, 0) − ut (x, 0) = a2
dx2
d2 u
p2 u(x, p) = a2 2 , since u(x, 0) = 0 and ut (x, 0) = 0
dx
d 2 u p2
− u = 0.
dx2 a2

The general solution is

px px
u(x, p) = Ae a + Be− a

Here
n
u(0, t) = A0 sin(nt) ⇒ L [u(0, t)] = A0 L [sin nt] ⇒ u(0, p) = A0 .
p2 + n2
u(x, p) is finite for x ≥ 0.
∴ A = 0.
n n
u(0, p) = A0 ⇒ B = A0 2
p2 +n 2 p + n2
n −px
∴ u(x, p) = A0 e a .
p2 +n 2

Taking inverse Laplace transform,

h 1 −px i
∴ u(x, t) = nA0 L −1 e a
p2 + n2

 
, t>
 x x
A sin n t −


 0 a a
u(x, t) = 



t< x

0,


a

This is the required solution.

Transform Techniques B.Sc.(Mathematics)-IDE(UNOM)-II Year-IV Sem


20.5. Exercises 507

20.5 Exercises

∂u ∂2 u
1. Solve the boundary value problem ∂t
= ∂x2
, 0 < x < 1, t > 0 where u(0, t) = 1 = u(1, t),
t > 0 and u(x, 0) = 1 + sin πx, 0 < x < 1.
∂2 u ∂2 u
2. Solve the boundary value problem ∂t2
= ∂x2
, 0 < x < 1, t > 0 where u(0, t) = 0 = u(1, t),
t > 0, u(x, 0) = sin πx and ut (x, 0) = − sin πx, 0 < x < 1.

3. Solve the boundary value problem ∂u ∂2 u π


 
∂t
= 3 ∂x 2 , u 2
, t = 0, u x (0, t) = 0 and u(x, 0) = 30 cos 5x.
∂2 u
= c2 ∂∂xu2 , 0 ≤ x < ∞, 0 ≤ t < ∞, u(0, t) = 0,
2
4. Solve the boundary value problem ∂t2

u(x, 0) = 0 and ut (x, 0) = 1.


∂2 u
= c2 ∂∂xu2 + sin πx, 0 < x < 1, t > 0, u(x, 0) = 0,
2
5. Solve the boundary value problem ∂t2

ut (x, 0) = 0, u(0, t) = 0 and u(1, t) = 0.

Answers
1. u(x, t) = 1 + e−π t sin πx.
2

2. u(x, t) = sin πx cos πt − sinππt .


 

3. u(x, t) = 30e−75t cos 5x.


h   
4. u(x, t) = t − t − cx H t − cx .

5. u(x, t) = 1
π2
(1 − cos πt) sin πx.

B.Sc.(Mathematics)-IDE(UNOM)-II Year-IV Sem Transform Techniques


508
UNIT-V

Lesson 21

APPLICATIONS OF FOURIER
TRANSFORMS TO INITIAL AND
BOUNDARY VALUE PROBLEMS
Learning Objectives
Upon completion of this lesson, students will be able to

• solve initial and boundary value problem by using Fourier transform

• solve heat equation by using finite Fourier transform and also give physical
interpretation of the problem

• apply Laplace transforms to integral equations.

21.1 Introduction

W
e will use Fourier transform to solve initial and boundary value problems. If the Fourier
transform is applied with respect to one of the variables in the partial differential equation,
then we obtain an ordinary differential equation in terms of the other variable. We solve this
differential equation. The solution to the boundary value problem (BVP)is then obtained by taking
inverse Fourier transform.

B.Sc.(Mathematics)-IDE(UNOM)-II Year-IV Sem 509 Transform Techniques


510 21.1. Introduction

Application of Infinite Fourier Transforms


If in the given problem, one of the variables ranges from −∞ to ∞ or 0 to ∞, we solve them
using infinite Fourier transforms on both sides. We then take the corresponding inverse Fourier
transform to the solution obtained.
Application of Infinite Sine or Cosine transform
We will use Sine or Cosine transforms depending on the form of boundary conditions at the lower
limit variable we want to exclude.
∂2 y
If we want to remove the term ∂x2
in a differential equation, we apply sine transformation to
obtain.


∂2 V ∂V ∂V
Z i∞ Z ∞
sin px 2 dx = sin px − p cos px. dx
0 ∂x ∂x 0
0 ∂x
∂V ∂V
Z ∞
= −p cos px. dx, i f → 0 as x → ∞
0 ∂x ∂x
 ∞ Z ∞
= −p V cos px + p V sin pxdx
0
0

= p(V) x=0 − p2 V s .

where V s is the sine transform of V and assuming that V → ∞ as x → ∞.


Again, applying cosine transform, we have


∂2 V ∂V ∂V
Z i∞ Z ∞
cos px 2 dx = cos px + p sin px. dx
0 ∂x ∂x 0
0 ∂x
 ∂V  Z ∞
∂V
= − + p[V sin px] − p 2
V cos px.dx, i f → 0 as x → ∞
∂x x=0 0 ∂x
 ∂V 
= − p2 V c .
∂X x=0

if V → 0 as x → ∞.
Thus, we observe that for the exclusion of ∂∂xV2 from a differential equation, we require (V) x=0 in
2

sine transform and in ∂V


 
∂x
cosine transformation.
x=0
Note.1 A derivative of odd order cannot be excluded from the given differential equation by

Transform Techniques B.Sc.(Mathematics)-IDE(UNOM)-II Year-IV Sem


21.2. Worked out Problems 511

Fourier sine transform or Fourier cosine transform.


2. Complex Fourier transform can be used to exclude a variable in a differential equation which
ranges from −∞ to ∞.

21.2 Worked out Problems

Example 21.2.1. Use the complex form of the Fourier transform to show that V =
R∞
f (u)e− 4t du is the solution of the boundary value problem ∂V = ∂∂xV2 , −∞ < x < ∞,
(x−u) 2
1√
2π t −∞ ∂x

t > 0, V = f (x), when t = 0.

Solution. Given equation


∂V ∂2 V
= 2
∂x ∂x
Taking Fourier transform on both sides, we get


∂V ipx ∂ V ipx
Z Z ∞ 2
1 1
√ e dx = √ e dx
2π −∞ ∂t 2π −∞ ∂x2
d
V(p) = (ip)2 V(p)
dt
d
V(p) = −p2 V(p)
dt

The solution is given by


2
V(p) = Ae−p t

When t = 0, we have A = V(p).


By the definition of Fourier transform
Z ∞
1
V(p) = √ V(u)eipu du.
2π −∞

Z ∞
1
V(p) = √ f (u)eipu du,
2π −∞

B.Sc.(Mathematics)-IDE(UNOM)-II Year-IV Sem Transform Techniques


512 21.2. Worked out Problems

since V = f (x) when t = 0


Z ∞
−p2 t 1
∴ V(p) = e √ f (u)eipu du.
2π −∞

Taking the inverse Fourier transform, we get

Z ∞ h −p2 t Z ∞
1 e i
V(x, t) = √ √ f (u)eipu du e−ipx d p

Z ∞−∞ h 2π −∞
1 2
i
= f (u) e−p t−ip(x−u) d p du
2π −∞
Z ∞ h i2 
1 h Z ∞ −t p+i (x−u)2 − (x−u)2 i
V(x, t) = f (u) e 2t 4t
d p du
2π −∞ −∞

Z ∞ ∞
h 2
i2 
1 hZ −t p+i (x−u) i (x−u)2
V(x, t) = f (u) e 2t
d p e− 4t du
2π −∞ −∞

√ i(x − u)  dy
Put t p+ =y so that d p = √ .
2t t

Z ∞ h 1 Z ∞ 2 i (x−u)2
1
V(x, t) = f (u) √ e−y dy e− 4t du
2π −∞ t −∞
Z ∞ Z ∞
1 1 √ − (x−u) 2
2 √
= f (u) √ πe 4t du, since e−y dy = π
2π −∞ t −∞
Z ∞
1 (x−u)2
∴V = √ f (u)e− 4t du.
2 πt −∞

∂U ∂2 U
Example 21.2.2. Solve ∂t
= ∂x2
, x > 0, t > 0 subject to the conditions
(i) U = 0, when x = 0, t > 0
(ii) U = 1, 0 < x < 1, when t = 0
(iii) U = 0, x ≥ 1.
(iv) U(x, t) is bounded.

Solution. Given that U = 0, when x = 0. Thus we will consider Fourier sine transform. Given

Transform Techniques B.Sc.(Mathematics)-IDE(UNOM)-II Year-IV Sem


21.2. Worked out Problems 513

partial differential equation


∂U ∂2 U
=
∂t ∂x2
Taking Fourier sine transform on both sides, we get
r Z ∞ r Z ∞
2 ∂U 2 ∂2 U
sin pxdx = sin pxdx
π 0 ∂t π 0 ∂x2
r r r Z ∞
2 h ∂U ∂U
Z ∞
2d i∞ 2
U sin pxdx = sin px − p cos pxdx
π dt 0 π ∂x 0 π 0 ∂x
r Z ∞
∂U s 2 ∂U ∂U
= −p cos pxdx, i f → 0 as x → ∞
∂t π 0 ∂x ∂x
r r Z ∞
∂U s 2h i∞ 2
= −p U(x, t) cos px − p 2
U sin pxdx
∂t π 0 π 0
r
2
= − pU(0, t) − p2 U s , assuming U → 0 as x → 0
π
r
d 2
∴ (U s ) + p U s =
2
pU(0, t)
dt π
d
(U s ) + p2 U s = 0, since U = 0 when x = 0, t > 0
dt

The solution of this equations is


2
U s = Ae−p t ,

where A is a constant.
When t = 0, we have the Fourier sine transform of U(x, t) is
r Z ∞
2
U s (p, 0) = U(x, 0) sin pxdx
π 0
r Z 1 r Z ∞
2 2
= 1. sin pxdx + 0. sin pxdx
π 0 π 1
r
2  cos px 1
= −
π p 0
r
2 1 − cos p
=
π p

B.Sc.(Mathematics)-IDE(UNOM)-II Year-IV Sem Transform Techniques


514 21.2. Worked out Problems

Since r
2 1 − cos p
U s (p, 0) = A ⇒ A =
π p
r
2 1 − cos p −p2 t
∴ Us = e .
π p
Applying inverse Fourier sine transform, we have
Z ∞
2 1 − cos p −p2 t
U(x, t) = e sin pxd p
π 0 p

This gives the required solution.

∂U
= 2 ∂∂xU2 , x > 0, t > 0 if U(0, t) = 0, U(x, 0) = e−x , x > 0, U(x, t)
2
Example 21.2.3. Solve ∂t

is bounded where x > 0, t > 0.

Solution. Given that U = 0, when x = 0. Thus we will consider Fourier sine transform. Given
partial differential equation
∂U ∂2 U
=2 2
∂t ∂x
Taking Fourier sine transform on both sides, we get
r Z ∞ r Z ∞
2 ∂U 2 ∂2 U
sin pxdx = 2 sin pxdx
π 0 ∂t π 0 ∂x2
r r r Z ∞
2 h ∂U ∂U
Z ∞
2d i∞ 2
U sin pxdx = 2 sin px − 2p cos pxdx
π dt 0 π ∂x 0 π 0 ∂x
r Z ∞
∂U s 2 ∂U ∂U
= −2p cos pxdx, i f → ∞ as x → ∞
∂t π 0 ∂x ∂x

r r Z ∞
∂U s 2h i∞ 2
= −2p U(x, t) cos px − 2p2 U sin pxdx
∂t π 0 π 0
r
2
= − pU(0, t) − p2 U s , assuming U → 0 as x → 0
π

Transform Techniques B.Sc.(Mathematics)-IDE(UNOM)-II Year-IV Sem


21.2. Worked out Problems 515
r
d 2
∴ (U s ) + 2p U s =
2
pU(0, t)
dt π
d
(U s ) + 2p2 U s = 0, since U = 0 when x = 0, t > 0
dt

The solution of this equations is


2
U s = Ae−2p t ,

where A is a constant.
But U(x, 0) = e−x
r Z ∞
2
U s (p, 0) = U(x, 0) sin pxdx
π 0
r Z ∞
2
= e−x . sin pxdx
π 0
r
2 h e−x iin f ty
= (− sin px − p cos px)
π 1 + p2 0
r
2 p
=
π 1 + p2

Since r
2 p
U s (p, 0) = A ⇒ A =
π 1 + p2
r
2 p 2
∴ Us = e−2p t .
π 1 + p62
Applying inverse Fourier sine transform, we have
Z ∞
2 p 2
U(x, t) = e−2p t sin pxd p
π 0 1+ p2

This gives the required solution.

Example 21.2.4. The temperature U in the semi infinite rod 0 ≤ x ≤ ∞ is determined by the
∂U
= k ∂∂xU2 subject to the conditions (i) U(x, 0) = 0 when t = 0, x ≥ 0; (ii)
2
differential equation ∂t

B.Sc.(Mathematics)-IDE(UNOM)-II Year-IV Sem Transform Techniques


516 21.2. Worked out Problems

∂U
∂x
= −u (a constant) when x = 0, t > 0. Making use of cosine transform show that
Z ∞
2u cos px −kp2 t
U(x, t) = (1 − e )d p
π 0 p2

Solution. Given partial differential equation

∂U ∂2 U
=k 2
∂t ∂x

Taking Fourier cosine transform on both sides, we get


r Z ∞ r Z ∞
2 ∂U 2 ∂2 U
cos pxdx = k cos pxdx
π 0 ∂t π 0 ∂x2
r r r Z ∞
2 h ∂U ∂U
Z ∞
2d i∞ 2
U cos pxdx = k cos px + kp sin pxdx
π dt 0 π ∂x 0 π 0 ∂x
r r Z ∞
∂U c 2  ∂U  2 ∂U ∂U
= −k + kp sin pxdx, i f → ∞ as x → ∞
∂t π ∂x x=0 π 0 ∂x ∂x
r r r Z ∞
∂U c 2  ∂U  2h i∞ 2
= −k + kp U(x, t) sin px − kp 2
U cos pxdx
∂t π ∂x x=0 π 0 π 0
r
2  ∂U 
= ku − kp2 U c , since = −u
π ∂x x=0
r
d 2
∴ (U c ) + kp2 U c = ku
dt π

This is a linear differential equation.

Z   
I.F = exp kp2 dt = exp kp2 t .

2
i.e., I.F = ekp t

Transform Techniques B.Sc.(Mathematics)-IDE(UNOM)-II Year-IV Sem


21.3. Application of Finite Fourier Transform 517

The general solution is

Z r
kp2 t k kp2 t
U ce = e dt + A
u
r
2 u kp2 t
= e +A
π p2

When t = 0, U c = 0 Therefore U = 0, when t = 0.


r r
2 u 2 u
∴0= ⇒A=− .
π p2 π p2
r
kp2 t 1 u kp2 t

∴e Uc = − 1 + e
π p2

r
2 u −kp2

i.e., U c = 1 − e
π p2
Applying inverse Fourier cosine transform, we have
r Z ∞ 2
2u ∞ 1 − e−kp t
Z
2
Uc = U c cos pxd p = cos pxd p
π 0 π 0 p2

This gives the required solution.

21.3 Application of Finite Fourier Transform

If in the given problems,range of one of the variables is finite, we will apply finite Fourier
transform.
Finite Fourier Transforms of the Partial Derivatives
∂U
(i) The finite Fourier sine and cosine transforms of ∂x
where U is a function of x and t for

B.Sc.(Mathematics)-IDE(UNOM)-II Year-IV Sem Transform Techniques


518 21.3. Application of Finite Fourier Transform

0 < x < l, t > 0.

 ∂U  ∂U
Z l
pπx
Fs = sin dx
∂x 0 ∂x l
pπx il pπ l
Z
h pπx
= U(x, t) sin − U cos dx
l 0 l 0 l
 ∂U  pπ
Fs = − Fc (U)
∂x l

Again

 ∂U  ∂U
Z l
pπx
Fc = cos dx
∂x 0 ∂x l
pπx il pπ l
Z
h pπx
= U(x, t) cos + U sin dx
l 0 l 0 l
 ∂U  pπ
Fc = F s (U)
∂x l
= U(0, t) − U(l, t) cos pπ

∂2 U
(ii) The finite Fourier sine and cosine transforms of ∂x2
where U is a function of x and t for
0 < x < l, t > 0

 ∂2 U  Z l
∂2 U pπx
Fs = sin dx
∂x2 0 ∂x
2 l
h ∂U pπx il pπ l ∂U
Z
pπx
= sin − cos dx
∂x l 0 l 0 ∂x l
pπ l ∂U
Z
pπx
= − cos dx
l 0 ∂x l
pπx il pπ l
Z
pπ h pπx
= − U cos + U sin dx
l l 0 l 0 l
pπ h pπ i
= − F s (U) − U(0, t) − U(l, t) cos px
l l
 ∂2 U  p2 π2 pπ
∴ Fs = − 2 F s (U) + [U(0, t) − U(l, t) cos pπ]
∂x2 l l

Transform Techniques B.Sc.(Mathematics)-IDE(UNOM)-II Year-IV Sem


21.4. Worked out Problems 519

and

 ∂2 U  ∂2 U
Z l
pπx
Fc = cos dx
∂x2 0 ∂x
2 l
h ∂U pπx il pπ l ∂U
Z
pπx
= cos + sin dx
∂x l 0 l 0 ∂x l
pπ  ∂U 
= Fs − [U x (0, t) − U x (l, t) cos pπ]
l ∂x
 ∂2 U  p2 π2
∴ Fc = − Fc (U) − [U x (0, t) − U x (l, t) cos pπ].
∂x2 l2

Choice of finite sine or cosine transform The choice of finite or cosine transform is decided by
form of the boundary conditions.

If U(0, t) and U(l, t) are given we apply finite Fourier sine transform.

If U x (0, t) and U x (l, t) are given we apply finite Fourier cosine transform.

21.4 Worked out Problems

∂u ∂2 u
Example 21.4.1. Use finite Fourier transforms to solve the equation ∂t
= ∂x2
, u(0, t) = 0 =
u(4, t), u(x, 0) = 2x where 0 < x < 4, t > 0. Give the physical interpretation of the problem.

Solution. Given equation is


∂u ∂2 u
= 2
∂t ∂x
Taking finite Fourier sine transform with l = 4, we get

4
∂u ∂u
Z Z 4 2
pπx pπx
sin dx = sin dx
0 ∂t 4 0 ∂x
2 4
d p2 π2 pπ
(u s ) = − us + [u(0, t) − u(4, t) cos pπ],
dt 4 4

where u is the finite Fourier sine transform of u. Given u(0, t) = 0 = u(4, t)

d p2 π2
us = − us
dt 16

B.Sc.(Mathematics)-IDE(UNOM)-II Year-IV Sem Transform Techniques


520 21.4. Worked out Problems

The solution is
p2 π2 t
u s = Ae− 16

Since u(x, 0) = 2x, 0 < x < 4 Taking finite Fourier sine transform, we have at t = 0,
Z 4
pπx
us = 2x sin
dx
0 4
h  cos pπx   sin pπx i4
= 2x − pπ 4
− 2 − p2 π24
0
4 16
32
= − cos pπ

32
us = (−1) p+1 .

32
∴ u(x, 0) = u s = A ⇒ A = (−1) p+1 .

32 −p2 π2 t
∴ us = (−1) p+1 e 16

Taking inverse finite Fourier sine transform, we have


2 X 32 −p2 π2 t pπx
u(x, t) = (−1) p+1 e 16 sin
4 p=1 pπ 4

16 X 1 −p2 π2 t pπx
u(x, t) = (−1) p+1 e 16 sin .
π p=1 p 4

This gives the desired solution.

Physical Interpretation Here u(x, t) represents the temperature at any point x at any time to in
the solid bounded by the planes x = 0, and x = 4 (or) a bar on the x− axis with the ends x = 0
and x = 4 whose surfaces insulated latterly. The conditions u(0, t) = 0 and u(4, t) = 0 imply
that the ends are kept at zero temperature. The condition u(x, 0) = 2x implies that the initial
temperature is a function of x.

∂U ∂2 U
Example 21.4.2. Solve the equation ∂t
= ∂x2
, 0 < x < 6, t > 0 subject to the conditions

Transform Techniques B.Sc.(Mathematics)-IDE(UNOM)-II Year-IV Sem


21.4. Worked out Problems 521

U(0, t) = 0, U(6, t) = 0, 
1, 0 < x < 3




U(x, 0) = 


0, 3 < x < 6


Also give the physical interpretation.

Solution. Given equation is


∂U ∂2 U
=
∂t ∂x2
Taking finite Fourier sine transform with l = 6, we get

6
∂U ∂U
Z Z 6 2
pπx pπx
sin dx = sin dx
0 ∂t 6 0 ∂x
2 6
d p2 π2 pπ
(U s ) = − Us + [U(0, t) − U(6, t) cos pπ],
dt 36 6

where U is the finite Fourier sine transform of U. Given U(0, t) = 0 = U(6, t)

d p2 π2
Us = − Us
dt 36

The solution is
p2 π2 t
U s = Ae− 36

Since u(x, 0) = 2x, 0 < x < 4 Now at t = 0,



1, 0 < x < 3




U(x, 0) = 


0, 3 < x < 6


B.Sc.(Mathematics)-IDE(UNOM)-II Year-IV Sem Transform Techniques


522 21.4. Worked out Problems

Taking finite Fourier transform, we have


Z 3
pπx
Us = 1. sin dx
0 6
 cos pπx
6 3

= − pπ 0
6
6 pπ 
Us = 1 − cos
pπ 2

6 pπ 
∴ U(x, 0) = U s = A ⇒ A = 1 − cos .
pπ 2
6 pπ  −p362 π2 t
∴ Us = 1 − cos e
pπ 2
Taking inverse finite Fourier sine transform, we have


2X 6  pπ  −p362 π2 t pπx
U(x, t) = 1 − cos e sin
6 p=1 pπ 2 6

2 X 1 pπ  −p362 π2 t pπx
U(x, t) = 1 − cos e sin
π p=1 p 2 6

This gives the desired solution.

Physical Interpretation

Here U(x, t) represents the temperature at any point x at any time to in the solid bounded by
the planes x = 0, and x = 6 kept at zero temperature which is insulated latterly. Initially the
temperature in the half bar from x = 0 to x = 3 is constant equal to one unit. The half bar from
x = 3 to x = 6 is at zero temperature.

∂U ∂2 U
Example 21.4.3. Solve the boundary value problem ∂t
= ∂x2
, U(0, t) = 1, U(π, t) = 3,
U(x, 0) = 1, 0 < x < π, t > 0.

Solution. Given equation is


∂U ∂2 U
=
∂t ∂x2

Transform Techniques B.Sc.(Mathematics)-IDE(UNOM)-II Year-IV Sem


21.4. Worked out Problems 523

Taking finite Fourier sine transform with l = π, we get

π Z π 2
∂U ∂U
Z
sin pxdx = sin pxdx
0 ∂t 0 ∂x
2

d
(U s ) = −p2 U s + p[U(0, t) − U(π, t) cos pπ],
dt

where U is the finite Fourier sine transform of U. Given U(0, t) = 0 = U(6, t)

d
U s = −p2 U s + p(1 − cos pπ)
dt

U s + p2 U s = p(1 − 3 cos pπ)

This is the linear differential equation,

Z 
I.F = exp p2 dt = exp(p2 t)

2
i.e., I.F = e p t .

Z
p2 t 2
∴ U se = p(1 − cos πp)e p t dt + c
1 − cos 3π p2 t
= e +c
p
1 − cos 3π 2
Us = + ce−p t
p

Given t = 0, U = 1. Taking finite Fourier transform, we have


Z pi
Us = 1. sin pxdx
0
1 − cos pπ
=
p

At t = 0, c = 2 cos pπ
p
1 − cos pπ 2 cos pπ −p2 t
∴ Us = + e
p p

B.Sc.(Mathematics)-IDE(UNOM)-II Year-IV Sem Transform Techniques


524 21.5. Applications of Laplace Transforms to Integral Equations

Taking inverse finite Fourier sine transform, we have

∞ ∞
2 X 1 − 3 cos pπ 2 X 2 cos pπ −p2 t
U(x, t) = sin px + e sin px
π p=1 p π p=1 p
∞ ∞
2 X 1 − 3(−1) p 4 X (−1) p −p2 t
U(x, t) = sin px + e sin px
π p=1 p π p=1 p

This gives the desired solution.

21.5 Applications of Laplace Transforms to Integral


Equations

Definition 21.5.1. An equation of the form, ∀t > 0,


Z b
Y(t) = F(t) + K(u, t)Y(u)du (21.5.1)
a

is called an integral equation where K(u, t) is the kernel of integral equation.

Note.1 If a, b are constants or functions of t then (21.5.1) is called Fredholm integral equation.
2. If a = 0, b = t then (21.5.1) is called Volterra’s equation.

Definition 21.5.2. Integral equation of convolution type:


An equation of the form Z t
Y(t) = F(t) + K(t − u)Y(u)du (21.5.2)
0

is called an integral equation of convolution type.

(21.5.2) can also written as


Y(t) = F(t) + K(t) ∗ Y(t)

The Laplace transform provides a useful technique for the solution of equation (21.5.2) in which
F(t), K(u, t) are known and Y(t) is to be determined.

Transform Techniques B.Sc.(Mathematics)-IDE(UNOM)-II Year-IV Sem


21.6. Exercises 525

Note.1 It is possible to convert every linear differential equation into integral equation. However
not every integral equation can be converted into a differential equation.

Example. Z t
Y(t) = cos t + ln(u + t)Y(u)du.
0

2. Volterra Integral Equation arises from linear differential equation where conditions are
specified at one point.

3. Fredholm Integral Equation arises from linear differential equation where the condition are
specified at two points.

Integro -Differential Equation

Definition 21.5.3. An integro - differential equation is an integral equation in which various


derivatives of the unknown functions Y(t) can also be present.

Example. Z t
Y (t) = Y(t) + sin t +
00
cos(t − u)Y(u)du.
0

Abel’s Integral Equation.

Definition 21.5.4. An equation of the form


Z t
Y(u)
du = G(t)
0 (t − u)n

is called Abel’s integral eqaution where G(t) is known and n is constant such that 0 < n < 1.

21.6 Exercises

∂u
= k ∂∂xu2 , −∞ < x < ∞, t > 0 subject to the conditions u(x, 0) = f (x), ∂u
2
1. Solve ∂t ∂x
and u
tends to zero as x tends to ±∞.

B.Sc.(Mathematics)-IDE(UNOM)-II Year-IV Sem Transform Techniques


526 21.6. Exercises

∂u
= k ∂∂xu2 , for x ≥ 0, t ≥ 0 subject to the conditions u(x, 0) = u0 , t > 0 with initial
2
2. Solve ∂t

condition u(x, 0) = 0, x ≥ 0.
∂u ∂2 u ∂u ∂u
3. Solve ∂t
= ∂x2
, 0 < x < 6, t > 0, given ∂x
(0, t) = 0, ∂x
(6, t) = 0 and u(x, 0) = 2x.
∂u ∂2 u
4. Solve ∂t
= ∂x2
, 0 < x < π, t > 0, given that u(0, t) = 0, u(π, t) = 0 for t > 0 and
u(x, 0) = sin3 x.
∂u
= 2 ∂∂xu2 , 0 < x < 1, t > 0, given that u(0, t) = 0, u(1, t) = 0 for t > 0 and
2
5. Solve ∂t

u(x, 0) = sin 3πx + sin πx.

Answers
R∞ √
1. u(x, t) = f (x − 2 kt φ)e−φ dφ.
2
√1
π −∞
R ∞  1−e−ks2 t 
2. u(x, t) = 2u0
π 0 s
sin pxd p.
2 2
(cos pπ−1) − p π t
 pπx 
3. u(x, t) = 6 + .
24 P∞
π2 p=1 p2
e 36 cos 6

4. u(x, t) = 34 e−t sin x − 14 e−9t sin 3x.

5. u(x, t) = e−2π t sin πx + e−18π t sin 3πx.


2 2

Transform Techniques B.Sc.(Mathematics)-IDE(UNOM)-II Year-IV Sem


MODEL QUESTION PAPER

B.Sc. Mathematics
Second Year - Fourth Semester
Core Paper-VII
TRANSFORM TECHNIQUES
Time: Three Hours Maximum: 75 Marks

PART-A ( 10 × 2 = 20 Marks)
Answer any TEN out of TWELVE questions
Each question carries TWO marks

1. Find the Laplace transform of sin 2t cos t.

t .
sin t
2. Find the Laplace transform of
2p+1
3. Find the inverse Laplace transform of p(p+1) .
h i
4. Find L −1 1
p2 +2p+5
.

5. State the sufficient conditions for the existence of Fourier series.


6. If x − x2 = − π3 + 4 cos
2
h i h i
1 2
x
− cos 2x
2 2 + cos 3x
3 2 − · · · + 2 sin x
1 − sin 2x
2 + sin 3x
3 − · · · in (−π, π), deduce the
P∞ (−1)n−1
value of n=1 n2 .

7. If f (p) is the complex


  Fourier transform of f (x), prove that
F [ f (ax)] = a1 f ap , a > 0.

8. State Fourier integral theorem.


d2 y dy dy
9. Find the Laplace transform of the differential equation dx2
+ 2 dx + 2y = 0, given y = dx = 1 when
t = 0.
10. Define Volterra’s integral equation.
11. State and prove the first shifting theorem.
12. Find the half - range sine series of f (x) = x, 0 < x < π.

B.Sc.(Mathematics)-IDE(UNOM)-II Year-IV Sem 527 Transform Techniques


528 Model Question Paper

PART - B (5 × 5 = 25Marks)
Answer any FIVE of SEVEN Questions.
Each question carries SIX marks.
h Rt i
13. Find L e−3t 0 1−cos at
t dt .

14. State and prove convolution theorem in Laplace transform.

15. Find the Fourier series to represent the function f (x) = | sin x|, in −π < x < π.
x2
16. Find the Fourier transform of f (x) defined by f (x) = e− 2 .
∂Y ∂Y
17. Solve ∂x − ∂t = 1 − e−t , 0 < x < 1, t > 0 given that Y(x, 0) = x.
p2
h i
18. Find L −1 (p+2)3 ] .

19. Find the Fourier cosine transform of the function f (x) defined by

0<x<a
(
cos x,
f (x) =
0, x ≥ a.

PART - C (3 × 10 = 30 Marks)
Answer any THREE out of FIVE questions.
Each question carries TEN marks.

20. Find L [F(t)], where F(t) is given by

0<t<b
(
t,
F(t) =
2b − t, b < t < 2b,

2b being the period of F(t).


h i
21. Find L −1 (p+1)21(p2 +4) .

22. Find the Fourier series of periodicity 3 for f (x) = 2x − x3 in 0 < x < 3.

23. Find the Fourier transform of

1 − |x|, when |x| < 1


(
f (x) =
0, when |x| > 1.
R ∞  sin x 4
Hence show that 0 x dx = π3 .

d2 x
dt − 12x = e
− 4 dx given that x(0) = 1, and x0 (0) = −2 using
3t
24. Solve the differential equation dt2
Laplace transform.

zzzzz

Transform Techniques B.Sc.(Mathematics)-IDE(UNOM)-II Year-IV Sem

You might also like